You are on page 1of 629

‫قَالُوا سُبِحَانَكَ لَا عِلِمَ َلنَا إِلَّا مَا عََّلمِ َتنَا‬

‫ۖ ِإنَّكَ َأنِتَ الِعَلِي ُ‬


‫م الِحَكِيمُ‬
‫‪‬إهداء‪‬‬
‫لكن آٍْ من كتاب اهلل حفظناها ًلكن معمٌمْ قسأناها ًنشَناها لعن اهلل حيمَنا من النشَان ًٍسدها إلَنا‬
‫لكن طخص دعٌنا لى يف ظوسالغَب لعن اهلل أن ٍشتجَب دعاٖنا‬
‫لكن مسٍض قد آملى ما أملى لعن اهلل ٍظفَى ًٍعَد إلَى صحتى ًعافَتى‬
‫لكن حلظْ تآمل بوا أحبتنا لعن اهلل ال ٍسٍوم اآلمل ًال ٍضَموم مسّ أخسٔ‬
‫لكن مٌقف احتجت بى عَناِ فمم تمبَانُ لعن اهلل ال ٍضسنُ بوما يف حَاتُ‬
‫لكن حامل صاحب أهداف ًمبادئ لعن اهلل حيكل لى مكاصدي حتٓ ٍسضَى ًٍسضٓ عنى‬
‫لكن دلتود لعن اهلل جيعن لى من التٌفَل أًفس حظ ً نصَب ًٍوٌن لى األسباب كموا من حَث ال ٍدزِ من غري ضسإ مضسّ ًال فتنْ مضمى‬
‫لكن من فكد طَٗا يف احلَاّ لعن اهلل ٍعَدي إلَى أً ٍعٌضى مبا ٍشعدي‬
‫لكن طخص ابتعدنا عنى لعن اهلل جيمعى مبن هم أفضن منا‬
‫ًأخريا ًلَص آخسا ألهمنا لعن اهلل جيعموا هلم صدقْ جازٍى تسد إلَوم قمَال من فضموم عمَنا ًتكٌن سببا ٍمم مشمنا حتت سكف ًاحد بعد أن‬
‫هجَّ َستنا احلسًب ًاملصاٖب‬

‫يقول أحد الصالحين‬


‫إذا زأٍت أن اهلل ًفكك إلخٌّ تعَنك عمٓ اخلري ‪ ..‬فأعمم أ َّن اهلل ٍسٍد بك خرياً !! ًإذا ًفكك اهلل لمدعإ ‪ ..‬فأعمم أنَّى ٍسٍد أنْ ٍعطَك !! ًإذا‬
‫ًفكك لكضإ حٌاٖج الناض فاعمم بأنى لن ٍتخمٓ عنك !!ًإذا ًفكك اهلل لركسي ‪ ..‬فأعمم أنَّى حيبَّك !!‪ً ..‬إذا أحبَّك أعزك ‪ً ..‬نصسك ‪ً ..‬أٍدك ‪..‬‬
‫ًأستجاب لدعاٖك !!‬
This issue of MrcpPART1 Made Easy 2017
contains the following:
- Full topic note.
- Around 2600 MCQs from PM 2017 bank covering 14
chapter in internal medicine
- External links for the related guidelines (such as NICE, SIGN, RCP …etc).
- Comments with illustrative pictures and links to external media.
- Most Recommended QBank for MRCP Part 1, Arab Board, Jordanian
Board and other internal medicine assessment exams.

Please Note that


Each chapter begins with notes section. These notes are repeated under the
Questions in the website. In these collections the notes are not repeated under
each question except in some to avoid unnecessary repetition. For clarity and
convenience the title of the related note is only mentioned after the comments
in case the reader want to refer back to the respective note in the notes section.

The illustration below is to show and Example


MrcpPART1 Made Easy 2017
Chapter Number of Q
Cardiology 292
Clinical haematology/oncology 171
Endocrinology 174
Gastroenterology 183
Infectious diseases and STIs 212
Nephrology 104
Neurology 247
Respiratory medicine 162
Rheumatology 134
Clinical pharmacology and
toxicology 273
Clinical sciences 410
Dermatology 131
Ophthalmology 56
Psychiatry 59

‫إعداد وتنسيق‬
M. Habayeb A. Murad
Msc Internal. Medicine
Cairo University

‫ال تىسىوا مه صالح دعائكم‬


‫ما كان مه خطأ فمه أوفسىا وما كان مه تىفيق فمه هللا‬
‫تم بحمد هللا وتىفيقه ومىه‬
‫جعل هللا عملىا متقبال خالصا لىجهه الكريم‬
Reference ranges
Reference ranges vary according to individual labs. All values are for adults unless otherwise stated

Full blood count

Haemoglobin Men: 13.5­18 g/dl Women: 11.5­16 g/dl

Mean cell volume 82­100 fl

Platelets 150­400 * 109/l

White blood cells 4­11 * 109/l|

Urea and electrolytes

Sodium 135­145 mmol/l

Potassium 3.5 ­ 5.0 mmol/l

Urea 2.0­7 mmol/l

Creatinine 55­120 umol/l

Bicarbonate 22­28 mmol/l

Chloride 95­105 mmol/l

Liver function tests

Bilirubin 3­17 umol/l

Alanine transferase (ALT) 3­40 iu/l

Aspartate transaminase (AST) 3­30 iu/l

Alkaline phosphatase (ALP) 30­100 umol/l

Gamma glutamyl transferase (yGT) 8­60 u/l

Albumin 35­50 g/l

Total protein 60­80 g/l

Other haematology

Erythrocyte sedimentation rate (ESR) Men: < (age / 2) mm/hr Women: < ((age + 10) / 2) mm/hr

Prothrombin time (PT) 10­14 secs

Activated partial thromboplastin time (APTT) 25­35 secs

Ferritin 20­230 ng/ml

Vitamin B12 200­900 ng/l

Folate 3.0 nmol/l

Reticulocytes 0.5­1.5%

D­Dimer < 400 ng/ml

Other biochemistry

Calcium 2.1­2.6 mmol/l

Phosphate 0.8­1.4 mmol/l

CRP < 10 mg/l

Thyroid stimulating hormone (TSH) 0.5­5.5 mu/l

Free thyroxine (T4) 9­18 pmol/l

Total thyroxine (T4) 70­140 nmol/l

Amylase 70­300 u/l


Uric acid 0.18­0.48 mmol/l

Arterial blood gases

pH 7.35 ­ 7.45

pCO2 4.5 ­ 6.0 kPa

pO2 10 ­ 14 kPa

Lipids

Desirable lipid values depend on other risk factors for cardiovascular disease, below is just a guide:

Total cholesterol < 5 mmol/l

Triglycerides < 2 mmol/l

HDL cholesterol > 1 mmol/l

LDL cholesterol < 3 mmol/l


Chapter: Neurology

Absence seizures Notes

Absent ankle jerks, extensor plantars Notes

Acoustic neuroma Notes

Acute confusional state Notes

Alzheimer's disease Notes

Aphasia Notes

Ataxic telangiectasia Notes

Atrial septal defects Notes

Autonomic neuropathy Notes

Bell's palsy Notes

Benign paroxysmal positional vertigo Notes

Brain lesions Notes

Carbamazepine Notes

Cataplexy Notes

Cerebellar syndrome Notes

Cerebrospinal fluid: raised lymphocytes Notes

Cerebrospinal fluid: raised protein Notes

Chorea Notes

Cluster headache Notes

Cranial nerves Notes

Creutzfeldt-Jakob disease Notes

Dementia Notes

Dementia: causes Notes

Dermatomes Notes

Diabetes insipidus Notes

Drugs causing peripheral neuropathy Notes

DVLA: neurological disorders Notes

1
MRCP part 1 Made Easy‫‏‬‎‫‏‬by M. Habayeb & A. Murad
Chapter: Neurology

Eclampsia Notes

Epilepsy in children: syndromes Notes

Epilepsy: classification Notes

Epilepsy: pregnancy and breast feeding Notes

Epilepsy: treatment Notes

Essential tremor Notes

Ethosuximide Notes

Facial nerve Notes

Friedreich's ataxia Notes

Frontotemporal lobar degeneration Notes

Guillain-Barre syndrome Notes

Guillain-Barre syndrome: features Notes

Guillain-Barre syndrome: management Notes

Guillain-Barre syndrome: prognosis Notes

Head injury: NICE guidance Notes

Head injury: types of traumatic brain injury Notes

Headache Notes

Hemiballism Notes

Herpes simplex encephalitis Notes

Hodgkin's lymphoma: histological classification and prognosis Notes

Horner's syndrome Notes

HSMN Notes

Huntington's disease Notes

Idiopathic intracranial hypertension Notes

Intracranial venous thrombosis Notes

Lambert-Eaton syndrome Notes

Lateral medullary syndrome Notes

2
MRCP part 1 Made Easy‫‏‬‎‫‏‬by M. Habayeb & A. Murad
Chapter: Neurology

Levodopa Notes

Lewy body dementia Notes

Medication overuse headache Notes

Meniere's disease Notes

Meralgia paraesthetica Notes

Migraine: diagnostic criteria Notes

Migraine: pregnancy, contraception and other hormonal Notes


factors

Miosis Notes

Motor neuron disease: features Notes

Motor neuron disease: management Notes

Motor neuron disease: types Notes

Multiple sclerosis: features Notes

Multiple sclerosis: management Notes

Multiple sclerosis: prognostic features Notes

Multiple system atrophy Notes

Myasthenia gravis Notes

Myasthenia gravis: exacerbating factors Notes

Myotonic dystrophy Notes

Nerve conduction studies Notes

Neuropathic pain Notes

Normal pressure hydrocephalus Notes

Nystagmus Notes

Paraneoplastic syndromes affecting nervous system Notes

Parkinson's disease: features Notes

Parkinson's disease: management Notes

Parkinsonism Notes

3
MRCP part 1 Made Easy‫‏‬‎‫‏‬by M. Habayeb & A. Murad
Chapter: Neurology

Peripheral neuropathy Notes

Peripheral neuropathy: demyelinating vs. axonal Notes

Pituitary apoplexy Notes

Post-lumbar puncture headache Notes

Progressive supranuclear palsy Notes

Ptosis Notes

Restless legs syndrome Notes

Reye's syndrome Notes

Rinne's and Weber's test Notes

Spastic paraparesis Notes

Spinal cord compression Notes

Spinal cord lesions Notes

Stroke by anatomy Notes

Stroke: management Notes

Stroke: types Notes

Subarachnoid haemorrhage Notes

Subdural haemorrhage Notes

Syringomyelia Notes

Tinnitus Notes

Transient global amnesia Notes

Transient ischaemic attack Notes

Trigeminal neuralgia Notes

Triptans Notes

Tuberous sclerosis Notes

Vigabatrin Notes

Visual field defects Notes

Von Hippel-Lindau syndrome Notes

4
MRCP part 1 Made Easy‫‏‬‎‫‏‬by M. Habayeb & A. Murad
Chapter: Neurology

Absence seizures

Absence seizures (petit mal) are a form of generalised epilepsy that is mostly seen in children.
The typical age of onset of 3-10 years old and girls are affected twice as commonly as boys

Features
 absences last a few seconds and are associated with a quick recovery
 seizures may be provoked by hyperventilation or stress
 the child is usually unaware of the seizure
 they may occur many times a day
 EEG: bilateral, symmetrical 3Hz spike and wave pattern

Management

 sodium valproate and ethosuximide are first-line treatment


 good prognosis - 90-95% become seizure free in adolescence

External Links:
NICE
2012 Epilepsy guidelines

Absent ankle jerks, extensor plantars

Typically caused by lesion producing both upper motor neuron (extensor plantars) and lower
motor neuron (absent ankle jerk) signs

Causes:
 subacute combined degeneration of the cord
 motor neuron disease
 Friedreich's ataxia
 syringomyelia
 taboparesis (syphilis)
 conus medullaris lesion

5
MRCP part 1 Made Easy‫‏‬‎‫‏‬by M. Habayeb & A. Murad
Chapter: Neurology

Acoustic neuroma

Acoustic neuromas (more correctly called vestibular schwannomas) account for


approximately five percent of intracranial tumours and 90 percent of cerebellopontine angle

Features can be predicted by the affected cranial nerves:


 cranial nerve VIII: hearing loss, vertigo, tinnitus
 cranial nerve V: absent corneal reflex
 cranial nerve VII: facial palsy

Bilateral acoustic neuromas are seen in neurofibromatosis type 2

MRI of the cerebellopontine angle is the investigation of choice

Acute confusional state

Acute confusional state is also known as delirium or acute organic brain syndrome. It affects
up to 30% of elderly patients admitted to hospital.

Features - wide variety of presentations


 memory disturbances (loss of short term > long term)
 may be very agitated or withdrawn
 disorientation
 mood change
 visual hallucinations
 disturbed sleep cycle
 poor attention
Management

 treatment of underlying cause

 modification of environment

 the 2006 Royal College of Physicians publication 'The prevention, diagnosis and
management of delirium in older people: concise guidelines' recommended
haloperidol 0.5 mg as the first-line sedative

 the 2010 NICE delirium guidelines advocate the use of haloperidol or olanzapine

6
MRCP part 1 Made Easy‫‏‬‎‫‏‬by M. Habayeb & A. Murad
Chapter: Neurology

External Links:

NICE

2010 Delirium guidelines

RCP

Delirium guidelines

Alzheimer's disease

Alzheimer's disease is a progressive degenerative disease of the brain accounting for the
majority of dementia seen in the UK

Genetics

 most cases are sporadic

 5% of cases are inherited as an autosomal dominant trait

 mutations in the amyloid precursor protein (chromosome 21), presenilin 1


(chromosome 14) and presenilin 2 (chromosome 1) genes are thought to cause the
inherited form

 apoprotein E allele E4 - encodes a cholesterol transport protein

Pathological changes

 macroscopic: widespread cerebral atrophy, particularly involving the cortex and


hippocampus

 microscopic: cortical plaques due to deposition of type A-Beta-amyloid protein and


intraneuronal neurofibrillary tangles caused by abnormal aggregation of the tau
protein

 biochemical: there is a deficit of acetylecholine from damage to an ascending


forebrain projection

Neurofibrillary tangles

 paired helical filaments are partly made from a protein called tau

 in AD are tau proteins are excessively phosphorylated.

7
MRCP part 1 Made Easy‫‏‬‎‫‏‬by M. Habayeb & A. Murad
Chapter: Neurology

Management

 NICE now recommend the three acetylcholinesterase inhibitors (donepezil,


galantamine and rivastigmine) as options for managing mild to moderate Alzheimer's
disease

 memantine (a NMDA receptor antagonist) is reserved for patients with moderate -


severe Alzheimer's

External Links:

NICE

2011 Dementia guidelines

Aphasia

The table below lists the major types of aphasia. Remember that dysarthria is different and
refers to a motor speech disorder.

Type of aphasia Notes


Wernicke's Due to a lesion of the superior temporal gyrus
(receptive)
aphasia This area 'forms' the speech before 'sending it' to
Broca's area. Lesions result in sentences that
make no sense, word substitution and
neologisms but speech remains fluent

Comprehension is impaired
Broca's Due to a lesion of the inferior frontal gyrus
(expressive)
aphasia Speech is non-fluent, laboured, and halting

Comprehension is normal
Conduction Classically due to a stroke affecting the arcuate
aphasia fasiculus - the connection between Wernicke's
and Broca's area

Speech is fluent but repetition is poor. Aware of


the errors they are making

Comprehension is normal
Global aphasia Large lesion affecting all 3 of the above areas
resulting in severe expressive and receptive
aphasia

8
MRCP part 1 Made Easy‫‏‬‎‫‏‬by M. Habayeb & A. Murad
Chapter: Neurology

Ataxic telangiectasia

Ataxic telangiectasia is an autosomal recessive disorder caused by a defect in the ATM gene
which encodes for DNA repair enzymes. It is one of the inherited combined
immunodeficiency disorders. It typical presents in early childhood with abnormal movements.

Features

 cerebellar ataxia

 telangiectasia (spider angiomas)

 IgA deficiency resulting in recurrent chest infections

 10% risk of developing malignancy, lymphoma or leukaemia, but also non-lymphoid


tumours

Comparison of Friedreich's ataxia and ataxic telangiectasia. Note in particular how ataxic
telangiectasia tends to present much earlier, often at the age of 1-2 years

9
MRCP part 1 Made Easy‫‏‬‎‫‏‬by M. Habayeb & A. Murad
Chapter: Neurology

Atrial septal defects

Atrial septal defects (ASDs) are the most likely congenital heart defect to be found in
adulthood. They carry a significant mortality, with 50% of patients being dead at 50 years.
Two types of ASDs are recognised, ostium secundum and ostium primum. Ostium secundum
are the most common

Features

 ejection systolic murmur, fixed splitting of S2


 embolism may pass from venous system to left side of heart causing a stroke

Ostium secundum (70% of ASDs)

 associated with Holt-Oram syndrome (tri-phalangeal thumbs)


 ECG: RBBB with RAD

Ostium primum

 present earlier than ostium secundum defects


 associated with abnormal AV valves
 ECG: RBBB with LAD, prolonged PR interval

Autonomic neuropathy

Features

 impotence, inability to sweat, postural hypotension


 postural hypotension e.g. drop of 30/15 mmHg
 loss of decrease in heart rate following deep breathing
 pupils: dilates following adrenaline instillation.

10
MRCP part 1 Made Easy‫‏‬‎‫‏‬by M. Habayeb & A. Murad
Chapter: Neurology

Causes
 diabetes
 Guillain-Barre syndrome
 multisystem atrophy (MSA), Shy-Drager syndrome
 Parkinson's
 infections: HIV, Chagas' disease, neurosyphilis
 drugs: antihypertensives, tricyclics
 craniopharyngioma

Bell's palsy

Bell's palsy may be defined as an acute, unilateral, idiopathic, facial nerve paralysis. The
aetiology is unknown although the role of the herpes simplex virus has been investigated
previously. The peak incidence is 20-40 years and the condition is more common in pregnant
women.

Features
 lower motor neuron facial nerve palsy - forehead affected*
 patients may also notice post-auricular pain (may precede paralysis), altered taste, dry
eyes, hyperacusis

Management
 in the past a variety of treatment options have been proposed including no treatment,
prednisolone only and a combination of aciclovir and prednisolone
 following a National Institute for Health randomised controlled trial it is now
recommended that prednisolone 1mg/kg for 10 days should be prescribed for patients
within 72 hours of onset of Bell's palsy. Adding in aciclovir gives no additional
benefit
 eye care is important - prescription of artificial tears and eye lubricants should be
considered

Prognosis
 if untreated around 15% of patients have permanent moderate to severe weakness

*upper motor neuron lesion 'spares' upper face

11
MRCP part 1 Made Easy‫‏‬‎‫‏‬by M. Habayeb & A. Murad
Chapter: Neurology

External Links:

Clinical Knowledge Summaries

Bell's palsy guidelines

Benign paroxysmal positional vertigo

♦Benign paroxysmal positional vertigo (BPPV) is one of the most common causes of vertigo
encountered. It is characterised by the sudden onset of dizziness and vertigo triggered by
changes in head position. The average age of onset is 55 years and it is less common in
younger patients.

Features

 vertigo triggered by change in head position (e.g. rolling over in bed or gazing
upwards)

 may be associated with nausea

 each episode typically lasts 10-20 seconds

 positive Dix-Hallpike manoeuvre

BPPV has a good prognosis and usually resolves spontaneously after a few weeks to months.
Symptomatic relief may be gained by:

 Epley manoeuvre (successful in around 80% of cases)

 teaching the patient exercises they can do themselves at home, for example Brandt-
Daroff exercises

Medication is often prescribed (e.g. Betahistine) but it tends to be of limited value.

External Links:

Clinical Knowledge Summaries

Benign paroxysmal positional vertigo guidelines

YouTube

Hallpike Test and Epley Maneuver

12
MRCP part 1 Made Easy‫‏‬‎‫‏‬by M. Habayeb & A. Murad
Chapter: Neurology

Brain lesions

The following neurological disorders/features may allow localisation of a brain lesion:


Gross anatomy

Parietal lobe lesions:


 sensory inattention
 apraxias
 astereognosis (tactile agnosia)
 inferior homonymous quadrantanopia
 Gerstmann's syndrome (lesion of dominant parietal): alexia, acalculia, finger agnosia
and right-left disorientation

Occipital lobe lesions:


 homonymous hemianopia (with macula sparing)
 cortical blindness
 visual agnosia

Temporal lobe lesion:


 Wernicke's aphasia: this area 'forms' the speech before 'sending it' to Brocas area.
Lesions result in word substituion, neologisms but speech remains fluent
 superior homonymous quadrantanopia
 auditory agnosia
 prosopagnosia (difficulty recognising faces)
Frontal lobes lesions:
 expressive (Broca's) aphasia: located on the posterior aspect of the frontal lobe, in the
inferior frontal gyrus. Speech is non-fluent, laboured, and halting
 disinhibition
 perseveration
 anosmia
 inability to generate a list.

Cerebellum lesions:
 midline lesions: gait and truncal ataxia
 hemisphere lesions: intention tremor, past pointing, dysdiadokinesis, nystagmus.
13
MRCP part 1 Made Easy‫‏‬‎‫‏‬by M. Habayeb & A. Murad
Chapter: Neurology

More specific areas

Area Associated conditions


Medial thalamus and Wernicke and Korsakoff syndrome
mammillary bodies of the
hypothalamus
Subthalamic nucleus of the Hemiballism
basal ganglia
Striatum (caudate nucleus) Huntington chorea
of the basal ganglia
Substantia nigra of the basal Parkinson's disease
ganglia
Amygdala Kluver-Bucy syndrome
(hypersexuality, hyperorality,
hyperphagia, visual agnosia

External Links:
American Speech-Language-Hearing Association
Common Classifications of Aphasia

Carbamazepine

♦Carbamazepine is chemically similar to the tricyclic antidepressant drugs. It is most


commonly used in the treatment of epilepsy, particularly partial seizures, where
carbamazepine remains a first-line medication. Other uses include:
 neuropathic pain (e.g. trigeminal neuralgia, diabetic neuropathy)
 bipolar disorder

Mechanism of action
 binds to sodium channels increases their refractory period

Adverse effects:
 P450 enzyme inducer
 dizziness and ataxia
 drowsiness
 headache
 visual disturbances (especially diplopia)
 Steven-Johnson syndrome
 leucopenia and agranulocytosis
 syndrome of inappropriate ADH secretion
14
MRCP part 1 Made Easy‫‏‬‎‫‏‬by M. Habayeb & A. Murad
Chapter: Neurology

Cataplexy

Cataplexy describes the sudden and transient loss of muscular tone caused by strong emotion
(e.g. laughter, being frightened). Around two-thirds of patients with narcolepsy have
cataplexy.

Features range from buckling knees to collapse.

Cerebellar syndrome

Unilateral cerebellar lesions cause ipsilateral signs.

Causes:

 Friedreich's ataxia, ataxic telangiectasia

 neoplastic: cerebellar haemangioma

 stroke

 alcohol

 multiple sclerosis

 hypothyroidism

 drugs: phenytoin, lead poisoning

 paraneoplastic e.g. secondary to lung cancer

Cerebrospinal fluid: raised lymphocytes

Normal values of cerebrospinal fluid (CSF) are as follows:


 pressure = 60-150 mm (patient recumbent)
 protein = 0.2-0.4 g/l
 glucose = > 2/3 blood glucose
 cells: red cells = 0, white cells < 5/mm³.

15
MRCP part 1 Made Easy‫‏‬‎‫‏‬by M. Habayeb & A. Murad
Chapter: Neurology

The following conditions are associated with raised lymphocytes:


 viral meningitis/encephalitis
 TB meningitis
 partially treated bacterial meningitis
 Lyme disease
 Behcet's, SLE
 lymphoma, leukaemia

Cerebrospinal fluid: raised protein

Normal values of cerebrospinal fluid (CSF) are as follows:


 pressure = 60-150 mm (patient recumbent)
 protein = 0.2-0.4 g/l
 glucose = > 2/3 blood glucose
 cells: red cells = 0, white cells < 5/mm³

The following conditions are associated with raised protein levels:


 Guillain-Barre syndrome
 tuberculous, fungal and bacterial meningitis
 spinal block (Froin's syndrome*)
 viral encephalitis
*describes an increase in CSF protein below a spinal canal blockage (e.g. tumour, disc,
infection)

16
MRCP part 1 Made Easy‫‏‬‎‫‏‬by M. Habayeb & A. Murad
Chapter: Neurology

Chorea

Chorea describes involuntary, rapid, jerky movements which often move from one part of the
body to another. Slower, sinuous movement of the limbs is termed athetosis. Chorea is caused
by damage to the basal ganglia, especially the caudate nucleus.

Causes of chorea

 Huntington's disease, Wilson's disease, ataxic telangiectasia

 SLE, anti-phospholipid syndrome

 rheumatic fever: Sydenham's chorea

 drugs: oral contraceptive pill, L-dopa, antipsychotics

 neuroacanthocytosis

 pregnancy: chorea gravidarum

 thyrotoxicosis

 polycythaemia rubra vera

 carbon monoxide poisoning

 cerebrovascular disease

External Links:

Postgraduate Medical Journal

Review of chorea

17
MRCP part 1 Made Easy‫‏‬‎‫‏‬by M. Habayeb & A. Murad
Chapter: Neurology

Cluster headache

Cluster headaches* are more common in men (5:1) and smokers.

Features:

 pain typical occurs once or twice a day, each episode lasting 15 mins - 2 hours

 clusters typically last 4-12 weeks

 intense pain around one eye (recurrent attacks 'always' affect same side)

 patient is restless during an attack

 accompanied by redness, lacrimation, lid swelling

 nasal stuffiness

 miosis and ptosis in a minority

Management:

 acute: 100% oxygen, subcutaneous or a nasal triptan

 prophylaxis: verapamil, prednisolone

 NICE recommend seeking specialist advice from a neurologist if a patient develops


cluster headaches with respect to neuroimaging

*some neurologists use the term trigeminal autonomic cephalgia to group a number of
conditions including cluster headache, paroxysmal hemicrania and short-lived unilateral
neuralgiform headache with conjunctival injection and tearing (SUNCT). It is recommended
such patients are referred for specialist assessment as specific treatment may be required, for
example it is known paroxysmal hemicrania responds very well to indomethacin

External Links:

NICE

2012 Headache guideline

SIGN

2008 Diagnosis and management of headache in adults

18
MRCP part 1 Made Easy‫‏‬‎‫‏‬by M. Habayeb & A. Murad
Chapter: Neurology

Cranial nerves

The table below lists the major characteristics of the 12 cranial nerves:

Nerve Functions Clinical Pathway/foramen


I (Olfactory) Smell Cribriform plate
II (Optic) Sight Optic canal
III (Oculomotor) Eye Palsy results in Superior orbital
movement fissure (SOF)
(MR, IO, SR,  ptosis
IR)  'down and out'
Pupil eye
constriction  dilated, fixed
Accomodation pupil
Eyelid
opening
IV (Trochlear) Eye Palsy results in defective SOF
movement downward gaze →
(SO) vertical diplopia
V (Trigeminal) Facial Lesions may cause: V1: SOF, V2:
sensation Foramen
Mastication  trigeminal rotundum,
neuralgia V3: Foramen ovale
 loss of corneal
reflex (afferent)
 loss of facial
sensation
 paralysis of
mastication
muscles
 deviation of jaw
to weak side
VI (Abducens) Eye Palsy results in defective SOF
movement abduction → horizontal
(LR) diplopia
VII (Facial) Facial Lesions may result in: Internal auditory
movement meatus
Taste (anterior  flaccid paralysis
2/3rds of of upper + lower
tongue) face
Lacrimation  loss of corneal
Salivation reflex (efferent)
 loss of taste
 hyperacusis
VIII Hearing, Hearing loss Internal auditory
(Vestibulocochlear) balance Vertigo, nystagmus meatus
Acoustic neuromas are
Schwann cell tumours of
the cochlear nerve

19
MRCP part 1 Made Easy‫‏‬‎‫‏‬by M. Habayeb & A. Murad
Chapter: Neurology

IX Taste Lesions may result in; Jugular foramen


(Glossopharyngeal) (posterior
1/3rd of  hypersensitive
tongue) carotid sinus
Salivation reflex
Swallowing  loss of gag reflex
Mediates (afferent)
input from
carotid body
& sinus
X (Vagus) Phonation Lesions may result in; Jugular foramen
Swallowing
Innervates  uvula deviates
viscera away from site of
lesion
 loss of gag reflex
(efferent)
XI (Accessory) Head and Lesions may result in; Jugular foramen
shoulder
movement  weakness turning
head to
contralateral side
XII (Hypoglossal) Tongue Tongue deviates towards Hypoglossal canal
movement side of lesion

Some cranial nerves are motor, some sensory and some are both. The most useful mnemonic
is given below.

CN I ----------------------------------------------------------------------→XII

Some Say Marry Money But My Brother Says Big Brains Matter Most

S = Sensory, M = Motor, B = Both

20
MRCP part 1 Made Easy‫‏‬‎‫‏‬by M. Habayeb & A. Murad
Chapter: Neurology

Image sourced from Wikipedia

View from the inferior surface of the brain showing the emergence of the cranial nerves

21
MRCP part 1 Made Easy‫‏‬‎‫‏‬by M. Habayeb & A. Murad
Chapter: Neurology

Image sourced from Wikipedia

Diagram showing the nuclei of the cranial nerves in the brainstem

Cranial nerve reflexes

Reflex Afferent limb Efferent limb


Corneal Ophthalmic nerve (V1) Facial nerve (VII)
Jaw jerk Mandibular nerve (V3) Mandibular nerve
(V3)
Gag Glossopharyngeal nerve Vagal nerve (X)
(IX)
Carotid sinus Glossopharyngeal nerve Vagal nerve (X)
(IX)
Pupillary Optic nerve (II) Oculomotor nerve
light (III)
Lacrimation Ophthalmic nerve (V1) Facial nerve (VII)

22
MRCP part 1 Made Easy‫‏‬‎‫‏‬by M. Habayeb & A. Murad
Chapter: Neurology

Creutzfeldt-Jakob disease

Creutzfeldt-Jakob disease (CJD) is rapidly progressive neurological condition caused by prion


proteins. These proteins induce the formation of amyloid folds resulting in tightly packed
beta-pleated sheets resistant to proteases.

Features
 dementia (rapid onset)
 myoclonus

Investigation
 CSF is usually normal
 EEG: biphasic, high amplitude sharp waves (only in sporadic CJD)
 MRI: hyperintense signals in the basal ganglia and thalamus

Sporadic CJD

 accounts for 85% of cases


 10-15% of cases are familial
 mean age of onset is 65 years

New variant CJD


 younger patients (average age of onset = 25 years)
 psychological symptoms such as anxiety, withdrawal and dysphonia are the most
common presenting features
 the 'prion protein' is encoded on chromosome 20 - it's role is not yet understood
 methionine homozygosity at codon 129 of the prion protein is a risk factor for
developing CJD - all patients who have so far died have had this
 median survival = 13 months

Other prion diseases

 kuru

 fatal familial insomnia

 Gerstmann Straussler-Scheinker disease

23
MRCP part 1 Made Easy‫‏‬‎‫‏‬by M. Habayeb & A. Murad
Chapter: Neurology

Dementia

Dementia is thought to affect over 700,000 people in the UK and accounts for a large amount
of health and social care spending. The most common cause of dementia in the UK is
Alzheimer's disease followed by vascular and Lewy body dementia. These conditions may
coexist.

Features

 diagnosis can be difficult and is often delayed

 assessment tools include the Abbreviated mental test score (AMTS), 6-Item cognitive
impairment test (6CIT), General practitioner assessment of cognition (GPCOG) and
the mini-mental state examination (MMSE) is widely used. A MMSE score of 24 or
less out of 30 suggests dementia

Management

 in primary care a blood screen is usually sent to exclude reversible causes (e.g.
Hypothyroidism). NICE recommend the following tests: FBC, U&E, LFTs, calcium,
glucose, TFTs, vitamin B12 and folate levels. Patients are now commonly referred on
to old-age psychiatrists (sometimes working in 'memory clinics').

 in secondary care neuroimaging is performed* to exclude other reversible conditions


(e.g. Subdural haematoma, normal pressure hydrocephalus) and help provide
information on aetiology to guide prognosis and management

*in the 2011 NICE guidelines structural imaging was said to be essential in the investigation
of dementia

External Links:

NICE

2011 Dementia guidelines

Alzheimers Society

Helping you to assess cognition - A practical toolkit for clinicians

24
MRCP part 1 Made Easy‫‏‬‎‫‏‬by M. Habayeb & A. Murad
Chapter: Neurology

Dementia: causes

Common causes

 Alzheimer's disease

 cerebrovascular disease: multi-infarct dementia (c. 10-20%)

 Lewy body dementia (c. 10-20%)

Rarer causes (c. 5% of cases):

 Huntington's

 CJD

 Pick's disease (atrophy of frontal and temporal lobes)

 HIV (50% of AIDS patients)

Important differentials, potentially treatable:

 hypothyroidism, Addison's

 B12/folate/thiamine deficiency

 syphilis

 brain tumour

 normal pressure hydrocephalus

 subdural haematoma

 depression

 chronic drug use e.g. Alcohol, barbiturates

25
MRCP part 1 Made Easy‫‏‬‎‫‏‬by M. Habayeb & A. Murad
Chapter: Neurology

Dermatomes

The table below lists the major dermatome landmarks:

Nerve Landmark Mnemonics


root
C2 Posterior half of the
skull (cap)
C3 High turtleneck shirt
C4 Low-collar shirt
C5, C6 Thumb + index finger Make a 6 with your left hand by
touching the tip of the thumb &
index finger together - C6
C7 Middle finger + palm
of hand
C8 Ring + little finger
T4 Nipples T4 at the Teat Pore
T5 Inframammary fold
T7 Xiphoid process
T10 Umbilicus BellybuT-TEN
L1 Inguinal ligament L for ligament, 1 for 1nguinal
L4 Knee caps Down on aLL fours - L4
L5 Big toe, dorsum of L5 = Largest of the 5 toes
foot (except lateral
aspect)
S1 Lateral foot, small toe S1 = the smallest one
S2, S3 Genitalia

26
MRCP part 1 Made Easy‫‏‬‎‫‏‬by M. Habayeb & A. Murad
Chapter: Neurology

27
MRCP part 1 Made Easy‫‏‬‎‫‏‬by M. Habayeb & A. Murad
Chapter: Neurology

28
MRCP part 1 Made Easy‫‏‬‎‫‏‬by M. Habayeb & A. Murad
Chapter: Neurology

Diabetes insipidus

Diabetes insipidus (DI) is a condition characterised by either a deficiency of antidiuretic


hormone, ADH, (cranial DI) or an insensitivity to antidiuretic hormone (nephrogenic DI).

Causes of cranial DI

 idiopathic

 post head injury

 pituitary surgery

 craniopharyngiomas

 histiocytosis X

 DIDMOAD is the association of cranial Diabetes Insipidus, Diabetes Mellitus, Optic


Atrophy and Deafness (also known as Wolfram's syndrome)

Causes of nephrogenic DI
 genetic: the more common form affects the vasopression (ADH) receptor, the less
common form results from a mutation in the gene that encodes the aquaporin 2
channel
 electrolytes: hypercalcaemia, hypokalaemia
 drugs: demeclocycline, lithium
 tubulo-interstitial disease: obstruction, sickle-cell, pyelonephritis

Features
 polyuria
 polydipsia

Investigation
 high plasma osmolality, low urine osmolality
 water deprivation test

External Links:‎

Merck manual

Nephrogenic diabetes insipidus

29
MRCP part 1 Made Easy‫‏‬‎‫‏‬by M. Habayeb & A. Murad
Chapter: Neurology

Drugs causing peripheral neuropathy

Drugs causing a peripheral neuropathy:


 antibiotics: nitrofurantoin, metronidazole
 amiodarone
 isoniazid
 vincristine

DVLA: neurological disorders

The guidelines below relate to car/motorcycle use unless specifically stated. For obvious
reasons, the rules relating to drivers of heavy goods vehicles tend to be much stricter
Specific rules
 first seizure: 6 months off driving*. For patients with established epilepsy they must
be fit free for 12 months before being able to drive
 stroke or TIA: 1 month off driving, may not need to inform DVLA if no residual
neurological deficit
 multiple TIAs over short period of times: 3 months off driving and inform DVLA
 craniotomy e.g. For meningioma: 1 year off driving**
 pituitary tumour: craniotomy: 6 months; trans-sphenoidal surgery 'can drive when
there is no debarring residual impairment likely to affect safe driving'
 narcolepsy/cataplexy: cease driving on diagnosis, can restart once 'satisfactory control
of symptoms'
 chronic neurological disorders e.g. multiple sclerosis, motor neuron disease: DVLA
should be informed, complete PK1 form (application for driving licence holders state
of health).

Syncope
 simple faint: no restriction
 single episode, explained and treated: 4 weeks off
 single episode, unexplained: 6 months off
 two or more episodes: 12 months off
*previously rule was 12 months. It is now 6 months off driving if the licence holder has
undergone assessment by an appropriate specialist and no relevant abnormality has been
identified on investigation, for example EEG and brain scan where indicated

**if the tumour is a benign meningioma and there is no seizure history, licence can be
reconsidered 6 months after surgery if remains seizure free.
30
MRCP part 1 Made Easy‫‏‬‎‫‏‬by M. Habayeb & A. Murad
Chapter: Neurology

External Links:
DVLA
Neurological disorder guidelines

Eclampsia

Eclampsia may be defined as the development of seizures in association pre-eclampsia. To


recap, pre-eclampsia is defined as:
 condition seen after 20 weeks gestation
 pregnancy-induced hypertension
 proteinuria
Magnesium sulphate is used to both prevent seizures in patients with severe pre-eclampsia
and treat seizures once they develop. Guidelines on its use suggest the following:
 should be given once a decision to deliver has been made
 in eclampsia an IV bolus of 4g over 5-10 minutes should be given followed by an
infusion of 1g / hour
 urine output, reflexes, respiratory rate and oxygen saturations should be monitored
during treatment
 treatment should continue for 24 hours after last seizure or delivery (around 40% of
seizures occur post-partum)
Other important aspects of treating severe pre-eclampsia/eclampsia include fluid restriction to
avoid the potentially serious consequences of fluid overload

External Links:‎
NICE
2010 Hypertension in pregnancy: The management of hypertensive disorders during
pregnancy

Epilepsy in children: syndromes

Infantile spasms (West's syndrome):


 brief spasms beginning in first few (4-6) months of life; M>F
1. Flexion of head, trunk, limbs → extension of arms (Salaam attack); last 1-2 secs,
repeat up to 50 times
2. Progressive mental handicap
3. EEG: hypsarrhythmia

31
MRCP part 1 Made Easy‫‏‬‎‫‏‬by M. Habayeb & A. Murad
Chapter: Neurology

 usually 2nd to serious neurological abnormality (e.g. TS, encephalitis, birth asphyxia)
or may be cryptogenic

 poor prognosis

 vigabatrin/steroids

Typical (petit mal) absence seizures:

 onset 4-8 yrs

 duration few-30 secs; no warning, quick recovery; often many per day

 EEG: 3Hz generalized, symmetrical

 sodium valproate, ethosuximide

 good prognosis: 90-95% become seizure free in adolescence

Lennox-Gastaut syndrome:

 may be extension of infantile spasms (50% have hx)

 onset 1-5 yrs

 atypical absences, falls, jerks

 90% moderate-severe mental handicap

 EEG: slow spike

 ketogenic diet may help

Benign rolandic epilepsy:


 most common in childhood, M>F
 paraesthesia (e.g. unilateral face), usually on waking up

Juvenile myoclonic epilepsy (Janz syndrome):


 onset: teens; F:M = 2:1
 1. Infrequent generalized seizures, often in morning
 2. Daytime absences
 3. Sudden, shock like myoclonic seizure
 usually good response to sodium valproate

32
MRCP part 1 Made Easy‫‏‬‎‫‏‬by M. Habayeb & A. Murad
Chapter: Neurology

Neonatal period - try vitamin B6:


 2nd: hypoglycaemia, meningitis, head trauma
 pyridoxine dependency (AR, IV B6)
 benign familial neonatal seizures (AD)
 benign neonatal convulsions (5th day)

Epilepsy: classification

Basics
 two main categories are generalised and partial seizures
 partial seizures may progress to general seizures
 other types: myoclonic, atypical absence, atonic and tonic seizures are usually seen in
childhood
Generalised: - no focal features, consciousness lost immediately
 grand mal (tonic-clonic)
 petit mal (absence seizures)
 myoclonic: brief, rapid muscle jerks
 partial seizures progressing to generalised seizures.
Partial: - focal features depending on location
 simple (no disturbance of consciousness or awareness)
 complex (consciousness is disturbed)
 temporal lobe → aura, déjà vu, jamais vu; motor → Jacksonian

External Links:‎
NICE
2012 Epilepsy guidelines

Epilepsy: pregnancy and breast feeding

The risks of uncontrolled epilepsy during pregnancy generally outweigh the risks of
medication to the fetus. All women thinking about becoming pregnant should be advised to
take folic acid 5mg per day well before pregnancy to minimise the risk of neural tube defects.
Around 1-2% of newborns born to non-epileptic mothers have congenital defects. This rises
to 3-4% if the mother takes antiepileptic medication.

33
MRCP part 1 Made Easy‫‏‬‎‫‏‬by M. Habayeb & A. Murad
Chapter: Neurology

Other points:
 aim for monotherapy
 there is no indication to monitor antiepileptic drug levels
 sodium valproate: associated with neural tube defects
 carbamazepine: often considered the least teratogenic of the older antiepileptics
 phenytoin: associated with cleft palate
 lamotrigine: studies to date suggest the rate of congenital malformations may be low.
The dose of lamotrigine may need to be increased in pregnancy
♦Breast feeding is generally considered safe for mothers taking antiepileptics with the
possible exception of the barbiturates.
♦It is advised that pregnant women taking phenytoin are given vitamin K in the last month of
pregnancy to prevent clotting disorders in the newborn.

Sodium valproate
The November 2013 issue of the Drug Safety Update also carried a warning about new
evidence showing a significant risk of neurodevelopmental delay in children following
maternal use of sodium valproate.

The update concludes that sodium valproate should not be used during pregnancy and in
women of childbearing age unless clearly necessary. Women of childbearing age should not
start treatment without specialist neurological or psychiatric advice.

External Links:‎
NICE
2012 Epilepsy guidelines
SIGN
2015 Diagnosis and management of epilepsy in adults

Epilepsy: treatment

Most neurologists now start antiepileptics following a second epileptic seizure. NICE
guidelines suggest starting antiepileptics after the first seizure if any of the following are
present:
 the patient has a neurological deficit
 brain imaging shows a structural abnormality
 the EEG shows unequivocal epileptic activity
 the patient or their family or carers consider the risk of having a further seizure
unacceptable.

34
MRCP part 1 Made Easy‫‏‬‎‫‏‬by M. Habayeb & A. Murad
Chapter: Neurology

♦Sodium valproate is considered the first line treatment for patients with generalised
seizures with carbamazepine used for partial seizures.

Generalized tonic-clonic seizures:

 sodium valproate

 second line: lamotrigine, carbamazepine

Absence seizures* (Petit mal):

 sodium valproate or ethosuximide

 sodium valproate particularly effective if co-existent tonic-clonic seizures in primary


generalised epilepsy

Myoclonic seizures:

 sodium valproate

 second line: clonazepam, lamotrigine

Partial seizures

 carbamazepine or lamotrigine

 second line: sodium valproate

*carbamazepine may actually exacerbate absence seizure

External Links:‎

NICE

2012 Epilepsy guidelines

Royal College of Physicians

2013 Modern management of epilepsy.

35
MRCP part 1 Made Easy‫‏‬‎‫‏‬by M. Habayeb & A. Murad
Chapter: Neurology

Essential tremor

Essential tremor (previously called benign essential tremor) is an autosomal dominant


condition which usually affects both upper limbs

Features

 postural tremor: worse if arms outstretched

 improved by alcohol and rest

 most common cause of titubation (head tremor)

Management

 propranolol is first-line

 primidone is sometimes used

Ethosuximide

Ethosuximide is an antiepileptic that is particularly indicated in patients with absence seizures


Mechanism of action:

 blocks T-type calcium channels in thalamic neurons

Facial nerve

Supply - 'face, ear, taste, tear'

 face: muscles of facial expression

 ear: nerve to stapedius

 taste: supplies anterior two-thirds of tongue

 tear: parasympathetic fibres to lacrimal glands, also salivary glands

Causes of bilateral facial nerve palsy

 sarcoidosis

 Guillain-Barre syndrome

 polio, Lyme disease

36
MRCP part 1 Made Easy‫‏‬‎‫‏‬by M. Habayeb & A. Murad
Chapter: Neurology

Causes of unilateral facial nerve palsy - as above plus

Lower motor neuron Upper motor


neuron
 Bell's palsy
 Ramsay-Hunt syndrome (due to  stroke
herpes zoster)
 acoustic neuroma
 parotid tumours
 HIV
 multiple sclerosis*
 diabetes mellitus
LMN vs. UMN

 upper motor neuron lesion 'spares' upper face i.e. forehead

 lower motor neuron lesion affects all facial muscles

*may also cause an UMN palsy

Friedreich's ataxia

♦Friedreich's ataxia is the most common of the early-onset hereditary ataxias. It is an


autosomal recessive, trinucleotide repeat disorder characterised by a GAA repeat in the X25
gene on chromosome 9 (frataxin). Friedreich's ataxia is unusual amongst trinucleotide repeat
disorders in not demonstrating the phenomenon of anticipation.

♦The typical age of onset is 10-15 years old. Gait ataxia and kyphoscoliosis are the most
common presenting features.

Neurological features:

 absent ankle jerks/extensor plantars

 cerebellar ataxia

 optic atrophy

 spinocerebellar tract degeneration

Other features

 hypertrophic obstructive cardiomyopathy (90%, most common cause of death)

 diabetes mellitus (10-20%)

 high-arched palate.

37
MRCP part 1 Made Easy‫‏‬‎‫‏‬by M. Habayeb & A. Murad
Chapter: Neurology

Comparison of Friedreich's ataxia and ataxic telangiectasia. Note in particular how ataxic
telangiectasia tends to present much earlier, often at the age of 1-2 years

Frontotemporal lobar degeneration

Frontotemporal lobar degeneration (FTLD) is the third most common type of cortical
dementia after Alzheimer's and Lewy body dementia.

There are three recognised types of FTLD:


 Frontotemporal dementia (Pick's disease).
 Progressive non fluent aphasia (chronic progressive aphasia, CPA).
 Semantic dementia.

Common features of frontotemporal lobar dementias


Onset before 65
Insidious onset
Relatively preserved memory and visuospatial skills
Personality change and social conduct problems

Pick's disease

♦This is the most common type and is characterised by personality change and impaired
social conduct. Other common features include hyperorality, disinhibition, increased appetite,
and perseveration behaviours.

♦Focal gyral atrophy with a knife-blade appearance is characteristic of Pick's disease.

♦Macroscopic changes seen in Pick's disease include:-

 Atrophy of the frontal and temporal lobes

38
MRCP part 1 Made Easy‫‏‬‎‫‏‬by M. Habayeb & A. Murad
Chapter: Neurology

♦Microscopic changes include:-


 Pick bodies - spherical aggregations of tau protein (silver-staining)
 Gliosis
 Neurofibrillary tangles
 Senile plaques

CPA
Here the chief factor is non fluent speech. They make short utterances that are agrammatic.
Comprehension is relatively preserved.

Semantic dementia
Here the patient has a fluent progressive aphasia. The speech is fluent but empty and conveys
little meaning. Unlike in Alzheimer's memory is better for recent rather than remote events.

Guillain-Barre syndrome

Guillain-Barre syndrome describes an immune mediated demyelination of the peripheral


nervous system often triggered by an infection (classically Campylobacter jejuni)

Pathogenesis:
 cross reaction of antibodies with gangliosides in the peripheral nervous system
 correlation between anti-ganglioside antibody (e.g. anti-GM1) and clinical features
has been demonstrated
 anti-GM1 antibodies in 25% of patients

Miller Fisher syndrome


 variant of Guillain-Barre syndrome
 associated with ophthalmoplegia, areflexia and ataxia. The eye muscles are typically
affected first
 usually presents as a descending paralysis rather than ascending as seen in other forms
of Guillain-Barre syndrome
 anti-GQ1b antibodies are present in 90% of cases

39
MRCP part 1 Made Easy‫‏‬‎‫‏‬by M. Habayeb & A. Murad
Chapter: Neurology

Guillain-Barre syndrome: features

Guillain-Barre syndrome describes an immune mediated demyelination of the peripheral


nervous system often triggered by an infection (classically Campylobacter jejuni).

The characteristic features of Guillain-Barre syndrome is progressive weakness of all four


limbs. The weakness is classically ascending i.e. the lower extremities are affected first,
however it tends to affect proximal muscles earlier than the distal ones. Sensory symptoms
tend to be mild (e.g. distal paraesthesia) with very few sensory signs. Some patients
experience back pain in the initial stages of the illness

Other features:
 areflexia
 cranial nerve involvement e.g. diplopia
 autonomic involvement: e.g. urinary retention

Less common findings


 papilloedema: thought to be secondary to reduced CSF resorption

External Links:‎
Patient.info
Guillain-Barre syndrome review

Guillain-Barre syndrome: management

Guillain-Barre syndrome describes an immune mediated demyelination of the peripheral


nervous system often triggered by an infection (classically Campylobacter jejuni).

Management:
 plasma exchange
 IV immunoglobulins (IVIG): as effective as plasma exchange. No benefit in
combining both treatments. IVIG may be easier to administer and tends to have fewer
side-effects
 steroids and immunosuppressants have not been shown to be beneficial
 FVC regularly to monitor respiratory function
Prognosis

 20% suffer permanent disability, 5% die

40
MRCP part 1 Made Easy‫‏‬‎‫‏‬by M. Habayeb & A. Murad
Chapter: Neurology

Guillain-Barre syndrome: prognosis

Guillain-Barre syndrome (GBS) describes an immune mediated demyelination of the


peripheral nervous system often triggered by an infection (classically Campylobacter jejuni)

Poor prognostic features:

 age > 40 years

 poor upper extremity muscle strength

 previous history of a diarrhoeal illness (specifically Campylobacter jejuni)

 high anti-GM1 antibody titre

 need for ventilatory support


There is currently contradictory evidence as to whether a gradual or rapid onset of
GBS is associated with a poor outcome

Head injury: NICE guidance

NICE has strict and clear guidance regarding which adult patients are safe to discharge and
which need further CT head imaging. The latter group are also divided into two further
cohorts, those who require an immediate CT head and those requiring CT head within 8
hours of injury:

CT head immediately

 GCS < 13 on initial assessment

 GCS < 15 at 2 hours post-injury

 suspected open or depressed skull fracture.

 any sign of basal skull fracture (haemotympanum, 'panda' eyes, cerebrospinal fluid
leakage from the ear or nose, Battle's sign).

 post-traumatic seizure.

 focal neurological deficit.

 more than 1 episode of vomiting.

41
MRCP part 1 Made Easy‫‏‬‎‫‏‬by M. Habayeb & A. Murad
Chapter: Neurology

CT head scan within 8 hours of the head injury - for adults with any of the following risk
factors who have experienced some loss of consciousness or amnesia since the injury:

 age 65 years or older

 any history of bleeding or clotting disorders

 dangerous mechanism of injury (a pedestrian or cyclist struck by a motor vehicle, an


occupant ejected from a motor vehicle or a fall from a height of greater than 1 metre or
5 stairs)

 more than 30 minutes' retrograde amnesia of events immediately before the head
injury

♦If a patient is on warfarin who have sustained a head injury with no other indications for a
CT head scan, perform a CT head scan within 8 hours of the injury.

External Links:‎

NICE

2014 Head injury guidelines

Head injury: types of traumatic brain injury

Basics

 primary brain injury may be focal (contusion/haematoma) or diffuse (diffuse axonal


injury)

 diffuse axonal injury occurs as a result of mechanical shearing following deceleration,


causing disruption and tearing of axons

 intra-cranial haematomas can be extradural, subdural or intracerebral, while


contusions may occur adjacent to (coup) or contralateral (contre-coup) to the side of
impact

 secondary brain injury occurs when cerebral oedema, ischaemia, infection, tonsillar or
tentorial herniation exacerbates the original injury. The normal cerebral auto
regulatory processes are disrupted following trauma rendering the brain more
susceptible to blood flow changes and hypoxia

 the Cushings reflex (hypertension and bradycardia) often occurs late and is usually a
pre terminal event.

42
MRCP part 1 Made Easy‫‏‬‎‫‏‬by M. Habayeb & A. Murad
Chapter: Neurology

Type of injury Notes


Extradural Bleeding into the space between the dura mater and the
(epidural) skull. Often results from acceleration-deceleration
haematoma trauma or a blow to the side of the head. The majority of
epidural haematomas occur in the temporal region where
skull fractures cause a rupture of the middle meningeal
artery.

Features

 features of raised intracranial pressure


 some patients may exhibit a lucid interval
Subdural Bleeding into the outermost meningeal layer. Most
haematoma commonly occur around the frontal and parietal lobes.

Risk factors include old age, alcoholism and


anticoagulation.

Slower onset of symptoms than a epidural haematoma.


Subarachnoid Usually occurs spontaneously in the context of a
haemorrhage ruptured cerebral aneurysm but may be seen in
association with other injuries when a patient has
sustained a traumatic brain injury

43
MRCP part 1 Made Easy‫‏‬‎‫‏‬by M. Habayeb & A. Murad
Chapter: Neurology

Image gallery

Extradural (epidural) haematoma:

© Image used on license from Radiopaedia

© Image used on license from Radiopaedia

44
MRCP part 1 Made Easy‫‏‬‎‫‏‬by M. Habayeb & A. Murad
Chapter: Neurology

© Image used on license from Radiopaedia

Subdural haematoma:

© Image used on license from Radiopaedia

45
MRCP part 1 Made Easy‫‏‬‎‫‏‬by M. Habayeb & A. Murad
Chapter: Neurology

© Image used on license from Radiopaedia

Subarachnoid haemorrhage:

© Image used on license from Radiopaedia

46
MRCP part 1 Made Easy‫‏‬‎‫‏‬by M. Habayeb & A. Murad
Chapter: Neurology

© Image used on license from Radiopaedia

External Links:‎
NICE
2014 Head injury guidelines

47
MRCP part 1 Made Easy‫‏‬‎‫‏‬by M. Habayeb & A. Murad
Chapter: Neurology

Headache

Headache accounts for a large proportion of medical consultations. The table below
summarises the main characteristics of common or important causes:

Migraine Recurrent, severe headache which is usually


unilateral and throbbing in nature
May be be associated with aura, nausea and
photosensitivity
Aggravated by, or causes avoidance of,
routine activities of daily living. Patients
often describe 'going to bed'.
In women may be associated with
menstruation
Tension Recurrent, non-disabling, bilateral headache,
headache often described as a 'tight-band'
Not aggravated by routine activities of daily
living
Cluster Pain typical occurs once or twice a day, each
headache* episode lasting 15 mins - 2 hours with clusters
typically lasting 4-12 weeks
Intense pain around one eye (recurrent attacks
'always' affect same side)
Patient is restless during an attack
Accompanied by redness, lacrimation, lid
swelling
More common in men and smokers
Temporal Typically patient > 60 years old
arteritis Usually rapid onset (e.g. < 1 month) of
unilateral headache
Jaw claudication (65%)
Tender, palpable temporal artery
Raised ESR
Medication Present for 15 days or more per month
overuse Developed or worsened whilst taking regular
headache symptomatic medication
Patients using opioids and triptans are at most
risk
May be psychiatric co-morbidity

Other causes of headache:

Acute single episode:


 meningitis
 encephalitis
 subarachnoid haemorrhage
 head injury
 sinusitis
48
MRCP part 1 Made Easy‫‏‬‎‫‏‬by M. Habayeb & A. Murad
Chapter: Neurology

 glaucoma (acute closed-angle)


 tropical illness e.g. Malaria

Chronic headache

 chronically raised ICP


 Paget's disease
 psychological

*some neurologists use the term trigeminal autonomic cephalgia to group a number of
conditions including cluster headache, paroxysmal hemicrania and short-lived unilateral
neuralgiform headache with conjunctival injection and tearing (SUNCT). It is recommended
such patients are referred for specialist assessment as specific treatment may be required, for
example it is known paroxysmal hemicrania responds very well to indomethacin

External Links:‎

NICE

2012 Headaches: Diagnosis and management of headaches in young people and adults

SIGN

2008 Diagnosis and management of headache in adults

Hemiballism

♦Hemiballism occurs following damage to the subthalamic nucleus. Ballisic movements are
involuntary, sudden, jerking movements which occur contralateral to the side of the lesion.
The ballisic movements primarily affect the proximal limb musculature whilst the distal
muscles may display more choreiform-like movements

♦Symptoms may decrease whilst the patient is asleep.

♦Antidopaminergic agents (e.g. Haloperidol) are the mainstay of treatment

49
MRCP part 1 Made Easy‫‏‬‎‫‏‬by M. Habayeb & A. Murad
Chapter: Neurology

Herpes simplex encephalitis

Herpes simplex (HSV) encephalitis is a common topic in the exam. The virus
characteristically affects the temporal lobes - questions may give the result of imaging or
describe temporal lobe signs e.g. aphasia

Features

 fever, headache, psychiatric symptoms, seizures, vomiting

 focal features e.g. aphasia

 peripheral lesions (e.g. cold sores) have no relation to presence of HSV encephalitis

Pathophysiology

 HSV-1 responsible for 95% of cases in adults

 typically affects temporal and inferior frontal lobes

Investigation

 CSF: lymphocytosis, elevated protein

 PCR for HSV

 CT: medial temporal and inferior frontal changes (e.g. petechial haemorrhages) -
normal in one-third of patients

 MRI is better

 EEG pattern: lateralised periodic discharges at 2 Hz

Treatment

 intravenous aciclovir

♦The prognosis is dependent on whether aciclovir is commenced early. If treatment is started


promptly the mortality is 10-20%. Left untreated the mortality approaches 80%.

50
MRCP part 1 Made Easy‫‏‬‎‫‏‬by M. Habayeb & A. Murad
Chapter: Neurology

© Image used on license from Radiopaedia

MRI of a patient with HSV encephalitis. There is hyperintensity of the affected white matter
and cortex in the medial temporal lobes and insular cortex.

Hodgkin's lymphoma: histological classification and prognosis

Hodgkin's lymphoma is a malignant proliferation of lymphocytes characterised by the


presence of the Reed-Sternberg cell. It has a bimodal age distributions being most common in
the third and seventh decades

Histological classification

Type Frequency Prognosis Notes


Nodular Most common Good More common in
sclerosing (around 70%) prognosis women. Associated
with lacunar cells
Mixed Around 20% Good Associated with a
cellularity prognosis large number of
Reed-Sternberg
cells
Lymphocyte A*round 5% Best
predominant prognosis
Lymphocyte Rare Worst
depleted prognosis

51
MRCP part 1 Made Easy‫‏‬‎‫‏‬by M. Habayeb & A. Murad
Chapter: Neurology

'B' symptoms also imply a poor prognosis:

 weight loss > 10% in last 6 months

 fever > 38ºC

 night sweats

Other factors associated with a poor prognosis identified in a 1998 NEJM paper
included:

 age > 45 years

 stage IV disease

 haemoglobin < 10.5 g/dl

 lymphocyte count < 600/µl or < 8%

 male

 albumin < 40 g/l

 white blood count > 15,000/µl

*Reed-Sternberg cells with nuclei surrounded by a clear space

Horner's syndrome

Features:

 miosis (small pupil)

 ptosis

 enophthalmos* (sunken eye)

 anhidrosis (loss of sweating one side)

Distinguishing between causes:

 heterochromia (difference in iris colour) is seen in congenital Horner's

 anhidrosis: see below

52
MRCP part 1 Made Easy‫‏‬‎‫‏‬by M. Habayeb & A. Murad
Chapter: Neurology

Central lesions Pre-ganglionic Post-ganglionic


lesions lesions
Anhidrosis of the face, Anhidrosis of the No anhidrosis
arm and trunk face
Stroke Pancoast's tumour Carotid artery
Syringomyelia Thyroidectomy dissection
Multiple sclerosis Trauma Carotid aneurysm
Tumour Cervical rib Cavernous sinus
Encephalitis thrombosis
Cluster headache

*in reality the appearance is due to a narrow palpebral aperture rather than true enophthalmos

HSMN

Hereditary sensorimotor neuropathy (HSMN) is a relatively new term which encompasses


Charcot-Marie-Tooth disease (also known as peroneal muscular atrophy). Over 7 types have
been characterised - however only 2 are common to clinical practice

 HSMN type I: primarily due to demyelinating pathology

 HSMN type II: primarily due to axonal pathology

HSMN type I

 autosomal dominant

 due to defect in PMP-22 gene (which codes for myelin)

 features often start at puberty

 motor symptoms predominate

 distal muscle wasting, pes cavus, clawed toes

 foot drop, leg weakness often first features

53
MRCP part 1 Made Easy‫‏‬‎‫‏‬by M. Habayeb & A. Murad
Chapter: Neurology

Huntington's disease

Huntington's disease is an inherited neurodegenerative condition. It is a progressive and


incurable condition that typically results in death 20 years after the initial symptoms develop.

Genetics
 autosomal dominant
 trinucleotide repeat disorder: repeat expansion of CAG
 results in degeneration of cholinergic and GABAergic neurons in the striatum of the
basal ganglia
 due to defect in huntingtin gene on chromosome 4

Features typical develop after 35 years of age


 chorea
 personality changes (e.g. irritability, apathy, depression) and intellectual impairment
 dystonia
 saccadic eye movements

Idiopathic intracranial hypertension

Idiopathic intracranial hypertension (also known as pseudotumour cerebri and formerly


benign intracranial hypertension) is a condition classically seen in young, overweight females.

Features
 headache
 blurred vision
 papilloedema (usually present)
 enlarged blind spot
 sixth nerve palsy may be present

Risk factors

 obesity

 female sex

 pregnancy

 drugs*: oral contraceptive pill, steroids, tetracycline, vitamin A, lithium


54
MRCP part 1 Made Easy‫‏‬‎‫‏‬by M. Habayeb & A. Murad
Chapter: Neurology

Management
 weight loss
 diuretics e.g. acetazolamide
 topiramate is also used, and has the added benefit of causing weight loss in most
patients
 repeated lumbar puncture
 surgery: optic nerve sheath decompression and fenestration may be needed to prevent
damage to the optic nerve. A lumboperitoneal or ventriculoperitoneal shunt may also
be performed to reduce intracranial pressure

*if intracranial hypertension is thought to occur secondary to a known causes (e.g.


Medication) then it is of course not idiopathic

External Links:‎
Patient.info
Idiopathic intracranial hypertension review.

Intracranial venous thrombosis

Overview
 can cause cerebral infarction, much lesson common than arterial causes
 50% of patients have isolated sagittal sinus thromboses - the remainder have
coexistent lateral sinus thromboses and cavernous sinus thromboses

Features
 headache (may be sudden onset)
 nausea & vomiting.

Sagittal sinus thrombosis


 may present with seizures and hemiplegia
 parasagittal biparietal or bifrontal haemorrhagic infarctions are sometimes seen.

55
MRCP part 1 Made Easy‫‏‬‎‫‏‬by M. Habayeb & A. Murad
Chapter: Neurology

Cavernous sinus thrombosis

 other causes of cavernous sinus syndrome: local infection (e.g. sinusitis), neoplasia,
trauma

 periorbital oedema

 ophthalmoplegia: 6th nerve damage typically occurs before 3rd & 4th

 trigeminal nerve involvement may lead to hyperaesthesia of upper face and eye pain

 central retinal vein thrombosis

Lateral sinus thrombosis

 6th and 7th cranial nerve palsies

© Image used on license from Radiopaedia

CT with contrast demonstating a superior sagittal sinus thrombosis showing the typical
empty delta sign. Look at the 'bottom' of the scan for the triangular shaped dural sinus. This
should normally be white due to it being filled with contrast. The empty delta sign occurs
when the thrombus fails to enhance within the dural sinus and is outlined by enhanced
collateral channels in the falx. This sign is seen in only about 25%-30% of cases but is highly
diagnostic for sagittal sinus thrombosis

56
MRCP part 1 Made Easy‫‏‬‎‫‏‬by M. Habayeb & A. Murad
Chapter: Neurology

External Links:‎
Webmed Central
Diagram of types of intracranial venous thrombosis

Lambert-Eaton syndrome

Lambert-Eaton myasthenic syndrome is seen in association with small cell lung cancer, and to
a lesser extent breast and ovarian cancer. It may also occur independently as an autoimmune
disorder. Lambert-Eaton myasthenic syndrome is caused by an antibody directed against pre-
synaptic voltage gated calcium channel in the peripheral nervous system

Features:
 repeated muscle contractions lead to increased muscle strength* (in contrast to
myasthenia gravis)
 limb girdle weakness (affects lower limbs first)
 hyporeflexia
 autonomic symptoms: dry mouth, impotence, difficultly micturating
 ophthalmoplegia and ptosis not commonly a feature (unlike in myasthenia gravis)

EMG
 incremental response to repetitive electrical stimulation

Management
 treatment of underlying cancer
 immunosuppression, for example with prednisolone and/or azathioprine
 3,4-diaminopyridine is currently being trialled**
 intravenous immunoglobulin therapy and plasma exchange may be beneficial

*in reality this is seen in only 50% of patients and following prolonged muscle use muscle
strength will eventually decrease

**works by blocking potassium channel efflux in the nerve terminal so that the action
potential duration is increased. Calcium channels can then be open for a longer time and allow
greater acetylcholine release to the stimulate muscle at the end plate

57
MRCP part 1 Made Easy‫‏‬‎‫‏‬by M. Habayeb & A. Murad
Chapter: Neurology

Lateral medullary syndrome

Lateral medullary syndrome, also known as Wallenberg's syndrome, occurs following


occlusion of the posterior inferior cerebellar artery

Cerebellar features

 ataxia

 nystagmus

Brainstem features

 ipsilateral: dysphagia, facial numbness, cranial nerve palsy e.g. Horner's

 contralateral: limb sensory loss

Levodopa

Overview

 usually combined with a decarboxylase inhibitor (e.g. carbidopa or benserazide) to


prevent peripheral metabolism of L-dopa to dopamine

 reduced effectiveness with time (usually by 2 years)

 no use in neuroleptic induced parkinsonism

Adverse effects

 dyskinesia

 'on-off' effect

 postural hypotension

 cardiac arrhythmias

 nausea & vomiting

 psychosis

 reddish discolouration of urine upon standing

58
MRCP part 1 Made Easy‫‏‬‎‫‏‬by M. Habayeb & A. Murad
Chapter: Neurology

Lewy body dementia

♦Lewy body dementia is an increasingly recognised cause of dementia, accounting for up to


20% of cases. The characteristic pathological feature is alpha-synuclein cytoplasmic
inclusions (Lewy bodies) in the substantia nigra, paralimbic and neocortical areas

♦The relationship between Parkinson's disease and Lewy body dementia is complicated,
particularly as dementia is often seen in Parkinson's disease. Also, up to 40% of patients with
Alzheimer's have Lewy bodies

♦Neuroleptics should be avoided in Lewy body dementia as patients are extremely sensitive
and may develop irreversible parkinsonism. Questions may give a history of a patient who has
deteriorated following the introduction of an antipsychotic agent

Features:

 progressive cognitive impairment

 parkinsonism

 visual hallucinations (other features such as delusions and non-visual hallucinations


may also be seen)

Diagnosis

 usually clinical

 single-photon emission computed tomography (SPECT) is increasingly used. It is


currently commercially known as a DaTscan. Dopaminergic iodine-123-radiolabelled
2-carbomethoxy-3-(4-iodophenyl)-N-(3-fluoropropyl) nortropane (123-I FP-CIT) is
used as the radioisotope. The sensitivity of SPECT in diagnosing Lewy body dementia
is around 90% with a specificity of 100%

59
MRCP part 1 Made Easy‫‏‬‎‫‏‬by M. Habayeb & A. Murad
Chapter: Neurology

Medication overuse headache

Medication overuse headache is one of the most common causes of chronic daily headache. It
may affect up to 1 in 50 people

Features:

 present for 15 days or more per month

 developed or worsened whilst taking regular symptomatic medication

 patients using opioids and triptans are at most risk

 may be psychiatric co-morbidity

Management (from 2008 SIGN guidelines):

 simple analgesics and triptans should be withdrawn abruptly (may initially worsen
headaches)

 opioid analgesics should be gradually withdrawn

External Links:‎

SIGN

2008 Diagnosis and management of headache in adults.

Meniere's disease

Meniere's disease is a disorder of the inner ear of unknown cause. It is characterised by


excessive pressure and progressive dilation of the endolymphatic system. It is more common
in middle-aged adults but may be seen at any age. Meniere's disease has a similar prevalence
in both men and women.

Features:

 recurrent episodes of vertigo, tinnitus and hearing loss (sensorineural). Vertigo is


usually the prominent symptom

 a sensation of aural fullness or pressure is now recognised as being common

 other features include nystagmus and a positive Romberg test

 episodes last minutes to hours

 typically symptoms are unilateral but bilateral symptoms may develop after a number
of years.
60
MRCP part 1 Made Easy‫‏‬‎‫‏‬by M. Habayeb & A. Murad
Chapter: Neurology

Natural history:

 symptoms resolve in the majority of patients after 5-10 years

 the majority of patients will be left with a degree of hearing loss

 psychological distress is common

Management

 ENT assessment is required to confirm the diagnosis

 patients should inform the DVLA. The current advice is to cease driving until
satisfactory control of symptoms is achieved

 acute attacks: buccal or intramuscular prochlorperazine. Admission is sometimes


required

 prevention: betahistine may be of benefit

External Links:‎

Clinical Knowledge Summaries

Meniere's disease guidelines.

Meralgia paraesthetica

Basics:

 caused by compression of lateral cutaneous nerve of thigh

 typically burning sensation over antero-lateral aspect of thigh

61
MRCP part 1 Made Easy‫‏‬‎‫‏‬by M. Habayeb & A. Murad
Chapter: Neurology

Migraine: diagnostic criteria

The International Headache Society has produced the following diagnostic criteria for
migraine without aura:

Point Criteria
A At least 5 attacks fulfilling criteria B-D
B Headache attacks lasting 4-72 hours* (untreated or
unsuccessfully treated)
C Headache has at least two of the following characteristics:

 1. unilateral location*
 2. pulsating quality (i.e., varying with the heartbeat)
 3. moderate or severe pain intensity
 4. aggravation by or causing avoidance of routine
physical activity (e.g., walking or climbing stairs)
D During headache at least one of the following:

 1. nausea and/or vomiting*


 2. photophobia and phonophobia
E Not attributed to another disorder (history and examination
do not suggest a secondary headache disorder or, if they do,
it is ruled out by appropriate investigations or headache
attacks do not occur for the first time in close temporal
relation to the other disorder)

*In children, attacks may be shorter-lasting, headache is more commonly bilateral, and
gastrointestinal disturbance is more prominent.

Migraine with aura (seen in around 25% of migraine patients) tends to be easier to diagnose
with a typical aura being progressive in nature and may occur hours prior to the headache.
Typical aura include a transient hemianopic disturbance or a spreading scintillating scotoma
('jagged crescent'). Sensory symptoms may also occur.

♦If we compare these guidelines to the NICE criteria the following points are noted:

 NICE suggests migraines may be unilateral or bilateral

 NICE also give more detail about typical auras:

♦Auras may occur with or without headache and:

 are fully reversible.

 develop over at least 5 minutes.

 last 5-60 minutes.

62
MRCP part 1 Made Easy‫‏‬‎‫‏‬by M. Habayeb & A. Murad
Chapter: Neurology

♦The following aura symptoms are atypical and may prompt further
investigation/referral:
 motor weakness
 double vision
 visual symptoms affecting only one eye
 poor balance
 decreased level of consciousness.

External Links:‎
British Association for the Study of Headache
Headache guidelines

Migraine: pregnancy, contraception and other hormonal factors

SIGN produced guidelines in 2008 on the management of migraine, the following is


selected highlights:

Migraine during pregnancy


 paracetamol 1g is first-line
 aspirin 300mg or ibuprofen 400mg can be used second-line in the first and second
trimester

Migraine and the combined oral contraceptive (COC) pill:


 if patients have migraine with aura then the COC is absolutely contraindicated due to
an increased risk of stroke (relative risk 8.72)
Migraine and menstruation
 many women find that the frequency and severity of migraines increase around the
time of menstruation
 SIGN recommends that women are treated with mefanamic acid or a combination of
aspirin, paracetamol and caffeine. Triptans are also recommended in the acute
situation

Migraine and hormone replacement therapy (HRT)


 safe to prescribe HRT for patients with a history of migraine but it may make
migraines worse

63
MRCP part 1 Made Easy‫‏‬‎‫‏‬by M. Habayeb & A. Murad
Chapter: Neurology

External Links:‎
SIGN
2008 Diagnosis and management of headache in adults

Miosis

Causes of miosis (small pupil):


 Horner's syndrome
 Argyll-Robertson pupil
 senile miosis
 pontine haemorrhage
 congenital
Drugs causes
 opiates
 parasympathomimetics: pilocarpine
 organophosphate toxicity

Motor neuron disease: features

Motor neuron disease is a neurological condition of unknown cause which can present with
both upper and lower motor neuron signs. It rarely presents before 40 years and various
patterns of disease are recognised including amyotrophic lateral sclerosis, progressive
muscular atrophy and bulbar palsy

There are a number of clues which point towards a diagnosis of motor neuron disease:
 fasciculation
 the absence of sensory signs/symptoms*
 the mixture of lower motor neurone and upper motor neurone signs
 wasting of the small hand muscles/tibialis anterior is common

Other features:
 doesn't affect external ocular muscles
 no cerebellar signs
 abdominal reflexes are usually preserved and sphincter dysfunction if present is a late
feature

64
MRCP part 1 Made Easy‫‏‬‎‫‏‬by M. Habayeb & A. Murad
Chapter: Neurology

♦The diagnosis of motor neuron disease is clinical, but nerve conduction studies will show
normal motor conduction and can help exclude a neuropathy. Electromyography shows a
reduced number of action potentials with an increased amplitude. MRI is usually performed to
exclude the differential diagnosis of cervical cord compression and myelopathy

*vague sensory symptoms may occur early in the disease (e.g. limb pain) but 'never' sensory
signs

External Links:‎
Royal College of Physicians
Motor neuron disease: diagnostic pitfalls
Postgraduate Medical Journal
Review of MND

Motor neuron disease: management

Motor neuron disease is a neurological condition of unknown cause which can present with
both upper and lower motor neuron signs. It rarely presents before 40 years and various
patterns of disease are recognised including amyotrophic lateral sclerosis, progressive
muscular atrophy and bulbar palsy

Riluzole
 prevents stimulation of glutamate receptors
 used mainly in amyotrophic lateral sclerosis
 prolongs life by about 3 months

Respiratory care
 non-invasive ventilation (usually BIPAP) is used at night
 studies have shown a survival benefit of around 7 months

Prognosis
 poor: 50% of patients die within 3 years

External Links:‎
NICE
2016 Motor neurone disease: assessment and management

65
MRCP part 1 Made Easy‫‏‬‎‫‏‬by M. Habayeb & A. Murad
Chapter: Neurology

Motor neuron disease: types

Motor neuron disease is a neurological condition of unknown cause which can present with
both upper and lower motor neuron signs. It rarely presents before 40 years and various
patterns of disease are recognised including amyotrophic lateral sclerosis, primary lateral
sclerosis, progressive muscular atrophy and progressive bulbar palsy. In some patients
however, there is a combination of clinical patterns

Amyotrophic lateral sclerosis (50% of patients):


 typically LMN signs in arms and UMN signs in legs
 in familial cases the gene responsible lies on chromosome 21 and codes for superoxide
dismutase

Primary lateral sclerosis


 UMN signs only

Progressive muscular atrophy

 LMN signs only


 affects distal muscles before proximal
 carries best prognosis

Progressive bulbar palsy

 palsy of the tongue, muscles of chewing/swallowing and facial muscles due to loss of
function of brainstem motor nuclei
 carries worst prognosis

External Links:‎
Postgraduate Medical Journal

Review of MND

66
MRCP part 1 Made Easy‫‏‬‎‫‏‬by M. Habayeb & A. Murad
Chapter: Neurology

Multiple sclerosis: features

Patient's with multiple sclerosis (MS) may present with non-specific features, for example
around 75% of patients have significant lethargy.

Visual

 optic neuritis: common presenting feature

 optic atrophy

 Uhthoff's phenomenon: worsening of vision following rise in body temperature

 internuclear ophthalmoplegia

Sensory

 pins/needles

 numbness

 trigeminal neuralgia

 Lhermitte's syndrome: paraesthesiae in limbs on neck flexion

Motor

 spastic weakness: most commonly seen in the legs

Cerebellar

 ataxia: more often seen during an acute relapse than as a presenting symptom

 tremor

Others

 urinary incontinence

 sexual dysfunction

 intellectual deterioration

67
MRCP part 1 Made Easy‫‏‬‎‫‏‬by M. Habayeb & A. Murad
Chapter: Neurology

External Links:‎
NICE
2014 Multiple sclerosis guidelines

Multiple sclerosis: management

Treatment in multiple sclerosis is focused at reducing the frequency and duration of relapses.
There is no cure.

Acute relapse

High dose steroids (e.g. oral or IV methylprednisolone) may be given for 5 days to shorten the
length of an acute relapse. It should be noted that steroids shorten the duration of a relapse
and do not alter the degree of recovery (i.e. whether a patient returns to baseline function)

Disease modifying drugs

Beta-interferon has been shown to reduce the relapse rate by up to 30%. Certain criteria have
to be met before it is used:

 relapsing-remitting disease + 2 relapses in past 2 years + able to walk 100m unaided


 secondary progressive disease + 2 relapses in past 2 years + able to walk 10m (aided
or unaided)
 reduces number of relapses and MRI changes, however doesn't reduce overall
disability

Other drugs used in the management of multiple sclerosis include:

 glatiramer acetate: immunomodulating drug - acts as an 'immune decoy'


 natalizumab: a recombinant monoclonal antibody that antagonises Alpha-4 Beta-1-
integrin found on the surface of leucocytes, thus inhibiting migration of leucocytes
across the endothelium across the blood-brain barrier
 fingolimod: sphingosine 1-phosphate receptor modulator, prevents lymphocytes from
leaving lymph nodes. An oral formulation is available

68
MRCP part 1 Made Easy‫‏‬‎‫‏‬by M. Habayeb & A. Murad
Chapter: Neurology

Some specific problems

Fatigue
 once other problems (e.g. anaemia, thyroid or depression) have been excluded NICE
recommend a trial of amantadine

 other options include mindfulness training and CBT

Spasticity

 baclofen and gabapentin are first-line. Other options include diazepam, dantrolene and
tizanidine

 physiotherapy is important

 cannabis and botox are undergoing evalulation

Bladder dysfunction

 may take the form of urgency, incontinence, overflow etc

 guidelines stress the importance of getting an ultrasound first to assess bladder


emptying - anticholinergics may worsen symptoms in some patients

 if significant residual volume → intermittent self-catheterisation

 if no significant residual volume → anticholinergics may improve urinary frequency

Oscillopsia (visual fields apper to oscillate)

 gabapentin is first-line

External Links:‎
NICE
2014 Multiple Sclerosis guidelines
Multiple Sclerosis Society
Primary care guidelines

69
MRCP part 1 Made Easy‫‏‬‎‫‏‬by M. Habayeb & A. Murad
Chapter: Neurology

Multiple sclerosis: prognostic features

Good prognosis features

 female sex

 young age of onset (i.e. 20s or 30s)

 relapsing-remitting disease

 sensory symptoms only

 long interval between first two relapses

 complete recovery between relapses

Ways of remembering prognostic features

 the typical patient carries a better prognosis than an atypical presentation

Multiple system atrophy

Shy-Drager syndrome is a type of multiple system atrophy

Features

 parkinsonism

 autonomic disturbance (atonic bladder, postural hypotension)

 cerebellar signs

70
MRCP part 1 Made Easy‫‏‬‎‫‏‬by M. Habayeb & A. Murad
Chapter: Neurology

Myasthenia gravis

Myasthenia gravis is an autoimmune disorder resulting in insufficient functioning


acetylcholine receptors. Antibodies to acetylcholine receptors are seen in 85-90% of cases*.
Myasthenia is more common in women (2:1)

The key feature is muscle fatigability - muscles become progressively weaker during periods
of activity and slowly improve after periods of rest:

 extraocular muscle weakness: diplopia


 proximal muscle weakness: face, neck, limb girdle
 ptosis
 dysphagia

Associations:
 thymomas in 15%
 autoimmune disorders: pernicious anaemia, autoimmune thyroid disorders,
rheumatoid, SLE
 thymic hyperplasia in 50-70%

Investigations
 single fibre electromyography: high sensitivity (92-100%)
 CT thorax to exclude thymoma
 CK normal
 autoantibodies: around 85-90% of patients have antibodies to acetylcholine receptors.
In the remaining patients, about about 40% are positive for anti-muscle-specific
tyrosine kinase antibodies
 Tensilon test: IV edrophonium reduces muscle weakness temporarily - not commonly
used anymore due to the risk of cardiac arrhythmia

Management
 long-acting anticholinesterase e.g. pyridostigmine
 immunosuppression: prednisolone initially
 thymectomy.

71
MRCP part 1 Made Easy‫‏‬‎‫‏‬by M. Habayeb & A. Murad
Chapter: Neurology

Management of myasthenic crisis


 plasmapheresis
 intravenous immunoglobulins

*antibodies are less commonly seen in disease limited to the ocular muscles

External Links:‎

Postgraduate Medical Journal

Review of myasthenia gravis

Myasthenia gravis: exacerbating factors

The most common exacerbating factor is exertion resulting in fatigability, which is the
hallmark feature of myasthenia gravis . Symptoms become more marked during the day

The following drugs may exacerbate myasthenia:

 penicillamine

 quinidine, procainamide

 beta-blockers

 lithium

 phenytoin

 antibiotics: gentamicin, macrolides, quinolones, tetracyclines

External Links:

Myasthenia Gravis Association

Drugs which may aggravate myasthenia gravis

Royal College of Physicians (Edin)

Myasthenia gravis review

72
MRCP part 1 Made Easy‫‏‬‎‫‏‬by M. Habayeb & A. Murad
Chapter: Neurology

Myotonic dystrophy

♦Myotonic dystrophy (also called dystrophia myotonica) is an inherited myopathy with


features developing at around 20-30 years old. It affects skeletal, cardiac and smooth muscle.
There are two main types of myotonic dystrophy, DM1 and DM2.

Genetics
 autosomal dominant
 a trinucleotide repeat disorder
 DM1 is caused by a CTG repeat at the end of the DMPK (Dystrophia Myotonica-
Protein Kinase) gene on chromosome 19
 DM2 is caused by a repeat expansion of the ZNF9 gene on chromosome 3

The key differences are listed in table below:

DM1 DM2
- DMPK gene on chromosome - ZNF9 gene on chromosome 3
19 - Proximal weakness more
- Distal weakness more prominent
prominent - Severe congenital form not seen

General features:
 myotonic facies (long, 'haggard' appearance)
 frontal balding
 bilateral ptosis
 cataracts
 dysarthria

Other features:
 myotonia (tonic spasm of muscle)
 weakness of arms and legs (distal initially)
 mild mental impairment
 diabetes mellitus
 testicular atrophy
 cardiac involvement: heart block, cardiomyopathy
 dysphagia.

73
MRCP part 1 Made Easy‫‏‬‎‫‏‬by M. Habayeb & A. Murad
Chapter: Neurology

Nerve conduction studies

Nerve conduction studies (NCS) are useful in determining between axonal and demyelinating
pathology

Axonal
 normal conduction velocity
 reduced amplitude

Demyelinating
 reduced conduction velocity
 normal amplitude

External Links:
The Bone School
Nerve Conduction Studies / EMG

Neuropathic pain

Neuropathic pain may be defined as pain which arises following damage or disruption of the
nervous system. It is often difficult to treat and responds poorly to standard analgesia.
Examples include:
 diabetic neuropathy
 post-herpetic neuralgia
 trigeminal neuralgia
 prolapsed intervertebral disc
NICE updated their guidance on the management of neuropathic pain in 2013:
 first-line treatment*: amitriptyline, duloxetine, gabapentin or pregabalin
 if the first-line drug treatment does not work try one of the other 3 drugs
 tramadol may be used as 'rescue therapy' for exacerbations of neuropathic pain
 topical capsaicin may be used for localised neuropathic pain (e.g. post-herpetic
neuralgia)
 pain management clinics may be useful in patients with resistant problems

*please note that for some specific conditions the guidance may vary. For example
carbamazepine is used first-line for trigeminal neuralgia.

74
MRCP part 1 Made Easy‫‏‬‎‫‏‬by M. Habayeb & A. Murad
Chapter: Neurology

External Links:
NICE
2013 Neuropathic pain guidelines

Normal pressure hydrocephalus

Normal pressure hydrocephalus is a reversible cause of dementia seen in elderly patients. It is


thought to be secondary to reduced CSF absorption at the arachnoid villi. These changes may
be secondary to head injury, subarachnoid haemorrhage or meningitis

A classical triad of features is seen


 urinary incontinence
 dementia and bradyphrenia
 gait abnormality (may be similar to Parkinson's disease)

Imaging
 hydrocephalus with an enlarged fourth ventricle
Management
 ventriculoperitoneal shunting

External Links:
YouTube
Typical NPH Gait

Nystagmus

Upbeat nystagmus
 cerebellar vermis lesions

Downbeat nystagmus - foramen magnum lesions


 Arnold-Chiari malformation

Image sourced from Wikipedia


Horizontal optokinetic nystagmus, a normal (physiological) form of nystagmus

75
MRCP part 1 Made Easy‫‏‬‎‫‏‬by M. Habayeb & A. Murad
Chapter: Neurology

Paraneoplastic syndromes affecting nervous system

Lambert-Eaton myasthenic syndrome

 associated with small cell lung cancer (also breast and ovarian)

 antibody directed against pre-synaptic voltage gated calcium channel in the peripheral
nervous system

 can also occur independently as autoimmune disorder

Anti-Hu

 associated with small cell lung carcinoma and neuroblastomas

 sensory neuropathy - may be painful

 cerebellar syndrome

 encephalomyelitis

Anti-Yo

 associated with ovarian and breast cancer

 cerebellar syndrome

Anti-GAD antibody

 associated with breast, colorectal and small cell lung carcinoma

 stiff person's syndrome or diffuse hypertonia

Anti-Ri

 associated with breast and small cell lung carcinoma

 ocular opsoclonus-myoclonus

76
MRCP part 1 Made Easy‫‏‬‎‫‏‬by M. Habayeb & A. Murad
Chapter: Neurology

Parkinson's disease: features

Parkinson's disease is a progressive neurodegenerative condition caused by degeneration of


dopaminergic neurons in the substantia nigra.. This results in a classic triad of features:
bradykinesia, tremor and rigidity. The symptoms of Parkinson's disease are characteristically
asymmetrical.

Epidemiology
 around twice as common in men
 mean age of diagnosis is 65 years

Bradykinesia
 poverty of movement also seen, sometimes referred to as hypokinesia
 short, shuffling steps with reduced arm swinging
 difficulty in initiating movement

Tremor
 most marked at rest, 3-5 Hz
 worse when stressed or tired
 typically 'pill-rolling', i.e. in the thumb and index finger

Rigidity
 lead pipe
 cogwheel: due to superimposed tremor

Other characteristic features


 mask-like facies
 flexed posture
 micrographia
 drooling of saliva
 psychiatric features: depression is the most common feature (affects about 40%);
dementia, psychosis and sleep disturbances may also occur
 impaired olfaction
 REM sleep behaviour disorder

77
MRCP part 1 Made Easy‫‏‬‎‫‏‬by M. Habayeb & A. Murad
Chapter: Neurology

Drug-induced parkinsonism has slightly different features to Parkinson's disease:


 motor symptoms are generally rapid onset and bilateral
 rigidity and rest tremor are uncommon

Image sourced from Wikipedia

A Lewy body (stained brown) in a brain cell of the substantia nigra in Parkinson's
disease. The brown colour is positive immunohistochemistry staining for alpha-
synuclein

Image sourced from Wikipedia

Discoloration of the substantia nigra due to loss of pigmented nerve cells.

78
MRCP part 1 Made Easy‫‏‬‎‫‏‬by M. Habayeb & A. Murad
Chapter: Neurology

Parkinson's disease: management

Currently accepted practice in the management of patients with Parkinson's disease (PD) is to
delay treatment until the onset of disabling symptoms and then to introduce a dopamine
receptor agonist. If the patient is elderly, levodopa is sometimes used as an initial treatment.
Dopamine receptor agonists

 e.g. Bromocriptine, ropinirole, cabergoline, apomorphine

 ergot-derived dopamine receptor agonists (bromocriptine, cabergoline, pergolide*)


have been associated with pulmonary, retroperitoneal and cardiac fibrosis. The
Committee on Safety of Medicines advice that an echocardiogram, ESR, creatinine
and chest x-ray should be obtained prior to treatment and patients should be closely
monitored

 patients should be warned about the potential for dopamine receptor agonists to cause
impulse control disorders and excessive daytime somnolence

 more likely than levodopa to cause hallucinations in older patients. Nasal congestion
and postural hypotension are also seen in some patients

Levodopa

 usually combined with a decarboxylase inhibitor (e.g. carbidopa or benserazide) to


prevent peripheral metabolism of levodopa to dopamine

 reduced effectiveness with time (usually by 2 years)

 unwanted effects: dyskinesia (involuntary writhing movements), 'on-off' effect, dry


mouth, anorexia, palpitations, postural hypotension, psychosis, drowsiness

 no use in neuroleptic induced parkinsonism

MAO-B (Monoamine Oxidase-B) inhibitors

 e.g. Selegiline

 inhibits the breakdown of dopamine secreted by the dopaminergic neurons

Amantadine

 mechanism is not fully understood, probably increases dopamine release and inhibits
its uptake at dopaminergic synapses

 side-effects include ataxia, slurred speech, confusion, dizziness and livedo reticularis
79
MRCP part 1 Made Easy‫‏‬‎‫‏‬by M. Habayeb & A. Murad
Chapter: Neurology

COMT (Catechol-O-Methyl Transferase) inhibitors:


 e.g. Entacapone, tolcapone
 COMT is an enzyme involved in the breakdown of dopamine, and hence may be used
as an adjunct to levodopa therapy
 used in conjunction with levodopa in patients with established PD

Antimuscarinics:
 block cholinergic receptors
 now used more to treat drug-induced parkinsonism rather than idiopathic Parkinson's
disease
 help tremor and rigidity
 e.g. procyclidine, benzotropine, trihexyphenidyl (benzhexol)

Image sourced from Wikipedia

Diagram showing the mechanism of action of Parkinson's drugs

*pergolide was withdrawn from the US market in March 2007 due to concern regarding
increased incidence of valvular dysfunction

80
MRCP part 1 Made Easy‫‏‬‎‫‏‬by M. Habayeb & A. Murad
Chapter: Neurology

External Links:
NICE
2006 Parkinson's disease guidelines
SIGN
2010 Parkinson's disease guidelines
Royal College of Physicians
How to treat Parkinson's disease in 2013

Parkinsonism

Causes of Parkinsonism:
 Parkinson's disease
 drug-induced e.g. antipsychotics, metoclopramide - see below
 progressive supranuclear palsy
 multiple system atrophy
 Wilson's disease
 post-encephalitis
 dementia pugilistica (secondary to chronic head trauma e.g. boxing)
 toxins: carbon monoxide, MPTP

Drugs causing Parkinsonism

 phenothiazines: e.g. chlorpromazine, prochlorperazine


 butyrophenones: haloperidol, droperidol
 metoclopramide

Domperidone does not cross the blood-brain barrier and therefore does not cause extra-
pyramidal side-effects.

81
MRCP part 1 Made Easy‫‏‬‎‫‏‬by M. Habayeb & A. Murad
Chapter: Neurology

Peripheral neuropathy

Peripheral neuropathy may be divided into conditions which predominately cause a motor or
sensory loss

Predominately motor loss:


 Guillain-Barre syndrome
 porphyria
 lead poisoning
 hereditary sensorimotor neuropathies (HSMN) - Charcot-Marie-Tooth
 chronic inflammatory demyelinating polyneuropathy (CIDP)
 diphtheria

Predominately sensory loss


 diabetes
 uraemia
 leprosy
 alcoholism
 vitamin B12 deficiency
 amyloidosis

Alcoholic neuropathy
 secondary to both direct toxic effects and reduced absorption of B vitamins
 sensory symptoms typically present prior to motor symptoms
Vitamin B12 deficiency
 subacute combined degeneration of spinal cord
 dorsal column usually affected first (joint position, vibration) prior to distal
paraesthesia

82
MRCP part 1 Made Easy‫‏‬‎‫‏‬by M. Habayeb & A. Murad
Chapter: Neurology

Peripheral neuropathy: demyelinating vs. axonal

Demyelinating pathology

 Guillain-Barre syndrome

 chronic inflammatory demyelinating polyneuropathy (CIDP)

 amiodarone

 hereditary sensorimotor neuropathies (HSMN) type I

 paraprotein neuropathy

Axonal pathology

 alcohol

 diabetes mellitus*

 vasculitis

 vitamin B12 deficiency*

 hereditary sensorimotor neuropathies (HSMN) type II

* may also cause a demyelinating picture

Pituitary apoplexy

Sudden enlargement of pituitary tumour secondary to haemorrhage or infarction

Features

 sudden onset headache similar to that seen in subarachnoid haemorrhage

 vomiting

 neck stiffness

 visual field defects: classically bitemporal superior quadrantic defect

 extraocular nerve palsies

 features of pituitary insufficiency e.g. Hypotension secondary to hypoadrenalism

83
MRCP part 1 Made Easy‫‏‬‎‫‏‬by M. Habayeb & A. Murad
Chapter: Neurology

External Links:

Postgraduate Medical Journal

Review of endocrine emergencies

Post-lumbar puncture headache

Headache following lumbar puncture (LP) occurs in approximately one-third of patients. The
pathophysiology of is unclear but may relate to a 'leak' of CSF following dural puncture. Post-
LP headaches are more common in young females with a low body mass index

Typical features:

 usually develops within 24-48 hours following LP but may occur up to one week later

 may last several days

 worsens with upright position

 improves with recumbent position

Factors which may contribute Factors which do not


to headache contribute to headache
Increased needle size Increased volume of CSF
Direction of bevel removed
Not replacing the stylet Bed rest following procedure
Increased number of LP Increased fluid intake post
attempts procedure
Opening pressure of CSF
Position of patient

Management

 supportive initially (analgesia, rest)

 if pain continues for more than 72 hours then specific treatment is indicated, to
prevent subdural haematoma

 treatment options include: blood patch, epidural saline and intravenous caffeine

84
MRCP part 1 Made Easy‫‏‬‎‫‏‬by M. Habayeb & A. Murad
Chapter: Neurology

Progressive supranuclear palsy

Overview
 aka Steele-Richardson-Olszewski syndrome
 a 'Parkinson Plus' syndrome

Features
 impairment of vertical gaze (down gaze worse than up gaze - patients may complain
of difficultly reading or descending stairs)
 parkinsonism
 falls
 slurring of speech
 cognitive impairment

Management
 poor response to L-dopa

Ptosis

Ptosis may be unilateral or bilateral

Causes of bilateral ptosis:


 myotonic dystrophy
 myasthenia gravis*
 syphilis
 congenital

Causes of unilateral ptosis, as above plus:


 third nerve palsy
 Horner's

*ptosis is much less common in Lambert-Eaton syndrome than myasthenia gravis

85
MRCP part 1 Made Easy‫‏‬‎‫‏‬by M. Habayeb & A. Murad
Chapter: Neurology

Restless legs syndrome

Restless legs syndrome (RLS) is a syndrome of spontaneous, continuous lower limb


movements that may be associated with paraesthesia. It is extremely common, affecting
between 2-10% of the general population. Males and females are equally affected and a
family history may be present.

Clinical features

 uncontrollable urge to move legs (akathisia). Symptoms initially occur at night but as
condition progresses may occur during the day. Symptoms are worse at rest

 paraesthesias e.g. 'crawling' or 'throbbing' sensations

 movements during sleep may be noted by the partner - periodic limb movements of
sleeps (PLMS)

Causes and associations

 there is a positive family history in 50% of patients with idiopathic RLS

 iron deficiency anaemia

 uraemia

 diabetes mellitus

 pregnancy

The diagnosis is clinical although bloods to exclude iron deficiency anaemia may be
appropriate.

Management

 simple measures: walking, stretching, massaging affected limbs

 treat any iron deficiency

 dopamine agonists are first-line treatment (e.g. Pramipexole, ropinirole)

 benzodiazepines

 gabapentin

86
MRCP part 1 Made Easy‫‏‬‎‫‏‬by M. Habayeb & A. Murad
Chapter: Neurology

Reye's syndrome

Reye's syndrome is a severe, progressive encephalopathy affecting children that is


accompanied by fatty infiltration of the liver, kidneys and pancreas. The aetiology of Reye's
syndrome is not fully understood although there is a known association with aspirin use and a
viral cause has been postulated

The peak incidence is 2 years of age, features include:


 may be history of preceding viral illness
 encephalopathy: confusion, seizures, cerebral oedema, coma
 fatty infiltration of the liver, kidneys and pancreas
 hypoglycaemia

Management is supportive

Although the prognosis has improved over recent years there is still a mortality rate of 15-
25%.

Rinne's and Weber's test

Performing both Rinne's and Weber's test allows differentiation of conductive and
sensorineural deafness.

Rinne's test
 tuning fork is placed over the mastoid process until the sound is no longer heard,
followed by repositioning just over external acoustic meatus
 air conduction (AC) is normally better than bone conduction (BC)
 if BC > AC then conductive deafness

Weber's test
 tuning fork is placed in the middle of the forehead equidistant from the patient's ears
 the patient is then asked which side is loudest
 in unilateral sensorineural deafness, sound is localised to the unaffected side
 in unilateral conductive deafness, sound is localised to the affected side

87
MRCP part 1 Made Easy‫‏‬‎‫‏‬by M. Habayeb & A. Murad
Chapter: Neurology

External Links:
ENT SHO
Guide to the ear examination

Spastic Para paresis

Spastic paraparesis describes a upper motor neuron pattern of weakness in the lower limbs
Causes:
 demyelination e.g. multiple sclerosis
 cord compression: trauma, tumour
 parasagittal meningioma
 tropical spastic paraparesis
 transverse myelitis e.g. HIV
 syringomyelia
 hereditary spastic paraplegia
 osteoarthritis of the cervical spine

Spinal cord compression

Spinal cord compression is an oncological emergency and affects up to 5% of cancer patients.


Extradural compression accounts for the majority of cases, usually due to vertebral body
metastases. It is more common in patients with lung, breast and prostate cancer

Features
 back pain - the earliest and most common symptom - may be worse on lying down and
coughing
 lower limb weakness
 sensory changes: sensory loss and numbness
 neurological signs depend on the level of the lesion. Lesions above L1 usually result
in upper motor neuron signs in the legs and a sensory level. Lesions below L1 usually
cause lower motor neuron signs in the legs and perianal numbness. Tendon reflexes
tend to be increased below the level of the lesion and absent at the level of the lesion.

Management
 high-dose oral dexamethasone.
 urgent oncological assessment for consideration of radiotherapy or surgery.

88
MRCP part 1 Made Easy‫‏‬‎‫‏‬by M. Habayeb & A. Murad
Chapter: Neurology

External Links:
NICE
2008 Metastatic spinal cord compression guidelines

Spinal cord lesions

The diagram belows shows cross-section view of the spinal cord:

Image sourced from Wikipedia

Motor lesions

Amyotrophic lateral sclerosis (motor neuron disease):


 affects both upper (corticospinal tracts) and lower motor neurons
 results in a combination of upper and lower motor neuron signs

Poliomyelitis:
 affects anterior horns resulting in lower motor neuron signs.

89
MRCP part 1 Made Easy‫‏‬‎‫‏‬by M. Habayeb & A. Murad
Chapter: Neurology

Combined motor and sensory lesions

Disorder Tracts affected Clinical notes


Brown-Sequard 1. Lateral 1. Ipsilateral spastic
syndrome (spinal corticospinal tract paresis below lesion
cord hemisection) 2. Dorsal columns 2. Ipsilateral loss of
3. Lateral proprioception and
spinothalamic tract vibration sensation
3. Contralateral loss
of pain and
temperature
sensation
Subacute combined 1. Lateral 1. Bilateral spastic
degeneration of the corticospinal tracts paresis
spinal cord (vitamin 2. Dorsal columns 2. Bilateral loss of
B12 & E deficiency) 3. Spinocerebellar proprioception and
tracts vibration sensation
3. Bilateral limb
ataxia
Friedrich's ataxia Same as subacute Same as subacute
combined combined
degeneration of the degeneration of the
spinal cord (see spinal cord (see
above) above)

In addition
cerebellar ataxia →
other features e.g.
intention tremor
Anterior spinal 1. Lateral 1. Bilateral spastic
artery occlusion corticospinal tracts paresis
2. Lateral 2. Bilateral loss of
spinothalamic tracts pain and
temperature
sensation
Syringomyelia 1. Ventral horns 1. Flacid paresis
2. Lateral (typically affecting
spinothalamic tract the intrinsic hand
muscles)
2. Loss of pain and
temperature
sensation
Multiple sclerosis Asymmetrical, Combination of
varying spinal tracts motor, sensory and
involved ataxia symptoms

90
MRCP part 1 Made Easy‫‏‬‎‫‏‬by M. Habayeb & A. Murad
Chapter: Neurology

Sensory lesions

Disorder Tracts Clinical notes


affected
Neurosyphilis (tabes 1. Dorsal 1. Loss of proprioception and
dorsalis) columns vibration sensation

Stroke by anatomy

Site of the lesion Associated effects


Anterior cerebral artery Contralateral hemiparesis
and sensory loss, lower
extremity > upper
Middle cerebral artery Contralateral hemiparesis
and sensory loss, upper
extremity > lower
Contralateral homonymous
hemianopia
Aphasia
Posterior cerebral artery Contralateral homonymous
hemianopia with macular
sparing
Visual agnosia
Weber's syndrome (branches of the Ipsilateral CN III palsy
posterior cerebral artery that supply Contralateral weakness of
the midbrain) upper and lower extremity
Posterior inferior cerebellar artery Ipsilateral: facial pain and
(lateral medullary syndrome, temperature loss
Wallenberg syndrome) Contralateral: limb/torso
pain and temperature loss
Ataxia, nystagmus
Anterior inferior cerebellar artery Symptoms are similar to
(lateral pontine syndrome) Wallenberg's (see above),
but:
Ipsilateral: facial paralysis
and deafness
Retinal/ophthalmic artery Amaurosis fugax
Basilar artery 'Locked-in' syndrome

Lacunar strokes

 present with either isolated hemiparesis, hemisensory loss or hemiparesis with limb
ataxia.

 strong association with hypertension.

 common sites include the basal ganglia, thalamus and internal capsule.

91
MRCP part 1 Made Easy‫‏‬‎‫‏‬by M. Habayeb & A. Murad
Chapter: Neurology

Stroke: management

The Royal College of Physicians (RCP) published guidelines on the diagnosis and
management of patients following a stroke in 2004. NICE also issued stroke guidelines in
2008, although they modified their guidance with respect to antiplatelet therapy in 2010.
Selected points relating to the management of acute stroke include:
 blood glucose, hydration, oxygen saturation and temperature should be maintained
within normal limits
 blood pressure should not be lowered in the acute phase unless there are complications
e.g. Hypertensive encephalopathy*
 aspirin 300mg orally or rectally should be given as soon as possible if a haemorrhagic
stroke has been excluded
 with regards to atrial fibrillation, the RCP state: 'anticoagulants should not be started
until brain imaging has excluded haemorrhage, and usually not until 14 days have
passed from the onset of an ischaemic stroke'
 if the cholesterol is > 3.5 mmol/l patients should be commenced on a statin. Many
physicians will delay treatment until after at least 48 hours due to the risk of
haemorrhagic transformation
Thrombolysis
Thrombolysis should only be given if:
 it is administered within 4.5 hours of onset of stroke symptoms (unless as part of a
clinical trial)
 haemorrhage has been definitively excluded (i.e. Imaging has been performed)
♦Alteplase is currently recommended by NICE.
Contraindications to thrombolysis:

Absolute Relative
- Previous intracranial haemorrhage - Concurrent anticoagulation
- Seizure at onset of stroke (INR >1.7)
- Intracranial neoplasm - Haemorrhagic diathesis
- Suspected subarachnoid - Active diabetic
haemorrhage haemorrhagic retinopathy
- Stroke or traumatic brain injury in - Suspected intracardiac
preceding 3 months thrombus
- Lumbar puncture in preceding 7 - Major surgery / trauma in
days preceding 2 weeks
- Gastrointestinal haemorrhage in
preceding 3 weeks
- Active bleeding
- Pregnancy
- Oesophageal varices
- Uncontrolled hypertension
>200/120mmHg

92
MRCP part 1 Made Easy‫‏‬‎‫‏‬by M. Habayeb & A. Murad
Chapter: Neurology

Secondary prevention
NICE also published a technology appraisal in 2010 on the use of clopidogrel and
dipyridamole.

Recommendations from NICE include:


 clopidogrel is now recommended by NICE ahead of combination use of aspirin plus
modified release (MR) dipyridamole in people who have had an ischaemic stroke
 aspirin plus MR dipyridamole is now recommended after an ischaemic stroke only if
clopidogrel is contraindicated or not tolerated, but treatment is no longer limited to 2
years' duration
 MR dipyridamole alone is recommended after an ischaemic stroke only if aspirin or
clopidogrel are contraindicated or not tolerated, again with no limit on duration of
treatment
With regards to carotid artery endarterectomy:
 recommend if patient has suffered stroke or TIA in the carotid territory and are not
severely disabled
 should only be considered if carotid stenosis > 70% according ECST** criteria or >
50% according to NASCET*** criteria
*the 2009 Controlling hypertension and hypotension immediately post-stroke (CHHIPS) trial
may change thinking on this but guidelines have yet to change to reflect this
**European Carotid Surgery Trialists' Collaborative Group
***North American Symptomatic Carotid Endarterectomy Trial

External Links
NICE
2008 Stroke guidelines
SIGN
2008 Stroke guidelines
RCP
Stroke guidelines
NICE
2010 Clopidogrel and dipyridamole guidelines
Age and Ageing
Interesting article on managing blood pressure during acute stroke
Clinical Knowledge Summaries
Stroke and TIA guidelines

93
MRCP part 1 Made Easy‫‏‬‎‫‏‬by M. Habayeb & A. Murad
Chapter: Neurology

Stroke: types

The Oxford Stroke Classification (also known as the Bamford Classification) classifies
strokes based on the initial symptoms. A summary is as follows:

The following criteria should be assessed:


 1. unilateral hemiparesis and/or hemisensory loss of the face, arm & leg
 2. homonymous hemianopia
 3. higher cognitive dysfunction e.g. dysphasia

Total anterior circulation infarcts (TACI, c. 15%)


 involves middle and anterior cerebral arteries
 all 3 of the above criteria are present

Partial anterior circulation infarcts (PACI, c. 25%):


 involves smaller arteries of anterior circulation e.g. upper or lower division of middle
cerebral artery
 2 of the above criteria are present

Lacunar infarcts (LACI, c. 25%)

 involves perforating arteries around the internal capsule, thalamus and basal ganglia
 presents with 1 of the following:
 1. unilateral weakness (and/or sensory deficit) of face and arm, arm and leg or all
three.
 2. pure sensory stroke.
 3. ataxic hemiparesis

Posterior circulation infarcts (POCI, c. 25%):


 involves vertebrobasilar arteries
 presents with 1 of the following:
1. cerebellar or brainstem syndromes
2. loss of consciousness
3. isolated homonymous hemianopia.

94
MRCP part 1 Made Easy‫‏‬‎‫‏‬by M. Habayeb & A. Murad
Chapter: Neurology

Other recognized patterns of stroke:

Lateral medullary syndrome (posterior inferior cerebellar artery)


 aka Wallenberg's syndrome
 ipsilateral: ataxia, nystagmus, dysphagia, facial numbness, cranial nerve palsy e.g.
Horner's
 contralateral: limb sensory loss

Weber's syndrome
 ipsilateral III palsy
 contralateral weakness

External Links
YouTube
Bamford Classification

Subarachnoid hemorrhage

Causes:
 85% are due to rupture of berry aneurysms (conditions associated with berry
aneurysms include adult polycystic kidney disease, Ehlers-Danlos syndrome and
coarctation of the aorta)
 AV malformations
 trauma
 tumours

Investigations:
 CT: negative in 5%
 lumbar puncture: done after 12 hrs (allowing time for xanthochromia to develop)

Complications:
 rebleeding (in 30%)
 obstructive hydrocephalus (due to blood in ventricles)
 vasospasm leading to cerebral ischaemia

95
MRCP part 1 Made Easy‫‏‬‎‫‏‬by M. Habayeb & A. Murad
Chapter: Neurology

Management
 neurosurgical opinion: no clear evidence over early surgical intervention against
delayed intervention

 post-operative nimodipine (e.g. 60mg / 4 hrly, if BP allows) has been shown to reduce
the severity of neurological deficits but doesn't reduce rebleeding*

© Image used on license from Radiopaedia

CT image shows diffuse subarachnoid haemorrhage in all basal cisterns, bilateral sylvian
fissures and the inter-hemispheric fissure. This case demonstrates the typical distribution that
takes the blood into the subarachnoid space in a subarachnoid hemorrhage.

*the way nimodipine works in subarachnoid haemorrhage is not fully understood. It has been
previously postulated that it reduces cerebral vasospasm (hence maintaining cerebral
perfusion) but this has not been demonstrated in studies

96
MRCP part 1 Made Easy‫‏‬‎‫‏‬by M. Habayeb & A. Murad
Chapter: Neurology

External Link

SIGN

2008 Diagnosis and management of headache in adults

Subdural hemorrhage

Basics

 most commonly secondary to trauma e.g. old person/alcohol falling over

 initial injury may be minor and is often forgotten

 caused by bleeding from damaged bridging veins between cortex and venous sinuses

Features

 headache

 classically fluctuating conscious level

 raised ICP

Treatment

 needs neurosurgical review ? burr hole

External Links

Postgraduate Medical Journal

Chronic subdural haematoma

97
MRCP part 1 Made Easy‫‏‬‎‫‏‬by M. Habayeb & A. Murad
Chapter: Neurology

Syringomyelia

Overview

 development of cavity (syrinx) within the spinal cord

 if extends into medulla then termed syringobulbia

 strongly associated with the Arnold-Chiari malformation

Features

 maybe asymmetrical initially

 slowly progressives, possibly over years

 motor: wasting and weakness of arms

 sensory: spinothalamic sensory loss (pain and temperature)

 loss of reflexes, bilateral upgoing plantars

 also seen: Horner's syndrome

Tinnitus

Causes of tinnitus include:

Meniere's Associated with hearing loss, vertigo, tinnitus


disease and sensation of fullness or pressure in one or
both ears
Otosclerosis Onset is usually at 20-40 years
Conductive deafness
Tinnitus
Normal tympanic membrane*
Positive family history
Acoustic Hearing loss, vertigo, tinnitus
neuroma Absent corneal reflex is important sign
Associated with neurofibromatosis type 2
Hearing loss Causes include excessive loud noise and
presbycusis
Drugs Aspirin
Aminoglycosides
Loop diuretics
Quinine

98
MRCP part 1 Made Easy‫‏‬‎‫‏‬by M. Habayeb & A. Murad
Chapter: Neurology

Other causes include:


 impacted ear wax
 chronic suppurative otitis media

*10% of patients may have a 'flamingo tinge', caused by hyperaemia

Transient global amnesia

Overview
 presents with transient loss of memory function
 patients may appear anxious and repeatedly ask the same question
 patients have no recall of events after the attack
 etiology is unknown, thought to be due to transient ischaemia to the thalamus (in
particular the amygdala and hippocampus)

Transient ischemic attack

NICE issued updated guidelines relating to stroke and transient ischaemic attack (TIA) in
2008. They advocated the use of the ABCD2 prognostic score for risk stratifying patients
who've had a suspected TIA:

Criteria Points
A Age >= 60 years 1
B Blood pressure >= 140/90 mmHg 1
C Clinical features
- Unilateral weakness 2
- Speech disturbance, no weakness 1
D Duration of symptoms
- > 60 minutes 2
- 10-59 minutes 1
Patient has diabetes 1

This gives a total score ranging from 0 to 7. People who have had a suspected TIA who are
at a higher risk of stroke (that is, with an ABCD2 score of 4 or above) should have:
 aspirin (300 mg daily) started immediately.
 specialist assessment and investigation within 24 hours of onset of symptoms.
 measures for secondary prevention introduced as soon as the diagnosis is confirmed,
including discussion of individual risk factors.

99
MRCP part 1 Made Easy‫‏‬‎‫‏‬by M. Habayeb & A. Murad
Chapter: Neurology

If the ABCD2 risk score is 3 or below:

 specialist assessment within 1 week of symptom onset, including decision on brain


imaging

 if vascular territory or pathology is uncertain, refer for brain imaging

People with crescendo TIAs (two or more episodes in a week) should be treated as being at
high risk of stroke, even though they may have an ABCD2 score of 3 or below.

Antithrombotic therapy:

 clopidogrel is recommended first-line (as for patients who've had a stroke)

 aspirin + dipyridamole should be given to patients who cannot tolerate clopidogrel

 these recommendations follow the 2012 Royal College of Physicians National clinical
guideline for stroke. Please see the link for more details (section 5.5)

 these guidelines may change following the CHANCE study (NEJM 2013;369:11).
This study looked at giving high-risk TIA patients aspirin + clopidogrel for the first 90
days compared to aspirin alone. 11.7% of aspirin only patients had a stroke over 90
days compared to 8.2% of dual antiplatelet patients

With regards to carotid artery endarterectomy:

 recommend if patient has suffered stroke or TIA in the carotid territory and are not
severely disabled

 should only be considered if carotid stenosis > 70% according ECST* criteria or >
50% according to NASCET** criteria

*European Carotid Surgery Trialists' Collaborative Group


**North American Symptomatic Carotid Endarterectomy Trial

External Links

NICE

2008 Stroke: Diagnosis and initial management of acute stroke and transient ischaemic attack
(TIA)

Royal College of Physicians

2016 National clinical guideline for stroke

100
MRCP part 1 Made Easy‫‏‬‎‫‏‬by M. Habayeb & A. Murad
Chapter: Neurology

Trigeminal neuralgia

Trigeminal neuralgia is a pain syndrome characterised by severe unilateral pain. The vast
majority of cases are idiopathic but compression of the trigeminal roots by tumours or
vascular problems may occur.

The International Headache Society defines trigeminal neuralgia as:


 a unilateral disorder characterised by brief electric shock-like pains, abrupt in onset
and termination, limited to one or more divisions of the trigeminal nerve
 the pain is commonly evoked by light touch, including washing, shaving, smoking,
talking, and brushing the teeth (trigger factors), and frequently occurs spontaneously
 small areas in the nasolabial fold or chin may be particularly susceptible to the
precipitation of pain (trigger areas)
 the pains usually remit for variable periods

Management:
 carbamazepine is first-line
 failure to respond to treatment or atypical features (e.g. < 50 years old) should prompt
referral to neurology

External Links
Clinical Knowledge Summaries
Trigeminal Neuralgia guidelines

Triptans

Triptans are specific 5-HT1 agonists used in the acute treatment of migraine. They are
generally used first-line in combination therapy with an NSAID or paracetamol.

Prescribing points:
 should be taken as soon as possible after the onset of headache, rather than at onset of
aura
 oral, orodispersible, nasal spray and subcutaneous injections are available

Adverse effects:
 'triptan sensations' - tingling, heat, tightness (e.g. throat and chest), heaviness, pressure

Contraindications:
 patients with a history of, or significant risk factors for, ischaemic heart disease or
cerebrovascular disease
101
MRCP part 1 Made Easy‫‏‬‎‫‏‬by M. Habayeb & A. Murad
Chapter: Neurology

External Links

Clinical Knowledge Summaries

Migraine guidelines

Tuberous sclerosis

Tuberous sclerosis (TS) is a genetic condition of autosomal dominant inheritance. Like


neurofibromatosis, the majority of features seen in TS are neuro-cutaneous

Cutaneous features:

 depigmented 'ash-leaf' spots which fluoresce under UV light

 roughened patches of skin over lumbar spine (Shagreen patches)

 adenoma sebaceum (angiofibromas): butterfly distribution over nose

 fibromata beneath nails (subungual fibromata)

 café-au-lait spots* may be seen

Neurological features

 developmental delay

 epilepsy (infantile spasms or partial)

 intellectual impairment

Also

 retinal hamartomas: dense white areas on retina (phakomata)

 rhabdomyomas of the heart

 gliomatous changes can occur in the brain lesions

 polycystic kidneys, renal angiomyolipomata

 lymphangioleiomyomatosis: multiple lung cysts

102
MRCP part 1 Made Easy‫‏‬‎‫‏‬by M. Habayeb & A. Murad
Chapter: Neurology

Comparison of neurofibromatosis and tuberous sclerosis. Note that whilst they are both
autosomal dominant neurocutaneous disorders there is little overlap otherwise

*these of course are more commonly associated with neurofibromatosis. However a 1998
study of 106 children with TS found café-au-lait spots in 28% of patients

External Links

Patient.info

Tuberous sclerosis review

Vigabatrin

Key points

 40% of patients develop visual field defects, which may be irreversible

 visual fields should be checked every 6 months

103
MRCP part 1 Made Easy‫‏‬‎‫‏‬by M. Habayeb & A. Murad
Chapter: Neurology

Visual field defects

The main points for the exam are:

 left homonymous hemianopia means visual field defect to the left, i.e. Lesion of right
optic tract

 homonymous quadrantanopias: PITS (Parietal-Inferior, Temporal-Superior)

 incongruous defects = optic tract lesion; congruous defects = optic radiation lesion or
occipital cortex

A congruous defect simply means complete or symmetrical visual field loss and conversely
an incongruous defect is incomplete or asymmetric. Please see the link for an excellent
diagram.

Homonymous hemianopia:

 incongruous defects: lesion of optic tract

 congruous defects: lesion of optic radiation or occipital cortex

 macula sparing: lesion of occipital cortex

Homonymous quadrantanopias*:

 superior: lesion of temporal lobe

 inferior: lesion of parietal lobe

 mnemonic = PITS (Parietal-Inferior, Temporal-Superior)

Bitemporal hemianopia:

 lesion of optic chiasm

 upper quadrant defect > lower quadrant defect = inferior chiasmal compression,
commonly a pituitary tumour

 lower quadrant defect > upper quadrant defect = superior chiasmal compression,
commonly a craniopharyngioma

*this is very much the 'exam answer'. Actual studies suggest that the majority of
quadrantanopias are caused by occipital lobe lesions. Please see the following link for more
details.

104
MRCP part 1 Made Easy‫‏‬‎‫‏‬by M. Habayeb & A. Murad
Chapter: Neurology

External Links

Patient.info

Visual field defects

PubMed

The localizing value of a quadrantanopia

Von Hippel-Lindau syndrome

Von Hippel-Lindau (VHL) syndrome is an autosomal dominant condition predisposing to


neoplasia. It is due to an abnormality in the VHL gene located on short arm of chromosome 3

Features

 cerebellar haemangiomas

 retinal haemangiomas: vitreous haemorrhage

 renal cysts (premalignant)

 phaeochromocytoma

 extra-renal cysts: epididymal, pancreatic, hepatic

 endolymphatic sac tumours

105
MRCP part 1 Made Easy‫‏‬‎‫‏‬by M. Habayeb & A. Murad
Chapter: Neurology

© Image used on license from Radiopaedia

CT scan showing a cerebellar haemangioma in a patient with Von Hippel-Lindau syndrome.

106
MRCP part 1 Made Easy‫‏‬‎‫‏‬by M. Habayeb & A. Murad
Chapter: Neurology

© Image used on license from Radiopaedia

MRI showing renal cysts in patient with known Von Hippel-Lindau syndrome.

107
MRCP part 1 Made Easy‫‏‬‎‫‏‬by M. Habayeb & A. Murad
Chapter: Neurology

Question 1 of 246

A 27-year-old man presents to the Emergency Department with 2 day history of severe headache and
pyrexia (38.2ºC). A CT scan is reported as follows:

CT: Brain

Petechial haemorrhages in the temporal and inferior frontal lobes. No mass effect. Brain parenchyma
otherwise normal

What is the most likely diagnosis?

Brain abscess
Meningococcal meningitis
Cerebral malaria
Herpes simplex encephalitis
New variant CJD

108

MRCP part 1 Made Easy‫‏‬‎‫‏‬by M. Habayeb & A. Murad


Chapter: Neurology

Question 1 of 246

A 27-year-old man presents to the Emergency Department with 2 day history of severe headache and
pyrexia (38.2ºC). A CT scan is reported as follows:

CT: Brain

Petechial haemorrhages in the temporal and inferior frontal lobes. No mass effect. Brain parenchyma
otherwise normal

What is the most likely diagnosis?

Brain abscess
Meningococcal meningitis
Cerebral malaria
Herpes simplex encephalitis
New variant CJD

CT head showing temporal lobe changes - think herpes simplex encephalitis

Discuss and give feedback

Herpes simplex encephalitis

109

MRCP part 1 Made Easy‫‏‬‎‫‏‬by M. Habayeb & A. Murad


Chapter: Neurology

Question 2 of 246

A 23-year-old man presents with recurrent headaches. These typically occur 2 or 3 times a month and
are characterised by severe, right-sided headaches which are throbbing in nature and last around 8-12
hours. When he gets the headaches he finds it hard to carry on working and tends to go and lie down in
a dark room. The headaches so far have responded poorly to paracetamol.

Which one of the following medications should be prescribed to help reduce the frequency of
these headaches?

Ibuprofen
Pizotifen
Propranolol
Amitriptyline
Carbamazepine

110

MRCP part 1 Made Easy‫‏‬‎‫‏‬by M. Habayeb & A. Murad


Chapter: Neurology

Question 2 of 246

A 23-year-old man presents with recurrent headaches. These typically occur 2 or 3 times a month and
are characterised by severe, right-sided headaches which are throbbing in nature and last around 8-12
hours. When he gets the headaches he finds it hard to carry on working and tends to go and lie down in
a dark room. The headaches so far have responded poorly to paracetamol.
Which one of the following medications should be prescribed to help reduce the frequency of
these headaches?

Ibuprofen
Pizotifen
Propranolol
Amitriptyline
Carbamazepine

Migraine

 acute: triptan + NSAID or triptan + paracetamol

 prophylaxis: topiramate or propranolol

This is a classic history of migraine, prophylaxis should be offered with propranolol or


topiramate.

Discuss and give feedback

Migraine: management

External links

SIGN 2008 Diagnosis and management of headache in adults

Clinical Knowledge Summaries Migraine guidelines

NICE 2012 Headache guidelines

British Association for the Study of Headache Headache guidelines

111

MRCP part 1 Made Easy‫‏‬‎‫‏‬by M. Habayeb & A. Murad


Chapter: Neurology

Question 3 of 246

A 29-year-old man presents complaining of visual disturbance. Examination reveals a right superior
homonymous quadrantanopia. Where is the lesion most likely to be?

Optic chiasm
Left temporal lobe
Right temporal lobe
Left optic nerve
Left parietal lobe

112

MRCP part 1 Made Easy‫‏‬‎‫‏‬by M. Habayeb & A. Murad


Chapter: Neurology

Question 3 of 246

A 29-year-old man presents complaining of visual disturbance. Examination reveals a right superior
homonymous quadrantanopia. Where is the lesion most likely to be?

Optic chiasm
Left temporal lobe
Right temporal lobe
Left optic nerve
Left parietal lobe

Visual field defects:

 left homonymous hemianopia means visual field defect to the left, i.e. lesion of right optic tract

 homonymous quadrantanopias: PITS (Parietal-Inferior, Temporal-Superior)

 incongruous defects = optic tract lesion; congruous defects= optic radiation lesion or occipital
cortex

Discuss and give feedback

Visual field defects


External links
Patient.info
Visual field defects
PubMed
The localizing value of a quadrantanopia

113

MRCP part 1 Made Easy‫‏‬‎‫‏‬by M. Habayeb & A. Murad


Chapter: Neurology

Question 4 of 246

A 64-year-old man with a history of Parkinson's disease is reviewed in clinic and a decision has been
made to start him on cabergoline. Which one of the following adverse effects is most strongly
associated with this drug?

Optic neuritis
Transient rise in liver function tests
Pulmonary fibrosis
Renal failure
Thrombocytopenia

114

MRCP part 1 Made Easy‫‏‬‎‫‏‬by M. Habayeb & A. Murad


Chapter: Neurology

Question 4 of 246

A 64-year-old man with a history of Parkinson's disease is reviewed in clinic and a decision has been
made to start him on cabergoline. Which one of the following adverse effects is most strongly
associated with this drug?

Optic neuritis
Transient rise in liver function tests
Pulmonary fibrosis
Renal failure
Thrombocytopenia

Discuss and give feedback

Parkinson's disease: management


External links
SIGN
2010 Parkinson's disease guidelines
NICE
2006 Parkinson's disease guidelines
Royal College of Physicians
How to treat Parkinson's disease in 2013

115

MRCP part 1 Made Easy‫‏‬‎‫‏‬by M. Habayeb & A. Murad


Chapter: Neurology

Question 5 of 246

A 76-year-old man is admitted with a right hemiparesis. He first noticed weakness on his right side
around six hours ago. A CT scan shows an ischaemic stroke and aspirin 300mg is commenced. terms
of further management in the acute phase, which one of the following values should not be
corrected?

BP 210/110
Blood glucose 11.2 mmol/l
Oxygen saturation 93%
Temp 38.3ºC
Blood glucose 3.5 mmol/l

116

MRCP part 1 Made Easy‫‏‬‎‫‏‬by M. Habayeb & A. Murad


Chapter: Neurology

Question 5 of 246
A 76-year-old man is admitted with a right hemiparesis. He first noticed weakness on his right side
around six hours ago. A CT scan shows an ischaemic stroke and aspirin 300mg is commenced. terms
of further management in the acute phase, which one of the following values should not be
corrected?

BP 210/110
Blood glucose 11.2 mmol/l
Oxygen saturation 93%
Temp 38.3ºC
Blood glucose 3.5 mmol/l

Hypertension should not be treated in the initial period following a stroke

Elevated blood pressure should not be treated in the acute phase following a stroke unless
complications develop. Other physiological parameters should be kept within normal limits - an
aggressive approach with respect to this has been shown to improve outcome
Discuss and give feedback

Stroke: management
External links
NICE 2010 Clopidogrel and dipyridamole guidelines
SIGN 2008 Stroke guidelines
Age and Ageing Interesting article on managing blood pressure during acute stroke
RCP Stroke guidelines
NICE 2008 TIA and stroke guidelines

Clinical Knowledge Summaries Stroke and TIA guidelines

117

MRCP part 1 Made Easy‫‏‬‎‫‏‬by M. Habayeb & A. Murad


Chapter: Neurology

Question 6 of 246

A 23-year-old man is referred to neurology clinic. He describes episodes of leg weakness following
bouts of laughing whilst out with friends. The following weekend his friends described a brief collapse
following a similar episode. What is the most likely diagnosis?

Stokes-Adams attack
Cataplexy
Hypokalaemic periodic paralysis
Absence seizure
Myasthenia gravis

118

MRCP part 1 Made Easy‫‏‬‎‫‏‬by M. Habayeb & A. Murad


Chapter: Neurology

Question 6 of 246

A 23-year-old man is referred to neurology clinic. He describes episodes of leg weakness following
bouts of laughing whilst out with friends. The following weekend his friends described a brief collapse
following a similar episode. What is the most likely diagnosis?

Stokes-Adams attack
Cataplexy
Hypokalaemic periodic paralysis
Absence seizure
Myasthenia gravis

Discuss and give feedback

Cataplexy

119

MRCP part 1 Made Easy‫‏‬‎‫‏‬by M. Habayeb & A. Murad


Chapter: Neurology

Question 7 of 246

A 44-year-old woman presents with pain in her right hand and forearm which has been getting worse
for the past few weeks. There is no history of trauma. The pain is concentrated around the thumb and
index finger and is often worse at night. Shaking her hand seems to provide some relief. On
examination there is weakness of the abductor pollicis brevis and reduced sensation to fine touch at the
index finger. What is the most likely diagnosis?

C6 entrapment neuropathy
Thoracic outlet syndrome
Carpal tunnel syndrome
Cervical rib
Pancoast's tumour

120

MRCP part 1 Made Easy‫‏‬‎‫‏‬by M. Habayeb & A. Murad


Chapter: Neurology

Question 7 of 246
A 44-year-old woman presents with pain in her right hand and forearm which has been getting worse
for the past few weeks. There is no history of trauma. The pain is concentrated around the thumb and
index finger and is often worse at night. Shaking her hand seems to provide some relief. On
examination there is weakness of the abductor pollicis brevis and reduced sensation to fine touch at the
index finger. What is the most likely diagnosis?

C6 entrapment neuropathy
Thoracic outlet syndrome
Carpal tunnel syndrome
Cervical rib
Pancoast's tumour

♦More proximal symptoms would be expected with a C6 entrapment neuropathy e.g. weakness of the
biceps muscle or reduced biceps reflex.

♦Patients with carpal tunnel syndrome often get relief from shaking their hands and this may be an
important clue in exam questions.
Discuss and give feedback

Carpal tunnel syndrome


Carpal tunnel syndrome is caused by compression of median nerve in the carpal tunnel.

History
 pain/pins and needles in thumb, index, middle finger
 unusually the symptoms may 'ascend' proximally
 patient shakes his hand to obtain relief, classically at night

 Examination
 weakness of thumb abduction (abductor pollicis brevis)
 wasting of thenar eminence (NOT hypothenar)
 Tinel's sign: tapping causes paraesthesia
 Phalen's sign: flexion of wrist causes symptoms

121

MRCP part 1 Made Easy‫‏‬‎‫‏‬by M. Habayeb & A. Murad


Chapter: Neurology

Causes

 idiopathic

 pregnancy

 oedema e.g. heart failure

 lunate fracture

 rheumatoid arthritis

Electrophysiology

 motor + sensory: prolongation of the action potential

Treatment

 corticosteroid injection

 wrist splints at night

 surgical decompression (flexor retinaculum division)

External media

Carpal tunnel syndrome

Osmosis - YouTube

122

MRCP part 1 Made Easy‫‏‬‎‫‏‬by M. Habayeb & A. Murad


Chapter: Neurology

Question 8 of 246

You are called to the obstetric ward to see a woman who is fitting. She is 34-weeks pregnant and
currently an inpatient for the treatment of severe pre-eclampsia. The anaesthetist has secured the
airway and is giving 100% oxygen. What is the most appropriate next step?

IV calcium gluconate
IV labetalol
IV methyldopa
IV lorazepam
IV magnesium sulphate

123

MRCP part 1 Made Easy‫‏‬‎‫‏‬by M. Habayeb & A. Murad


Chapter: Neurology

Question 8 of 246

You are called to the obstetric ward to see a woman who is fitting. She is 34-weeks pregnant and
currently an inpatient for the treatment of severe pre-eclampsia. The anaesthetist has secured the
airway and is giving 100% oxygen. What is the most appropriate next step?

IV calcium gluconate
IV labetalol
IV methyldopa
IV lorazepam
IV magnesium sulphate

Eclampsia - give magnesium sulphate first-line

Discuss and give feedback

Eclampsia

External links

NICE

2010 Hypertension in pregnancy

124

MRCP part 1 Made Easy‫‏‬‎‫‏‬by M. Habayeb & A. Murad


Chapter: Neurology

Question 9 of 246

A 56-year-old woman presents with facial asymmetry. Whilst brushing her teeth this morning she
noted that the right hand corner of her mouth was drooping. She is generally well but noted some pain
behind her right ear yesterday and says her right eye is becoming dry. On examination she has a
complete paralysis of the facial nerve on the right side, extending from the forehead to the mouth. Ear,
nose and throat examination is normal. Clinical examination of the peripheral nervous system is
normal. What is the most likely diagnosis?

Ramsey-Hunt syndrome
Bell's palsy
Stroke
Multiple sclerosis
Parotid tumour

125

MRCP part 1 Made Easy‫‏‬‎‫‏‬by M. Habayeb & A. Murad


Chapter: Neurology

Question 9 of 246

A 56-year-old woman presents with facial asymmetry. Whilst brushing her teeth this morning she
noted that the right hand corner of her mouth was drooping. She is generally well but noted some pain
behind her right ear yesterday and says her right eye is becoming dry. On examination she has a
complete paralysis of the facial nerve on the right side, extending from the forehead to the mouth. Ear,
nose and throat examination is normal. Clinical examination of the peripheral nervous system is
normal. What is the most likely diagnosis?

Ramsey-Hunt syndrome
Bell's palsy
Stroke
Multiple sclerosis
Parotid tumour

The pain around the ear raises the possibility of Ramsey-Hunt syndrome but this is actually quite
common in Bell's palsy - some studies suggest it is seen in 50% of patients. The normal ear exam also
goes against this diagnosis.

Discuss and give feedback

Bell's palsy

External links

Clinical Knowledge Summaries

Bell's palsy guidelines

126

MRCP part 1 Made Easy‫‏‬‎‫‏‬by M. Habayeb & A. Murad


Chapter: Neurology

Question 10 of 246

A 29-year-old woman with a past history of hypothyroidism presents to the surgery complaining of
weakness, particularly of her arms, for the past four months. She has also developed double vision
towards the end of the day, despite having a recent normal examination at the opticians. What is the
most likely diagnosis?

LambertEaton myasthenic syndrome


Polymyositis
Polymyalgia rheumatica
Multiple sclerosis
Myasthenia gravis

127

MRCP part 1 Made Easy‫‏‬‎‫‏‬by M. Habayeb & A. Murad


Chapter: Neurology

Question 10 of 246

A 29-year-old woman with a past history of hypothyroidism presents to the surgery complaining of
weakness, particularly of her arms, for the past four months. She has also developed double vision
towards the end of the day, despite having a recent normal examination at the opticians. What is the
most likely diagnosis?

LambertEaton myasthenic syndrome


Polymyositis
Polymyalgia rheumatica
Multiple sclerosis
Myasthenia gravis

Discuss and give feedback

Myasthenia gravis

External links

Postgraduate Medical Journal

Review of myasthenia gravis

External media

Myasthenia gravis - causes, symptoms, treatment, pathology

Osmosis – YouTube

128

MRCP part 1 Made Easy‫‏‬‎‫‏‬by M. Habayeb & A. Murad


Chapter: Neurology

Question 11 of 246

Which one of the following statements regarding the development of a headache following
lumbar puncture is NOT correct?

Occurs in one-third of patients


May last several days
More common in females
Usually develops within the first 12 hours
Worsens with upright position

129

MRCP part 1 Made Easy‫‏‬‎‫‏‬by M. Habayeb & A. Murad


Chapter: Neurology

Question 11 of 246

Which one of the following statements regarding the development of a headache following
lumbar puncture is NOT correct?

Occurs in one-third of patients


May last several days
More common in females
Usually develops within the first 12 hours
Worsens with upright position

Discuss and give feedback

Post-lumbar puncture headache

130

MRCP part 1 Made Easy‫‏‬‎‫‏‬by M. Habayeb & A. Murad


Chapter: Neurology

Question 12 of 246

A 70-year-old man who presented with repetitive large involuntary movements of right his lower limb
and right upper limb has been diagnosed with hemiballismus. This affects which part of the brain?

Medial thalamus and mammillary bodies of the hypothalamus


Subthalamic nucleus of the basal ganglia
Amygdala
Substantia nigra of the basal ganglia
Striatum (caudate nucleus) of the basal ganglia

131

MRCP part 1 Made Easy‫‏‬‎‫‏‬by M. Habayeb & A. Murad


Chapter: Neurology

Question 12 of 246

A 70-year-old man who presented with repetitive large involuntary movements of right his lower limb
and right upper limb has been diagnosed with hemiballismus. This affects which part of the brain?

Medial thalamus and mammillary bodies of the hypothalamus


Subthalamic nucleus of the basal ganglia
Amygdala
Substantia nigra of the basal ganglia
Striatum (caudate nucleus) of the basal ganglia

♦Hemiballism is a type of chorea which is caused by a decreased activity in the subthalamic nucleus of
the basal ganglia in most cases.

♦The other areas of the brain are associated with other conditions. Wernicke and Korsakoff syndrome
localises to the medial thalamus and mammillary bodies of the hypothalamus. Huntington chorea is
localised to the striatum (caudate nucleus) of the basal ganglia. Parkinson's disease is caused by
disease of the substantia nigra of the basal ganglia. Kluver-Bucy syndrome is caused by damage both
temporal lobes including the amygdala.

Discuss and give feedback

Brain lesion

External links

American Speech-Language-Hearing Association

Common Classifications of Aphasia

132

MRCP part 1 Made Easy‫‏‬‎‫‏‬by M. Habayeb & A. Murad


Chapter: Neurology

Question 13 of 246

A 34-year-old female presents with vomiting preceded by an occipital headache of acute onset. On
examination she was conscious and alert with photophobia but no neck stiffness. CT brain is reported
as normal. What is the most appropriate further management?

CT brain with contrast


Repeat CT brain in 24h
CSF examination
Cerebral angiography
MRI brain

133

MRCP part 1 Made Easy‫‏‬‎‫‏‬by M. Habayeb & A. Murad


Chapter: Neurology

Question 13 of 246

A 34-year-old female presents with vomiting preceded by an occipital headache of acute onset. On
examination she was conscious and alert with photophobia but no neck stiffness. CT brain is reported
as normal. What is the most appropriate further management?

CT brain with contrast


Repeat CT brain in 24h
CSF examination
Cerebral angiography
MRI brain

If the CSF examination revealed xanthochromia, or there was still a high level of clinical suspicion,
then cerebral angiography would be the next step.

Discuss and give feedback

Subarachnoid haemorrhage

External links

SIGN

2008 Diagnosis and management of headache in adults

134

MRCP part 1 Made Easy‫‏‬‎‫‏‬by M. Habayeb & A. Murad


Chapter: Neurology

Question 14 of 246

In patients with Guillain-Barre syndrome, respiratory function should be monitored with:

Oxygen saturations
PEFR
Flow volume loop
Arterial blood gases
Forced vital capacity

135

MRCP part 1 Made Easy‫‏‬‎‫‏‬by M. Habayeb & A. Murad


Chapter: Neurology

question 14 of 246

In patients with Guillain-Barre syndrome, respiratory function should be monitored with:

Oxygen saturations
PEFR
Flow volume loop
Arterial blood gases
Forced vital capacity

FVC is used to monitor respiratory function in Guillain-Barre syndrome

Discuss and give feedback

Guillain-Barre syndrome: management

136

MRCP part 1 Made Easy‫‏‬‎‫‏‬by M. Habayeb & A. Murad


Chapter: Neurology

Question 15 of 246

A 73-year-old female with a history of recurrent falls at home and alcohol excess is brought to the
Emergency Department due to episodes of confusion over the past 5 days. Between these episodes she
is apparently her normal self. On examination her GCS is 14/15 and she has nystagmus on left lateral
gaze. What is the most likely diagnosis?

Subdural haemorrhage
Subarachnoid haemorrhage
Meningitis
Herpes simplex encephalitis
Alzheimer's disease

137

MRCP part 1 Made Easy‫‏‬‎‫‏‬by M. Habayeb & A. Murad


Chapter: Neurology

Question 15 of 246

A 73-year-old female with a history of recurrent falls at home and alcohol excess is brought to the
Emergency Department due to episodes of confusion over the past 5 days. Between these episodes she
is apparently her normal self. On examination her GCS is 14/15 and she has nystagmus on left lateral
gaze. What is the most likely diagnosis?

Subdural haemorrhage
Subarachnoid haemorrhage
Meningitis
Herpes simplex encephalitis
Alzheimer's disease

Fluctuating consciousness = subdural haemorrhage

♦The combination of falls, alcohol excess, fluctuating episodes of confusion and focal neurology
points towards a diagnosis of subdural haemorrhage.

♦The phrase 'fluctuating conscious level' is common in questions and should always bring to mind
subdural haemorrhage

Discuss and give feedback

Subdural hemorrhage

External links

Postgraduate Medical Journal

Chronic subdural haematoma

138

MRCP part 1 Made Easy‫‏‬‎‫‏‬by M. Habayeb & A. Murad


Chapter: Neurology

Question 16 of 246

A 61-year-old man with a chronic cough and gradual weight loss presents to the clinic with gradually
worsening proximal myopathy over the past few weeks. A hilar mass has been identified on chest x-
ray and he is awaiting referral to the respiratory oncology clinic. Neurological testing confirms
weakness which is improved by reinforcement and movement repetition.

Which of the following autoantibodies is most likely to be found?

Anti-AChR
Anti-Hu
Anti-Ri
Anti-Yo
Anti-VGCC

139

MRCP part 1 Made Easy‫‏‬‎‫‏‬by M. Habayeb & A. Murad


Chapter: Neurology

Question 16 of 246

A 61-year-old man with a chronic cough and gradual weight loss presents to the clinic with gradually
worsening proximal myopathy over the past few weeks. A hilar mass has been identified on chest x-
ray and he is awaiting referral to the respiratory oncology clinic. Neurological testing confirms
weakness which is improved by reinforcement and movement repetition.

Which of the following autoantibodies is most likely to be found?

Anti-AChR
Anti-Hu
Anti-Ri
Anti-Yo
Anti-VGCC

♦The most likely diagnosis is Lambert-Eaton syndrome, related to antibodies directed against voltage-
gated calcium channels, resulting in proximal myopathy. A key difference from myasthenia gravis is
that muscle power increases with reinforcement. There is a strong association with underlying
bronchial carcinoma and symptoms often improve when the underlying tumour is treated.

♦Anti-AChR antibodies are associated with the development of myasthenia gravis. Anti-Hu antibodies
are associated with paraneoplastic encephalomyelitis, also associated with bronchial carcinoma. Anti-
Ri antibodies are associated with ocular opsoclonus. Anti-Yo antibodies are associated with
paraneoplastic cerebellar degeneration.

Discuss and give feedback

Paraneoplastic syndromes affecting nervous system

140

MRCP part 1 Made Easy‫‏‬‎‫‏‬by M. Habayeb & A. Murad


Chapter: Neurology

Question 17 of 246

A 47-year-old man with a known history of schizophrenia is admitted to the Emergency Department
due to confusion. A bottle of procyclidine tablets are found in his pocket. On examination the
temperature is 38.1ºC with a blood pressure of 155/100 mmHg. Neurological examination reveals a
GCS of 13/15 but assessment of his peripheral nervous system is difficult due to generalised increased
muscle tone. What is the most likely diagnosis?

Neuroleptic malignant syndrome


Procyclidine overdose
Catatonic schizophrenia
Clozapine induced agranulocytosis
Quetiapine induced rhabdomyolysis

141

MRCP part 1 Made Easy‫‏‬‎‫‏‬by M. Habayeb & A. Murad


Chapter: Neurology

Question 17 of 246
A 47-year-old man with a known history of schizophrenia is admitted to the Emergency Department
due to confusion. A bottle of procyclidine tablets are found in his pocket. On examination the
temperature is 38.1ºC with a blood pressure of 155/100 mmHg. Neurological examination reveals a
GCS of 13/15 but assessment of his peripheral nervous system is difficult due to generalised increased
muscle tone. What is the most likely diagnosis?

Neuroleptic malignant syndrome


Procyclidine overdose
Catatonic schizophrenia
Clozapine induced agranulocytosis
Quetiapine induced rhabdomyolysis
Discuss and give feedback

Neuroleptic malignant syndrome


Neuroleptic malignant syndrome is a rare but dangerous condition seen in patients taking antipsychotic
medication. It carries a mortality of up to 10% and can also occur with atypical antipsychotics. It may
also occur with dopaminergic drugs (such as levodopa) for Parkinson's disease, usually when the drug
is suddenly stopped or the dose reduced.

Features
 more common in young male patients
 onset usually in first 10 days of treatment or after increasing dose
 pyrexia
 rigidity
 tachycardia
♦ A raised creatine kinase is present in most cases. A leukocytosis may also be seen.
Management
 stop antipsychotic
 IV fluids to prevent renal failure
 dantrolene* may be useful in selected cases
 bromocriptine, dopamine agonist, may also be used

*thought to work by decreasing excitation-contraction coupling in skeletal muscle by binding to the


ryanodine receptor, and decreasing the release of calcium from the sarcoplasmic reticulum

External links
Patient.info
Neuroleptic Malignant Syndrome
The Mental Health Clinician
Differentiating MN

142

MRCP part 1 Made Easy‫‏‬‎‫‏‬by M. Habayeb & A. Murad


Chapter: Neurology

Question 18 of 246
You review a 25-year-old man who is complaining of leg weakness. Other than a bout of diarrhoea
three weeks ago he has been feeling fit and well and has no significant medical history. On
examination you note reduced power in his legs, normal sensation and reduced knee and ankle
reflexes. His pulse is 78/min and blood pressure is 122/84 mmHg (standing), 100/64 mmHg (sitting).
What is the most likely diagnosis?

Botulism food poisoning


Guillain-Barre syndrome
Cauda equina syndrome
Myasthenia gravis
Transverse myelitis

143

MRCP part 1 Made Easy‫‏‬‎‫‏‬by M. Habayeb & A. Murad


Chapter: Neurology

Question 18 of 246

You review a 25-year-old man who is complaining of leg weakness. Other than a bout of diarrhoea
three weeks ago he has been feeling fit and well and has no significant medical history. On
examination you note reduced power in his legs, normal sensation and reduced knee and ankle
reflexes. His pulse is 78/min and blood pressure is 122/84 mmHg (standing), 100/64 mmHg (sitting).
What is the most likely diagnosis?

Botulism food poisoning


Guillain-Barre syndrome
Cauda equina syndrome
Myasthenia gravis
Transverse myelitis

Discuss and give feedback

Guillain-Barre syndrome: features

External links

Patient.info

Guillain-Barre syndrome review

144

MRCP part 1 Made Easy‫‏‬‎‫‏‬by M. Habayeb & A. Murad


Chapter: Neurology

Question 19 of 246

A 27-year-old man presents with a history of fits consistent with tonic-clonic seizures. What is the
most suitable first-line treatment?

Gabapentin
Lamotrigine
Sodium valproate
Carbamazepine
Phenytoin

145

MRCP part 1 Made Easy‫‏‬‎‫‏‬by M. Habayeb & A. Murad


Chapter: Neurology

Question 19 of 246

A 27-year-old man presents with a history of fits consistent with tonic-clonic seizures. What is the
most suitable first-line treatment?

Gabapentin
Lamotrigine
Sodium valproate
Carbamazepine
Phenytoin

Epilepsy medication: first-line

 generalised seizure: sodium valproate

 partial seizure: carbamazepine

Discuss and give feedback

Epilepsy: treatment

External links

NICE

2012 Epilepsy guidelines

Royal College of Physicians

2013 Modern management of epilepsy

146

MRCP part 1 Made Easy‫‏‬‎‫‏‬by M. Habayeb & A. Murad


Chapter: Neurology

Question 20 of 246

Which of the following features is least likely to be found in a patient with tuberous sclerosis?

Adenoma sebaceum
Cafe-au-lait spots
Retinal hamartomas
'Ash-leaf' spots
Lisch nodules

147

MRCP part 1 Made Easy‫‏‬‎‫‏‬by M. Habayeb & A. Murad


Chapter: Neurology

Question 20 of 246

Which of the following features is least likely to be found in a patient with tuberous sclerosis?

Adenoma sebaceum
Cafe-au-lait spots
Retinal hamartomas
'Ash-leaf' spots
Lisch nodules

Lisch nodules are seen in neurofibromatosis

Discuss and give feedback

Tuberous sclerosis

External links

Patient.info

Tuberous sclerosis review

148

MRCP part 1 Made Easy‫‏‬‎‫‏‬by M. Habayeb & A. Murad


Chapter: Neurology

Question 21 of 246

Which of the following visual field changes would be most consistent with a left parietal lobe
lesion?

Right homonymous hemianopia


Left inferior homonymous quadrantanopia
Left superior homonymous quadrantanopia
Right superior homonymous quadrantanopia
Right inferior homonymous quadrantanopia

149

MRCP part 1 Made Easy‫‏‬‎‫‏‬by M. Habayeb & A. Murad


Chapter: Neurology

Question 21 of 246

Which of the following visual field changes would be most consistent with a left parietal lobe
lesion?

Right homonymous hemianopia


Left inferior homonymous quadrantanopia
Left superior homonymous quadrantanopia
Right superior homonymous quadrantanopia
Right inferior homonymous quadrantanopia

Visual field defects:

 left homonymous hemianopia means visual field defect to the left, i.e. lesion of right optic tract

 homonymous quadrantanopias: PITS (Parietal-Inferior, Temporal-Superior)

 incongruous defects = optic tract lesion; congruous defects= optic radiation lesion or occipital
cortex

Discuss and give feedback

Visual field defects

150

MRCP part 1 Made Easy‫‏‬‎‫‏‬by M. Habayeb & A. Murad


Chapter: Neurology

Question 22 of 246

A 52-year-old man is prescribed apomorphine. What type of receptors does apomorphine act
on?

Opioid receptors
GABA receptors
Cholinergic receptors
Dopamine receptors
Muscarinic receptors

151

MRCP part 1 Made Easy‫‏‬‎‫‏‬by M. Habayeb & A. Murad


Chapter: Neurology

Question 22 of 246

A 52-year-old man is prescribed apomorphine. What type of receptors does apomorphine act on?

Opioid receptors
GABA receptors
Cholinergic receptors
Dopamine receptors
Muscarinic receptors

Apomorphine is one of the older dopamine receptor agonists. Newer agents such as ropinirole and
cabergoline have since been developed

Discuss and give feedback

Parkinson's disease: management

External links

SIGN

2010 Parkinson's disease guidelines

NICE

2006 Parkinson's disease guidelines

Royal College of Physicians

How to treat Parkinson's disease in 2013

External media

Parkinson's disease

Osmosis – YouTube

152

MRCP part 1 Made Easy‫‏‬‎‫‏‬by M. Habayeb & A. Murad


Chapter: Neurology

Question 23 of 246

A 67-year-old man is reviewed in the falls clinic. Over the past few months he has sustained a number
of falls. His daughter reports that he is starting to 'shuffle around the house' and has particular
problems going up and down stairs. She also notes that he appears to be confused at times and often
forgets his grandchildren's names.

On examination he appears to move and follow commands slowly. There is a resting tremor in the left
hand more so than the right. Some rigidity is also noted when examining his arms. Examination of the
cranial nerves is unremarkable other than a problem following movement in the vertical plane.

What is the most likely diagnosis?

Parkinson's disease
Motor neuron disease
Multiple system atrophy
Dementia with Lewy bodies
Progressive supranuclear palsy

153

MRCP part 1 Made Easy‫‏‬‎‫‏‬by M. Habayeb & A. Murad


Chapter: Neurology

Question 23 of 246

A 67-year-old man is reviewed in the falls clinic. Over the past few months he has sustained a number
of falls. His daughter reports that he is starting to 'shuffle around the house' and has particular
problems going up and down stairs. She also notes that he appears to be confused at times and often
forgets his grandchildren's names.

On examination he appears to move and follow commands slowly. There is a resting tremor in the left
hand more so than the right. Some rigidity is also noted when examining his arms. Examination of the
cranial nerves is unremarkable other than a problem following movement in the vertical plane.

What is the most likely diagnosis?

Parkinson's disease
Motor neuron disease
Multiple system atrophy
Dementia with Lewy bodies
Progressive supranuclear palsy

Progressive supranuclear palsy: parkinsonism, impairment of vertical gaze

Discuss and give feedback

Progressive supranuclear palsy

154

MRCP part 1 Made Easy‫‏‬‎‫‏‬by M. Habayeb & A. Murad


Chapter: Neurology

Question 24 of 246

A 72-year-old man becomes confused at the end of a charity walk for Alzheimer's. He is brought to the
hospital by paramedics because he repeatedly asked other attendees on the walk why he was there and
how he had got there. There is no past medical history of note and he takes no regular medication. He
is orientated in time and person and knows he has been brought to the hospital. His blood pressure is
123/82 mmHg, his pulse is 70 beats per minute and regular. The neurological exam is unremarkable,
routine blood tests and CT head are normal. He gradually recovers over the course of 3hrs.

What is the most appropriate intervention?

Aspirin
Clopidogrel
Reassurance
Rivaroxaban
Warfarin

155

MRCP part 1 Made Easy‫‏‬‎‫‏‬by M. Habayeb & A. Murad


Chapter: Neurology

Question 24 of 246

A 72-year-old man becomes confused at the end of a charity walk for Alzheimer's. He is brought to the
hospital by paramedics because he repeatedly asked other attendees on the walk why he was there and
how he had got there. There is no past medical history of note and he takes no regular medication. He
is orientated in time and person and knows he has been brought to the hospital. His blood pressure is
123/82 mmHg, his pulse is 70 beats per minute and regular. The neurological exam is unremarkable,
routine blood tests and CT head are normal. He gradually recovers over the course of 3hrs.

What is the most appropriate intervention?

Aspirin
Clopidogrel
Reassurance
Rivaroxaban
Warfarin

The most likely diagnosis here, with acute onset of retrograde amnesia with preserved orientation and
consciousness, is transient global amnesia, (TGA). The exact cause of TGA is unclear, although it may
be similar in aetiology to migraines. There is no evidence in prospective cohort studies to suggest an
association between TGA and increased risk of stroke.

Aspirin and clopidogrel, both antiplatelet agents, are not indicated here because TGA does not mark
out the patient as being at increased risk of ischaemic stroke. Rivaroxaban and warfarin are both
indicated for anticoagulation in patients with atrial fibrillation, which hasn't been identified here.

Discuss and give feedback

Transient global amnesia

156

MRCP part 1 Made Easy‫‏‬‎‫‏‬by M. Habayeb & A. Murad


Chapter: Neurology

Question 25 of 246

An obese 24-year-old female presents with headaches and blurred vision. Examination reveals
bilateral blurring of the optic discs but is otherwise unremarkable with no other neurological signs.
Blood pressure is 130/74 and she is apyrexial. What is the most likely underlying diagnosis?

Multiple sclerosis
Meningococcal meningitis
Brain abscess
Normal pressure hydrocephalus
Idiopathic intracranial hypertension

157

MRCP part 1 Made Easy‫‏‬‎‫‏‬by M. Habayeb & A. Murad


Chapter: Neurology

Question 25 of 246

An obese 24-year-old female presents with headaches and blurred vision. Examination reveals
bilateral blurring of the optic discs but is otherwise unremarkable with no other neurological signs.
Blood pressure is 130/74 and she is apyrexial. What is the most likely underlying diagnosis?

Multiple sclerosis
Meningococcal meningitis
Brain abscess
Normal pressure hydrocephalus
Idiopathic intracranial hypertension

Obese, young female with headaches / blurred vision think idiopathic intracranial hypertension

The combination of a young, obese female with papilloedema but otherwise normal neurology makes
idiopathic intracranial hypertension the most likely diagnosis

Discuss and give feedback

Idiopathic intracranial hypertension

External links

Patient.info

Idiopathic intracranial hypertension review

158

MRCP part 1 Made Easy‫‏‬‎‫‏‬by M. Habayeb & A. Murad


Chapter: Neurology

Question 26 of 246

A 78-year-old female has been diagnosed with mild to moderate dementia. Which of the following is
an effect of cholinesterase inhibitors?

Improvement in physical function


Improvement in activities of daily living
Longer time before entering residential care
Improved mortality
Minimize the progression of dementia

159

MRCP part 1 Made Easy‫‏‬‎‫‏‬by M. Habayeb & A. Murad


Chapter: Neurology

Question 26 of 246

A 78-year-old female has been diagnosed with mild to moderate dementia. Which of the following is
an effect of cholinesterase inhibitors?

Improvement in physical function


Improvement in activities of daily living
Longer time before entering residential care
Improved mortality
Minimize the progression of dementia

♦Patients with Alzheimer disease have reduced production of choline acetyl transferase, leading to a
decrease in acetylcholine synthesis and impaired cortical cholinergic functioning.

♦The only role for cholinesterase inhibitors is to improve some cognitive function and
improvement in activities of daily living. There is no role for cholinesterase inhibitors in
advanced Alzheimer's disease.

Discuss and give feedback

Alzheimer's disease

External links

NICE

2011 Dementia guidelines

External media

Alzheimer's disease

Osmosis – YouTube

160

MRCP part 1 Made Easy‫‏‬‎‫‏‬by M. Habayeb & A. Murad


Chapter: Neurology

Question 27 of 246

A 60-year-old woman presents with a tremor. Which one of the following features would suggest a
diagnosis of essential tremor rather than Parkinson's disease?

Difficulty in initiating movement


Tremor is worse following alcohol
Postural instability
Unilateral symptoms
Tremor is worse when the arms are outstretched

161

MRCP part 1 Made Easy‫‏‬‎‫‏‬by M. Habayeb & A. Murad


Chapter: Neurology

Question 27 of 246

A 60-year-old woman presents with a tremor. Which one of the following features would suggest a
diagnosis of essential tremor rather than Parkinson's disease?

Difficulty in initiating movement


Tremor is worse following alcohol
Postural instability
Unilateral symptoms
Tremor is worse when the arms are outstretched

Difficulty in initiating movement (bradykinesia), postural instability and unilateral symptoms


(initially) are typical of Parkinson's. Essential tremor symptoms are usually eased by alcohol.

Discuss and give feedback

Essential tremor

162

MRCP part 1 Made Easy‫‏‬‎‫‏‬by M. Habayeb & A. Murad


Chapter: Neurology

Question 28 of 246

A 24-year-old female presents with a headache. She has a past history of epilepsy and is known to
suffer from migraines, but has previously managed attacks with a combination of paracetamol and
metoclopramide. This combination is however not working for the current episode.

What second line medication is it most appropriate to use?

Codeine + paracetamol
Pizotifen
Zolmitriptan + paracetamol
Methysergide
Ergotamine

163

MRCP part 1 Made Easy‫‏‬‎‫‏‬by M. Habayeb & A. Murad


Chapter: Neurology

Question 28 of 246

A 24-year-old female presents with a headache. She has a past history of epilepsy and is known to
suffer from migraines, but has previously managed attacks with a combination of paracetamol and
metoclopramide. This combination is however not working for the current episode.

What second line medication is it most appropriate to use?

Codeine + paracetamol
Pizotifen
Zolmitriptan + paracetamol
Methysergide
Ergotamine

Epilepsy is not a contraindication to the use of triptans. Opioids are not recommended in the
management of migraine

Discuss and give feedback

Migraine: management

External links

SIGN

2008 Diagnosis and management of headache in adults

Clinical Knowledge Summaries

Migraine guidelines

NICE

2012 Headache guidelines

British Association for the Study of Headache

Headache guidelines

164

MRCP part 1 Made Easy‫‏‬‎‫‏‬by M. Habayeb & A. Murad


Chapter: Neurology

Question 29 of 246

A 45-year-old man presents with dizziness and right-sided hearing loss. Which one of the following
tests would most likely indicate an acoustic neuroma?

Jerky nystagmus
Left homonymous hemianopia
Tongue deviated to the left
Fasciculation of the tongue
Absent corneal reflex

165

MRCP part 1 Made Easy‫‏‬‎‫‏‬by M. Habayeb & A. Murad


Chapter: Neurology

Question 29 of 246

A 45-year-old man presents with dizziness and right-sided hearing loss. Which one of the following
tests would most likely indicate an acoustic neuroma?

Jerky nystagmus
Left homonymous hemianopia
Tongue deviated to the left
Fasciculation of the tongue
Absent corneal reflex

Loss of corneal reflex - think acoustic neuroma

Discuss and give feedback

Acoustic neuroma

166

MRCP part 1 Made Easy‫‏‬‎‫‏‬by M. Habayeb & A. Murad


Chapter: Neurology

Question 30 of 246

A 57-year-old woman presents with an 8 week history of intermittent dizziness. These episodes
typically occur when she suddenly moves her head and are characterised by the sensation that the
room is 'spinning'. Most attacks last around one minute before dissipating. Neurological examination
is unremarkable. What is the most likely diagnosis?

Benign paroxysmal positional vertigo


Meniere disease
Crescendo transient ischaemic attacks
Multiple sclerosis
Viral labyrinthitis

167

MRCP part 1 Made Easy‫‏‬‎‫‏‬by M. Habayeb & A. Murad


Chapter: Neurology

Question 30 of 246

A 57-year-old woman presents with an 8 week history of intermittent dizziness. These episodes
typically occur when she suddenly moves her head and are characterised by the sensation that the
room is 'spinning'. Most attacks last around one minute before dissipating. Neurological examination
is unremarkable. What is the most likely diagnosis?

Benign paroxysmal positional vertigo


Meniere disease
Crescendo transient ischaemic attacks
Multiple sclerosis
Viral labyrinthitis

♦Viral labyrinthitis typically causes constant symptoms of a shorter duration. Patients with Meniere
disease usually have associated hearing loss and tinnitus. Also, the vertigo associated with Meniere
disease typically lasts much longer.

Discuss and give feedback

Benign paroxysmal positional vertigo

External links

YouTube

Hallpike Test and Epley Maneuver

Clinical Knowledge Summaries

Benign paroxysmal positional vertigo guidelines

168

MRCP part 1 Made Easy‫‏‬‎‫‏‬by M. Habayeb & A. Murad


Chapter: Neurology

Question 31 of 246

A 58-year-old lady presents to clinic with a 6-month history of limb weakness and falls. She has
hypercholesterolaemia and takes simvastatin.

On examination, there is wasting of the intrinsic hand muscles and left biceps with fasciculations in
the left biceps and shoulder girdle. There is a generalised weakness with brisk left biceps and bilateral
knee reflexes and a right extensor plantar response. The sensation is normal throughout. Examination
of the cranial nerves was normal.

What is the most likely diagnosis?

Compressive cervical myelopathy


Motor neurone disease
Multifocal motor neuropathy with conduction block
Myopathy
Myasthenia gravis

169

MRCP part 1 Made Easy‫‏‬‎‫‏‬by M. Habayeb & A. Murad


Chapter: Neurology

Question 31 of 246

A 58-year-old lady presents to clinic with a 6-month history of limb weakness and falls. She has
hypercholesterolaemia and takes simvastatin.

On examination, there is wasting of the intrinsic hand muscles and left biceps with fasciculations in
the left biceps and shoulder girdle. There is a generalised weakness with brisk left biceps and bilateral
knee reflexes and a right extensor plantar response. The sensation is normal throughout. Examination
of the cranial nerves was normal.
What is the most likely diagnosis?

Compressive cervical myelopathy


Motor neurone disease
Multifocal motor neuropathy with conduction block
Myopathy
Myasthenia gravis

♦The clinical description is a subacute progressive weakness with a mixture of lower and upper motor
neurone signs without sensory involvement.

♦The diagnosis is motor neurone disease. You would suspect motor neurone disease in any patient
with a mixture of lower and upper motor neurone signs without sensory involvement. Some
pathologies can cause lower and upper motor neurone signs in different parts of the body (e.g.
syringomyelia). The fact that these paradoxical signs co-exist in the same myotome clinches the
diagnosis.

♦A cervical myelopathy would not cause lower motor neurone signs.


Multifocal motor neuropathy with conduction block, myopathy and myasthenia gravis would not cause
upper motor neurone signs
Discuss and give feedback

Motor neuron disease: features

External links

Royal College of Physicians

Motor neuron disease: diagnostic pitfalls

Postgraduate Medical Journal

Review of MND

170

MRCP part 1 Made Easy‫‏‬‎‫‏‬by M. Habayeb & A. Murad


Chapter: Neurology

Question 32 of 246

Which one of the following is least associated with normal pressure hydrocephalus?

Papilloedema
Dementia
Urinary incontinence
Gait abnormality
Enlarged fourth ventricle

171

MRCP part 1 Made Easy‫‏‬‎‫‏‬by M. Habayeb & A. Murad


Chapter: Neurology

Question 32 of 246

Which one of the following is least associated with normal pressure hydrocephalus?

Papilloedema
Dementia
Urinary incontinence
Gait abnormality
Enlarged fourth ventricle

Urinary incontinence + gait abnormality + dementia = normal pressure hydrocephalus

Discuss and give feedback

Normal pressure hydrocephalus

External links

YouTube

Typical NPH Gait

172

MRCP part 1 Made Easy‫‏‬‎‫‏‬by M. Habayeb & A. Murad


Chapter: Neurology

Question 33 of 246

A 33-year-old man presents complaining of visual disturbance. Examination reveals a bitemporal


hemianopia with predominately the upper quadrants being affected. What is the most likely lesion?

Craniopharyngioma
Brainstem lesion
Pituitary macroadenoma
Frontal lobe lesion
Right occipital lesion

173

MRCP part 1 Made Easy‫‏‬‎‫‏‬by M. Habayeb & A. Murad


Chapter: Neurology

Question 33 of 246

A 33-year-old man presents complaining of visual disturbance. Examination reveals a bitemporal


hemianopia with predominately the upper quadrants being affected. What is the most likely lesion?

Craniopharyngioma
Brainstem lesion
Pituitary macroadenoma
Frontal lobe lesion
Right occipital lesion

Bitemporal hemianopia

 lesion of optic chiasm

 upper quadrant defect > lower quadrant defect = inferior chiasmal compression, commonly a
pituitary tumour

 lower quadrant defect > upper quadrant defect = superior chiasmal compression, commonly a
craniopharyngioma

♦An upper quadrant defect implies inferior chiasmal compression making a pituitary macroadenoma
the most likely diagnosis.

Discuss and give feedback

Visual field defects

External links

Patient.info

Visual field defects

PubMed

The localizing value of a quadrantanopia

174

MRCP part 1 Made Easy‫‏‬‎‫‏‬by M. Habayeb & A. Murad


Chapter: Neurology

Question 34 of 246

A 40-year-old man with no past medical history presents to the emergency department with a 1-week
history of progressive walking difficulties. On examination, there is a loss of pin-prick sensation in the
lower limbs from mid-thigh distally and in the upper limbs from MCP joints distally.

There is a bilateral weakness of ankle dorsiflexion to 3/5 and knee flexion and extension is weak to 4/5
bilaterally. Power in upper and lower limbs is otherwise normal. Knee and ankle deep tendon reflexes
are absent. What are the most likely findings on CSF analysis?

Normal values:

 Protein - 0.2-0.4 g/L

 WCC - 0-5 per mm cubed

 RCC - 0-10 per mm cubed

Protein 1.2 g/L WCC 0 per mm cubed RCC 0 per mm cubed


Protein 8.2 g/L WCC 0 per mm cubed RCC 0 per mm cubed
Protein 1.2 g/L WCC 20 per mm cubed RCC 5 per mm cubed
Protein 0.3 g/L WCC 3 per mm cubed RCC 2 per mm cubed
Protein 0.3 g/L WCC 20 per mm cubed RCC 5 per mm cubed

175

MRCP part 1 Made Easy‫‏‬‎‫‏‬by M. Habayeb & A. Murad


Chapter: Neurology

Question 34 of 246

A 40-year-old man with no past medical history presents to the emergency department with a 1-week
history of progressive walking difficulties. On examination, there is a loss of pin-prick sensation in the
lower limbs from mid-thigh distally and in the upper limbs from MCP joints distally.

There is a bilateral weakness of ankle dorsiflexion to 3/5 and knee flexion and extension is weak to 4/5
bilaterally. Power in upper and lower limbs is otherwise normal. Knee and ankle deep tendon reflexes
are absent. What are the most likely findings on CSF analysis?

Normal values:

 Protein - 0.2-0.4 g/L

 WCC - 0-5 per mm cubed

 RCC - 0-10 per mm cubed

Protein 1.2 g/L WCC 0 per mm cubed RCC 0 per mm cubed


Protein 8.2 g/L WCC 0 per mm cubed RCC 0 per mm cubed
Protein 1.2 g/L WCC 20 per mm cubed RCC 5 per mm cubed
Protein 0.3 g/L WCC 3 per mm cubed RCC 2 per mm cubed
Protein 0.3 g/L WCC 20 per mm cubed RCC 5 per mm cubed

♦The clinical description is in keeping with Guillain-Barre syndrome

♦The most likely findings on CSF would be an isolated raised protein without an abnormal cell count.
Both 1 and 2 conform to this description, however, a marked rise in CSF protein would be more
suggestive of a CSF blockage as seen in a spinal cord compression

Discuss and give feedback

Guillain-Barre syndrome: features

External links

Patient.info

Guillain-Barre syndrome review

176

MRCP part 1 Made Easy‫‏‬‎‫‏‬by M. Habayeb & A. Murad


Chapter: Neurology

Question 35 of 246

A 28-year-old man develops nausea and a severe headache whilst trekking in Nepal. Within the next
hour he becomes ataxic and confused. A diagnosis of high altitude cerebral oedema is suspected. Other
than descent and oxygen, what is the most important treatment?

Acetazolamide
Dexamethasone
Burr hole
Mannitol
Frusemide

177

MRCP part 1 Made Easy‫‏‬‎‫‏‬by M. Habayeb & A. Murad


Chapter: Neurology

Question 35 of 246
A 28-year-old man develops nausea and a severe headache whilst trekking in Nepal. Within the next
hour he becomes ataxic and confused. A diagnosis of high altitude cerebral oedema is suspected. Other
than descent and oxygen, what is the most important treatment?

Acetazolamide
Dexamethasone
Burr hole
Mannitol
Frusemide

Acetazolamide is used more in the prevention of high altitude cerebral oedema.


Altitude related disorders
There are three main types of altitude related disorders: acute mountain sickness (AMS), which may
progress to high altitude pulmonary edema (HAPE) or high altitude cerebral edema (HACE). All three
conditions are due to the chronic hypobaric hypoxia which develops at high altitudes

Acute mountain sickness is generally a self-limiting condition. Features of AMS start to occur above
2,500 - 3,000m, developing gradually over 6-12 hours and potentially last a number of days:
 headache
 nausea
 fatigue
Prevention and treatment of AMS
 the risk of AMS may actually be positively correlated to physical fitness
 gain altitude at no more than 500 m per day
 acetazolamide (a carbonic anhydrase inhibitor) is widely used to prevent AMS and has a
supporting evidence base
 treatment: descent

A minority of people above 4,000m go onto develop high altitude pulmonary oedema (HAPE) or high
altitude cerebral oedema (HACE), potentially fatal conditions.
 HAPE presents with classical pulmonary oedema features
 HACE presents with headache, ataxia, papilloedema

Management of HACE
 descent
 dexamethasone
Management of HAPE
 descent
 nifedipine, dexamethasone, acetazolamide, phosphodiesterase type V inhibitors*
 oxygen if available

*the relative merits of these different treatments has only been studied in small trials. All seem to work
by reducing systolic pulmonary artery pressure
178

MRCP part 1 Made Easy‫‏‬‎‫‏‬by M. Habayeb & A. Murad


Chapter: Neurology

Question 36 of 246

A patient is referred due to the development of a third nerve palsy associated with a headache. On
examination meningism is present. Which one of the following diagnoses needs to be urgently
excluded?

Weber's syndrome
Internal carotid artery aneurysm
Multiple sclerosis
Posterior communicating artery aneurysm
Anterior communicating artery aneurysm

179

MRCP part 1 Made Easy‫‏‬‎‫‏‬by M. Habayeb & A. Murad


Chapter: Neurology

Question 36 of 246

A patient is referred due to the development of a third nerve palsy associated with a headache. On
examination meningism is present. Which one of the following diagnoses needs to be urgently
excluded?

Weber's syndrome
Internal carotid artery aneurysm
Multiple sclerosis
Posterior communicating artery aneurysm
Anterior communicating artery aneurysm

Painful third nerve palsy = posterior communicating artery aneurysm

Given the combination of a headache and third nerve palsy it is important to exclude a posterior
communicating artery aneurysm

Discuss and give feedback

Third nerve palsy

180

MRCP part 1 Made Easy‫‏‬‎‫‏‬by M. Habayeb & A. Murad


Chapter: Neurology

Question 37 of 246

Which one of the following features is most associated with frontal lobe lesions?

Wernicke's aphasia
Gerstmann's syndrome
Perseveration
Cortical blindness
Superior homonymous quadrantanopia

181

MRCP part 1 Made Easy‫‏‬‎‫‏‬by M. Habayeb & A. Murad


Chapter: Neurology

Question 37 of 246

Which one of the following features is most associated with frontal lobe lesions?

Wernicke's aphasia
Gerstmann's syndrome
Perseveration
Cortical blindness
Superior homonymous quadrantanopia

Discuss and give feedback

Brain lesions

External links

American Speech-Language-Hearing Association

Common Classifications of Aphasia

182

MRCP part 1 Made Easy‫‏‬‎‫‏‬by M. Habayeb & A. Murad


Chapter: Neurology

Question 38 of 246

A 21-year-old female is seen in the first seizure clinic in the outpatient department. Both the EEG and
MRI brain are normal. A decision is made not to start her on anti-epileptic medication. What
restrictions on driving should she be informed about?

No restrictions but inform DVLA


No restrictions, no need to inform DVLA if not on medication
Cannot drive for 1 month from date of seizure
Cannot drive for 6 months from date of seizure
Cannot drive for 1 year from date of seizure

183

MRCP part 1 Made Easy‫‏‬‎‫‏‬by M. Habayeb & A. Murad


Chapter: Neurology

Question 38 of 246

A 21-year-old female is seen in the first seizure clinic in the outpatient department. Both the EEG and
MRI brain are normal. A decision is made not to start her on anti-epileptic medication. What
restrictions on driving should she be informed about?

No restrictions but inform DVLA


No restrictions, no need to inform DVLA if not on medication
Cannot drive for 1 month from date of seizure
Cannot drive for 6 months from date of seizure
Cannot drive for 1 year from date of seizure

Patients cannot drive for 6 months following a seizure

Discuss and give feedback

DVLA: neurological disorders

External links

DVLA

Fitness to drive guidelines

184

MRCP part 1 Made Easy‫‏‬‎‫‏‬by M. Habayeb & A. Murad


Chapter: Neurology

Question 39 of 246

A 75-year-old female presents with weakness of her left hand. On examination wasting of the
hypothenar eminence is seen and there is weakness of finger abduction. Thumb adduction is also
weak. Where is the lesion most likely to be?

C7
Median nerve
Radial nerve
Anterior interosseous nerve
Ulnar nerve

185

MRCP part 1 Made Easy‫‏‬‎‫‏‬by M. Habayeb & A. Murad


Chapter: Neurology

Question 39 of 246

A 75-year-old female presents with weakness of her left hand. On examination wasting of the
hypothenar eminence is seen and there is weakness of finger abduction. Thumb adduction is also
weak. Where is the lesion most likely to be?

C7
Median nerve
Radial nerve
Anterior interosseous nerve
Ulnar nerve

Ulnar nerve
Overview
 arises from medial cord of brachial plexus (C8, T1)

Motor to:
 medial two lumbricals
 aDductor pollicis
 interossei
 hypothenar muscles: abductor digiti minimi, flexor digiti minimi
 flexor carpi ulnaris

Sensory to:
 medial 1 1/2 fingers (palmar and dorsal aspects)

Path<
 posteromedial aspect of upper arm to flexor compartment of forearm, then along the ulnar.
Passes beneath the flexor carpi ulnaris muscle, then superficially through the flexor
retinaculum into the palm of the hand.

186

MRCP part 1 Made Easy‫‏‬‎‫‏‬by M. Habayeb & A. Murad


Chapter: Neurology

Branches
Branch Supplies
Muscular branch Flexor carpi ulnaris
Medial half of the flexor digitorum profundus
Palmar cutaneous branch (Arises near the Skin on the medial part of the palm
middle of the forearm)
Dorsal cutaneous branch Dorsal surface of the medial part of the hand
Superficial branch Cutaneous fibres to the anterior surfaces of the
medial one and one-half digits
Deep branch Hypothenar muscles
All the interosseous muscles
Third and fourth lumbricals
Adductor pollicis
Medial head of the flexor pollicis brevis

187

MRCP part 1 Made Easy‫‏‬‎‫‏‬by M. Habayeb & A. Murad


Chapter: Neurology

Patterns of damage

Damage at wrist

 'claw hand' - hyperextension of the metacarpophalangeal joints and flexion at the distal and
proximal interphalangeal joints of the 4th and 5th digits
 wasting and paralysis of intrinsic hand muscles (except lateral two lumbricals)
 wasting and paralysis of hypothenar muscles
 sensory loss to the medial 1 1/2 fingers (palmar and dorsal aspects)

Damage at elbow

 as above (however, ulnar paradox - clawing is more severe in distal lesions)


 radial deviation of wrist

188

MRCP part 1 Made Easy‫‏‬‎‫‏‬by M. Habayeb & A. Murad


Chapter: Neurology

Question 40 of 246

Which type of motor neuron disease carries the worst prognosis?

Relapsing-remitting
Progressive bulbar palsy
Progressive muscular atrophy
Spinocerebellar ataxia
Amyotrophic lateral sclerosis

189

MRCP part 1 Made Easy‫‏‬‎‫‏‬by M. Habayeb & A. Murad


Chapter: Neurology

Question 40 of 246

Which type of motor neuron disease carries the worst prognosis?

Relapsing-remitting
Progressive bulbar palsy
Progressive muscular atrophy
Spinocerebellar ataxia
Amyotrophic lateral sclerosis

Discuss and give feedback

Motor neuron disease: types

External links

Postgraduate Medical Journal

Review of MND

190

MRCP part 1 Made Easy‫‏‬‎‫‏‬by M. Habayeb & A. Murad


Chapter: Neurology

Question 41 of 246

A 65-year-old man is referred to the neurology outpatient clinic due to a resting tremor of his right
hand. A diagnosis of Parkinson's disease is made. He is otherwise well and is not currently disabled by
his symptoms. What is the most appropriate treatment?

Selegiline
No treatment
New generation dopamine receptor agonist e.g. ropinirole
Conventional dopamine receptor agonist e.g. bromocriptine
Antimuscarinics

191

MRCP part 1 Made Easy‫‏‬‎‫‏‬by M. Habayeb & A. Murad


Chapter: Neurology

Question 41 of 246

A 65-year-old man is referred to the neurology outpatient clinic due to a resting tremor of his right
hand. A diagnosis of Parkinson's disease is made. He is otherwise well and is not currently disabled by
his symptoms. What is the most appropriate treatment?

Selegiline
No treatment
New generation dopamine receptor agonist e.g. ropinirole
Conventional dopamine receptor agonist e.g. bromocriptine
Antimuscarinics

Discuss and give feedback

Parkinson's disease: management

192

MRCP part 1 Made Easy‫‏‬‎‫‏‬by M. Habayeb & A. Murad


Chapter: Neurology

Question 42 of 246

Which one of the following features is most associated with temporal lobe lesions?

Astereognosis
Auditory agnosia
Visual agnosia
Disinhibition
Expressive (Broca's) aphasia

193

MRCP part 1 Made Easy‫‏‬‎‫‏‬by M. Habayeb & A. Murad


Chapter: Neurology

Question 42 of 246

Which one of the following features is most associated with temporal lobe lesions?

Astereognosis
Auditory agnosia
Visual agnosia
Disinhibition
Expressive (Broca's) aphasia

Discuss and give feedback

Brain lesions

External links

American Speech-Language-Hearing Association

Common Classifications of Aphasia

194

MRCP part 1 Made Easy‫‏‬‎‫‏‬by M. Habayeb & A. Murad


Chapter: Neurology

Question 43 of 246

Each of the following features are seen in myotonic dystrophy, except:

Mild mental impairment


Round face
Frontal balding
Myotonia
Cataracts

195

MRCP part 1 Made Easy‫‏‬‎‫‏‬by M. Habayeb & A. Murad


Chapter: Neurology

Question 43 of 246

Each of the following features are seen in myotonic dystrophy, except:

Mild mental impairment


Round face
Frontal balding
Myotonia
Cataracts

Dystrophia myotonica - DM1

 distal weakness initially

 autosomal dominant

 diabetes

 dysarthria

Discuss and give feedback

Myotonic dystrophy

196

MRCP part 1 Made Easy‫‏‬‎‫‏‬by M. Habayeb & A. Murad


Chapter: Neurology

Question 44 of 246

A 25-year-old female with a history of bilateral vitreous haemorrhage is referred due to progressive
ataxia. What is the likely diagnosis?

Neurofibromatosis type I
Neurofibromatosis type II
Tuberose sclerosis
Von Hippel-Lindau syndrome
Sarcoidosis

197

MRCP part 1 Made Easy‫‏‬‎‫‏‬by M. Habayeb & A. Murad


Chapter: Neurology

Question 44 of 246

A 25-year-old female with a history of bilateral vitreous haemorrhage is referred due to progressive
ataxia. What is the likely diagnosis?

Neurofibromatosis type I
Neurofibromatosis type II
Tuberose sclerosis
Von Hippel-Lindau syndrome
Sarcoidosis

Retinal and cerebellar haemangiomas are key features of Von Hippel-Lindau syndrome. Retinal
haemangiomas are bilateral in 25% of patients and may lead to vitreous haemorrhage

Discuss and give feedback

Von Hippel-Lindau syndrome

198

MRCP part 1 Made Easy‫‏‬‎‫‏‬by M. Habayeb & A. Murad


Chapter: Neurology

Question 45 of 246

A 78-year-old man is seen in the Memory clinic. His daughter reports that for the past 12 months he
has become increasingly forgetful and has now started to wander around at night. A mini-mental test is
performed and he scores 18 out of 30. Neurological examination is unremarkable. A full blood screen
is also requested, all of which comes back as normal. What is the most appropriate next step?

Arrange a MRI head


Perform carotid Dopplers
Give practical advice + advise family to contact Alzheimer's Society
Prescribe aspirin + simvastatin
Prescribe donepezil

199

MRCP part 1 Made Easy‫‏‬‎‫‏‬by M. Habayeb & A. Murad


Chapter: Neurology

Question 45 of 246

A 78-year-old man is seen in the Memory clinic. His daughter reports that for the past 12 months he
has become increasingly forgetful and has now started to wander around at night. A mini-mental test is
performed and he scores 18 out of 30. Neurological examination is unremarkable. A full blood screen
is also requested, all of which comes back as normal. What is the most appropriate next step?

Arrange a MRI head


Perform carotid Dopplers
Give practical advice + advise family to contact Alzheimer's Society
Prescribe aspirin + simvastatin
Prescribe donepezil

Neuroimaging is required to diagnose dementia

Discuss and give feedback

Dementia

External links

NICE

2011 Dementia guidelines

Alzheimers Society

Helping you to assess cognition - A practical toolkit for clinicians

200

MRCP part 1 Made Easy‫‏‬‎‫‏‬by M. Habayeb & A. Murad


Chapter: Neurology

Question 46 of 246

A 55-year-old woman complains of neck and right arm pain for the past two months. The pain is often
triggered by flexing her neck. Her past medical history includes osteoarthritis of her knee, obesity and
depression. On examination there is no obvious muscle atrophy or weakness of the right arm. There is
however some sensory loss over the middle finger and palm of the hand. Which nerve root is most
likely to be affected by the impingement?

C4
C5
C6
C7
C8

201

MRCP part 1 Made Easy‫‏‬‎‫‏‬by M. Habayeb & A. Murad


Chapter: Neurology

Question 46 of 246

A 55-year-old woman complains of neck and right arm pain for the past two months. The pain is often
triggered by flexing her neck. Her past medical history includes osteoarthritis of her knee, obesity and
depression. On examination there is no obvious muscle atrophy or weakness of the right arm. There is
however some sensory loss over the middle finger and palm of the hand. Which nerve root is most
likely to be affected by the impingement?

C4
C5
C6
C7
C8

Discuss and give feedback

Dermatomes

202

MRCP part 1 Made Easy‫‏‬‎‫‏‬by M. Habayeb & A. Murad


Chapter: Neurology

Question 47 of 246

A 78-year-old right-handed female is admitted with an acute onset stroke of 2 hours duration. The
decision was taken by the stroke team for thrombolysis which cures her symptoms. She is
subsequently transferred to the high dependency unit for closer monitoring. Overnight, she has three
bouts of vomiting and is seen by an FY2 who detects a new onset right-left disorientation and
acalculia.

An urgent CT scan is requested which reveals an intracerebral haemorrhage. Which area of the brain
is most likely to have been affected?

Left temporal lobe


Left parietal lobe
Right frontal lobe
Right temporal lobe
Right parietal lobe

203

MRCP part 1 Made Easy‫‏‬‎‫‏‬by M. Habayeb & A. Murad


Chapter: Neurology

Question 47 of 246

A 78-year-old right-handed female is admitted with an acute onset stroke of 2 hours duration. The
decision was taken by the stroke team for thrombolysis which cures her symptoms. She is
subsequently transferred to the high dependency unit for closer monitoring. Overnight, she has three
bouts of vomiting and is seen by an FY2 who detects a new onset right-left disorientation and
acalculia.

An urgent CT scan is requested which reveals an intracerebral haemorrhage. Which area of the brain
is most likely to have been affected?

Left temporal lobe


Left parietal lobe
Right frontal lobe
Right temporal lobe
Right parietal lobe

This is a case of Gerstmann syndrome. Gerstmann syndrome is a constellation of acalculia, right-left


disorientation, finger agnosia and agraphia. This occurs as a result of a deficit in the angular and
supramarginal gyri between the dominant parietal and temporal lobes. This can occur with space
occupying lesions or in adults following a stroke.

Discuss and give feedback

Brain lesions

External links

American Speech-Language-Hearing Association

Common Classifications of Aphasia

204

MRCP part 1 Made Easy‫‏‬‎‫‏‬by M. Habayeb & A. Murad


Chapter: Neurology

Question 48 of 246

A 59-year-old female continues to have chronic neuropathic pain for 12 months following
improvement of a dermatomal vesicular rash. She experiences minimal relief with paracetamol and
NSAIDs. What is the next best treatment option?

Sertraline
Gabapentin
Aciclovir
Oxycontin
Morphine

205

MRCP part 1 Made Easy‫‏‬‎‫‏‬by M. Habayeb & A. Murad


Chapter: Neurology

Question 48 of 246

A 59-year-old female continues to have chronic neuropathic pain for 12 months following
improvement of a dermatomal vesicular rash. She experiences minimal relief with paracetamol and
NSAIDs. What is the next best treatment option?

Sertraline
Gabapentin
Aciclovir
Oxycontin
Morphine

Postherpetic neuralgia results from reactivation of the varicella-zoster virus acquired during the
primary varicella infection, or chickenpox. Although postherpetic neuralgia is generally a self-limiting
condition, it can last indefinitely and can be resistant to the usual pain medications. Anti-epileptic
medications such as tricyclic antidepressants and gabapentin are useful in people with neurogenic
pain. Tricyclic's have a worse side effect profile compared to gabapentin.

Discuss and give feedback

Neuropathic pain

External links

NICE

2013 Neuropathic pain guidelines

206

MRCP part 1 Made Easy‫‏‬‎‫‏‬by M. Habayeb & A. Murad


Chapter: Neurology

Question 49 of 246

You want to prescribe an antiemetic to a 19-year-old female who is having a migraine attack. Which
one of the following medications is most likely to precipitate extrapyramidal side-effects?

Meptazinol
Ondansetron
Domperidone
Cyclizine
Metoclopramide

207

MRCP part 1 Made Easy‫‏‬‎‫‏‬by M. Habayeb & A. Murad


Chapter: Neurology

Question 49 of 246

You want to prescribe an antiemetic to a 19-year-old female who is having a migraine attack. Which
one of the following medications is most likely to precipitate extrapyramidal side-effects?

Meptazinol
Ondansetron
Domperidone
Cyclizine
Metoclopramide

Extrapyramidal side-effects are particularly common in children and young adults.

Discuss and give feedback

Migraine: management

External links

NICE

2012 Headache guidelines

British Association for the Study of Headache

Headache guidelines

208

MRCP part 1 Made Easy‫‏‬‎‫‏‬by M. Habayeb & A. Murad


Chapter: Neurology

Question 50 of 246

A 55-year-old man presents due to an uncontrollable urge to move his legs during the night-time. He
has also experience the sensation of spiders crawling over his legs. Simple measures such as walking
and massaging the affected limb have not alleviated the problem. What is the most appropriate
medical therapy?

Selective serotonin reuptake inhibitor


Low-dose tricyclic antidepressant
Dopamine agonist
5-HT3 antagonist
Dopamine antagonist

209

MRCP part 1 Made Easy‫‏‬‎‫‏‬by M. Habayeb & A. Murad


Chapter: Neurology

Question 50 of 246

A 55-year-old man presents due to an uncontrollable urge to move his legs during the night-time. He
has also experience the sensation of spiders crawling over his legs. Simple measures such as walking
and massaging the affected limb have not alleviated the problem. What is the most appropriate
medical therapy?

Selective serotonin reuptake inhibitor


Low-dose tricyclic antidepressant
Dopamine agonist
5-HT3 antagonist
Dopamine antagonist

Restless leg syndrome - management includes dopamine agonists such as ropinirole

Discuss and give feedback

Restless legs syndrome

210

MRCP part 1 Made Easy‫‏‬‎‫‏‬by M. Habayeb & A. Murad


Chapter: Neurology

Question 51 of 246

A 34-year-old man is reviewed in the neurology clinic. He has been established on sodium valproate
for primary generalised epilepsy. Despite now taking a therapeutic dose he continues to have seizures
and is troubled by weight gain since starting sodium valproate. He asks to stop his current medication
and try a different drug. Which one of the following drugs would be the most appropriate second-
line treatment?

Lamotrigine
Ethosuximide
Pregabalin
Gabapentin
Tiagabine

211

MRCP part 1 Made Easy‫‏‬‎‫‏‬by M. Habayeb & A. Murad


Chapter: Neurology

Question 51 of 246

A 34-year-old man is reviewed in the neurology clinic. He has been established on sodium valproate
for primary generalised epilepsy. Despite now taking a therapeutic dose he continues to have seizures
and is troubled by weight gain since starting sodium valproate. He asks to stop his current medication
and try a different drug. Which one of the following drugs would be the most appropriate second-line
treatment?

Lamotrigine
Ethosuximide
Pregabalin
Gabapentin
Tiagabine

Monotherapy with another drug should be attempted before combination therapy is started. Caution
should be exercised when combining sodium valproate and lamotrigine as serious skin rashes such as
Steven-Johnson's syndrome may be provoked.

Discuss and give feedback

Epilepsy: treatment

External links

NICE

2012 Epilepsy guidelines

Royal College of Physicians

2013 Modern management of epilepsy

212

MRCP part 1 Made Easy‫‏‬‎‫‏‬by M. Habayeb & A. Murad


Chapter: Neurology

Question 52 of 246

A 69-year-old man who is known to have Alzheimer's disease is reviewed in clinic. His latest Mini
Mental State Examination (MMSE) score is 18 out of 30. What is the most appropriate
management?

Supportive care + memantine


Supportive care + trial of citalopram
Supportive care
Supportive care + donepezil + low-dose aspirin
Supportive care + donepezil

213

MRCP part 1 Made Easy‫‏‬‎‫‏‬by M. Habayeb & A. Murad


Chapter: Neurology

Question 52 of 246

A 69-year-old man who is known to have Alzheimer's disease is reviewed in clinic. His latest Mini
Mental State Examination (MMSE) score is 18 out of 30. What is the most appropriate
management?

Supportive care + memantine


Supportive care + trial of citalopram
Supportive care
Supportive care + donepezil + low-dose aspirin
Supportive care + donepezil

Discuss and give feedback

Alzheimer's disease

External links

NICE

2011 Dementia guidelines

External media

Alzheimer's disease

Osmosis - YouTube

214

MRCP part 1 Made Easy‫‏‬‎‫‏‬by M. Habayeb & A. Murad


Chapter: Neurology

Question 53 of 246

A 76-year-old man is reviewed in the Elderly Medicine clinic. He is concerned about his increasing
forgetfulness over the past six months. His daughter notes he has generally 'slowed down' and
struggles to follow conversations. Over the past month he has noted increasingly frequent episodes of
urinary incontinence. He has also had one episode of faecal incontinence in the past week. On
examination he is noted to have brisk reflexes and a short, shuffling gait. No cerebellar signs are
noted. What is the most likely diagnosis?

Multiple system atrophy


Parkinson's disease
Normal pressure hydrocephalus
Urinary tract infection
Pick's disease

215

MRCP part 1 Made Easy‫‏‬‎‫‏‬by M. Habayeb & A. Murad


Chapter: Neurology

Question 53 of 246

A 76-year-old man is reviewed in the Elderly Medicine clinic. He is concerned about his increasing
forgetfulness over the past six months. His daughter notes he has generally 'slowed down' and
struggles to follow conversations. Over the past month he has noted increasingly frequent episodes of
urinary incontinence. He has also had one episode of faecal incontinence in the past week. On
examination he is noted to have brisk reflexes and a short, shuffling gait. No cerebellar signs are
noted. What is the most likely diagnosis?

Multiple system atrophy


Parkinson's disease
Normal pressure hydrocephalus
Urinary tract infection
Pick's disease

Urinary incontinence + gait abnormality + dementia = normal pressure hydrocephalus

The presence of dementia and absence of cerebellar signs point away from a diagnosis of multiple
system atrophy

Discuss and give feedback

Normal pressure hydrocephalus

External links

YouTube

Typical NPH Gait

216

MRCP part 1 Made Easy‫‏‬‎‫‏‬by M. Habayeb & A. Murad


Chapter: Neurology

Question 54 of 246

A 55-year-old man is diagnosed with amyotrophic lateral sclerosis. Which one of the following drugs
has been shown to confer a survival benefit?

Rituximab
Riluzole
Interferon-beta
Cyclophosphamide
Interferon-alpha

217

MRCP part 1 Made Easy‫‏‬‎‫‏‬by M. Habayeb & A. Murad


Chapter: Neurology

Question 54 of 246

A 55-year-old man is diagnosed with amyotrophic lateral sclerosis. Which one of the following drugs
has been shown to confer a survival benefit?

Rituximab
Riluzole
Interferon-beta
Cyclophosphamide
Interferon-alpha

Motor neuron disease - riluzole

Discuss and give feedback

Motor neuron disease: management

External links

NICE

2016 Motor neurone disease: assessment and management

218

MRCP part 1 Made Easy‫‏‬‎‫‏‬by M. Habayeb & A. Murad


Chapter: Neurology

Question 55 of 246

Which one of the following is least associated with the development of chorea?

Haemochromatosis
Ataxic telangiectasia
Carbon monoxide poisoning
SLE
Huntington's disease

219

MRCP part 1 Made Easy‫‏‬‎‫‏‬by M. Habayeb & A. Murad


Chapter: Neurology

Question 55 of 246

Which one of the following is least associated with the development of chorea?

Haemochromatosis
Ataxic telangiectasia
Carbon monoxide poisoning
SLE
Huntington's disease

Discuss and give feedback

Chorea

External links

Postgraduate Medical Journal

Review of chorea

220

MRCP part 1 Made Easy‫‏‬‎‫‏‬by M. Habayeb & A. Murad


Chapter: Neurology

Question 56 of 246

A 33-year-old female with multiple sclerosis complains that her vision becomes blurred during a hot
bath. What is this an example of?

Uhthoff's phenomenon
Oppenheim's sign
Werdnig-Hoffman's sign
Lambert's sign
Lhermitte's sign

221

MRCP part 1 Made Easy‫‏‬‎‫‏‬by M. Habayeb & A. Murad


Chapter: Neurology

Question 56 of 246
A 33-year-old female with multiple sclerosis complains that her vision becomes blurred during a hot
bath. What is this an example of?

Uhthoff's phenomenon
Oppenheim's sign
Werdnig-Hoffman's sign
Lambert's sign
Lhermitte's sign

♦This is Uhthoff's phenomenon. Lhermitte's sign describes paraesthesiae in the limbs on neck flexion.
♦Oppenheim's sign is seen when scratching of the inner side of leg leads to extension of the toes. It is a
sign of cerebral irritation and is not related to multiple sclerosis.
♦Werdnig-Hoffman's disease is also known as spinal muscular atrophy.
Discuss and give feedback

Multiple sclerosis: features


External links
NICE
2014 Multiple sclerosis guidelines

External media

Multiple sclerosis - causes, symptoms, diagnosis, treatment, pathology


Osmosis - YouTube

Multiple sclerosis
Podmedics - YouTube

222

MRCP part 1 Made Easy‫‏‬‎‫‏‬by M. Habayeb & A. Murad


Chapter: Neurology

Question 57 of 246

A 23-year-old woman undergoes a planned lumbar puncture (LP) as part of neurological investigations
for possible multiple sclerosis. During the consent process, she expresses concern about a post-LP
headache. What is the mechanism of post-LP headaches?

Vertebral body injury


Nerve injury
Bleeding into cerebrospinal fluid
Leaking cerebrospinal fluid from the dura
Too much cerebrospinal fluid removed

223

MRCP part 1 Made Easy‫‏‬‎‫‏‬by M. Habayeb & A. Murad


Chapter: Neurology

Question 57 of 246

A 23-year-old woman undergoes a planned lumbar puncture (LP) as part of neurological investigations
for possible multiple sclerosis. During the consent process, she expresses concern about a post-LP
headache. What is the mechanism of post-LP headaches?

Vertebral body injury


Nerve injury
Bleeding into cerebrospinal fluid
Leaking cerebrospinal fluid from the dura
Too much cerebrospinal fluid removed

♦Leaking of cerebrospinal fluid from the dura is the most likely explanation for post-lumbar puncture
headaches. It is thought that ongoing leak of cerebrospinal fluid (CSF) through the puncture site
causes ongoing CSF loss, leading to low pressure. A post-LP headache is typically frontal or occipital
and occurs within three days. It is normally associated with worsening on standing and improvement
when lying down. Treatment in severe cases includes an epidural blood patch, but most resolve on
their own.

♦Vertebral body injury, nerve injury and bleeding would not cause headaches, whilst the loss of
cerebrospinal fluid volume during lumbar puncture does not cause pain.

Discuss and give feedback

Post-lumbar puncture headache

224

MRCP part 1 Made Easy‫‏‬‎‫‏‬by M. Habayeb & A. Murad


Chapter: Neurology

Question 58 of 246

A 25-year-old female is found to have a left hemiparesis following a deep vein thrombosis. An ECG
shows RBBB with right axis deviation. What is the most likely underlying diagnosis?

Ventricular septal defect


Patent ductus arteriosus
Ostium primum atrial septal defect
Ostium secundum atrial septal defect
Tetralogy of Fallot

225

MRCP part 1 Made Easy‫‏‬‎‫‏‬by M. Habayeb & A. Murad


Chapter: Neurology

Question 58 of 246

A 25-year-old female is found to have a left hemiparesis following a deep vein thrombosis. An ECG
shows RBBB with right axis deviation. What is the most likely underlying diagnosis?

Ventricular septal defect


Patent ductus arteriosus
Ostium primum atrial septal defect
Ostium secundum atrial septal defect
Tetralogy of Fallot

♦The ostium secundum in this patient has allowed passage of an embolus from the right-sided
circulation to the left causing a stroke

Discuss and give feedback

Atrial septal defects

External media

Atrial septal defect

Osmosis – YouTube

Atrial septal defect murmur - REQUIRES HEADPHONES

Thinklabs – YouTube

226

MRCP part 1 Made Easy‫‏‬‎‫‏‬by M. Habayeb & A. Murad


Chapter: Neurology

Question 59 of 246

You are reviewing a 22-year-old man who has developed headaches. Which one of the following
features is most typical of migraines?

Pain on neck flexion


Phonophobia
Epiphora
Recent viral illness
Bilateral, 'tight-band' like pain

227

MRCP part 1 Made Easy‫‏‬‎‫‏‬by M. Habayeb & A. Murad


Chapter: Neurology

Question 59 of 246

You are reviewing a 22-year-old man who has developed headaches. Which one of the following
features is most typical of migraines?

Pain on neck flexion


Phonophobia
Epiphora
Recent viral illness
Bilateral, 'tight-band' like pain

Phonophobia occurs in around three-quarters of patients.

Discuss and give feedback

Migraine: diagnostic criteria

External links

British Association for the Study of Headache

Headache guidelines

External media

Migraine

Podmedics - YouTube

228

MRCP part 1 Made Easy‫‏‬‎‫‏‬by M. Habayeb & A. Murad


Chapter: Neurology

Question 60 of 246

A 22-year-old man complains of hearing problems. You perform an examination of his auditory
system including Rinne's and Weber's test:

Rinne's test: Left ear: bone conduction > air conduction


Right ear: air conduction > bone conduction
Weber's test: Lateralises to the left side

What do these tests imply?

Normal hearing
Left conductive deafness
Right conductive deafness
Left sensorineural deafness
Right sensorineural deafness

229

MRCP part 1 Made Easy‫‏‬‎‫‏‬by M. Habayeb & A. Murad


Chapter: Neurology

Question 60 of 246

A 22-year-old man complains of hearing problems. You perform an examination of his auditory
system including Rinne's and Weber's test:

Rinne's test: Left ear: bone conduction > air conduction


Right ear: air conduction > bone conduction
Weber's test: Lateralises to the left side

What do these tests imply?

Normal hearing
Left conductive deafness
Right conductive deafness
Left sensorineural deafness
Right sensorineural deafness

Discuss and give feedback

Rinne's and Weber's test

External links

ENT SHO

Guide to the ear examination

230

MRCP part 1 Made Easy‫‏‬‎‫‏‬by M. Habayeb & A. Murad


Chapter: Neurology

Question 61 of 246

A 12-year-old boy is brought to the Emergency Department. He was hit on the side the head by a
cricket ball during a match. His teacher describes him initially collapsing to the ground and
complaining of a sore head. After two minutes he got up, said he felt OK and continued playing. After
30 minutes he suddenly collapsed to the ground and lost consciousness. What type of injury is he
most likely to have sustained?

Cerebral contusion
Subarachnoid haemorrhage
Intraventricular haemorrhage
Extradural haematoma
Subdural haematoma

231

MRCP part 1 Made Easy‫‏‬‎‫‏‬by M. Habayeb & A. Murad


Chapter: Neurology

Question 61 of 246

A 12-year-old boy is brought to the Emergency Department. He was hit on the side the head by a
cricket ball during a match. His teacher describes him initially collapsing to the ground and
complaining of a sore head. After two minutes he got up, said he felt OK and continued playing. After
30 minutes he suddenly collapsed to the ground and lost consciousness. What type of injury is he
most likely to have sustained?

Cerebral contusion
Subarachnoid haemorrhage
Intraventricular haemorrhage
Extradural haematoma
Subdural haematoma

Epidural haematoma - lucid interval

Discuss and give feedback

Head injury: types of traumatic brain injury

External links

NICE

2014 Head injury guidelines

232

MRCP part 1 Made Easy‫‏‬‎‫‏‬by M. Habayeb & A. Murad


Chapter: Neurology

Question 62 of 246

A 50-year-old gentleman presents to clinic with a 20-year history of bilateral upper limb tremor. He
has no past medical history. On examination, he has a bilateral symmetrical upper limb postural tremor
with no rest or intention tremor. His father had a similar tremor. He has noticed improvement with
alcohol. It has become more severe in the past 2 years and he would like treatment. What is the best
first line therapy?

L-DOPA
Deep brain stimulation
Primidone
Propranolol
Botulinum toxin

233

MRCP part 1 Made Easy‫‏‬‎‫‏‬by M. Habayeb & A. Murad


Chapter: Neurology

Question 62 of 246

A 50-year-old gentleman presents to clinic with a 20-year history of bilateral upper limb tremor. He
has no past medical history. On examination, he has a bilateral symmetrical upper limb postural tremor
with no rest or intention tremor. His father had a similar tremor. He has noticed improvement with
alcohol. It has become more severe in the past 2 years and he would like treatment. What is the best
first line therapy?

L-DOPA
Deep brain stimulation
Primidone
Propranolol
Botulinum toxin

♦This is a chronic tremor which is confined to the upper limbs, does not occur at rest, responds to
alcohol and is associated with a positive family history. The diagnosis is Essential tremor.

♦L-DOPA is a treatment for Parkinson's disease and has no role here.

●Answers 2-5 are all treatments for essential tremor. Propranolol is the first line treatment.

Discuss and give feedback

Essential tremor

234

MRCP part 1 Made Easy‫‏‬‎‫‏‬by M. Habayeb & A. Murad


Chapter: Neurology

Question 63 of 246

A 45-year-old man presents to the Emergency Department following the sudden onset of pain in the
right side of his face whilst hammering a nail into the wall. The pain is described as severe and
constant. On examination he has a mild right ptosis and small right pupil. What is the most likely
diagnosis?

Trigeminal neuralgia
Glaucoma
Carotid artery dissection
Syringomyelia
Migraine

235

MRCP part 1 Made Easy‫‏‬‎‫‏‬by M. Habayeb & A. Murad


Chapter: Neurology

Question 63 of 246

A 45-year-old man presents to the Emergency Department following the sudden onset of pain in the
right side of his face whilst hammering a nail into the wall. The pain is described as severe and
constant. On examination he has a mild right ptosis and small right pupil. What is the most likely
diagnosis?

Trigeminal neuralgia
Glaucoma
Carotid artery dissection
Syringomyelia
Migraine

This patient has Horner's syndrome caused by a carotid artery dissection. This may be caused by
relatively benign trauma to the neck such as hyperextension whilst doing DIY. Cluster headache
would be a differential diagnosis

Discuss and give feedback

Horner's syndrome

236

MRCP part 1 Made Easy‫‏‬‎‫‏‬by M. Habayeb & A. Murad


Chapter: Neurology

Question 64 of 246

A 17-year-old man is referred to dermatology. He has around 10 hyperpigmented macules on his torso
which vary in size from 1.5-5 cm in size. His GP also noted some freckles in the groin region. He is
also currently under orthopaedic review due to a worsening scoliosis of the spine. His father suffered
from similar problems before having a fatal myocardial infarction two years ago. Which chromosome
is most likely to have a gene defect?

Chromosome 4
Chromosome 11
Chromosome 16
Chromosome 17
Chromosome 22

237

MRCP part 1 Made Easy‫‏‬‎‫‏‬by M. Habayeb & A. Murad


Chapter: Neurology

Question 64 of 246

A 17-year-old man is referred to dermatology. He has around 10 hyperpigmented macules on his torso
which vary in size from 1.5-5 cm in size. His GP also noted some freckles in the groin region. He is
also currently under orthopaedic review due to a worsening scoliosis of the spine. His father suffered
from similar problems before having a fatal myocardial infarction two years ago. Which chromosome
is most likely to have a gene defect?

Chromosome 4
Chromosome 11
Chromosome 16
Chromosome 17
Chromosome 22

●This patient meets the diagnostic criteria for NF1.

One of our registered users sent the following mnemonic to us:

 NF1: chromosome 17 - as neurofibromatosis has 17 characters

 NF2: chromosome 22 - all the 2's

Discuss and give feedback

Neurofibromatosis

There are two types of neurofibromatosis, NF1 and NF2. Both are inherited in an autosomal dominant
fashion

●NF1 is also known as von Recklinghausen's syndrome. It is caused by a gene mutation on


chromosome 17 which encodes neurofibromin and affects around 1 in 4,000

●NF2 is caused by gene mutation on chromosome 22 and affects around 1 in 100,000

238

MRCP part 1 Made Easy‫‏‬‎‫‏‬by M. Habayeb & A. Murad


Chapter: Neurology

Features

NF1 NF2
Café-au-lait spots (>= 6, 15 mm in Bilateral acoustic neuromas
diameter) Multiple intracranial schwannomas, mengiomas and
Axillary/groin freckles ependymomas
Peripheral neurofibromas
Iris hamatomas (Lisch nodules) in >
90%
Scoliosis
Pheochromocytomas

Comparison of neurofibromatosis and tuberous sclerosis. Note that whilst they are both
autosomal dominant neurocutaneous disorders there is little overlap otherwise

External links

National Center for Biotechnology Information

Guidelines for the diagnosis and management of individuals with neurofibromatosis 1

239

MRCP part 1 Made Easy‫‏‬‎‫‏‬by M. Habayeb & A. Murad


Chapter: Neurology

Question 65 of 246

A 54 year-old woman presents with severe headache after a collapse while at a wedding reception. She
had been dancing when she abruptly fell to the ground. On examination she is neurologically intact,
but clearly finds it difficult to concentrate and is in some discomfort due to her headache. She is
afebrile and denies chest pain. She reports that she had about 3 glasses of wine tonight and she
normally drinks 10-12 units weekly. Her husband reports that she was unconscious for about one
minute and was drowsy and confused on waking.

ECG Normal sinus rhythm

What is the most appropriate management from the options below?

Urgent CT brain
Referral to first-fit clinic
Intravenous ceftriaxone and aciclovir
Referral for outpatient ambulatory ECG monitoring
Discharge with alcohol cessation advice

240

MRCP part 1 Made Easy‫‏‬‎‫‏‬by M. Habayeb & A. Murad


Chapter: Neurology

Question 65 of 246

A 54 year-old woman presents with severe headache after a collapse while at a wedding reception. She
had been dancing when she abruptly fell to the ground. On examination she is neurologically intact,
but clearly finds it difficult to concentrate and is in some discomfort due to her headache. She is
afebrile and denies chest pain. She reports that she had about 3 glasses of wine tonight and she
normally drinks 10-12 units weekly. Her husband reports that she was unconscious for about one
minute and was drowsy and confused on waking.

ECG Normal sinus rhythm

What is the most appropriate management from the options below?

Urgent CT brain
Referral to first-fit clinic
Intravenous ceftriaxone and aciclovir
Referral for outpatient ambulatory ECG monitoring
Discharge with alcohol cessation advice

♦This patient has collapsed in the community with onset of severe headache. Although seizure and
cardiogenic syncope enter the differential, it is vital to exclude subarachnoid haemorrhage as a cause.
Referral for first-fit clinic or outpatient ECG monitoring risk discharging a patient with an
intracerebral bleed and are inappropriate. The normal ECG and absence of chest pain also make a
cardiogenic cause less likely. Discharging the patient with alcohol cessation advice in the context of a
safe level of alcohol consumption is also incorrect.

♦IV ceftriaxone and aciclovir would be treatment for meningitis/encephalitis. This is an important
differential in severe headache but does not typically cause a collapse with sudden-onset headache.

Discuss and give feedback

Subarachnoid hemorrhage

External links

SIGN

2008 Diagnosis and management of headache in adults

241

MRCP part 1 Made Easy‫‏‬‎‫‏‬by M. Habayeb & A. Murad


Chapter: Neurology

Question 66 of 246

A 41-year-old man presents with a two week history of headaches around the left side of his face
associated with watery eyes. He describes having about two episodes a day each lasting around 30
minutes. On examination he has a red left eye and a partial left ptosis. There is no past medical history
of note other than migraines as a child. What is the likely diagnosis?

Atypical migraine
Cluster headache
Trigeminal neuralgia
Acute angle closure glaucoma
Cavernous sinus thrombosis

242

MRCP part 1 Made Easy‫‏‬‎‫‏‬by M. Habayeb & A. Murad


Chapter: Neurology

Question 66 of 246

A 41-year-old man presents with a two week history of headaches around the left side of his face
associated with watery eyes. He describes having about two episodes a day each lasting around 30
minutes. On examination he has a red left eye and a partial left ptosis. There is no past medical history
of note other than migraines as a child. What is the likely diagnosis?

Atypical migraine
Cluster headache
Trigeminal neuralgia
Acute angle closure glaucoma
Cavernous sinus thrombosis

Episodic eye pain, lacrimation, nasal stuffiness occurring daily - cluster headache

Discuss and give feedback

Cluster headache

External links

NICE

2012 Headache guidelines

SIGN

2008 Diagnosis and management of headache in adults

243

MRCP part 1 Made Easy‫‏‬‎‫‏‬by M. Habayeb & A. Murad


Chapter: Neurology

Question 67 of 246

A 55-year-old man is referred to the neurology clinic due to a resting tremor and an abnormal gait
characterised by short, shuffling steps. Which one of the following features would point towards a
diagnosis of Parkinson's disease rather than parkinsonism of another cause?

Asymmetrical tremor
Bradykinesia
Impairment of vertical gaze
Confusion
Poor response to levodopa therapy

244

MRCP part 1 Made Easy‫‏‬‎‫‏‬by M. Habayeb & A. Murad


Chapter: Neurology

Question 67 of 246

A 55-year-old man is referred to the neurology clinic due to a resting tremor and an abnormal gait
characterised by short, shuffling steps. Which one of the following features would point towards a
diagnosis of Parkinson's disease rather than parkinsonism of another cause?

Asymmetrical tremor
Bradykinesia
Impairment of vertical gaze
Confusion
Poor response to levodopa therapy

Asymmetrical symptoms suggests idiopathic Parkinson's

Discuss and give feedback

Parkinson's disease: features

245

MRCP part 1 Made Easy‫‏‬‎‫‏‬by M. Habayeb & A. Murad


Chapter: Neurology

Question 68 of 246

A 76 year-old man is brought in to the Emergency Department with confusion, a temperature of 39.9
degrees Celsius and rigidity. From a recent clinic letter you see he lives alone at home and depends on
help from his daughter, who has recently had a prolonged stay in hospital for appendicitis. Upon her
discharge she went to see him, found him unwell and called 999. She had called him and he had been
well the day before. He has a background of hypertension and Parkinson's disease for which he takes
ramipril, amlodipine and 'a Parkinson's tablet' which he does not have with him. His chest x-ray and
urine dip are clear and he is unable to give a history. A CT brain shows no acute abnormality.

What is the most appropriate intervention?

Broad spectrum antibiotics


Neurosurgical referral
Bromocriptine
Intravenous aciclovir
Cyproheptadine

246

MRCP part 1 Made Easy‫‏‬‎‫‏‬by M. Habayeb & A. Murad


Chapter: Neurology

Question 68 of 246

A 76 year-old man is brought in to the Emergency Department with confusion, a temperature of 39.9
degrees Celsius and rigidity. From a recent clinic letter you see he lives alone at home and depends on
help from his daughter, who has recently had a prolonged stay in hospital for appendicitis. Upon her
discharge she went to see him, found him unwell and called 999. She had called him and he had been
well the day before. He has a background of hypertension and Parkinson's disease for which he takes
ramipril, amlodipine and 'a Parkinson's tablet' which he does not have with him. His chest x-ray and
urine dip are clear and he is unable to give a history. A CT brain shows no acute abnormality.

What is the most appropriate intervention?

Broad spectrum antibiotics


Neurosurgical referral
Bromocriptine
Intravenous aciclovir
Cyproheptadine

♦The presentation is characteristic of neuroleptic malignant syndrome, but without a history of


neuroleptic drug use. In a patient with Parkinson's disease, this raises the question of Parkinsonian
malignant syndrome - a similar clinical picture triggered by abrupt withdrawal from antiparkinsonian
medication. In this case, the patient is dependent on his daughter bringing in his prescriptions and has
run out during her stay in hospital - hence why he does not have the anti-Parkinsonian medication with
him. Although the exact pathogenesis of malignant syndrome is not understood, reinitiation of
Parkinson's therapy is curative.

♦Piperacillin and tazobactam therapy would cover a possible sepsis, but is not a typical treatment for a
potential central nervous system infection and therefore would be the incorrect choice even if sepsis
was the underlying cause of this patient's symptoms. Neurosurgical referral is clearly inappropriate
without any clear operative treatment. Intravenous aciclovir is the treatment for herpes simplex
encephalitis, which is a differential in this case. It is made less likely only by a lack of any prodromal
illness. Cyproheptadine would be a treatment option in serotonin syndrome, but this man is not on any
serotonergic drugs and has no reason to have taken any, making this less likely.

247

MRCP part 1 Made Easy‫‏‬‎‫‏‬by M. Habayeb & A. Murad


Chapter: Neurology

Discuss and give feedback

Neuroleptic malignant syndrome

Neuroleptic malignant syndrome is a rare but dangerous condition seen in patients taking antipsychotic
medication. It carries a mortality of up to 10% and can also occur with atypical antipsychotics. It may
also occur with dopaminergic drugs (such as levodopa) for Parkinson's disease, usually when the drug
is suddenly stopped or the dose reduced.

Features

 more common in young male patients

 onset usually in first 10 days of treatment or after increasing dose

 pyrexia

 rigidity

 tachycardia

♦A raised creatine kinase is present in most cases. A leukocytosis may also be seen.
Management

 stop antipsychotic

 IV fluids to prevent renal failure

 dantrolene* may be useful in selected cases

 bromocriptine, dopamine agonist, may also be used


*thought to work by decreasing excitation-contraction coupling in skeletal muscle by binding
to the ryanodine receptor, and decreasing the release of calcium from the sarcoplasmic
reticulum

External links

Patient.info

Neuroleptic Malignant Syndrome

The Mental Health Clinician

Differentiating MNS and SS

248

MRCP part 1 Made Easy‫‏‬‎‫‏‬by M. Habayeb & A. Murad


Chapter: Neurology

Question 69 of 246

A 60-year-old woman presents to her general practice with signs of thirst and tiredness. On further
questioning, she also complains of polyuria and nocturia. As a result a 24-hour urine is collected,
which returns showing a urine osmolality of 189 mOsm/kg (500-800 mOsm/kg).

The general practitioner suspects a diagnosis of diabetes insipidus.

Which of the following is a recognised cause of nephrogenic diabetes insipidus?

Hyperkalaemia
Hypercalcaemia
Histiocytosis X
Hypermagnesemia
Hypernatraemia

249

MRCP part 1 Made Easy‫‏‬‎‫‏‬by M. Habayeb & A. Murad


Chapter: Neurology

Question 69 of 246

A 60-year-old woman presents to her general practice with signs of thirst and tiredness. On further
questioning, she also complains of polyuria and nocturia. As a result a 24-hour urine is collected,
which returns showing a urine osmolality of 189 mOsm/kg (500-800 mOsm/kg).
The general practitioner suspects a diagnosis of diabetes insipidus.

Which of the following is a recognised cause of nephrogenic diabetes insipidus?

Hyperkalaemia
Hypercalcaemia
Histiocytosis X
Hypermagnesemia
Hypernatraemia

The polyuria and polydipsia, in combination with a dilute urine suggest diabetes insipidus. It is
hypokalaemia, not hyperkalaemia, that causes nephrogenic diabetes insipidus whereas Histiocytosis X
causes cranial diabetes insipidus.

Discuss and give feedback

Diabetes insipidus

External links

Merck manual

Nephrogenic diabetes insipidus

250

MRCP part 1 Made Easy‫‏‬‎‫‏‬by M. Habayeb & A. Murad


Chapter: Neurology

Question 70 of 246

A 40-year-old woman presents with recurrent episode of vertigo associated with a feeling or 'fullness'
and 'pressure' in her ears. She thinks her hearing is worse during the attacks. Clinical examination is
unremarkable. What is the most likely diagnosis?

Meniere's disease
Benign paroxysmal positional vertigo
Acoustic neuroma
Cholesteatoma
Somatisation

251

MRCP part 1 Made Easy‫‏‬‎‫‏‬by M. Habayeb & A. Murad


Chapter: Neurology

Question 70 of 246

A 40-year-old woman presents with recurrent episode of vertigo associated with a feeling or 'fullness'
and 'pressure' in her ears. She thinks her hearing is worse during the attacks. Clinical examination is
unremarkable. What is the most likely diagnosis?

Meniere's disease
Benign paroxysmal positional vertigo
Acoustic neuroma
Cholesteatoma
Somatisation

Discuss and give feedback

Meniere's disease

External links

Clinical Knowledge Summaries

Meniere's disease guidelines

252

MRCP part 1 Made Easy‫‏‬‎‫‏‬by M. Habayeb & A. Murad


Chapter: Neurology

Question 71 of 246

A 64-year-old man presents with a eight-month history of generalised weakness. On examination he


has fasciculation and weakness in both arms with absent reflexes. Examination of the lower limbs
reveal increased tone and exaggerated reflexes. Sensation was normal and there were no cerebellar
signs. What is the most likely diagnosis?

Progressive muscular atrophy


Amyotrophic lateral sclerosis
Vitamin B12 deficiency
Syringomyelia
Multiple sclerosis

253

MRCP part 1 Made Easy‫‏‬‎‫‏‬by M. Habayeb & A. Murad


Chapter: Neurology

Question 71 of 246

A 64-year-old man presents with a eight-month history of generalised weakness. On examination he


has fasciculation and weakness in both arms with absent reflexes. Examination of the lower limbs
reveal increased tone and exaggerated reflexes. Sensation was normal and there were no cerebellar
signs. What is the most likely diagnosis?

Progressive muscular atrophy


Amyotrophic lateral sclerosis
Vitamin B12 deficiency
Syringomyelia
Multiple sclerosis

'Fasciculations' - think motor neuron disease

These symptoms are typical of amyotrophic lateral sclerosis, the most common type of motor neuron
disease.

Discuss and give feedback

Motor neuron disease: features

External links

Royal College of Physicians

Motor neuron disease: diagnostic pitfalls

Postgraduate Medical Journal

Review of MND

254

MRCP part 1 Made Easy‫‏‬‎‫‏‬by M. Habayeb & A. Murad


Chapter: Neurology

Question 72 of 246

A 72-year-old woman with a past history of treated hypertension presents to the Emergency
Department. Yesterday she had a 2 hour episode where she couldn't find the right word when
speaking. This has never happened before and there were no associated features. Neurological
examination is unremarkable and blood pressure was 150/100 mmHg. Her only current medication is
amlodipine. What is the most appropriate management?

Aspirin 300mg immediately + specialist review within 2 weeks


Specialist review within 2 weeks
Aspirin 300mg immediately + specialist review within 24 hours
Aspirin 75mg + outpatient CT brain
Specialist review within 24 hours

255

MRCP part 1 Made Easy‫‏‬‎‫‏‬by M. Habayeb & A. Murad


Chapter: Neurology

Question 72 of 246

A 72-year-old woman with a past history of treated hypertension presents to the Emergency
Department. Yesterday she had a 2 hour episode where she couldn't find the right word when
speaking. This has never happened before and there were no associated features. Neurological
examination is unremarkable and blood pressure was 150/100 mmHg. Her only current medication is
amlodipine. What is the most appropriate management?

Aspirin 300mg immediately + specialist review within 2 weeks


Specialist review within 2 weeks
Aspirin 300mg immediately + specialist review within 24 hours
Aspirin 75mg + outpatient CT brain
Specialist review within 24 hours

♦This patients age, blood pressure and duration of symptoms would put her in a higher risk category.
Current guidelines advocate specialist review within 24 hours.

♦If a patient's symptoms have not fully resolved then aspirin should be withheld until an
haemohorragic stroke has been excluded. As this is a transient ischaemic attack (symptoms last less
than 24 hours) aspirin should be given as soon as possible.

Discuss and give feedback

Transient ischaemic attack

External links

Royal College of Physicians

2012 National clinical guideline for stroke

NICE

2008 Stroke: Diagnosis and initial management of acute stroke and transient ischaemic attack (TIA)

256

MRCP part 1 Made Easy‫‏‬‎‫‏‬by M. Habayeb & A. Murad


Chapter: Neurology

Question 73 of 246

Which one of the following drugs is used in the management of multiple sclerosis?

Beta-interferon
Gamma-interferon
Infliximab
Rituximab
Alpha-interferon

257

MRCP part 1 Made Easy‫‏‬‎‫‏‬by M. Habayeb & A. Murad


Chapter: Neurology

Question 73 of 246

Which one of the following drugs is used in the management of multiple sclerosis?

Beta-interferon
Gamma-interferon
Infliximab
Rituximab
Alpha-interferon

Discuss and give feedback

Multiple sclerosis: management

External links

Multiple Sclerosis Society

Primary care guidelines

NICE

2014 Multiple Sclerosis guidelines

258

MRCP part 1 Made Easy‫‏‬‎‫‏‬by M. Habayeb & A. Murad


Chapter: Neurology

Question 74 of 246

A 34-year-old man from West Africa is admitted due to confusion associated with left-sided weakness
and ataxia. He is known to be HIV positive but is not on anti-retroviral treatment. The following
results are obtained:

CD4 43 u/l
CT head Low attenuation diffusely.
No mass effect or enhancement

What is the most likely diagnosis?

Toxoplasmosis
Tuberculosis
Progressive multifocal leukoencephalopathy
Cryptococcus
Cerebral lymphoma

259

MRCP part 1 Made Easy‫‏‬‎‫‏‬by M. Habayeb & A. Murad


Chapter: Neurology

Question 74 of 246

A 34-year-old man from West Africa is admitted due to confusion associated with left-sided weakness
and ataxia. He is known to be HIV positive but is not on anti-retroviral treatment. The following
results are obtained:

CD4 43 u/l
CT head Low attenuation diffusely.
No mass effect or enhancement

What is the most likely diagnosis?

Toxoplasmosis
Tuberculosis
Progressive multifocal leukoencephalopathy
Cryptococcus
Cerebral lymphoma

Discuss and give feedback


HIV: neurocomplications

Focal neurological lesions

Toxoplasmosis

 accounts for around 50% of cerebral lesions in patients with HIV


 constitutional symptoms, headache, confusion, drowsiness
 CT: usually single or multiple ring enhancing lesions, mass effect may be seen
 management: sulfadiazine and pyrimethamine

260

MRCP part 1 Made Easy‫‏‬‎‫‏‬by M. Habayeb & A. Murad


Chapter: Neurology

© Image used on license from Radiopaedia


Cerebral toxoplasmosis: CT scan with contrast showing multiple ring enhancing lesions

© Image used on license from Radiopaedia


Cerebral toxoplasmosis: MRI (T1 C+) demonstrates multiple small peripherally enhancing nodules
located predominantly in the basal ganglia as well as the central portions of the cerebellar
hemispheres. Only a small amount of surrounding oedema is present.

Primary CNS lymphoma

 accounts for around 30% of cerebral lesions


 associated with the Epstein-Barr virus
 CT: single or multiple homogenous enhancing lesions

261

MRCP part 1 Made Easy‫‏‬‎‫‏‬by M. Habayeb & A. Murad


Chapter: Neurology

 treatment generally involves steroids (may significantly reduce tumour size), chemotherapy
(e.g. methotrexate) + with or without whole brain irradiation. Surgical may be considered for
lower grade tumours

© Image used on license from Radiopaedia


Primary CNS lymphoma: Non-contrast CT demonstrates a hyper-attenuating mass adjacent to the left
lateral ventricle, with no calcification or haemorrhage.

262

MRCP part 1 Made Easy‫‏‬‎‫‏‬by M. Habayeb & A. Murad


Chapter: Neurology

© Image used on license from Radiopaedia


Primary CNS lymphoma: MRI (T1 C+) demonstrates a large multilobulated mass in the right frontal
lobe. It homogeneously enhances and extends to involve the caudate and the periventricular area.
There is significant mass effect.

Differentiating between toxoplasmosis and lymphoma is a common clinical scenario in HIV patients.
It is clearly important given the vastly different treatment strategies. The table below gives some
general differences. Please see the Radiopaedia link for more details.

Toxoplasmosis Lymphoma
Multiple lesions Single lesion
Ring or nodular enhancement Solid (homogenous) enhancement
Thallium SPECT negative Thallium SPECT positive

Tuberculosis

 much less common than toxoplasmosis or primary CNS lymphoma


 CT: single enhancing lesion.

263

MRCP part 1 Made Easy‫‏‬‎‫‏‬by M. Habayeb & A. Murad


Chapter: Neurology

Generalised neurological disease

Encephalitis

 may be due to CMV or HIV itself


 HSV encephalitis but is relatively rare in the context of HIV
 CT: oedematous brain

Cryptococcus

 most common fungal infection of CNS


 headache, fever, malaise, nausea/vomiting, seizures, focal neurological deficit
 CSF: high opening pressure, India ink test positive
 CT: meningeal enhancement, cerebral oedema
 meningitis is typical presentation but may occasionally cause a space occupying lesion

Progressive multifocal leukoencephalopathy (PML)

 widespread demyelination
 due to infection of oligodendrocytes by JC virus (a polyoma DNA virus)
 symptoms, subacute onset : behavioural changes, speech, motor, visual impairment
 CT: single or multiple lesions, no mass effect, don't usually enhance. MRI is better - high-
signal demyelinating white matter lesions are seen

AIDS dementia complex

 caused by HIV virus itself


 symptoms: behavioural changes, motor impairment
 CT: cortical and subcortical atrophy

External links
Radiopaedia
Toxoplasmosis vs lymphoma

264

MRCP part 1 Made Easy‫‏‬‎‫‏‬by M. Habayeb & A. Murad


Chapter: Neurology

Question 75 of 246

Which one of the following is least characteristic of Wernicke's encephalopathy?

Ataxia
Confusion
Ophthalmoplegia
Confabulation
Nystagmus

265

MRCP part 1 Made Easy‫‏‬‎‫‏‬by M. Habayeb & A. Murad


Chapter: Neurology

Question 75 of 246

Which one of the following is least characteristic of Wernicke's encephalopathy?

Ataxia
Confusion
Ophthalmoplegia
Confabulation
Nystagmus

An inability to acquire new memories and confabulation suggests the development of Korsakoff's
syndrome

Discuss and give feedback

Wernicke's encephalopathy

Wernicke's encephalopathy is a neuropsychiatric disorder caused by thiamine deficiency which is most


commonly seen in alcoholics. Rarer causes include: persistent vomiting, stomach cancer, dietary
deficiency. A classic triad of ophthalmoplegia/nystagmus, ataxia and confusion may occur. In
Wernicke's encephalopathy petechial haemorrhages occur in a variety of structures in the brain
including the mamillary bodies and ventricle walls

Features

 nystagmus (the most common ocular sign)

 ophthalmoplegia

 ataxia

 confusion, altered GCS

 peripheral sensory neuropathy

Investigations

 decreased red cell transketolase

 MRI

266

MRCP part 1 Made Easy‫‏‬‎‫‏‬by M. Habayeb & A. Murad


Chapter: Neurology

Treatment is with urgent replacement of thiamine

Relationship with Korsakoff syndrome

If not treated Korsakoff's syndrome may develop as well. This is termed Wernicke-Korsakoff
syndrome and is characterised by the addition of antero- and retrograde amnesia and confabulation in
addition to the above symptoms.

267

MRCP part 1 Made Easy‫‏‬‎‫‏‬by M. Habayeb & A. Murad


Chapter: Neurology

Question 76 of 246

A 54-year-old female with known breast cancer develops progressive motor disorder with impairment
of right arm function. She is clumsy with all movements and exaggerated 'flinging' actions are noted
on examination. There is a similar pattern in the right leg with to a much lesser extent. There are no
other abnormalities on neurological examination.

Where is the lesion most likely to be?

Caudate nucleus
Lateral cerebellum
Pre-motor cortex
Sub-thalamic nucleus
Temporal lobe

268

MRCP part 1 Made Easy‫‏‬‎‫‏‬by M. Habayeb & A. Murad


Chapter: Neurology

Question 76 of 246

A 54-year-old female with known breast cancer develops progressive motor disorder with impairment
of right arm function. She is clumsy with all movements and exaggerated 'flinging' actions are noted
on examination. There is a similar pattern in the right leg with to a much lesser extent. There are no
other abnormalities on neurological examination.

Where is the lesion most likely to be?

Caudate nucleus
Lateral cerebellum
Pre-motor cortex
Sub-thalamic nucleus
Temporal lobe

This is the clinical presentation of hemiballismus. Hemiballismus is a movement disorder


characterised by very violent movements of the arm or limbs. People can seriously injure themselves.
Caused by a stroke or other lesion of the subthalamic nucleus.

Movement disorders in order of least speed to fastest (DACB)

 Dystonia - fixated position

 Athetosis - Snake-like writhing (slow)

 Choreiform - Like a dance choreographer

 Ballistic/Ballismus/Hemiballismus - Fast flinging movements, can injure themselves or others


'like a ballistic missile' (memorisation method)

Discuss and give feedback

Hemiballism

269

MRCP part 1 Made Easy‫‏‬‎‫‏‬by M. Habayeb & A. Murad


Chapter: Neurology

Question 77 of 246

A 77-year-old lady is admitted by the emergency department complaining of difficulty coping at


home. She is unable to mobilise independently and has a poor appetite due to difficulty swallowing.
She has a diagnosis of oesophageal cancer but is not thought to be a candidate for chemotherapy. Her
GP recently started her on nitrofurantoin for a urinary tract infection.

On examination she is a thin, frail lady who is alert and oriented. There is no neurological deficit in the
upper limbs. She has weakness of hip flexion and knee extension in both legs, but markedly more so
on the right. You are able to elicit some loss of pinprick sensation on the anterior thigh. Her reflexes
are brisk with an upgoing plantar on the right.

Her blood results are as follows:

Hb 101 g/l
Platelets 440 * 109/l
WBC 8.4 * 109/l
MCV 99 fL
Na+ 136 mmol/l
K+ 4.8 mmol/l
Urea 3.7 mmol/l
Creatinine 52 µmol/l

What is the next most appropriate step in this patient's management?

Transfer to hospice
Refer for physiotherapy
MRI imaging of the spinal cord
Check B12 and folate levels
Stop nitrofurantoin

270

MRCP part 1 Made Easy‫‏‬‎‫‏‬by M. Habayeb & A. Murad


Chapter: Neurology

Question 77 of 246

A 77-year-old lady is admitted by the emergency department complaining of difficulty coping at


home. She is unable to mobilise independently and has a poor appetite due to difficulty swallowing.
She has a diagnosis of oesophageal cancer but is not thought to be a candidate for chemotherapy. Her
GP recently started her on nitrofurantoin for a urinary tract infection.

On examination she is a thin, frail lady who is alert and oriented. There is no neurological deficit in the
upper limbs. She has weakness of hip flexion and knee extension in both legs, but markedly more so
on the right. You are able to elicit some loss of pinprick sensation on the anterior thigh. Her reflexes
are brisk with an upgoing plantar on the right.

Her blood results are as follows:

Hb 101 g/l
Platelets 440 * 109/l
WBC 8.4 * 109/l
MCV 99 fL
Na+ 136 mmol/l
K+ 4.8 mmol/l
Urea 3.7 mmol/l
Creatinine 52 µmol/l

What is the next most appropriate step in this patient's management?

Transfer to hospice
Refer for physiotherapy
MRI imaging of the spinal cord
Check B12 and folate levels
Stop nitrofurantoin

A patient with new lower limb neurology and a history of cancer should raise the suspicion of
metastatic spinal cord compression, which is best demonstrated on MRI.

Although nitrofurantoin and B12 deficiency could cause a peripheral neuropathy, both are less urgent
problems than cord compression.

Discuss and give feedback

Spinal cord compression

271

MRCP part 1 Made Easy‫‏‬‎‫‏‬by M. Habayeb & A. Murad


Chapter: Neurology

Question 78 of 246

A 19-year-old female presents complaining of visual disturbance. Examination reveals a bitemporal


hemianopia with predominately the lower quadrants being affected. What is the most likely lesion?

Brainstem lesion
Craniopharyngioma
Frontal lobe lesion
Pituitary macroadenoma
Right occipital lesion

272

MRCP part 1 Made Easy‫‏‬‎‫‏‬by M. Habayeb & A. Murad


Chapter: Neurology

Question 78 of 246

A 19-year-old female presents complaining of visual disturbance. Examination reveals a bitemporal


hemianopia with predominately the lower quadrants being affected. What is the most likely lesion?

Brainstem lesion
Craniopharyngioma
Frontal lobe lesion
Pituitary macroadenoma
Right occipital lesion

Bitemporal hemianopia

 lesion of optic chiasm

 upper quadrant defect > lower quadrant defect = inferior chiasmal compression, commonly a
pituitary tumour

 lower quadrant defect > upper quadrant defect = superior chiasmal compression, commonly a
craniopharyngioma

Discuss and give feedback

Visual field defects

273

MRCP part 1 Made Easy‫‏‬‎‫‏‬by M. Habayeb & A. Murad


Chapter: Neurology

Question 79 of 246

An 80-year-old man is investigated for progressive cognitive impairment. Which one of the following
features is most suggestive of Lewy body dementia?

Disinhibition
Emotional lability
Symptoms worsen with neuroleptics
Urinary incontinence
Paucity of extrapyramidal signs

274

MRCP part 1 Made Easy‫‏‬‎‫‏‬by M. Habayeb & A. Murad


Chapter: Neurology

Question 79 of 246

An 80-year-old man is investigated for progressive cognitive impairment. Which one of the following
features is most suggestive of Lewy body dementia?

Disinhibition
Emotional lability
Symptoms worsen with neuroleptics
Urinary incontinence
Paucity of extrapyramidal signs

Discuss and give feedback

Lewy body dementia

275

MRCP part 1 Made Easy‫‏‬‎‫‏‬by M. Habayeb & A. Murad


Chapter: Neurology

Question 80 of 246

A 20-year-old man presents to the neurology clinic with a 6 month history of deteriorating gait.

On examination he has a wide based gait, with past pointing and high arched feet. Knee and ankle
reflexes are absent, but he has an extensor plantar response bilaterally. Fundoscopy reveals a pale optic
disc. There is no impairment of cognition.

What is the most likely diagnosis?

Wilson's disease
Friedrich's ataxia
Charcot-Marie-Tooth disease
Motor neuron disease
Bardet-Biedl syndrome

276

MRCP part 1 Made Easy‫‏‬‎‫‏‬by M. Habayeb & A. Murad


Chapter: Neurology

Question 80 of 246

A 20-year-old man presents to the neurology clinic with a 6 month history of deteriorating gait.
On examination he has a wide based gait, with past pointing and high arched feet. Knee and ankle
reflexes are absent, but he has an extensor plantar response bilaterally. Fundoscopy reveals a pale optic
disc. There is no impairment of cognition.
What is the most likely diagnosis?

Wilson's disease
Friedrich's ataxia
Charcot-Marie-Tooth disease
Motor neuron disease
Bardet-Biedl syndrome

The question describes someone with cerebellar signs, mixed lower motor neuron and upper motor
neuron signs, pes cavus, optic atrophy with a normal IQ.

All of which would be present in Friedrich's ataxia. This normally presents in childhood and is
autosomal recessive. Global spinal cord and cerebellar degeneration give a mixed patten of
degeneration. Retinal degeneration is common, as are cardiomyopathies and diabetes.

Wilson's disease can give ataxia due to excess copper deposition, however over neurological features
may be Parkinsonian in nature e.g. resting tremor and bradykinesia. Wilson's does not give peripheral
neuropathy. A typical Kayser-Fleischer ring of copper may be seen in the iris of patients, no retinal
changes are seen.

Charcot-Marie-Tooth (also known as hereditary motor and sensory neuropathy), presents with pes
cavus and peripheral motor/sensory neuropathy, but would not give cerebellar or visual symptoms.

Motor neuron disease presents with mixed upper and lower motor neuron weakness - as is seen here
with absent tendon reflexes and extensor plantars, however it does not give cerebellar signs or affect
the retina.

Bardet-Biedl syndrome is a rare autosomal recessive condition that gives retinitis pigmentosa, but also
presents with obesity, polydactyly and frequently mental retardation.

Discuss and give feedback

Friedreich's ataxia

277

MRCP part 1 Made Easy‫‏‬‎‫‏‬by M. Habayeb & A. Murad


Chapter: Neurology

Question 81 of 246

A 54-year-old man with small cell lung cancer complains of muscle weakness. Each one of the
following are features of Lambert-Eaton syndrome, except:

Proximal muscles more commonly affected


Hyporeflexia
Dry mouth
Repeated muscle contractions lead to decreased muscle strength
Impotence

278

MRCP part 1 Made Easy‫‏‬‎‫‏‬by M. Habayeb & A. Murad


Chapter: Neurology

Question 81 of 246

A 54-year-old man with small cell lung cancer complains of muscle weakness. Each one of the
following are features of Lambert-Eaton syndrome, except:

Proximal muscles more commonly affected


Hyporeflexia
Dry mouth
Repeated muscle contractions lead to decreased muscle strength
Impotence

In myasthenia gravis repeated muscle contractions lead to reduced muscle strength. The opposite is
however classically seen in the related disorder Lambert-Eaton syndrome

Discuss and give feedback

Lambert-Eaton syndrome

279

MRCP part 1 Made Easy‫‏‬‎‫‏‬by M. Habayeb & A. Murad


Chapter: Neurology

Question 82 of 246

A 30-year-old lady with no past medical history presents to the emergency department with a 2-day
history of pins and needles in the lower limbs and progressive walking difficulties. She offers a history
of a diarrhoeal illness 1 week ago. On examination, there is a loss of pinprick sensation in the lower
limbs from mid-thigh distally and in the upper limbs from MCP joints distally. There is a bilateral
weakness of ankle dorsiflexion to 3/5 and knee flexion and extension is weak to 4/5 bilaterally. Power
in upper and lower limbs is otherwise normal. Knee and ankle deep tendon reflexes are absent. What
is the most likely diagnosis?

Multiple sclerosis
Guillain-Barre syndrome
Functional neurological syndrome
Chronic inflammatory demyelinating polyneuropathy
Botulism

280

MRCP part 1 Made Easy‫‏‬‎‫‏‬by M. Habayeb & A. Murad


Chapter: Neurology

Question 82 of 246

A 30-year-old lady with no past medical history presents to the emergency department with a 2-day
history of pins and needles in the lower limbs and progressive walking difficulties. She offers a history
of a diarrhoeal illness 1 week ago. On examination, there is a loss of pinprick sensation in the lower
limbs from mid-thigh distally and in the upper limbs from MCP joints distally. There is a bilateral
weakness of ankle dorsiflexion to 3/5 and knee flexion and extension is weak to 4/5 bilaterally. Power
in upper and lower limbs is otherwise normal. Knee and ankle deep tendon reflexes are absent. What
is the most likely diagnosis?

Multiple sclerosis
Guillain-Barre syndrome
Functional neurological syndrome
Chronic inflammatory demyelinating polyneuropathy
Botulism

The correct answer is Guillain-Barre syndrome

♦Functional neurological syndrome can be discounted due to the presence of hard neurological signs

♦MS can be excluded due to the presence of lower motor neuron signs (i.e. areflexia) and absence of
upper motor neuron signs

♦Chronic inflammatory demyelinating polyneuropathy (CIDP) is the chronic form of Guillain-barre


syndrome (also known as acute inflammatory demyelinating polyneuropathy). Guillain-barre
syndrome reaches its nadir within three to four weeks, whereas CIDP is defined by progression for
longer than 8 weeks.

♦The diarrhoeal illness may be relevant. Prior Campylobacter infection confers a poorer prognosis
Discuss and give feedback

Guillain-Barre syndrome: features

281

MRCP part 1 Made Easy‫‏‬‎‫‏‬by M. Habayeb & A. Murad


Chapter: Neurology

Question 83 of 246

A 39-year-old lady admitted complaining of severe headache and floppy limbs. She says she has some
blurry vision

She is well known to the cardiology firm due to her post partum cardiomyopathy Previous
investigations for for blurry vision were inconclusive with no particular diagnosis reached.

On examination she has 5/5 power and sensation but tires easily. There is an upgoing left plantar. The
patient has a body mass index of 41 kg/m2. Her ECG shows normal sinus rhythm.

Results from a lumbar puncture are as follows:

White Cells 2 cells/L


Red Cells 3 cells/L
Glucose 75% of plasma level
Xanthochromia negative
Opening pressure 36 cm H2O

The on call radiologist has authorised this report:

CT Head No acute intracranial bleed. Skull base intact. Chronic changes to right hemisphere

Which is the next most appropriate investigation?

Ultrasound doppler scan of the carotids


Autoimmune screen
CT arterial angiogram
Echocardiogram
MRI brain with venogram

282

MRCP part 1 Made Easy‫‏‬‎‫‏‬by M. Habayeb & A. Murad


Chapter: Neurology

Question 83 of 246

A 39-year-old lady admitted complaining of severe headache and floppy limbs. She says she has some
blurry vision
She is well known to the cardiology firm due to her post partum cardiomyopathy Previous
investigations for for blurry vision were inconclusive with no particular diagnosis reached.
On examination she has 5/5 power and sensation but tires easily. There is an upgoing left plantar. The
patient has a body mass index of 41 kg/m2. Her ECG shows normal sinus rhythm.
Results from a lumbar puncture are as follows:

White Cells 2 cells/L


Red Cells 3 cells/L
Glucose 75% of plasma level
Xanthochromia negative
Opening pressure 36 cm H2O

The on call radiologist has authorised this report:

CT Head No acute intracranial bleed. Skull base intact. Chronic changes to right hemisphere

Which is the next most appropriate investigation?

Ultrasound doppler scan of the carotids


Autoimmune screen
CT arterial angiogram
Echocardiogram
MRI brain with venogram

There are features suggestive of idiopathic intracranial hypertension (e.g., raised opening pressure.
However, the upgoing plantar and curious CT head report must make you suspicious of a
cerebrovascular event, such as an infarction or even an intracranial sinus thrombosis. In this situation
MRI imaging of the head would be the best test to rule out these differentials. Remember that a CT
head is not perfectly sensitive for acute ischaemia, and its use in the acute scenario is to rule out a
haemorrhagic stroke
The other tests may be useful later when once the primary pathology (in this case, an acute stroke) has
been established. CT angiography would only be of use if you suspected a carotid or vertebral
dissection - there is little in the history to support this.

Discuss and give feedback

Intracranial venous thrombosis


283

MRCP part 1 Made Easy‫‏‬‎‫‏‬by M. Habayeb & A. Murad


Chapter: Neurology

Question 84 of 246

A 54-year-old man is admitted to the Emergency Department with a left hemiplegia. His symptoms
started around 5 hours ago and he has had no headache, visual disturbance or loss of consciousness.
On examination a dense left hemiplegia is noted. Blood pressure is 120/78 mmHg, GCS is 15/15 and
pupils are equal and reactive to light. An urgent CT scan is performed shortly after his arrival. This
demonstrates no abnormality. What is the most appropriate initial management?

Enoxaparin
Alteplase
Dexamethasone
Warfarin
Aspirin

284

MRCP part 1 Made Easy‫‏‬‎‫‏‬by M. Habayeb & A. Murad


Chapter: Neurology

Question 84 of 246

A 54-year-old man is admitted to the Emergency Department with a left hemiplegia. His symptoms
started around 5 hours ago and he has had no headache, visual disturbance or loss of consciousness.
On examination a dense left hemiplegia is noted. Blood pressure is 120/78 mmHg, GCS is 15/15 and
pupils are equal and reactive to light. An urgent CT scan is performed shortly after his arrival. This
demonstrates no abnormality. What is the most appropriate initial management?

Enoxaparin
Alteplase
Dexamethasone
Warfarin
Aspirin

Stroke thrombolysis - only consider if less than 4.5 hours and haemorrhage excluded

This patient has had an ischaemic stroke. He is however outside the thrombolytic window and should
therefore be treated with aspirin
Discuss and give feedback

Stroke: management
External links
NICE 2010 Clopidogrel and dipyridamole guidelines
SIGN 2008 Stroke guidelines
Age and Ageing Interesting article on managing blood pressure during acute stroke
RCP Stroke guidelines
NICE 2008 TIA and stroke guidelines

Clinical Knowledge Summaries Stroke and TIA guidelines

285

MRCP part 1 Made Easy‫‏‬‎‫‏‬by M. Habayeb & A. Murad


Chapter: Neurology

Question 85 of 246

A 54-year-old man presents concerned about leg weakness. On examination he is noted to have
increased tone in both legs, brisk reflexes and weakness in both lower limbs. Examination of his upper
limbs is normal. Which one of the following is least likely to produce these symptoms?

HIV
Amyloidosis
Hereditary spastic paraplegia
Multiple sclerosis
Parasagittal meningioma

286

MRCP part 1 Made Easy‫‏‬‎‫‏‬by M. Habayeb & A. Murad


Chapter: Neurology

Question 85 of 246

A 54-year-old man presents concerned about leg weakness. On examination he is noted to have
increased tone in both legs, brisk reflexes and weakness in both lower limbs. Examination of his upper
limbs is normal. Which one of the following is least likely to produce these symptoms?

HIV
Amyloidosis
Hereditary spastic paraplegia
Multiple sclerosis
Parasagittal meningioma

Amyloidosis is the least likely of the above options to result in a spastic paraparesis

Discuss and give feedback

Spastic paraparesis

287

MRCP part 1 Made Easy‫‏‬‎‫‏‬by M. Habayeb & A. Murad


Chapter: Neurology

Question 86 of 246

Which one of the following is most likely to cause a bilateral facial nerve palsy?

Acoustic neuroma
Cholesteatoma
Bell's palsy
Sarcoidosis
Amyloidosis

288

MRCP part 1 Made Easy‫‏‬‎‫‏‬by M. Habayeb & A. Murad


Chapter: Neurology

Question 86 of 246
Which one of the following is most likely to cause a bilateral facial nerve palsy?

Acoustic neuroma
Cholesteatoma
Bell's palsy
Sarcoidosis
Amyloidosis
Facial nerve
Supply - 'face, ear, taste, tear'
 face: muscles of facial expression
 ear: nerve to stapedius
 taste: supplies anterior two-thirds of tongue
 tear: parasympathetic fibres to lacrimal glands, also salivary glands

Causes of bilateral facial nerve palsy


 sarcoidosis

 Guillain-Barre syndrome

 polio, Lyme disease

Causes of unilateral facial nerve palsy - as above plus

Lower motor neuron Upper motor neuron


 Bell's palsy
 Ramsay-Hunt syndrome (due to herpes zoster)  stroke
 acoustic neuroma
 parotid tumours
 HIV
 multiple sclerosis*
 diabetes mellitus

LMN vs. UMN

 upper motor neuron lesion 'spares' upper face i.e. forehead

 lower motor neuron lesion affects all facial muscles

*may also cause an UMN palsy

289

MRCP part 1 Made Easy‫‏‬‎‫‏‬by M. Habayeb & A. Murad


Chapter: Neurology

Question 87 of 246

A 63-year-old man is prescribed ropinirole for Parkinson's disease. What is the mechanism of
action?

MAO-B inhibitor
Antimuscarinic
Dopamine receptor agonist
Dopamine receptor antagonist
Decarboxylase inhibitor

290

MRCP part 1 Made Easy‫‏‬‎‫‏‬by M. Habayeb & A. Murad


Chapter: Neurology

Question 87 of 246

A 63-year-old man is prescribed ropinirole for Parkinson's disease. What is the mechanism of
action?

MAO-B inhibitor
Antimuscarinic
Dopamine receptor agonist
Dopamine receptor antagonist
Decarboxylase inhibitor

Ropinirole - dopamine receptor agonist

Discuss and give feedback

Parkinson's disease: management

291

MRCP part 1 Made Easy‫‏‬‎‫‏‬by M. Habayeb & A. Murad


Chapter: Neurology

Question 88 of 246

A 59-year-old man with no significant past medical history is admitted to hospital following an
ischaemic stroke. He presented outside of the thrombolysis window and is treated with aspirin for the
first few days. His blood pressure is 130/80 mmHg, fasting glucose is 5.6 mmol/l and fasting
cholesterol is 3.9 mmol/l. He makes a good recovery and has regained nearly all of his previous
functions upon discharge. Following recent NICE guidelines, which of the following medications
should he be taking upon discharge (i.e. after 14 days)?

Aspirin + statin
Aspirin + dipyridamole + statin + ramipril
Clopidogrel + statin
Aspirin + dipyridamole
Aspirin + dipyridamole + statin

292

MRCP part 1 Made Easy‫‏‬‎‫‏‬by M. Habayeb & A. Murad


Chapter: Neurology

question 88 of 246

A 59-year-old man with no significant past medical history is admitted to hospital following an
ischaemic stroke. He presented outside of the thrombolysis window and is treated with aspirin for the
first few days. His blood pressure is 130/80 mmHg, fasting glucose is 5.6 mmol/l and fasting
cholesterol is 3.9 mmol/l. He makes a good recovery and has regained nearly all of his previous
functions upon discharge. Following recent NICE guidelines, which of the following medications
should he be taking upon discharge (i.e. after 14 days)?

Aspirin + statin
Aspirin + dipyridamole + statin + ramipril
Clopidogrel + statin
Aspirin + dipyridamole
Aspirin + dipyridamole + statin

Discuss and give feedback

Stroke: management

293

MRCP part 1 Made Easy‫‏‬‎‫‏‬by M. Habayeb & A. Murad


Chapter: Neurology

Question 89 of 246

Which one of the following infections is most strongly associated with the development of
Guillain-Barre syndrome?

Shigella
Salmonella
E. coli H7:0157
Herpes simplex
Campylobacter jejuni

294

MRCP part 1 Made Easy‫‏‬‎‫‏‬by M. Habayeb & A. Murad


Chapter: Neurology

Question 89 of 246

Which one of the following infections is most strongly associated with the development of
Guillain-Barre syndrome?

Shigella
Salmonella
E. coli H7:0157
Herpes simplex
Campylobacter jejuni

Campylobacter jejuni is strongly associated with the development of Guillain-Barre syndrome

Discuss and give feedback

Guillain-Barre syndrome

295

MRCP part 1 Made Easy‫‏‬‎‫‏‬by M. Habayeb & A. Murad


Chapter: Neurology

Question 90 of 246

Lateral medullary syndrome is caused by occlusion of which one of the following blood vessels?

Anterior inferior cerebellar artery


Posterior cerebral artery
Lateral sinus thrombosis
Middle cerebral artery
Posterior inferior cerebellar artery

296

MRCP part 1 Made Easy‫‏‬‎‫‏‬by M. Habayeb & A. Murad


Chapter: Neurology

Question 90 of 246

Lateral medullary syndrome is caused by occlusion of which one of the following blood vessels?

Anterior inferior cerebellar artery


Posterior cerebral artery
Lateral sinus thrombosis
Middle cerebral artery
Posterior inferior cerebellar artery

Lateral medullary syndrome - PICA lesion - cerebellar signs, contralateral sensory loss &
ipsilateral Horner's

Discuss and give feedback

Lateral medullary syndrome

297

MRCP part 1 Made Easy‫‏‬‎‫‏‬by M. Habayeb & A. Murad


Chapter: Neurology

Question 91 of 246

A 29-year-old female presents complaining of weakness in her arms, leading to increasing difficulties
at work. On examination she has a bilateral ptosis and loss of the red-reflex in both eyes. Urine testing
also reveals glycosuria. What is the most likely diagnosis?

Myotonic dystrophy
Homocystinuria
Multiple sclerosis
Myasthenia gravis
HIV

298

MRCP part 1 Made Easy‫‏‬‎‫‏‬by M. Habayeb & A. Murad


Chapter: Neurology

Question 91 of 246

A 29-year-old female presents complaining of weakness in her arms, leading to increasing difficulties
at work. On examination she has a bilateral ptosis and loss of the red-reflex in both eyes. Urine testing
also reveals glycosuria. What is the most likely diagnosis?

Myotonic dystrophy
Homocystinuria
Multiple sclerosis
Myasthenia gravis
HIV

Dystrophia myotonica - DM1

 distal weakness initially

 autosomal dominant

 diabetes

 dysarthria

These features are typical of myotonic dystrophy. The red-reflex is lost due to bilateral cataracts

Discuss and give feedback

Myotonic dystrophy

299

MRCP part 1 Made Easy‫‏‬‎‫‏‬by M. Habayeb & A. Murad


Chapter: Neurology

Question 92 of 246

A 61 year-old man presents to the respiratory clinic with a 2-month history of progressive weakness
and shortness of breath. He finds it difficult to stand from sitting, and struggles climbing stairs. He is
an ex-smoker with chronic obstructive pulmonary disease (COPD). He had a recent exacerbation one
month ago for which he was treated by the GP with a course of oral prednisolone, during which time
his weakness transiently improved. On examination you note a left-sided monophonic wheeze and
reduced breath sounds at the left lung base. Blood tests and a chest x-ray are requested.

Hb 145 g/L
WCC 10.9 109/l
Na+ 138 mmol/L
K+ 4.3 mmol/L
Urea 6.8 mmol/L
Creatinine 93 mmol/L
Calcium 2.62 mmol/L
Phosphate 1.44 mmol/L

Chest x-ray Hyperexpanded lungs, left lower lobe collapse, bulky left hilum

What is the most likely cause of this patient's weakness?

Myasthenia gravis
Steroid-induced myopathy
Lambert-Eaton myasthenic syndrome
Hypercalcaemia
Motor neurone disease

300

MRCP part 1 Made Easy‫‏‬‎‫‏‬by M. Habayeb & A. Murad


Chapter: Neurology

Question 92 of 246
A 61 year-old man presents to the respiratory clinic with a 2-month history of progressive weakness
and shortness of breath. He finds it difficult to stand from sitting, and struggles climbing stairs. He is
an ex-smoker with chronic obstructive pulmonary disease (COPD). He had a recent exacerbation one
month ago for which he was treated by the GP with a course of oral prednisolone, during which time
his weakness transiently improved. On examination you note a left-sided monophonic wheeze and
reduced breath sounds at the left lung base. Blood tests and a chest x-ray are requested.

Hb 145 g/L
WCC 10.9 109/l
Na+ 138 mmol/L
K+ 4.3 mmol/L
Urea 6.8 mmol/L
Creatinine 93 mmol/L
Calcium 2.62 mmol/L
Phosphate 1.44 mmol/L

Chest x-ray Hyperexpanded lungs, left lower lobe collapse, bulky left hilum

What is the most likely cause of this patient's weakness?


Myasthenia gravis
Steroid-induced myopathy
Lambert-Eaton myasthenic syndrome
Hypercalcaemia
Motor neurone disease

♦This man has a small-cell lung cancer (SCLC) and associated Lambert-Eaton myasthenic syndrome -
a well-recognised paraneoplastic manifestation of SCLC. This classically effects the proximal
muscles, especially in the legs, causing difficulty in standing from a seated position and climbing
stairs. In contrast to myasthenia gravis, eye involvement is uncommon.
♦Treatment with steroids is often helpful, which explains his transient symptomatic improvement
during treatment for his COPD exacerbation.
Steroid myopathy does not fit as the symptoms started well before his course of prednisolone.
Although the patient is mildly hypercalcaemic this would not be sufficient to produce his presenting
symptoms, although it does reinforce the suspicion of lung malignancy. Motor neurone disease would
be unlikely in this context and would not improve with steroids. Myasthenia gravis could produce
these symptoms, but in the context of a new lung mass is a less viable diagnosis.
Discuss and give feedback
Lambert-Eaton syndrome

301

MRCP part 1 Made Easy‫‏‬‎‫‏‬by M. Habayeb & A. Murad


Chapter: Neurology

Question 93 of 246

A 56-year-old man presents to the Emergency Department after developing trouble talking after
waking up this morning. The symptoms are consistent with expressive dysphasia and lasted about 90
minutes before resolving completely. Neurological examination is unremarkable. A diagnosis of
transient ischaemic attack (TIA) is made.

His past medical history includes ischaemic heart disease for which he is prescribed aspirin,
simvastatin and atenolol. Which one of the following factors is most associated with an increased
risk of going on to have a stroke?

History of ischaemic heart disease


History of aspirin use
Duration of this TIA
Expressive dysphasia during this TIA
His age

302

MRCP part 1 Made Easy‫‏‬‎‫‏‬by M. Habayeb & A. Murad


Chapter: Neurology

Question 93 of 246

A 56-year-old man presents to the Emergency Department after developing trouble talking after
waking up this morning. The symptoms are consistent with expressive dysphasia and lasted about 90
minutes before resolving completely. Neurological examination is unremarkable. A diagnosis of
transient ischaemic attack (TIA) is made.

His past medical history includes ischaemic heart disease for which he is prescribed aspirin,
simvastatin and atenolol. Which one of the following factors is most associated with an increased
risk of going on to have a stroke?

History of ischaemic heart disease


History of aspirin use
Duration of this TIA
Expressive dysphasia during this TIA
His age

This TIA lasted greater than 60 minutes which scores 2 as part of the ABCD2 prognostic scoring
system. It is therefore the most significant factor which would increase his risk of going on to have a
stroke.

Discuss and give feedback

Transient ischemic attack

External links

Royal College of Physicians

2012 National clinical guideline for stroke

NICE

2008 Stroke: Diagnosis and initial management of acute stroke and transient ischaemic attack
(TIA)

303

MRCP part 1 Made Easy‫‏‬‎‫‏‬by M. Habayeb & A. Murad


Chapter: Neurology

Question 94 of 246

A 14-year-old male is noted to have optic atrophy on fundoscopy. Neurological exam reveals
dysarthric speech and nystagmus. Knee and ankle jerks are absent but there is an extensor plantar
response. What is the likely diagnosis?

Leber's optic atrophy


Ataxic telangiectasia
Friedreich's ataxia
Subacute combined degeneration of the cord
Multiple sclerosis

304

MRCP part 1 Made Easy‫‏‬‎‫‏‬by M. Habayeb & A. Murad


Chapter: Neurology

Question 94 of 246

A 14-year-old male is noted to have optic atrophy on fundoscopy. Neurological exam reveals
dysarthric speech and nystagmus. Knee and ankle jerks are absent but there is an extensor plantar
response. What is the likely diagnosis?

Leber's optic atrophy


Ataxic telangiectasia
Friedreich's ataxia
Subacute combined degeneration of the cord
Multiple sclerosis

♦Multiple sclerosis would be unlikely at this age

Discuss and give feedback

Friedreich's ataxia

305

MRCP part 1 Made Easy‫‏‬‎‫‏‬by M. Habayeb & A. Murad


Chapter: Neurology

Question 95 of 246

A 46-year-old female presents with a burning sensation over the antero-lateral aspect of her right
thigh. A diagnosis of meralgia paraesthetica is suspected. Which nerve is most likely to be affected?

Common peroneal nerve


Anterior cutaneous nerve of thigh
Posterior cutaneous nerve of thigh
Lateral cutaneous nerve of thigh
Sciatic nerve

306

MRCP part 1 Made Easy‫‏‬‎‫‏‬by M. Habayeb & A. Murad


Chapter: Neurology

Question 95 of 246

A 46-year-old female presents with a burning sensation over the antero-lateral aspect of her right
thigh. A diagnosis of meralgia paraesthetica is suspected. Which nerve is most likely to be affected?

Common peroneal nerve


Anterior cutaneous nerve of thigh
Posterior cutaneous nerve of thigh
Lateral cutaneous nerve of thigh
Sciatic nerve

Burning thigh pain - ? meralgia paraesthetica - lateral cutaneous nerve of thigh compression

Discuss and give feedback

Meralgia paraesthetica

307

MRCP part 1 Made Easy‫‏‬‎‫‏‬by M. Habayeb & A. Murad


Chapter: Neurology

Question 96 of 246

A 72-year-old man who is being treated for Parkinson's disease is reviewed. Which one of the
following features should prompt you to consider an alternative diagnosis?

Micrographia
Impaired olfaction
REM sleep behaviour disorder
Diplopia
Psychosis

308

MRCP part 1 Made Easy‫‏‬‎‫‏‬by M. Habayeb & A. Murad


Chapter: Neurology

Question 96 of 246

A 72-year-old man who is being treated for Parkinson's disease is reviewed. Which one of the
following features should prompt you to consider an alternative diagnosis?

Micrographia
Impaired olfaction
REM sleep behaviour disorder
Diplopia
Psychosis

Diplopia is not common in Parkinson's disease and may suggest an alternative cause of parkinsonism
such as progressive supranuclear palsy.

Discuss and give feedback

Parkinson's disease: features

309

MRCP part 1 Made Easy‫‏‬‎‫‏‬by M. Habayeb & A. Murad


Chapter: Neurology

Question 97 of 246

A 52-year-old woman presents with a two week history of dizziness when she rolls over in bed. She
says it feels like the room is spinning around her. Examination of her ears and cranial nerves is
unremarkable. Given the likely diagnosis of benign paroxysmal positional vertigo what is the most
appropriate management?

Trial of prochlorperazine
Request MRI brain
Advise review by an optician
Perform Epley manoeuvre
Trial of cinnarizine

310

MRCP part 1 Made Easy‫‏‬‎‫‏‬by M. Habayeb & A. Murad


Chapter: Neurology

Question 97 of 246

A 52-year-old woman presents with a two week history of dizziness when she rolls over in bed. She
says it feels like the room is spinning around her. Examination of her ears and cranial nerves is
unremarkable. Given the likely diagnosis of benign paroxysmal positional vertigo what is the most
appropriate management?

Trial of prochlorperazine
Request MRI brain
Advise review by an optician
Perform Epley manoeuvre
Trial of cinnarizine

Discuss and give feedback

Benign paroxysmal positional vertigo

External links

YouTube

Hallpike Test and Epley Maneuver

Clinical Knowledge Summaries

Benign paroxysmal positional vertigo guidelines

311

MRCP part 1 Made Easy‫‏‬‎‫‏‬by M. Habayeb & A. Murad


Chapter: Neurology

Question 98 of 246

A 62-year-old man is referred to the neurology clinic with worsening symptoms over the past few
months. The neurologist suspects the patient has progressive supranuclear palsy. Which one of the
following features is least likely to be seen in this patient?

Poor response to L-dopa


Impairment of horizontal gaze
Falls
Cognitive impairment
Slurring of speech

312

MRCP part 1 Made Easy‫‏‬‎‫‏‬by M. Habayeb & A. Murad


Chapter: Neurology

Question 98 of 246

A 62-year-old man is referred to the neurology clinic with worsening symptoms over the past few
months. The neurologist suspects the patient has progressive supranuclear palsy. Which one of the
following features is least likely to be seen in this patient?

Poor response to L-dopa


Impairment of horizontal gaze
Falls
Cognitive impairment
Slurring of speech

Progressive supranuclear palsy: parkinsonism, impairment of vertical gaze

Impairment of vertical gaze is seen in progressive supranuclear palsy. Horizontal gaze impairment is
sometimes seen later as the disease progresses, but would be atypical in a newly diagnosed patient.

Discuss and give feedback

Progressive supranuclear palsy

313

MRCP part 1 Made Easy‫‏‬‎‫‏‬by M. Habayeb & A. Murad


Chapter: Neurology

Question 99 of 246

A 65-year-old gentleman is admitted to the medical take with a progressively worsening headache for
3 weeks. On closer questioning, he reveals it is worse first thing in the morning but also exacerbated
by recumbency and coughing. He is also complaining of intermittent visual disturbances and on
fundoscopic examination, there is papilloedema of the right disc but optic atrophy on the left.

A subsequent CT scan is performed revealing a space occupying lesion. Where is the space occupying
lesion most likely situated?

Left temporal
Right frontal
Right temporal
Left frontal
Left parietal

314

MRCP part 1 Made Easy‫‏‬‎‫‏‬by M. Habayeb & A. Murad


Chapter: Neurology

Question 99 of 246

A 65-year-old gentleman is admitted to the medical take with a progressively worsening headache for
3 weeks. On closer questioning, he reveals it is worse first thing in the morning but also exacerbated
by recumbency and coughing. He is also complaining of intermittent visual disturbances and on
fundoscopic examination, there is papilloedema of the right disc but optic atrophy on the left.

A subsequent CT scan is performed revealing a space occupying lesion. Where is the space
occupying lesion most likely situated?

Left temporal
Right frontal
Right temporal
Left frontal
Left parietal

This is an interesting case of Foster-Kennedy syndrome. This syndrome reflects a frontal lobe tumour
- usually a meningioma in this age group - leading to ipsilateral optic atrophy and papilloedema of the
contralateral optic nerve. The reason for the optic atrophy is as a result of direct damage from the
space occupying lesion. Other causes are AVMs and juvenile nasopharyngeal angiofibroma.

Discuss and give feedback

Brain lesions

315

MRCP part 1 Made Easy‫‏‬‎‫‏‬by M. Habayeb & A. Murad


Chapter: Neurology

Question 100 of 246

A 56-year-old woman comes for review. Around 4 weeks ago she had a blistering rash under her right
breast which extended around to the back. A diagnosis of shingles was made. Unfortunately since that
time she has been experiencing severe 'shooting' pains. The skin is also very tender to touch. Neither
paracetamol nor ibuprofen have helped her symptoms. What is the most appropriate next step in
management?

Lidocaine patch
Tramadol
Amitriptyline
Carbamazepine
Diclofenac

316

MRCP part 1 Made Easy‫‏‬‎‫‏‬by M. Habayeb & A. Murad


Chapter: Neurology

Question 100 of 246

A 56-year-old woman comes for review. Around 4 weeks ago she had a blistering rash under her right
breast which extended around to the back. A diagnosis of shingles was made. Unfortunately since that
time she has been experiencing severe 'shooting' pains. The skin is also very tender to touch. Neither
paracetamol nor ibuprofen have helped her symptoms. What is the most appropriate next step in
management?

Lidocaine patch
Tramadol
Amitriptyline
Carbamazepine
Diclofenac

This lady has developed post-herpetic neuralgia. NICE recommend using amitriptyline, duloxetine,
gabapentin or pregabalin first-line.

Discuss and give feedback

Neuropathic pain

External links

NICE

2013 Neuropathic pain guidelines

317

MRCP part 1 Made Easy‫‏‬‎‫‏‬by M. Habayeb & A. Murad


Chapter: Neurology

Question 101 of 246

A patient is noted to have absent ankle jerks combined with extensor plantars on examination. Which one of
the following is least likely?

Multiple sclerosis
Subacute combined degeneration of the cord
Syringomyelia
Syphilis
Motor neuron disease

MRCP part 1 Made Easy‫‏‬‎‫‏‬by M. Habayeb & A. Murad

318
Chapter: Neurology

Question 101 of 246

A patient is noted to have absent ankle jerks combined with extensor plantars on examination. Which one of
the following is least likely?

Multiple sclerosis
Subacute combined degeneration of the cord
Syringomyelia
Syphilis
Motor neuron disease

Multiple sclerosis would not give this combination of physical signs

Discuss and give feedback

Absent ankle jerks, extensor plantars

MRCP part 1 Made Easy‫‏‬‎‫‏‬by M. Habayeb & A. Murad

319
Chapter: Neurology

Question 102 of 246

A 42-year-old woman presents as she has noticed a 'droop' in the right side of her face since she woke up this
morning. There is no associated limb weakness, dysphagia or visual disturbance. On examination you notice
right-sided upper and lower facial paralysis. Which one of the following features would be most consistent
with a diagnosis of Bell's palsy?

Vesicular rash around the ear


Hyperacusis
Sensory loss over the distribution of the facial nerve
Pins and needles in the right arm
Rhinorrhoea

MRCP part 1 Made Easy‫‏‬‎‫‏‬by M. Habayeb & A. Murad

320
Chapter: Neurology

Question 102 of 246

A 42-year-old woman presents as she has noticed a 'droop' in the right side of her face since she woke up this
morning. There is no associated limb weakness, dysphagia or visual disturbance. On examination you notice
right-sided upper and lower facial paralysis. Which one of the following features would be most consistent
with a diagnosis of Bell's palsy?

Vesicular rash around the ear


Hyperacusis
Sensory loss over the distribution of the facial nerve
Pins and needles in the right arm
Rhinorrhoea

♦A vesicular rash around the ear would suggest a diagnosis of Ramsey Hunt syndrome. Hyperacusis is seen in
around a third of patients.

Discuss and give feedback

Bell's palsy

External links

Clinical Knowledge Summaries

Bell's palsy guidelines

MRCP part 1 Made Easy‫‏‬‎‫‏‬by M. Habayeb & A. Murad

321
Chapter: Neurology

Question 103 of 246

A 47-year-old man presents to the Emergency Department with a three day history of severe headache
associated with vomiting. There is no past medical history of note. On examination blood pressure is 98/62
mmHg, pulse is 108 bpm and temperature is 37.0ºC. There is mild neck stiffness and a partial third nerve
palsy of the left eye. Blood rests reveal:

Hb 14.8 g/dl
Plt 373 * 109/l
WBC 13.6 * 109/l

Na+ 132 mmol/l


K+ 5.2 mmol/l
Urea 4.2 mmol/l
Creatinine 99 µmol/l

Free T4 9 pmol/l
(range 10-22)

What is the most likely diagnosis?

Subarachnoid haemorrhage
Cavernous sinus thrombosis
Meningitis
Pituitary apoplexy
Lateral sinus thrombosis

MRCP part 1 Made Easy‫‏‬‎‫‏‬by M. Habayeb & A. Murad

322
Chapter: Neurology

Question 103 of 246

A 47-year-old man presents to the Emergency Department with a three day history of severe headache
associated with vomiting. There is no past medical history of note. On examination blood pressure is 98/62
mmHg, pulse is 108 bpm and temperature is 37.0ºC. There is mild neck stiffness and a partial third nerve
palsy of the left eye. Blood rests reveal:

Hb 14.8 g/dl
Plt 373 * 109/l
WBC 13.6 * 109/l

Na+ 132 mmol/l


K+ 5.2 mmol/l
Urea 4.2 mmol/l
Creatinine 99 µmol/l

Free T4 9 pmol/l
(range 10-22)

What is the most likely diagnosis?

Subarachnoid haemorrhage
Cavernous sinus thrombosis
Meningitis
Pituitary apoplexy
Lateral sinus thrombosis

The hypotension, electrolytes and low free T4 point towards hypopituitarism. Clinically, pituitary apoplexy
can mimic a subarachnoid haemorrhage

Discuss and give feedback

Pituitary apoplexy

External links

Postgraduate Medical Journal

Review of endocrine emergencies

MRCP part 1 Made Easy‫‏‬‎‫‏‬by M. Habayeb & A. Murad

323
Chapter: Neurology

Question 104 of 246

Which one of the following is a contraindication to the use of a triptan in the management of migraine?

Concurrent pizotifen use


Patients older than 55 years
A history of epilepsy
Previous intracranial tumour
A history of ischaemic heart disease

MRCP part 1 Made Easy‫‏‬‎‫‏‬by M. Habayeb & A. Murad

324
Chapter: Neurology

Question 104 of 246

Which one of the following is a contraindication to the use of a triptan in the management of migraine?

Concurrent pizotifen use


Patients older than 55 years
A history of epilepsy
Previous intracranial tumour
A history of ischaemic heart disease

Discuss and give feedback

Triptans

External links

Clinical Knowledge Summaries

Migraine guidelines

MRCP part 1 Made Easy‫‏‬‎‫‏‬by M. Habayeb & A. Murad

325
Chapter: Neurology

Question 105 of 246

A 24-year-old man is seen in the 'First Seizure' clinic. He has been referred by the local the Emergency
Department following a single episode of a witnessed seizure. Which one of the following factors would be
least relevant when deciding whether to start anti-epileptic drugs after a single seizure?

Brain imaging shows a structural abnormality


The patient has a neurological deficit
The EEG shows unequivocal epileptic activity
The patient is less than 45 years old
The patient considers the risk of having a further seizure unacceptable

MRCP part 1 Made Easy‫‏‬‎‫‏‬by M. Habayeb & A. Murad

326
Chapter: Neurology

Question 105 of 246

A 24-year-old man is seen in the 'First Seizure' clinic. He has been referred by the local the Emergency
Department following a single episode of a witnessed seizure. Which one of the following factors would be
least relevant when deciding whether to start anti-epileptic drugs after a single seizure?

Brain imaging shows a structural abnormality


The patient has a neurological deficit
The EEG shows unequivocal epileptic activity
The patient is less than 45 years old
The patient considers the risk of having a further seizure unacceptable

MRCP part 1 Made Easy‫‏‬‎‫‏‬by M. Habayeb & A. Murad

327
Chapter: Neurology

Question 105 of 246

A 24-year-old man is seen in the 'First Seizure' clinic. He has been referred by the local the Emergency
Department following a single episode of a witnessed seizure. Which one of the following factors would be
least relevant when deciding whether to start anti-epileptic drugs after a single seizure?

Brain imaging shows a structural abnormality


The patient has a neurological deficit
The EEG shows unequivocal epileptic activity
The patient is less than 45 years old
The patient considers the risk of having a further seizure unacceptable

Discuss and give feedback

Epilepsy: treatment

External links

NICE

2012 Epilepsy guidelines

Royal College of Physicians

2013 Modern management of epilepsy

MRCP part 1 Made Easy‫‏‬‎‫‏‬by M. Habayeb & A. Murad

328
Chapter: Neurology

Question 106 of 246

A 23-year-old man is admitted following the sudden onset of an occipital headache. On examination GCS is
15/15, neurological examination is unremarkable but neck stiffness is noted. A subarachnoid haemorrhage is
suspected but the CT scan is normal. At what time should a lumbar puncture be done to exclude the
diagnosis?

Immediately
2 hours post-headache
4 hours post-headache
12 hours post-headache
24 hours post-headache

MRCP part 1 Made Easy‫‏‬‎‫‏‬by M. Habayeb & A. Murad

329
Chapter: Neurology

Question 106 of 246

A 23-year-old man is admitted following the sudden onset of an occipital headache. On examination GCS is
15/15, neurological examination is unremarkable but neck stiffness is noted. A subarachnoid haemorrhage is
suspected but the CT scan is normal. At what time should a lumbar puncture be done to exclude the
diagnosis?

Immediately
2 hours post-headache
4 hours post-headache
12 hours post-headache
24 hours post-headache

A lumbar puncture should not be done until 12 hours after the onset of the headache to allow time for
xathochromia to develop.

SIGN guidelines state the following:


Subarachnoid blood degrades rapidly. Performing CT brain imaging as soon as possible maximises the
chance of accurate diagnosis. Even timely CT brain imaging may not pick up subarachnoid blood, so
lumbar puncture is also required. Lumbar puncture should be delayed till 12 hours after headache onset.

If the patient was acutely unwell or had an altered GCS then discussion with neurosurgery may be appropriate
rather than waiting 12 hours.
Discuss and give feedback

Subarachnoid haemorrhage

External links

SIGN

2008 Diagnosis and management of headache in adults

MRCP part 1 Made Easy‫‏‬‎‫‏‬by M. Habayeb & A. Murad

330
Chapter: Neurology

Question 107 of 246

A 24-year-old lady presents with a two-week history of frontal headache associated with blurred vision and
nausea. She has a history of asthma and acne. She uses a salbutamol inhaler as needed, a topical retinoid and
is halfway through a three-month course of doxycycline. On examination, she has no neurological deficits but
is noted to have bilateral gross papilloedema. Her body mass index is 26kg/m². A CT scan shows no
hydrocephalus or mass lesions.

Which of the following is the most likely cause of her symptoms?

Migraine
Retinoid use
Venous sinus thrombosis
Subarachnoid haemorrhage
Tetracycline use

MRCP part 1 Made Easy‫‏‬‎‫‏‬by M. Habayeb & A. Murad

331
Chapter: Neurology

Question 107 of 246


A 24-year-old lady presents with a two-week history of frontal headache associated with blurred vision and
nausea. She has a history of asthma and acne. She uses a salbutamol inhaler as needed, a topical retinoid and
is halfway through a three-month course of doxycycline. On examination, she has no neurological deficits but
is noted to have bilateral gross papilloedema. Her body mass index is 26kg/m². A CT scan shows no
hydrocephalus or mass lesions.

Which of the following is the most likely cause of her symptoms?

Migraine
Retinoid use
Venous sinus thrombosis
Subarachnoid haemorrhage
Tetracycline use

This patient has symptoms and signs consistent with idiopathic intracranial hypertension. This condition is
known to be associated with tetracycline use.

Explanation for other options:


 1. The finding of bilateral papilloedema is not consistent with a history of migraines.
 2. Retinoids are not associated with raised intracranial hypertension.
 3. Venous sinus thrombosis is a possible cause of these symptoms but would usually have some focal
neurological signs, and the history of tetracycline use makes idiopathic intracranial hypertension more
likely.
 4. Subarachnoid haemorrhage tends to present with a sudden onset 'thunderclap headache' rather than
this patient's more gradual symptoms.
Discuss and give feedback

Idiopathic intracranial hypertension


External links
Patient.info
Idiopathic intracranial hypertension review

MRCP part 1 Made Easy‫‏‬‎‫‏‬by M. Habayeb & A. Murad

332
Chapter: Neurology

Question 108 of 246

Which one of the following causes of peripheral neuropathy is most associated with demyelination?

Vasculitis
Alcohol
Hereditary sensorimotor neuropathies (HSMN) type I
Vitamin B12 deficiency
Diabetes mellitus

MRCP part 1 Made Easy‫‏‬‎‫‏‬by M. Habayeb & A. Murad

333
Chapter: Neurology

Question 108 of 246

Which one of the following causes of peripheral neuropathy is most associated with demyelination?

Vasculitis
Alcohol
Hereditary sensorimotor neuropathies (HSMN) type I
Vitamin B12 deficiency
Diabetes mellitus

Discuss and give feedback

Peripheral neuropathy: demyelinating vs. axonal

MRCP part 1 Made Easy‫‏‬‎‫‏‬by M. Habayeb & A. Murad

334
Chapter: Neurology

Question 109 of 246

A 19-year-old man is admitted following a generalised seizure. No past history is available as the man is
currently in a postictal state. On examination it is noted that he has three patches of hypopigmented skin and
fibromata under two of his finger nails. What is the most likely diagnosis?

Neurofibromatosis
Lennox-Gastaut Syndrome
Multiple endocrine neoplasia type 1
Birt-Hogg-Dube syndrome
Tuberous sclerosis

MRCP part 1 Made Easy‫‏‬‎‫‏‬by M. Habayeb & A. Murad

335
Chapter: Neurology

Question 109 of 246

A 19-year-old man is admitted following a generalised seizure. No past history is available as the man is
currently in a postictal state. On examination it is noted that he has three patches of hypopigmented skin and
fibromata under two of his finger nails. What is the most likely diagnosis?

Neurofibromatosis
Lennox-Gastaut Syndrome
Multiple endocrine neoplasia type 1
Birt-Hogg-Dube syndrome
Tuberous sclerosis

This man has a neurocutaneous syndrome which raises the possibility of neurofibromatosis or tuberous
sclerosis. Given the areas of hypopigmentation and subungual fibromas the most likely diagnosis is tuberous
sclerosis

Discuss and give feedback

Tuberous sclerosis

External links

Patient.info

Tuberous sclerosis review

MRCP part 1 Made Easy‫‏‬‎‫‏‬by M. Habayeb & A. Murad

336
Chapter: Neurology

Question 110 of 246

A 65-year-old gentleman presents with progressive dementia and behavioural abnormalities. On closer
questioning, he seems to have deficits in concentration, memory and judgement difficulties. There is a family
history of psychosis. During the consultation, you notice the patient's gait is ataxic with a notable jerk of his
left hand and general hypokinesia. There is also nystagmus during horizontal gaze. Romberg's and Dix-
Hallpike tests are negative. What is the most likely diagnosis?

Vascular dementia
Lewy body dementia
Sporadic Creutzfeldt-Jakob disease
Alzheimer's disease
Frontotemporal dementia

MRCP part 1 Made Easy‫‏‬‎‫‏‬by M. Habayeb & A. Murad

337
Chapter: Neurology

Question 110 of 246

A 65-year-old gentleman presents with progressive dementia and behavioural abnormalities. On closer
questioning, he seems to have deficits in concentration, memory and judgement difficulties. There is a family
history of psychosis. During the consultation, you notice the patient's gait is ataxic with a notable jerk of his
left hand and general hypokinesia. There is also nystagmus during horizontal gaze. Romberg's and Dix-
Hallpike tests are negative. What is the most likely diagnosis?

Vascular dementia
Lewy body dementia
Sporadic Creutzfeldt-Jakob disease
Alzheimer's disease
Frontotemporal dementia

The clue here is the 'notable jerk' found on examination consistent with myoclonus. Two cardinal features of
sporadic Creutzfeldt-Jakob disease (sCJD) are rapidly progressive mental deterioration and myoclonus
(usually provoked by startle). Extrapyramidal and cerebellar features occur in two-thirds of cases. The
question hints at a familial link which is present in 10-15% of cases, or a general family history of psychosis
(OR 9.9) has been shown to be associated with the disease.

Lewy body dementia, Alzheimer's disease, and vascular dementia would not account for the myoclonus nor
the cerebellar features. Frontotemporal dementia tends to present with a history of disinhibition, apathy, loss
of sympathy/empathy, compulsive behaviours and hyperorality.

Discuss and give feedback

Creutzfeldt-Jakob disease

MRCP part 1 Made Easy‫‏‬‎‫‏‬by M. Habayeb & A. Murad

338
Chapter: Neurology

Question 111 of 246

A 45-year-old woman presents complaining of visual disturbance. Examination reveals a left congruous
homonymous hemianopia. Where is the lesion most likely to be?

Optic chiasm
Left occipital cortex
Right optic tract
Right occipital cortex
Left optic tract

MRCP part 1 Made Easy‫‏‬‎‫‏‬by M. Habayeb & A. Murad

339
Chapter: Neurology

Question 111 of 246

A 45-year-old woman presents complaining of visual disturbance. Examination reveals a left congruous
homonymous hemianopia. Where is the lesion most likely to be?

Optic chiasm
Left occipital cortex
Right optic tract
Right occipital cortex
Left optic tract

Visual field defects:

 left homonymous hemianopia means visual field defect to the left, i.e. lesion of right optic tract

 homonymous quadrantanopias: PITS (Parietal-Inferior, Temporal-Superior)

 incongruous defects = optic tract lesion; congruous defects= optic radiation lesion or occipital cortex

Discuss and give feedback

Visual field defects

MRCP part 1 Made Easy‫‏‬‎‫‏‬by M. Habayeb & A. Murad

340
Chapter: Neurology

Question 112 of 246

A 5-year-old boy is diagnosed as having absence seizures. What is the chance he will be seizure free by the
age of 16-years-old?

5-10%
20-25%
40-45%
65-70%
90-95%

MRCP part 1 Made Easy‫‏‬‎‫‏‬by M. Habayeb & A. Murad

341
Chapter: Neurology

Question 112 of 246

A 5-year-old boy is diagnosed as having absence seizures. What is the chance he will be seizure free by the
age of 16-years-old?

5-10%
20-25%
40-45%
65-70%
90-95%

Absence seizures - good prognosis: 90-95% become seizure free in adolescence

Discuss and give feedback

Absence seizures

External links

NICE

2012 Epilepsy guidelines

MRCP part 1 Made Easy‫‏‬‎‫‏‬by M. Habayeb & A. Murad

342
Chapter: Neurology

Question 113 of 246

A 67-year-old man is reviewed in the neurology clinic due to concerns about increasing clumsiness.
Examination reveals an ataxic gait and increased upper limb tone with cog-wheel rigidity. Blood pressure is
135/80 lying and 95/70 standing. What is the most likely diagnosis?

Motor neuron disease


Progressive supranuclear palsy
Parkinson's disease
Multiple sclerosis
Multiple system atrophy

MRCP part 1 Made Easy‫‏‬‎‫‏‬by M. Habayeb & A. Murad

343
Chapter: Neurology

Question 113 of 246

A 67-year-old man is reviewed in the neurology clinic due to concerns about increasing clumsiness.
Examination reveals an ataxic gait and increased upper limb tone with cog-wheel rigidity. Blood pressure is
135/80 lying and 95/70 standing. What is the most likely diagnosis?

Motor neuron disease


Progressive supranuclear palsy
Parkinson's disease
Multiple sclerosis
Multiple system atrophy

Whilst postural hypotension may be seen in Parkinson's disease the ataxic gait point towards a diagnosis of
multiple system atrophy.

Discuss and give feedback

Multiple system atrophy

MRCP part 1 Made Easy‫‏‬‎‫‏‬by M. Habayeb & A. Murad

344
Chapter: Neurology

Question 114 of 246

A 66-year-old woman is investigated for ascites and found to have ovarian cancer. She presents due to
'unsteadiness'. On examination there is evidence of nystagmus and past-pointing. Which one of the following
antibodies is most likely to be present?

Anti-Hu
Anti-Yo
Anti-Ri
Anti-Ro
Anti-La

MRCP part 1 Made Easy‫‏‬‎‫‏‬by M. Habayeb & A. Murad

345
Chapter: Neurology

Question 114 of 246

A 66-year-old woman is investigated for ascites and found to have ovarian cancer. She presents due to
'unsteadiness'. On examination there is evidence of nystagmus and past-pointing. Which one of the following
antibodies is most likely to be present?

Anti-Hu
Anti-Yo
Anti-Ri
Anti-Ro
Anti-La

This lady has developed cerebellar syndrome secondary to anti-Yo antibodies.

Discuss and give feedback

Paraneoplastic syndromes affecting nervous system

MRCP part 1 Made Easy‫‏‬‎‫‏‬by M. Habayeb & A. Murad

346
Chapter: Neurology

Question 115 of 246

Which one of the following is least recognised as a cause of autonomic neuropathy?

Guillain-Barre syndrome
New variant CJD
Diabetes
Parkinson's
HIV

MRCP part 1 Made Easy‫‏‬‎‫‏‬by M. Habayeb & A. Murad

347
Chapter: Neurology

Question 115 of 246

Which one of the following is least recognised as a cause of autonomic neuropathy?

Guillain-Barre syndrome
New variant CJD
Diabetes
Parkinson's
HIV

Discuss and give feedback

Autonomic neuropathy

MRCP part 1 Made Easy‫‏‬‎‫‏‬by M. Habayeb & A. Murad

348
Chapter: Neurology

Question 116 of 246

A 29-year-old female presents complaining of double vision and unsteadiness. She has no past medical
history of note. On examination she has limited movement of her eyes in all directions. Pupils are 3 mm,
equal and reactive to light. Examination of the peripheral nervous system is normal other than reduced
reflexes and the plantars are down going. Some past-pointing is also noted. What is the most likely
diagnosis?

Multiple sclerosis
Conversion disorder
Miller Fisher syndrome
Ataxic telangiectasia
Friedreich's ataxia

MRCP part 1 Made Easy‫‏‬‎‫‏‬by M. Habayeb & A. Murad

349
Chapter: Neurology

Question 116 of 246

A 29-year-old female presents complaining of double vision and unsteadiness. She has no past medical
history of note. On examination she has limited movement of her eyes in all directions. Pupils are 3 mm,
equal and reactive to light. Examination of the peripheral nervous system is normal other than reduced
reflexes and the plantars are down going. Some past-pointing is also noted. What is the most likely
diagnosis?

Multiple sclerosis
Conversion disorder
Miller Fisher syndrome
Ataxic telangiectasia
Friedreich's ataxia

Miller Fisher syndrome - areflexia, ataxia, ophthalmoplegia

This patient has Miller Fisher syndrome, a variant of Guillain-Barre syndrome

Discuss and give feedback

Guillain-Barre syndrome

MRCP part 1 Made Easy‫‏‬‎‫‏‬by M. Habayeb & A. Murad

350
Chapter: Neurology

Question 117 of 246

In the treatment of migraine, sumatriptan is an example of a:

Beta-blocker
Alpha-blocker and a partial 5-HT2 agonist
Specific 5-HT1 agonist
5-HT2 antagonist
Tricyclic antidepressant

MRCP part 1 Made Easy‫‏‬‎‫‏‬by M. Habayeb & A. Murad

351
Chapter: Neurology

Question 117 of 246

In the treatment of migraine, sumatriptan is an example of a:

Beta-blocker
Alpha-blocker and a partial 5-HT2 agonist
Specific 5-HT1 agonist
5-HT2 antagonist
Tricyclic antidepressant

Migraine

 acute: triptan + NSAID or triptan + paracetamol

 prophylaxis: topiramate or propranolol

Discuss and give feedback

Migraine: management

It should be noted that as a general rule 5-HT receptor agonists are used in the acute treatment of migraine
whilst 5-HT receptor antagonists are used in prophylaxis. NICE produced guidelines in 2012 on the
management of headache, including migraines..

Acute treatment

 first-line: offer combination therapy with an oral triptan and an NSAID, or an oral triptan and
paracetamol

 for young people aged 12-17 years consider a nasal triptan in preference to an oral triptan

 if the above measures are not effective or not tolerated offer a non-oral preparation of
metoclopramide* or prochlorperazine and consider adding a non-oral NSAID or triptan.

MRCP part 1 Made Easy‫‏‬‎‫‏‬by M. Habayeb & A. Murad

352
Chapter: Neurology

Prophylaxis
 prophylaxis should be given if patients are experiencing 2 or more attacks per month. Modern
treatment is effective in about 60% of patients.
 NICE advise either topiramate or propranolol 'according to the person's preference, comorbidities and
risk of adverse events'. Propranolol should be used in preference to topiramate in women of child
bearing age as it may be teratogenic and it can reduce the effectiveness of hormonal contraceptives
 if these measures fail NICE recommend 'a course of up to 10 sessions of acupuncture over 5-8 weeks'
or gabapentin
 NICE recommend: 'Advise people with migraine that riboflavin (400 mg once a day) may be effective
in reducing migraine frequency and intensity for some people'
 for women with predictable menstrual migraine treatment NICE recommend either frovatriptan (2.5
mg twice a day) or zolmitriptan (2.5 mg twice or three times a day) as a type of 'mini-prophylaxis'
 pizotifen is no longer recommend. Adverse effects such as weight gain & drowsiness are common
*caution should be exercised with young patients as acute dystonic reactions may develop

External links

SIGN 2008 Diagnosis and management of headache in adults

Clinical Knowledge Summaries Migraine guidelines

NICE 2012 Headache guidelines

British Association for the Study of Headache Headache guidelines

External media

Migraine

Podmedics - YouTube

MRCP part 1 Made Easy‫‏‬‎‫‏‬by M. Habayeb & A. Murad

353
Chapter: Neurology

Question 118 of 246

A 34-year-old female is reviewed in the neurology clinic due to a number of 'funny-dos'. She describes a
sensation that her surroundings are unreal, 'like a dream'. Following this she has been told that she starts to
smack her lips, although she has no recollection of doing this. What is the most likely diagnosis?

Myoclonic seizure
Simple partial seizure
Complex partial seizure
Partial seizure progressing to generalised seizure
Absence seizure

MRCP part 1 Made Easy‫‏‬‎‫‏‬by M. Habayeb & A. Murad

354
Chapter: Neurology

Question 118 of 246

A 34-year-old female is reviewed in the neurology clinic due to a number of 'funny-dos'. She describes a
sensation that her surroundings are unreal, 'like a dream'. Following this she has been told that she starts to
smack her lips, although she has no recollection of doing this. What is the most likely diagnosis?

Myoclonic seizure
Simple partial seizure
Complex partial seizure
Partial seizure progressing to generalised seizure
Absence seizure

With simple partial seizures there is no disturbance of consciousness or awareness. Lip smacking is an
example of an automatism - an automatic, repetitive act

Discuss and give feedback

Epilepsy: classification

External links

NICE

2012 Epilepsy guidelines

MRCP part 1 Made Easy‫‏‬‎‫‏‬by M. Habayeb & A. Murad

355
Chapter: Neurology

Question 119 of 246

A 25-year-old female presents 5 days after discharge from hospital following an admission for suspected
meningitis. A lumbar puncture was performed which showed no evidence of infection. Unfortunately she
developed a headache 48 hours after discharge. This has now lasted 3 days and has failed to settle with
analgesia. Which one of the following treatment options should be considered?

Intrathecal steroids
Repeat lumbar puncture
Course of oral prednisolone
Blood patch
Intravenous fluids on top of oral fluid intake

MRCP part 1 Made Easy‫‏‬‎‫‏‬by M. Habayeb & A. Murad

356
Chapter: Neurology

Question 119 of 246

A 25-year-old female presents 5 days after discharge from hospital following an admission for suspected
meningitis. A lumbar puncture was performed which showed no evidence of infection. Unfortunately she
developed a headache 48 hours after discharge. This has now lasted 3 days and has failed to settle with
analgesia. Which one of the following treatment options should be considered?

Intrathecal steroids
Repeat lumbar puncture
Course of oral prednisolone
Blood patch
Intravenous fluids on top of oral fluid intake

Discuss and give feedback

Post-lumbar puncture headache

MRCP part 1 Made Easy‫‏‬‎‫‏‬by M. Habayeb & A. Murad

357
Chapter: Neurology

Question 120 of 246

An 18-year-old male gives a history of early morning jerking movements of his arm. After a night of heavy
drinking and sleep deprivation, he has a generalised tonic-clonic seizure at 5am. An EEG reveals generalised
spike and wave discharges. What is the most appropriate choice of anti-epileptic?

Carbamazepine
Ethosuximide
Sodium valproate
Gabapentin
Phenytoin

MRCP part 1 Made Easy‫‏‬‎‫‏‬by M. Habayeb & A. Murad

358
Chapter: Neurology

Question 120 of 246

An 18-year-old male gives a history of early morning jerking movements of his arm. After a night of heavy
drinking and sleep deprivation, he has a generalised tonic-clonic seizure at 5am. An EEG reveals generalised
spike and wave discharges. What is the most appropriate choice of anti-epileptic?

Carbamazepine
Ethosuximide
Sodium valproate
Gabapentin
Phenytoin

Juvenile myoclonic epilepsy (the diagnosis here), is a common form of idiopathic generalised epilepsy,
representing 10% of all patients with epilepsy.

This disorder typically first manifests itself between the ages of 10 and 18 with brief episodes of involuntary
muscle twitching occurring early in the morning. Most patients also have generalised seizures that affect the
entire brain and others have absence seizures. The most effective medication for this disorder is sodium
valproate.

Discuss and give feedback

Epilepsy in children: syndromes

MRCP part 1 Made Easy‫‏‬‎‫‏‬by M. Habayeb & A. Murad

359
Chapter: Neurology

Question 121 of 246

A 62-year-old man presents with left-sided eye pain and diplopia for the past 2 days. Examination of his eyes
shows his pupils equal and reactive to light with no proptosis. There is however an apparent palsy of the 6th
cranial nerve associated with a partial 3rd nerve palsy on the left side. Examining the remaining cranial
demonstrates hyperaesthesia of the upper face on the left side. Where is the likely lesion?

Cavernous sinus
Orbital apex
Pons
Cerebropontine angle
Medulla

MRCP part 1 Made Easy‫‏‬‎‫‏‬by M. Habayeb & A. Murad

360
Chapter: Neurology

Question 121 of 246

A 62-year-old man presents with left-sided eye pain and diplopia for the past 2 days. Examination of his eyes
shows his pupils equal and reactive to light with no proptosis. There is however an apparent palsy of the 6th
cranial nerve associated with a partial 3rd nerve palsy on the left side. Examining the remaining cranial
demonstrates hyperaesthesia of the upper face on the left side. Where is the likely lesion?

Cavernous sinus
Orbital apex
Pons
Cerebropontine angle
Medulla

Discuss and give feedback

Intracranial venous thrombosis

External links

Webmed Central

Diagram of types of intracranial venous thrombosis

MRCP part 1 Made Easy‫‏‬‎‫‏‬by M. Habayeb & A. Murad

361
Chapter: Neurology

Question 122 of 246

A 67-year-old male attends the Emergency department with sudden onset dizziness and vomiting, which has
been present for the past 2 hours. He has a background of hypertension and hypercholesterolaemia for which
he takes ramipril and simvastatin. Examination reveals vertical nystagmus and difficulty standing without
support. What is the most appropriate next step in this patient's management?

Prochlorperazine 12.5 mg IM
Arrange immediate for immediate thrombolysis
Urgent CT head
Sumatriptan 50 mg PO
Perform the Epley manoeuvre

MRCP part 1 Made Easy‫‏‬‎‫‏‬by M. Habayeb & A. Murad

362
Chapter: Neurology

Question 122 of 246

A 67-year-old male attends the Emergency department with sudden onset dizziness and vomiting, which has
been present for the past 2 hours. He has a background of hypertension and hypercholesterolaemia for which
he takes ramipril and simvastatin. Examination reveals vertical nystagmus and difficulty standing without
support. What is the most appropriate next step in this patient's management?

Prochlorperazine 12.5 mg IM
Arrange immediate for immediate thrombolysis
Urgent CT head
Sumatriptan 50 mg PO
Perform the Epley manoeuvre

This history is suggestive of a cerebellar stroke. Circulation to the cerebellum is impaired due to a lesion of
the superior cerebellar artery, anterior inferior cerebellar artery or the posterior inferior cerebellar artery (also
known as lateral medullary syndrome). The first step would be determine any evidence of cerebellar
haemorrhage, which would contraindicate thrombolysis. MRI will be better able to visualise cerebellar
infarction.

Cerebellar stroke may present in a similar fashion to vestibular neuritis. Clinically, vertical nystagmus is
suggestive of a central cause of vertigo. Additionally, patients usually cannot stand without support, even with
the eyes open, whereas a patient with acute vestibular neuritis is usually able to do so.

Benign paroxysmal positional vertigo (BPPV) is a very common cause of vertigo in older people. It is
characterised by severe, brief paroxysms of rotational vertigo provoked by positional changes. In contrast to
this case, vertigo usually lasts a few seconds to a minute (typically less than 30 seconds).

Migraine is a another common cause of vertigo, however given this patient's age and cardiovascular risk
factors, stroke is more likely and the important diagnosis not to miss.

Discuss and give feedback

Stroke: types

MRCP part 1 Made Easy‫‏‬‎‫‏‬by M. Habayeb & A. Murad

363
Chapter: Neurology

Question 123 of 246

A 29-year-old man with a history of schizophrenia is taken to the local Emergency Department as he is
generally unwell. He is currently taking olanzapine and citalopram. On examination he is noted to have a
temperature of 37.0ºC and his blood pressure is 170/100 mmHg. Which other examination finding would best
support a diagnosis of neuroleptic malignant syndrome?

Ataxia
Hyperreflexia
Muscle rigidity
Tremor
Papilloedema

MRCP part 1 Made Easy‫‏‬‎‫‏‬by M. Habayeb & A. Murad

364
Chapter: Neurology

Question 123 of 246

A 29-year-old man with a history of schizophrenia is taken to the local Emergency Department as he is
generally unwell. He is currently taking olanzapine and citalopram. On examination he is noted to have a
temperature of 37.0ºC and his blood pressure is 170/100 mmHg. Which other examination finding would
best support a diagnosis of neuroleptic malignant syndrome?

Ataxia
Hyperreflexia
Muscle rigidity
Tremor
Papilloedema
Discuss and give feedback

Neuroleptic malignant syndrome


Neuroleptic malignant syndrome is a rare but dangerous condition seen in patients taking antipsychotic
medication. It carries a mortality of up to 10% and can also occur with atypical antipsychotics. It may also
occur with dopaminergic drugs (such as levodopa) for Parkinson's disease, usually when the drug is suddenly
stopped or the dose reduced.
Features
 more common in young male patients
 onset usually in first 10 days of treatment or after increasing dose
 pyrexia
 rigidity
 tachycardia
A raised creatine kinase is present in most cases. A leukocytosis may also be seen.

Management:
 stop antipsychotic
 IV fluids to prevent renal failure
 dantrolene* may be useful in selected cases
 bromocriptine, dopamine agonist, may also be used

*thought to work by decreasing excitation-contraction coupling in skeletal muscle by binding to the ryanodine
receptor, and decreasing the release of calcium from the sarcoplasmic reticulum

External links
Patient.info
Neuroleptic Malignant Syndrome

The Mental Health Clinician


Differentiating MNS and SS

MRCP part 1 Made Easy‫‏‬‎‫‏‬by M. Habayeb & A. Murad

365
Chapter: Neurology

Question 124 of 246

Which one of the following side-effects is least associated with the use of levodopa?

Psychosis
'On-off' effect
Postural hypotension
Cardiac arrhythmias
Galactorrhoea

MRCP part 1 Made Easy‫‏‬‎‫‏‬by M. Habayeb & A. Murad

366
Chapter: Neurology

Question 124 of 246

Which one of the following side-effects is least associated with the use of levodopa?

Psychosis
'On-off' effect
Postural hypotension
Cardiac arrhythmias
Galactorrhoea

Discuss and give feedback

Levodopa

MRCP part 1 Made Easy‫‏‬‎‫‏‬by M. Habayeb & A. Murad

367
Chapter: Neurology

Question 125 of 246

Which one of the following is not a recognised causes of miosis?

Old age
Pontine haemorrhage
Holmes-Adie pupil
Argyll-Robertson pupil
Horner's syndrome

MRCP part 1 Made Easy‫‏‬‎‫‏‬by M. Habayeb & A. Murad

368
Chapter: Neurology

Question 125 of 246

Which one of the following is not a recognised causes of miosis?

Old age
Pontine haemorrhage
Holmes-Adie pupil
Argyll-Robertson pupil
Horner's syndrome

♦Holmes-Adie pupil is a benign condition most commonly seen in women. It is one of the differentials of a
dilated pupil

Discuss and give feedback

Miosis

MRCP part 1 Made Easy‫‏‬‎‫‏‬by M. Habayeb & A. Murad

369
Chapter: Neurology

Question 126 of 246

A 26-year-old male is admitted with sudden onset pain and weakness in both legs just after eating at a Chinese
restaurant.

He has no significant past medical history. He works on a construction site and on careful questioning admits
using cocaine several times a week.

Examination of the cranial nerves and upper limbs is normal. In the lower limbs his reflexes are present only
with reinforcement and the plantar response is ambiguous. There is an ascending pattern of weakness. There
is no impairment of light touch sensation or of proprioception.

A CT head scan is reported as normal, as is MRI imaging of the spine done 24 hours after admission normal.
He has a mildly raised CRP and a screen for anti-GM1 antibodies is negative.

Which of the following is the most likely diagnosis?

Prolapsed lumbar disc


Bacillary dysentery
Conversion disorder
Anterior spinal artery syndrome
Guillain-Barre syndrome

MRCP part 1 Made Easy‫‏‬‎‫‏‬by M. Habayeb & A. Murad

370
Chapter: Neurology

Question 126 of 246


A 26-year-old male is admitted with sudden onset pain and weakness in both legs just after eating at a Chinese
restaurant.

He has no significant past medical history. He works on a construction site and on careful questioning admits
using cocaine several times a week.

Examination of the cranial nerves and upper limbs is normal. In the lower limbs his reflexes are present only
with reinforcement and the plantar response is ambiguous. There is an ascending pattern of weakness. There
is no impairment of light touch sensation or of proprioception.

A CT head scan is reported as normal, as is MRI imaging of the spine done 24 hours after admission normal.
He has a mildly raised CRP and a screen for anti-GM1 antibodies is negative.

Which of the following is the most likely diagnosis?

Prolapsed lumbar disc


Bacillary dysentery
Conversion disorder
Anterior spinal artery syndrome
Guillain-Barre syndrome

Sudden onset weakness points to a vascular cause of neurological impairment. Cocaine and other drugs can
cause vasospasm which in this case appears to have affected his anterior spinal artery. Recall that during the
initial phase of a neurovascular insult, there may be an element of spinal shock which dampens the tendon
reflexes.
GBS has a much more sub-acute onset. Bacillary dysentery would not present like this.

Spinal cord lesions


External media

Spinal Pathways 1 - Spinal Cord Anatomy and Organisation

Handwritten Tutorials - YouTube

MRCP part 1 Made Easy‫‏‬‎‫‏‬by M. Habayeb & A. Murad

371
Chapter: Neurology

Spinal Pathways 4 - Corticospinal Tract

Handwritten Tutorials - YouTube00

Spinal Pathways 2 - Dorsal Column - Medial Lemniscus Pathway

Handwritten Tutorials - YouTube 00

Spinal Pathways 3 - Spinothalamic Tract

Handwritten Tutorials - YouTube

MRCP part 1 Made Easy‫‏‬‎‫‏‬by M. Habayeb & A. Murad

372
Chapter: Neurology

Question 127 of 246

Which of the following drugs is least likely to cause peripheral neuropathy?

Amiodarone
Vincristine
Trimethoprim
Isoniazid
Nitrofurantoin

MRCP part 1 Made Easy‫‏‬‎‫‏‬by M. Habayeb & A. Murad

373
Chapter: Neurology

Question 127 of 246

Which of the following drugs is least likely to cause peripheral neuropathy?

Amiodarone
Vincristine
Trimethoprim
Isoniazid
Nitrofurantoin

Trimethoprim is not listed in the BNF as causing peripheral neuropathy

Discuss and give feedback

Drugs causing peripheral neuropathy

MRCP part 1 Made Easy‫‏‬‎‫‏‬by M. Habayeb & A. Murad

374
Chapter: Neurology

Question 128 of 246

A 23-year-old man with difficult to control epilepsy is reviewed in clinic, four months after a change in his
antiepileptic medication. He has remained seizure free but has gained 5 kg in weight since last reviewed.
Which one of the following antiepileptic drugs is most associated with weight gain?

Ethosuximide
Sodium valproate
Levetiracetam
Carbamazepine
Lamotrigine

MRCP part 1 Made Easy‫‏‬‎‫‏‬by M. Habayeb & A. Murad

375
Chapter: Neurology

Question 128 of 246

A 23-year-old man with difficult to control epilepsy is reviewed in clinic, four months after a change in his
antiepileptic medication. He has remained seizure free but has gained 5 kg in weight since last reviewed.
Which one of the following antiepileptic drugs is most associated with weight gain?

Ethosuximide
Sodium valproate
Levetiracetam
Carbamazepine
Lamotrigine

Discuss and give feedback

Sodium valproate

Sodium valproate is used in the management of epilepsy and is first line therapy for generalised seizures. It
works by increasing GABA activity.

Adverse effects

 gastrointestinal: nausea

 increased appetite and weight gain

 alopecia: regrowth may be curly

 ataxia

 tremor

 hepatitis

 pancreatitis

 thromobcytopaenia

 teratogenic

 hyponatraemia

MRCP part 1 Made Easy‫‏‬‎‫‏‬by M. Habayeb & A. Murad

376
Chapter: Neurology

Question 129 of 246

A 48-year-old female with left sided pins and needles in her left thumb, index and middle finger for several
months, has been diagnosed with carpal tunnel syndrome. What features in her past medical history has
predisposed her to the condition?

High blood pressure


Prednisolone use
Obesity
Hormone replacement therapy
Previous contraceptive pill use

MRCP part 1 Made Easy‫‏‬‎‫‏‬by M. Habayeb & A. Murad

377
Chapter: Neurology

Question 129 of 246


A 48-year-old female with left sided pins and needles in her left thumb, index and middle finger for several
months, has been diagnosed with carpal tunnel syndrome. What features in her past medical history has
predisposed her to the condition?

High blood pressure


Prednisolone use
Obesity
Hormone replacement therapy
Previous contraceptive pill use

Obesity is a proven independent risk factor in those under 63 years. The others are not proven risk factors. ref:
Bland JD. The relationship of obesity, age, and carpal tunnel syndrome: more complex than was thought?
Muscle Nerve. 2005 Oct;32(4):527-32.

Prednisolone use, hormone replacement therapy and using the contraceptive pill are not proven risk factors
and no connection has been found between them and increased risk carpal tunnel syndrome. ref: Geoghegan
JM1, Clark DI, Bainbridge LC, Smith C, Hubbard R. Risk factors in carpal tunnel syndrome.J Hand Surg Br.
2004 Aug;29(4):315-20 and ref: American Academy of Orthopaedic Surgeons. Management of Carpal Tunnel
Syndrome Evidence-Based Clinical Practice Guideline. www.aaos.org/ctsguideline. Published February
29, 2016
Discuss and give feedback

Carpal tunnel syndrome


Carpal tunnel syndrome is caused by compression of median nerve in the carpal tunnel.

History
 pain/pins and needles in thumb, index, middle finger
 unusually the symptoms may 'ascend' proximally
 patient shakes his hand to obtain relief, classically at night
Examination
 weakness of thumb abduction (abductor pollicis brevis)

 wasting of thenar eminence (NOT hypothenar)

 Tinel's sign: tapping causes paraesthesia

 Phalen's sign: flexion of wrist causes symptoms

MRCP part 1 Made Easy‫‏‬‎‫‏‬by M. Habayeb & A. Murad

378
Chapter: Neurology

Causes

 idiopathic

 pregnancy

 oedema e.g. heart failure

 lunate fracture

 rheumatoid arthritis

Electrophysiology

 motor + sensory: prolongation of the action potential

Treatment:

 corticosteroid injection.

 wrist splints at night.

 surgical decompression (flexor retinaculum division).

External media

Carpal tunnel syndrome

Osmosis – YouTube

MRCP part 1 Made Easy‫‏‬‎‫‏‬by M. Habayeb & A. Murad

379
Chapter: Neurology

Question 130 of 246

A 15-year-old boy is reviewed. He has been referred by his GP with ptosis, diplopia and night blindness. On
examination he is noted to have a degree of ophthalmoplegia, bilateral partial ptosis and evidence of retinitis
pigmentosa during fundoscopy. His mother developed a similar problem when she was 18-years-old. What is
the most likely diagnosis?

Kearns-Sayre syndrome
Alport's syndrome
Usher syndrome
Refsum disease
Lawrence-Moon-Biedl syndrome

MRCP part 1 Made Easy‫‏‬‎‫‏‬by M. Habayeb & A. Murad

380
Chapter: Neurology

Question 130 of 246


A 15-year-old boy is reviewed. He has been referred by his GP with ptosis, diplopia and night blindness. On
examination he is noted to have a degree of ophthalmoplegia, bilateral partial ptosis and evidence of retinitis
pigmentosa during fundoscopy. His mother developed a similar problem when she was 18-years-old. What is
the most likely diagnosis?

Kearns-Sayre syndrome
Alport's syndrome
Usher syndrome
Refsum disease
Lawrence-Moon-Biedl syndrome
Kearns-Sayre syndrome
 mitochondrial inheritance
 onset < 20-years-old
 external ophthalmoplegia
 retinitis pigmentosa

Mitochondrial diseases
Whilst most DNA is found in the cell nucleus, a small amount of double-stranded DNA is present in the
mitochondria. It encodes protein components of the respiratory chain and some special types of RNA
Mitochondrial inheritance has the following characteristics:
 inheritance is only via the maternal line as the sperm contributes no cytoplasm to the zygote
 all children of affected males will not inherit the disease
 all children of affected females will inherit it
 generally encode rare neurological diseases
 poor genotype:phenotype correlation - within a tissue or cell there can be different mitochondrial
populations - this is known as heteroplasmy)

Histology
 muscle biopsy classically shows 'red, ragged fibres' due to increased number of mitochondria

Examples include:
 Leber's optic atrophy
 MELAS syndrome: mitochondrial encephalomyopathy lactic acidosis and stroke-like episodes
 MERRF syndrome: myoclonus epilepsy with ragged-red fibres
 Kearns-Sayre syndrome: onset in patients < 20 years old, external ophthalmoplegia, retinitis
pigmentosa. Ptosis may be seen
 sensorineural hearing loss

External links
Royal College of Physicians
2013 How to spot mitochondrial disease in adults

MRCP part 1 Made Easy‫‏‬‎‫‏‬by M. Habayeb & A. Murad

381
Chapter: Neurology

Question 131 of 246

A 34-year-old female with a history of primary generalised epilepsy asks for advice in the neurology clinic as
she plans to start a family. She currently takes sodium valproate as monotherapy. What advice should be
given regarding the prevention of neural tube defects?

Folic acid 400 mcg per day once pregnancy has been confirmed
Folic acid 1 mg per day once pregnancy has been confirmed
Folic acid 5 mg per day starting now
Folic acid 10 mg per day starting now
Folic acid 400 mcg per day starting now

MRCP part 1 Made Easy‫‏‬‎‫‏‬by M. Habayeb & A. Murad

382
Chapter: Neurology

Question 131 of 246

A 34-year-old female with a history of primary generalised epilepsy asks for advice in the neurology clinic as
she plans to start a family. She currently takes sodium valproate as monotherapy. What advice should be
given regarding the prevention of neural tube defects?

Folic acid 400 mcg per day once pregnancy has been confirmed
Folic acid 1 mg per day once pregnancy has been confirmed
Folic acid 5 mg per day starting now
Folic acid 10 mg per day starting now
Folic acid 400 mcg per day starting now

Epilepsy + pregnancy = 5mg folic acid

Discuss and give feedback

Epilepsy: pregnancy and breast feeding

External links

NICE

2012 Epilepsy guidelines

SIGN

2015 Diagnosis and management of epilepsy in adults

MRCP part 1 Made Easy‫‏‬‎‫‏‬by M. Habayeb & A. Murad

383
Chapter: Neurology

Question 132 of 246

Which one of the following is least likely to cause a raised protein level in the cerebrospinal fluid?

Tuberculous meningitis
Guillain-Barre syndrome
Fungal meningitis
Spinal block
Systemic lupus erythematous

MRCP part 1 Made Easy‫‏‬‎‫‏‬by M. Habayeb & A. Murad

384
Chapter: Neurology

Question 132 of 246

Which one of the following is least likely to cause a raised protein level in the cerebrospinal fluid?

Tuberculous meningitis
Guillain-Barre syndrome
Fungal meningitis
Spinal block
Systemic lupus erythematous

Discuss and give feedback

Cerebrospinal fluid: raised protein

MRCP part 1 Made Easy‫‏‬‎‫‏‬by M. Habayeb & A. Murad

385
Chapter: Neurology

Question 133 of 246

A 52-year-old gentleman with no past medical history presents to the emergency department accompanied by
his wife. His wife reported finding him confused earlier in the day. Repetitive questioning was a prominent
feature. He was able to recognise his wife and correctly reported his address and date of birth to paramedics.
On arrival to the emergency department, he had recovered and was back to his normal self. The event lasted
approximately 3 hours. A full neurological examination was normal. What is the most likely diagnosis?

Epilepsy
Transient psychogenic amnesia
Transient global amnesia
Brain tumour
Transient ischaemic attack

MRCP part 1 Made Easy‫‏‬‎‫‏‬by M. Habayeb & A. Murad

386
Chapter: Neurology

Question 133 of 246

A 52-year-old gentleman with no past medical history presents to the emergency department accompanied by
his wife. His wife reported finding him confused earlier in the day. Repetitive questioning was a prominent
feature. He was able to recognise his wife and correctly reported his address and date of birth to paramedics.
On arrival to the emergency department, he had recovered and was back to his normal self. The event lasted
approximately 3 hours. A full neurological examination was normal. What is the most likely diagnosis?

Epilepsy
Transient psychogenic amnesia
Transient global amnesia
Brain tumour
Transient ischaemic attack

♦Transient global amnesia is a clinical syndrome of uncertain aetiology, characterised by: a discrete
episode lasting for a few hours (always less than 24 hours) of anterograde amnesia, retrograde amnesia,
repetitive questioning with an absence of other cognitive or neurological impairments.

♦Diagnostic criteria (in addition to the above features) are as follows; reliable witness to episode, the
absence of head trauma or loss of consciousness at the onset, preserved personal identity and absence of
epileptic features.

♦Epilepsy can present with discreet episodes of amnesia. This syndrome is called transient epileptic
amnesia. Features that suggest epilepsy are; shorter duration (should be less than 1 hour), multiple attacks,
onset on waking from sleep and accompanying epileptic features - e.g. motor automatism, stereotyped
behaviours, limb shaking.

♦There are a small number of case reports where a brain tumour has been implicated in transient global
amnesia, however, this is very rare. A transient ischaemic attack can present with isolated amnesia, however,
this again is exceedingly rare.

♦Distinguishing transient global amnesia from transient psychogenic amnesia can be very difficult, however,
transient global amnesia is the most likely diagnosis in this case due to the clinical features conforming to the
classical description of transient global amnesia so closely.

Discuss and give feedback

Transient global amnesia

MRCP part 1 Made Easy‫‏‬‎‫‏‬by M. Habayeb & A. Murad

387
Chapter: Neurology

Question 134 of 246

Which one of the following features is most associated with parietal lobe lesions?

Gerstmann's syndrome
Perseveration
Cortical blindness
Superior homonymous quadrantanopia
Wernicke's aphasia

MRCP part 1 Made Easy‫‏‬‎‫‏‬by M. Habayeb & A. Murad

388
Chapter: Neurology

Question 134 of 246

Which one of the following features is most associated with parietal lobe lesions?

Gerstmann's syndrome
Perseveration
Cortical blindness
Superior homonymous quadrantanopia
Wernicke's aphasia

Discuss and give feedback

Brain lesions

MRCP part 1 Made Easy‫‏‬‎‫‏‬by M. Habayeb & A. Murad

389
Chapter: Neurology

Question 135 of 246

You are asked to perform a neurological exam of the lower limbs on a patient with multiple sclerosis. Which
one of the following findings is least typical?

Decreased tone
Patellar clonus
Upgoing plantars
Weakness
Brisk reflexes

MRCP part 1 Made Easy‫‏‬‎‫‏‬by M. Habayeb & A. Murad

390
Chapter: Neurology

Question 135 of 246

You are asked to perform a neurological exam of the lower limbs on a patient with multiple sclerosis. Which
one of the following findings is least typical?

Decreased tone
Patellar clonus
Upgoing plantars
Weakness
Brisk reflexes

In multiple sclerosis there is demyelination of the central nervous system and hence upper motor neuron signs
are seen

Discuss and give feedback

Multiple sclerosis: features

External links

NICE

2014 Multiple sclerosis guidelines

MRCP part 1 Made Easy‫‏‬‎‫‏‬by M. Habayeb & A. Murad

391
Chapter: Neurology

Question 136 of 246

Which of the following is least associated with Parkinsonism?

Chlorpromazine
Progressive supranuclear palsy
Dementia pugilistica
Lead poisoning
Wilson's disease

MRCP part 1 Made Easy‫‏‬‎‫‏‬by M. Habayeb & A. Murad

392
Chapter: Neurology

Question 136 of 246

Which of the following is least associated with Parkinsonism?

Chlorpromazine
Progressive supranuclear palsy
Dementia pugilistica
Lead poisoning
Wilson's disease

Discuss and give feedback

Parkinsonism

MRCP part 1 Made Easy‫‏‬‎‫‏‬by M. Habayeb & A. Murad

393
Chapter: Neurology

Question 137 of 246

You review a 70-year-old woman four days after she was admitted with a suspected stroke. Unfortunately she
has been left with right sided sensory loss affecting her arms more than the legs and a right sided
homonymous hemianopia. Functionally she has difficulty dressing her self. Examination of her cranial nerves
is unremarkable. What area is the stroke most likely to have affected?

Middle cerebral artery


Lacunar
Anterior cerebral artery
Posterior cerebral artery
Posterior inferior cerebellar artery

MRCP part 1 Made Easy‫‏‬‎‫‏‬by M. Habayeb & A. Murad

394
Chapter: Neurology

Question 137 of 246

You review a 70-year-old woman four days after she was admitted with a suspected stroke. Unfortunately she
has been left with right sided sensory loss affecting her arms more than the legs and a right sided
homonymous hemianopia. Functionally she has difficulty dressing her self. Examination of her cranial nerves
is unremarkable. What area is the stroke most likely to have affected?

Middle cerebral artery


Lacunar
Anterior cerebral artery
Posterior cerebral artery
Posterior inferior cerebellar artery

Discuss and give feedback

Stroke by anatomy

External media

Stroke

Podmedics – YouTube

MRCP part 1 Made Easy‫‏‬‎‫‏‬by M. Habayeb & A. Murad

395
Chapter: Neurology

Question 138 of 246

A 45-year-old male with a history of alcoholic liver disease presents with increasing confusion. On
assessment, he is noted to be obtunded with a Glasgow coma scale of 14 out of 15, has a wide-based gait and
nystagmus. There is no history or signs of recent trauma.

What is the most specific finding on brain MRI for this patient's condition?

Left-sided subdural haematoma


Generalised cerebral atrophy
Hydrocephalus
Enhancement of the mamillary bodies
Right-sided cerebellar infarction

MRCP part 1 Made Easy‫‏‬‎‫‏‬by M. Habayeb & A. Murad

396
Chapter: Neurology

Question 138 of 246

A 45-year-old male with a history of alcoholic liver disease presents with increasing confusion. On
assessment, he is noted to be obtunded with a Glasgow coma scale of 14 out of 15, has a wide-based gait and
nystagmus. There is no history or signs of recent trauma.

What is the most specific finding on brain MRI for this patient's condition?

Left-sided subdural haematoma


Generalised cerebral atrophy
Hydrocephalus
Enhancement of the mamillary bodies
Right-sided cerebellar infarction

This patient is suffering from Wernicke's encephalopathy as indicated by the history of alcoholic liver disease
with confusion, ataxia and ophthalmoplegia. The MRI finding of enhancement of the mamillary bodies due to
petechial haemorrhages is specific for Wernicke's encephalopathy, although sensitivity is only 50%.

Explanation for other options:

 1. A spontaneous subdural haematoma is a common cause of neurological deterioration in alcoholic


patients but this patient's signs are more suggestive of Wernicke's rather than a localising brain lesion.

 2. This would likely also be present but would not account for the acute presentation.

 3. There is nothing in the history to suggest a cause of hydrocephalus.

 5. This would cause ataxia and nystagmus but would be more likely to present with lateralizing signs
and would not be associated with acute confusion.

Discuss and give feedback

Wernicke's encephalopathy
Wernicke's encephalopathy is a neuropsychiatric disorder caused by thiamine deficiency which is most
commonly seen in alcoholics. Rarer causes include: persistent vomiting, stomach cancer, dietary deficiency. A
classic triad of ophthalmoplegia/nystagmus, ataxia and confusion may occur. In Wernicke's encephalopathy
petechial haemorrhages occur in a variety of structures in the brain including the mamillary bodies and
ventricle walls

MRCP part 1 Made Easy‫‏‬‎‫‏‬by M. Habayeb & A. Murad

397
Chapter: Neurology

Features
•nystagmus (the most common ocular sign)
 ophthalmoplegia
 ataxia
 confusion, altered GCS
 peripheral sensory neuropathy

Investigations
 decreased red cell transketolase
 MRI

Treatment is with urgent replacement of thiamine


Relationship with Korsakoff syndrome

If not treated Korsakoff's syndrome may develop as well. This is termed Wernicke-Korsakoff syndrome and is
characterised by the addition of antero- and retrograde amnesia and confabulation in addition to the above
symptoms.

MRCP part 1 Made Easy‫‏‬‎‫‏‬by M. Habayeb & A. Murad

398
Chapter: Neurology

Question 139 of 246

A 28-year-old woman with a history of systemic lupus erythematosus (SLE) presents with jerky, irregular
movements which seem to move from one limb to another. Her symptoms are continuous and there are no
other neurological features such as impairement of consciousness. Where is the neurological lesion most
likely to be?

Cerebellum
Temporal lobe
Hippocampus
Thalamus
Caudate nucleus

MRCP part 1 Made Easy‫‏‬‎‫‏‬by M. Habayeb & A. Murad

399
Chapter: Neurology

Question 139 of 246

A 28-year-old woman with a history of systemic lupus erythematosus (SLE) presents with jerky, irregular
movements which seem to move from one limb to another. Her symptoms are continuous and there are no
other neurological features such as impairement of consciousness. Where is the neurological lesion most
likely to be?

Cerebellum
Temporal lobe
Hippocampus
Thalamus
Caudate nucleus

Chorea is caused by damage to the basal ganglia, in particular the Caudate nucleus

These symptoms are consistent with chorea, which may have a number of causes including SLE. Temporal
lobe epilepsy would not cause continuous symptoms.

Discuss and give feedback

Chorea

External links

Postgraduate Medical Journal

Review of chorea

MRCP part 1 Made Easy‫‏‬‎‫‏‬by M. Habayeb & A. Murad

400
Chapter: Neurology

Question 140 of 246

Which one of the following antibodies are associated with painful sensory neuropathy in patients with
small cell lung cancer?

Anti-Ri
Anti-GAD
Anti-Ro
Anti-Hu
Anti-Yo

MRCP part 1 Made Easy‫‏‬‎‫‏‬by M. Habayeb & A. Murad

401
Chapter: Neurology

Question 140 of 246

Which one of the following antibodies are associated with painful sensory neuropathy in patients with
small cell lung cancer?

Anti-Ri
Anti-GAD
Anti-Ro
Anti-Hu
Anti-Yo

Discuss and give feedback

Paraneoplastic syndromes affecting nervous system

MRCP part 1 Made Easy‫‏‬‎‫‏‬by M. Habayeb & A. Murad

402
Chapter: Neurology

Question 141 of 246

A 68-year-old woman presents with a two month history of electric shock like pains on the right side of her
face. She describes having around 10-20 episodes a day which, each lasting for around 30-60 seconds. A
recent dental check was normal. Neurological examination is unremarkable. What is the most suitable first-
line management?

Amitriptyline
Sodium valproate
Carbamazepine
Atenolol
Zolmitriptan

MRCP part 1 Made Easy‫‏‬‎‫‏‬by M. Habayeb & A. Murad

403
Chapter: Neurology

Question 141 of 246

A 68-year-old woman presents with a two month history of electric shock like pains on the right side of her
face. She describes having around 10-20 episodes a day which, each lasting for around 30-60 seconds. A
recent dental check was normal. Neurological examination is unremarkable. What is the most suitable first-
line management?

Amitriptyline
Sodium valproate
Carbamazepine
Atenolol
Zolmitriptan

Trigeminal neuralgia - carbamazepine is first-line

Discuss and give feedback

Trigeminal neuralgia

External links

Clinical Knowledge Summaries

Trigeminal Neuralgia guidelines

MRCP part 1 Made Easy‫‏‬‎‫‏‬by M. Habayeb & A. Murad

404
Chapter: Neurology

Question 142 of 246

How long should a patient stop driving for following a stroke?

No restriction unless physical/visual impairment


1 month
3 month
6 months
12 months

MRCP part 1 Made Easy‫‏‬‎‫‏‬by M. Habayeb & A. Murad

405
Chapter: Neurology

Question 142 of 246

How long should a patient stop driving for following a stroke?

No restriction unless physical/visual impairment


1 month
3 month
6 months
12 months

DVLA advice post CVA: cannot drive for 1 month

Discuss and give feedback

DVLA: neurological disorders

External links

DVLA

Fitness to drive guidelines

MRCP part 1 Made Easy‫‏‬‎‫‏‬by M. Habayeb & A. Murad

406
Chapter: Neurology

Question 143 of 246

A 60-year-old man is diagnosed with Bell's palsy. What is the current evidenced base approach to the
management of this condition?

Refer for urgent surgical decompression


Aciclovir
No treatment
Aciclovir + prednisolone
Prednisolone

MRCP part 1 Made Easy‫‏‬‎‫‏‬by M. Habayeb & A. Murad

407
Chapter: Neurology

Question 143 of 246

A 60-year-old man is diagnosed with Bell's palsy. What is the current evidenced base approach to the
management of this condition?

Refer for urgent surgical decompression


Aciclovir
No treatment
Aciclovir + prednisolone
Prednisolone

Discuss and give feedback

Bell's palsy

MRCP part 1 Made Easy‫‏‬‎‫‏‬by M. Habayeb & A. Murad

408
Chapter: Neurology

Question 144 of 246

Which one of the following is most associated with a good prognosis in Guillain-Barre syndrome?

Age > 40 years


Female sex
No history of a diarrhoeal illness
High anti-GM1 antibody titre
Low peak expiratory flow rate

MRCP part 1 Made Easy‫‏‬‎‫‏‬by M. Habayeb & A. Murad

409
Chapter: Neurology

Question 144 of 246

Which one of the following is most associated with a good prognosis in Guillain-Barre syndrome?

Age > 40 years


Female sex
No history of a diarrhoeal illness
High anti-GM1 antibody titre
Low peak expiratory flow rate

Preceding gastrointestinal infections are associated with a poor prognosis in Guillain-Barre syndrome. The
sex of the patient has not been shown to correlate with outcome

Discuss and give feedback

Guillain-Barre syndrome: prognosis

MRCP part 1 Made Easy‫‏‬‎‫‏‬by M. Habayeb & A. Murad

410
Chapter: Neurology

Question 145 of 246

A 19-year-old man is diagnosed as having myoclonic seizures. What is the most appropriate first-line
antiepileptic?

Sodium valproate
Carbamazepine
Topiramate
Clonazepam
Ethosuximide

MRCP part 1 Made Easy‫‏‬‎‫‏‬by M. Habayeb & A. Murad

411
Chapter: Neurology

Question 145 of 246

A 19-year-old man is diagnosed as having myoclonic seizures. What is the most appropriate first-line
antiepileptic?

Sodium valproate
Carbamazepine
Topiramate
Clonazepam
Ethosuximide

Discuss and give feedback

Epilepsy: treatment

External links

NICE

2012 Epilepsy guidelines

Royal College of Physicians

2013 Modern management of epilepsy

MRCP part 1 Made Easy‫‏‬‎‫‏‬by M. Habayeb & A. Murad

412
Chapter: Neurology

Question 146 of 246

A 27-year-old woman is reviewed due to sudden loss of vision in her left eye. She is known to have severe
rheumatoid arthritis and is treated currently with methotrexate, infliximab and prednisolone. She has in the
past also used sulfasalazine and hydroxychloroquine. For the past 6 weeks she has developed troublesome
headaches. Examination demonstrates bilateral papilloedema. Which one of the following is most likely to
be responsible for this presentation?

Chloroquine retinopathy
Prednisolone
Infliximab
Methotrexate
Keratoconjunctivitis sicca

MRCP part 1 Made Easy‫‏‬‎‫‏‬by M. Habayeb & A. Murad

413
Chapter: Neurology

Question 146 of 246

A 27-year-old woman is reviewed due to sudden loss of vision in her left eye. She is known to have severe
rheumatoid arthritis and is treated currently with methotrexate, infliximab and prednisolone. She has in the
past also used sulfasalazine and hydroxychloroquine. For the past 6 weeks she has developed troublesome
headaches. Examination demonstrates bilateral papilloedema. Which one of the following is most likely to
be responsible for this presentation?

Chloroquine retinopathy
Prednisolone
Infliximab
Methotrexate
Keratoconjunctivitis sicca

This patient has developed intracranial hypertension probably secondary to prednisolone. Patients may lose
sight suddenly if the optic nerve becomes compressed

Discuss and give feedback

Idiopathic intracranial hypertension

External links

Patient.info

Idiopathic intracranial hypertension review

MRCP part 1 Made Easy‫‏‬‎‫‏‬by M. Habayeb & A. Murad

414
Chapter: Neurology

Question 147 of 246

A man is recovering after having an operation to remove a meningioma in his left temporal lobe. What sort
of visual field defect is he at risk of having following the procedure?

Right inferior homonymous quadrantanopia


Right superior homonymous quadrantanopia
Left inferior homonymous quadrantanopia
Right homonymous hemianopia with macula sparing
Left superior homonymous quadrantanopia

MRCP part 1 Made Easy‫‏‬‎‫‏‬by M. Habayeb & A. Murad

415
Chapter: Neurology

Question 147 of 246

A man is recovering after having an operation to remove a meningioma in his left temporal lobe. What sort
of visual field defect is he at risk of having following the procedure?

Right inferior homonymous quadrantanopia


Right superior homonymous quadrantanopia
Left inferior homonymous quadrantanopia
Right homonymous hemianopia with macula sparing
Left superior homonymous quadrantanopia

Visual field defects:

 left homonymous hemianopia means visual field defect to the left, i.e. lesion of right optic tract
 homonymous quadrantanopias: PITS (Parietal-Inferior, Temporal-Superior)
 incongruous defects = optic tract lesion; congruous defects= optic radiation lesion or occipital cortex

Discuss and give feedback

Visual field defects

MRCP part 1 Made Easy‫‏‬‎‫‏‬by M. Habayeb & A. Murad

416
Chapter: Neurology

Question 148 of 246

A 45-year-old male presents to the acute medical take with a three day history of progressive bilateral leg
weakness. Which of the following constellation of signs on examination is the most consistent with a
diagnosis of Guillain-Barre Syndrome?

Bradycardia with lower limb hyporeflexia and flaccid paralysis


Tachycardia with lower limb hyporeflexia and spastic paralysis
Bradycardia with lower limb hyperreflexia and spastic paralysis
Tachycardia with lower limb hyporeflexia and flaccid paralysis
Tachycardia with lower limb hyperreflexia and flaccid paralysis

MRCP part 1 Made Easy‫‏‬‎‫‏‬by M. Habayeb & A. Murad

417
Chapter: Neurology

Question 148 of 246

A 45-year-old male presents to the acute medical take with a three day history of progressive bilateral leg
weakness. Which of the following constellation of signs on examination is the most consistent with a
diagnosis of Guillain-Barre Syndrome?

Bradycardia with lower limb hyporeflexia and flaccid paralysis


Tachycardia with lower limb hyporeflexia and spastic paralysis
Bradycardia with lower limb hyperreflexia and spastic paralysis
Tachycardia with lower limb hyporeflexia and flaccid paralysis
Tachycardia with lower limb hyperreflexia and flaccid paralysis

♦Absent or depressed deep tendon reflexes are classical findings in Guillain-Barre Syndrome (GBS).
Hyperreflexia is seen in a GBS variant known as Bickerstaff's encephalitis.

♦The paralysis in GBS is flaccid.

♦Autonomic symptoms are common in GBS. The most frequently encountered are tachycardia and urinary
retention. Although autonomic dysfunction may manifest as hypertension, hypotension, bradycardia, or ileus,
these are not as commonly seen.

Discuss and give feedback

Guillain-Barre syndrome: features

External links

Patient.info

Guillain-Barre syndrome review

MRCP part 1 Made Easy‫‏‬‎‫‏‬by M. Habayeb & A. Murad

418
Chapter: Neurology

Question 149 of 246

A 63-year-old man is prescribed selegiline for Parkinson's disease. What is the mechanism of action?

Dopamine receptor antagonist


Dopamine receptor agonist
Monoamine Oxidase-B inhibitor
Antimuscarinic
Catechol-O-Methyl Transferase inhibitor

MRCP part 1 Made Easy‫‏‬‎‫‏‬by M. Habayeb & A. Murad

419
Chapter: Neurology

Question 149 of 246

A 63-year-old man is prescribed selegiline for Parkinson's disease. What is the mechanism of action?

Dopamine receptor antagonist


Dopamine receptor agonist
Monoamine Oxidase-B inhibitor
Antimuscarinic
Catechol-O-Methyl Transferase inhibitor

Discuss and give feedback

Parkinson's disease: management

MRCP part 1 Made Easy‫‏‬‎‫‏‬by M. Habayeb & A. Murad

420
Chapter: Neurology

Question 150 of 246

Which one of the following is most associated with downbeat nystagmus?

Arnold-Chiari malformation
Pseudobulbar palsy
Jugular foramen syndrome
Acoustic neuroma
Cerebellar vermis lesions

MRCP part 1 Made Easy‫‏‬‎‫‏‬by M. Habayeb & A. Murad

421
Chapter: Neurology

Question 150 of 246

Which one of the following is most associated with downbeat nystagmus?

Arnold-Chiari malformation
Pseudobulbar palsy
Jugular foramen syndrome
Acoustic neuroma
Cerebellar vermis lesions

Discuss and give feedback

Nystagmus

MRCP part 1 Made Easy‫‏‬‎‫‏‬by M. Habayeb & A. Murad

422
Chapter: Neurology

Question 151 of 246

A 31-year-old female with progressive leg weakness has nerve conduction studies for suspected Guillain-
Barre syndrome. Which one of the following findings would be most consistent with this diagnosis?

Reduced conduction velocity


Extended series of repetitive discharges lasting up to 30 seconds
Increased conduction velocity
Diminished response to repetitive stimulation
Reduced wave amplitude

MRCP part 1 Made Easy‫‏‬‎‫‏‬by M. Habayeb & A. Murad

423
Chapter: Neurology

Question 151 of 246

A 31-year-old female with progressive leg weakness has nerve conduction studies for suspected Guillain-
Barre syndrome. Which one of the following findings would be most consistent with this diagnosis?

Reduced conduction velocity


Extended series of repetitive discharges lasting up to 30 seconds
Increased conduction velocity
Diminished response to repetitive stimulation
Reduced wave amplitude

Slowing of the nerve conduction velocity usually indicates there is damage to the myelin sheath, as in
Guillain-Barre syndrome

Discuss and give feedback

Nerve conduction studies

External links

The Bone School

Nerve Conduction Studies / EMG

MRCP part 1 Made Easy‫‏‬‎‫‏‬by M. Habayeb & A. Murad

424
Chapter: Neurology

Question 152 of 246

A 34-year-old woman who presents with confusion, headache and fever is admitted to the Emergency
Department. Shortly after admission she has a seizure. A MRI scan is performed which shows patchy
haemorrhagic changes in the temporal lobe. Given the likely diagnosis, what is the treatment of choice?

Supportive treatment + intravenous cefotaxime


Supportive treatment + intravenous aciclovir
Supportive treatment + intravenous amphotericin B
Supportive treatment alone
Supportive treatment + intravenous immunoglobulin

MRCP part 1 Made Easy‫‏‬‎‫‏‬by M. Habayeb & A. Murad

425
Chapter: Neurology

Question 152 of 246

A 34-year-old woman who presents with confusion, headache and fever is admitted to the Emergency
Department. Shortly after admission she has a seizure. A MRI scan is performed which shows patchy
haemorrhagic changes in the temporal lobe. Given the likely diagnosis, what is the treatment of choice?

Supportive treatment + intravenous cefotaxime


Supportive treatment + intravenous aciclovir
Supportive treatment + intravenous amphotericin B
Supportive treatment alone
Supportive treatment + intravenous immunoglobulin

Discuss and give feedback

Herpes simplex encephalitis

MRCP part 1 Made Easy‫‏‬‎‫‏‬by M. Habayeb & A. Murad

426
Chapter: Neurology

Question 153 of 246

A 52-year-old man is reviewed in the neurology clinic. He has been referred due to the development of
difficultly in finding the right words whilst speaking. His comprehension of normal conversation has however
remained normal. Where is the likely lesion?

Anterior temporal lobe


Posterior temporal lobe
Parietal lobe
Posterior frontal lobe
Anterior frontal lobe

MRCP part 1 Made Easy‫‏‬‎‫‏‬by M. Habayeb & A. Murad

427
Chapter: Neurology

Question 153 of 246

A 52-year-old man is reviewed in the neurology clinic. He has been referred due to the development of
difficultly in finding the right words whilst speaking. His comprehension of normal conversation has however
remained normal. Where is the likely lesion?

Anterior temporal lobe


Posterior temporal lobe
Parietal lobe
Posterior frontal lobe
Anterior frontal lobe

This man has expressive aphasia due to a lesion in Broca's area, located on the posterior aspect of the frontal
lobe, in the inferior frontal gyrus

Discuss and give feedback

Brain lesions

MRCP part 1 Made Easy‫‏‬‎‫‏‬by M. Habayeb & A. Murad

428
Chapter: Neurology

Question 154 of 246

A 27 year old male with polycystic kidney disease presents with sudden onset headache and collapse. On
admission to emergency department his blood pressure 190/105 mmHg, tachycardic with a Glasgow Coma
Score of 7/15. He is intubated and transferred for a CT scan. The scan reveals a subarachnoid haemorrhage.
He is transferred to the critical care unit for monitoring. Which medication should be prescribed to reduce
the chance of any acute complications?

Labetalol
Mannitol
Ramipril
Furosemide
Nimodipine

MRCP part 1 Made Easy‫‏‬‎‫‏‬by M. Habayeb & A. Murad

429
Chapter: Neurology

Question 154 of 246

A 27 year old male with polycystic kidney disease presents with sudden onset headache and collapse. On
admission to emergency department his blood pressure 190/105 mmHg, tachycardic with a Glasgow Coma
Score of 7/15. He is intubated and transferred for a CT scan. The scan reveals a subarachnoid haemorrhage.
He is transferred to the critical care unit for monitoring. Which medication should be prescribed to reduce
the chance of any acute complications?

Labetalol
Mannitol
Ramipril
Furosemide
Nimodipine

Patients presenting following sub arachnoid haemorrhage may suffer from cerebral vasospasm. Vasospasm
occurs in approximately 30% of patients. In the sub population that this occurs in, it may result in further
ischemia due to a reduction in distal blood flow. All patients are prescribed a calcium channel blocker to
prophylactically prevent this from occurring.
In these patients we do not want to decrease the blood pressure acutely, this is because a higher blood pressure
may be required to drive the same cerebral perfusion pressure.

Guidelines on the management of Subarachnoid Haemorrhage


http://stroke.ahajournals.org/content/40/3/994.full
Discuss and give feedback

Subarachnoid haemorrhage

External links

SIGN

2008 Diagnosis and management of headache in adults

MRCP part 1 Made Easy‫‏‬‎‫‏‬by M. Habayeb & A. Murad

430
Chapter: Neurology

Question 155 of 246

A 34-year-old accountant presents with a one week history of pain around his right eye occurring once or
twice a day. They are described as being very severe and lasting between 10-30 minutes each. He also
describes a feeling of a blocked nose. What is the treatment of choice to treat this current episode?

Ibuprofen
Acetazolamide + topical pilocarpine
Prednisolone
Subcutaneous sumatriptan
Ergotamine

MRCP part 1 Made Easy‫‏‬‎‫‏‬by M. Habayeb & A. Murad

431
Chapter: Neurology

Question 155 of 246

A 34-year-old accountant presents with a one week history of pain around his right eye occurring once or
twice a day. They are described as being very severe and lasting between 10-30 minutes each. He also
describes a feeling of a blocked nose. What is the treatment of choice to treat this current episode?

Ibuprofen
Acetazolamide + topical pilocarpine
Prednisolone
Subcutaneous sumatriptan
Ergotamine

Cluster headache - acute treatment: subcutaneous sumatriptan + 100% O2

Standard analgesia is rarely effective in cluster headaches. 100% oxygen may also be used

Discuss and give feedback

Cluster headache

External links

NICE

2012 Headache guidelines

SIGN

2008 Diagnosis and management of headache in adults

MRCP part 1 Made Easy‫‏‬‎‫‏‬by M. Habayeb & A. Murad

432
Chapter: Neurology

Question 156 of 246

A 54-year-old man presents with a persistent tremor. On examination there is 6-8 Hz tremor of the arms
which is worse when his arms are outstretched. His father suffered from a similar complaint. What is the
most suitable first-line treatment?

Amitriptyline
Propranolol
D-penicillamine
Levodopa
Diazepam

MRCP part 1 Made Easy‫‏‬‎‫‏‬by M. Habayeb & A. Murad

433
Chapter: Neurology

Question 156 of 246

A 54-year-old man presents with a persistent tremor. On examination there is 6-8 Hz tremor of the arms
which is worse when his arms are outstretched. His father suffered from a similar complaint. What is the
most suitable first-line treatment?

Amitriptyline
Propranolol
D-penicillamine
Levodopa
Diazepam

Essential tremor is an AD condition that is made worse when arms are outstretched, made better by
alcohol and propranolol

This patient has a typical history of essential tremor. Propranolol is generally considered the first-line
treatment

Discuss and give feedback

Essential tremor

MRCP part 1 Made Easy‫‏‬‎‫‏‬by M. Habayeb & A. Murad

434
Chapter: Neurology

Question 157 of 246

A 49-year-old man presents to the Emergency Department complaining of visual disturbance. Examination
reveals a right incongruous homonymous hemianopia. Where is the lesion most likely to be?

Left optic tract


Left optic radiation
Right optic tract
Right optic radiation
Optic chiasm

MRCP part 1 Made Easy‫‏‬‎‫‏‬by M. Habayeb & A. Murad

435
Chapter: Neurology

Question 157 of 246

A 49-year-old man presents to the Emergency Department complaining of visual disturbance. Examination
reveals a right incongruous homonymous hemianopia. Where is the lesion most likely to be?

Left optic tract


Left optic radiation
Right optic tract
Right optic radiation
Optic chiasm

Discuss and give feedback

Visual field defects

MRCP part 1 Made Easy‫‏‬‎‫‏‬by M. Habayeb & A. Murad

436
Chapter: Neurology

Question 158 of 246

Each one of the following is associated with Friedreich's ataxia, except:

Increased risk of deep vein thrombosis


Optic atrophy
Cardiomyopathy
Nystagmus
High-arched palate

MRCP part 1 Made Easy‫‏‬‎‫‏‬by M. Habayeb & A. Murad

437
Chapter: Neurology

Question 158 of 246

Each one of the following is associated with Friedreich's ataxia, except:

Increased risk of deep vein thrombosis


Optic atrophy
Cardiomyopathy
Nystagmus
High-arched palate

Discuss and give feedback

Friedreich's ataxia

MRCP part 1 Made Easy‫‏‬‎‫‏‬by M. Habayeb & A. Murad

438
Chapter: Neurology

Question 159 of 246

A 72-year-old man presents to the Emergency Department. Whilst walking back from a friends house he
slipped on some ice and feel backwards, landing on his right arm and banging his head on the kerb in the
process. His past medical history includes atrial fibrillation for which he takes bisoprolol and warfarin. A
routine INR taken four days ago was 2.2. There are no signs of any external injury to his right arm or scalp.
What is the most appropriate course of action with relation to his head injury?

Arrange a CT head scan to be performed within 8 hours


Discharge with standard head injury advice
Admit for 24 hours of observation
Admit for 8 hours of observation
Discharge with standard head injury advice + advise he stops warfarin for 5 days

MRCP part 1 Made Easy‫‏‬‎‫‏‬by M. Habayeb & A. Murad

439
Chapter: Neurology

Question 159 of 246

A 72-year-old man presents to the Emergency Department. Whilst walking back from a friends house he
slipped on some ice and feel backwards, landing on his right arm and banging his head on the kerb in the
process. His past medical history includes atrial fibrillation for which he takes bisoprolol and warfarin. A
routine INR taken four days ago was 2.2. There are no signs of any external injury to his right arm or scalp.
What is the most appropriate course of action with relation to his head injury?

Arrange a CT head scan to be performed within 8 hours


Discharge with standard head injury advice
Admit for 24 hours of observation
Admit for 8 hours of observation
Discharge with standard head injury advice + advise he stops warfarin for 5 days

Patients who've had a head injury and are on warfarin need to have a CT scan, regardless of whether they have
risk factors for an intracranial injury.

Discuss and give feedback

Head injury: NICE guidance

MRCP part 1 Made Easy‫‏‬‎‫‏‬by M. Habayeb & A. Murad

440
Chapter: Neurology

Question 160 of 246

A 61-year-old woman presents with bilateral tinnitus. She reports no change in her hearing or other ear-related
symptoms. Ear and cranial nerve examination is unremarkable. Which medication is she most likely to have
recently started?

Ciprofloxacin
Nifedipine
Repaglinide
Quinine
Bendroflumethiazide

MRCP part 1 Made Easy‫‏‬‎‫‏‬by M. Habayeb & A. Murad

441
Chapter: Neurology

Question 160 of 246

A 61-year-old woman presents with bilateral tinnitus. She reports no change in her hearing or other ear-related
symptoms. Ear and cranial nerve examination is unremarkable. Which medication is she most likely to have
recently started?

Ciprofloxacin
Nifedipine
Repaglinide
Quinine
Bendroflumethiazide

Discuss and give feedback

Tinnitus

MRCP part 1 Made Easy‫‏‬‎‫‏‬by M. Habayeb & A. Murad

442
Chapter: Neurology

Question 161 of 246

What is the most common clinical pattern seen in motor neuron disease?

Progressive muscular atrophy


Progressive bulbar palsy
Spinocerebellar ataxia
Relapsing-remitting
Amyotrophic lateral sclerosis

MRCP part 1 Made Easy‫‏‬‎‫‏‬by M. Habayeb & A. Murad

443
Chapter: Neurology

Question 161 of 246

What is the most common clinical pattern seen in motor neuron disease?

Progressive muscular atrophy


Progressive bulbar palsy
Spinocerebellar ataxia
Relapsing-remitting
Amyotrophic lateral sclerosis

Discuss and give feedback

Motor neuron disease: types

External links

Postgraduate Medical Journal

Review of MND

MRCP part 1 Made Easy‫‏‬‎‫‏‬by M. Habayeb & A. Murad

444
Chapter: Neurology

Question 162 of 246

Each one of the following is associated with ataxic telangiectasia, except:

Telangiectasia
Cerebellar ataxia
Autosomal dominant inheritance
Recurrent chest infections
Increased risk of malignancy

MRCP part 1 Made Easy‫‏‬‎‫‏‬by M. Habayeb & A. Murad

445
Chapter: Neurology

Question 162 of 246

Each one of the following is associated with ataxic telangiectasia, except:

Telangiectasia
Cerebellar ataxia
Autosomal dominant inheritance
Recurrent chest infections
Increased risk of malignancy

Discuss and give feedback

Ataxic telangiectasia

MRCP part 1 Made Easy‫‏‬‎‫‏‬by M. Habayeb & A. Murad

446
Chapter: Neurology

Question 163 of 246

You are a CT1 in Acute Medicine covering a general medical ward at night. You are asked to see a 60-year-
old patient with known myasthenia gravis who is complaining of increased breathlessness and fatigue. Which
of the following antibiotics is most likely to be responsible for a myasthenic crisis?

Metronidazole
Doxycycline
Gentamicin
Clarithromycin
Amoxicillin

MRCP part 1 Made Easy‫‏‬‎‫‏‬by M. Habayeb & A. Murad

447
Chapter: Neurology

Question 163 of 246

You are a CT1 in Acute Medicine covering a general medical ward at night. You are asked to see a 60-year-
old patient with known myasthenia gravis who is complaining of increased breathlessness and fatigue. Which
of the following antibiotics is most likely to be responsible for a myasthenic crisis?

Metronidazole
Doxycycline
Gentamicin
Clarithromycin
Amoxicillin

♦This question asks which antibiotic is the most likely cause of an exacerbation. Although macrolides,
tetracyclines, and metronidazole are occasionally associated with exacerbation in myasthenic patients, they
are usually well tolerated in this population.
♦Aminoglycosides, including gentamicin, are relatively contra-indicated in myasthenia as they are more
commonly associated with exacerbations. The mechanism is thought to be competitive inhibition of the
release of acetylcholine at the presynaptic membrane of the neuromuscular junction.
♦Amoxicillin is generally considered to be safe in myasthenia gravis.
Discuss and give feedback

Myasthenia gravis: exacerbating factors

External links

Royal College of Physicians (Edin)

Myasthenia gravis review

Myasthenia Gravis Association

Drugs which may aggravate myasthenia gravis

External media

Myasthenia gravis - causes, symptoms, treatment, pathology

Osmosis - YouTube

MRCP part 1 Made Easy‫‏‬‎‫‏‬by M. Habayeb & A. Murad

448
Chapter: Neurology

Question 164 of 246

During a routine cranial nerve examination the following findings are observed:

Rinne's test: Air conduction > bone conduction in both ears

Weber's test: Localises to the right side

What do these tests imply?

Left conductive deafness


Normal hearing
Right conductive deafness
Right sensorineural deafness
Left sensorineural deafness

MRCP part 1 Made Easy‫‏‬‎‫‏‬by M. Habayeb & A. Murad

449
Chapter: Neurology

Question 164 of 246

During a routine cranial nerve examination the following findings are observed:

Rinne's test: Air conduction > bone conduction in both ears

Weber's test: Localises to the right side

What do these tests imply?

Left conductive deafness


Normal hearing
Right conductive deafness
Right sensorineural deafness
Left sensorineural deafness

In Weber's test if there is a sensorineural problem the sound is localised to the unaffected side (right)
indicating a problem on the left side

Discuss and give feedback

Rinne's and Weber's test

External links

ENT SHO

Guide to the ear examination

MRCP part 1 Made Easy‫‏‬‎‫‏‬by M. Habayeb & A. Murad

450
Chapter: Neurology

Question 165 of 246

A 40-year-old man undergoes a temporal lobectomy after the discovery of a brain tumour. Which one of the
following consequences would be least likely to develop?

Prosopagnosia
Astereognosis
Wernicke's aphasia
Superior homonymous quadrantanopia
Auditory agnosia

MRCP part 1 Made Easy‫‏‬‎‫‏‬by M. Habayeb & A. Murad

451
Chapter: Neurology

Question 165 of 246

A 40-year-old man undergoes a temporal lobectomy after the discovery of a brain tumour. Which one of the
following consequences would be least likely to develop?

Prosopagnosia
Astereognosis
Wernicke's aphasia
Superior homonymous quadrantanopia
Auditory agnosia

Discuss and give feedback

Brain lesions

MRCP part 1 Made Easy‫‏‬‎‫‏‬by M. Habayeb & A. Murad

452
Chapter: Neurology

Question 166 of 246

Which one of the following is least associated with the development of chorea?

Ataxic telangiectasia
SLE
Wilson's disease
Pregnancy
Infective endocarditis

MRCP part 1 Made Easy‫‏‬‎‫‏‬by M. Habayeb & A. Murad

453
Chapter: Neurology

Question 166 of 246

Which one of the following is least associated with the development of chorea?

Ataxic telangiectasia
SLE
Wilson's disease
Pregnancy
Infective endocarditis

Chorea can be a very rare manifestation of infective endocarditis, following embolisation to the basal ganglia.
It is however the least likely of the above five options

Discuss and give feedback

Chorea

External links

Postgraduate Medical Journal

Review of chorea

MRCP part 1 Made Easy‫‏‬‎‫‏‬by M. Habayeb & A. Murad

454
Chapter: Neurology

Question 167 of 246

A 43-year-old woman with multiple sclerosis presents for review. She is having increasing problems with
painful involuntary contractions of the leg muscles. What is the most appropriate first-line therapy?

Referral for relaxation therapy


Baclofen
Diazepam
Dantrolene
Natalizumab

MRCP part 1 Made Easy‫‏‬‎‫‏‬by M. Habayeb & A. Murad

455
Chapter: Neurology

Question 167 of 246

A 43-year-old woman with multiple sclerosis presents for review. She is having increasing problems with
painful involuntary contractions of the leg muscles. What is the most appropriate first-line therapy?

Referral for relaxation therapy


Baclofen
Diazepam
Dantrolene
Natalizumab

Discuss and give feedback

Multiple sclerosis: management

External links

Multiple Sclerosis Society

Primary care guidelines

NICE

2014 Multiple Sclerosis guidelines

External media

Multiple sclerosis - causes, symptoms, diagnosis, treatment, pathology

MRCP part 1 Made Easy‫‏‬‎‫‏‬by M. Habayeb & A. Murad

456
Chapter: Neurology

Question 168 of 246

A 19-year-old man presents with a two-day history of a diffuse headache and sore throat. He is pyrexial at
37.8ºC and is reluctant to have a fundoscopy due to photophobia. A lumbar puncture is performed:

Serum glucose 5.9 mmol/l

Lumbar puncture reveals:

Appearance Clear
Glucose 4.1 mmol/l
Protein 0.3 g/l
White cells lymphocytes 2 /mm³
polymorphs 0 /mm³

What is the most likely diagnosis?

Guillain-Barre syndrome
Viral meningitis
Bacterial meningitis
Cerebral malaria
Normal CSF result

MRCP part 1 Made Easy‫‏‬‎‫‏‬by M. Habayeb & A. Murad

457
Chapter: Neurology

Question 168 of 246


A 19-year-old man presents with a two-day history of a diffuse headache and sore throat. He is pyrexial at
37.8ºC and is reluctant to have a fundoscopy due to photophobia. A lumbar puncture is performed:

Serum glucose 5.9 mmol/l

Lumbar puncture reveals:

Appearance Clear
Glucose 4.1 mmol/l
Protein 0.3 g/l
White cells lymphocytes 2 /mm³
polymorphs 0 /mm³

What is the most likely diagnosis?

Guillain-Barre syndrome
Viral meningitis
Bacterial meningitis
Cerebral malaria
Normal CSF result
There results are consistent with normal CSF - an alternative diagnosis should be considered

Meningitis: CSF analysis


The table below summarises the characteristic cerebrospinal fluid (CSF) findings in meningitis:

Bacterial Viral Tuberculous


Appearance Cloudy Clear/cloudy Slight cloudy, fibrin web
Glucose Low (< 1/2 plasma) 60-80% of plasma glucose* Low (< 1/2 plasma)
Protein High (> 1 g/l) Normal/raised High (> 1 g/l)
White cells 10 - 5,000 15 - 1,000 lymphocytes/mm³ 10 - 1,000
polymorphs/mm³ lymphocytes/mm³

The Ziehl-Neelsen stain is only 20% sensitive in the detection of tuberculous meningitis and therefore PCR is
sometimes used (sensitivity = 75%)

*mumps is unusual in being associated with a low glucose level in a proportion of cases. A low glucose may
also be seen in herpes encephalitis

External links
Medscape
Meningitis: CSF analysis
MRCP part 1 Made Easy‫‏‬‎‫‏‬by M. Habayeb & A. Murad

458
Chapter: Neurology

Question 169 of 246

An 84-year-old female is admitted for a urinary tract infection. On the second night of admission she is found
wandering outside the ward in an agitated state. Despite appropriate antibiotic therapy, nursing care and
modification of her environment she remains agitated and aggressive and it is judged a potential danger to
herself. What is the most appropriate management?

Haloperidol 5 mg orally
Lorazepam 2 mg intramuscularly
Haloperidol 0.5 mg orally
Lorazepam 0.5 mg orally
Ask for on-call psychiatric opinion for consideration of section under the Mental Health
Act

MRCP part 1 Made Easy‫‏‬‎‫‏‬by M. Habayeb & A. Murad

459
Chapter: Neurology

Question 169 of 246

An 84-year-old female is admitted for a urinary tract infection. On the second night of admission she is found
wandering outside the ward in an agitated state. Despite appropriate antibiotic therapy, nursing care and
modification of her environment she remains agitated and aggressive and it is judged a potential danger to
herself. What is the most appropriate management?

Haloperidol 5 mg orally
Lorazepam 2 mg intramuscularly
Haloperidol 0.5 mg orally
Lorazepam 0.5 mg orally
Ask for on-call psychiatric opinion for consideration of section under the Mental Health
Act

Whilst many doctors may use oral lorazepam in this situation the Royal College of Physicians recommend
haloperidol as the first-line sedative. NICE also advocate the use of olanzapine.

Discuss and give feedback

Acute confusional state

External links

RCP

Delirium guidelines

NICE

2010 Delirium guidelines

MRCP part 1 Made Easy‫‏‬‎‫‏‬by M. Habayeb & A. Murad

460
Chapter: Neurology

Question 170 of 246

Which one of the following causes of Horner's syndrome is due to a lesion in the post-ganglionic part of
the nerve supply?

Internal carotid aneurysm


Stroke
Syringomyelia
Pancoast's tumour
Thyroidectomy

MRCP part 1 Made Easy‫‏‬‎‫‏‬by M. Habayeb & A. Murad

461
Chapter: Neurology

Question 170 of 246

Which one of the following causes of Horner's syndrome is due to a lesion in the post-ganglionic part of the
nerve supply?

Internal carotid aneurysm


Stroke
Syringomyelia
Pancoast's tumour
Thyroidectomy

Horner's syndrome - anhydrosis determines site of lesion:

 head, arm, trunk = central lesion: stroke, syringomyelia

 just face = pre-ganglionic lesion: Pancoast's, cervical rib

 absent = post-ganglionic lesion: carotid artery

Discuss and give feedback

Horner's syndrome

MRCP part 1 Made Easy‫‏‬‎‫‏‬by M. Habayeb & A. Murad

462
Chapter: Neurology

Question 171 of 246

A 31-year-old man presents around four weeks after a non-specific viral illness characterised by fever,
lethargy and sore throat. For the past week he has noticed increasing weakness in his legs which has now
started to extend to his arms. On examination he has reduced power, reflexes and slightly reduced sensation in
his lower limbs. A few days after admission he becomes short-of-breath. His forced vital capicity (FVC) starts
to fall and he is transferred to ITU. Given the likely diagnosis, what is the treatment of choice?

Neostigmine
Intravenous corticosteroids
Haemofilitration
Intravenous immunoglobulin
Riluzole

MRCP part 1 Made Easy‫‏‬‎‫‏‬by M. Habayeb & A. Murad

463
Chapter: Neurology

Question 171 of 246

A 31-year-old man presents around four weeks after a non-specific viral illness characterised by fever,
lethargy and sore throat. For the past week he has noticed increasing weakness in his legs which has now
started to extend to his arms. On examination he has reduced power, reflexes and slightly reduced sensation in
his lower limbs. A few days after admission he becomes short-of-breath. His forced vital capicity (FVC) starts
to fall and he is transferred to ITU. Given the likely diagnosis, what is the treatment of choice?

Neostigmine
Intravenous corticosteroids
Haemofilitration
Intravenous immunoglobulin
Riluzole

This patient has developed Guillain-Barre syndrome (GBS) secondary to a viral illness, possibly the Epstein-
Barr virus. The ascending weakness and areflexia point to a diagnosis of GBS.

Discuss and give feedback

Guillain-Barre syndrome: management

MRCP part 1 Made Easy‫‏‬‎‫‏‬by M. Habayeb & A. Murad

464
Chapter: Neurology

Question 172 of 246

A 62-year-old man is admitted to the Emergency Department with a left hemiplegia. His symptoms started
around 5 hours but he initially thought he had slept in an awkward position. He has no past medical history of
note but on examination is found to have and irregular pulse of 150 / min. The ECG confirms atrial
fibrillation. A CT head is immediately arranged and reported as normal. What is the most appropriate
initial management?

Aspirin
Aspirin + dipyridamole
Alteplase
Warfarin
Aspirin + warfarin

MRCP part 1 Made Easy‫‏‬‎‫‏‬by M. Habayeb & A. Murad

465
Chapter: Neurology

Question 172 of 246


A 62-year-old man is admitted to the Emergency Department with a left hemiplegia. His symptoms started
around 5 hours but he initially thought he had slept in an awkward position. He has no past medical history of
note but on examination is found to have and irregular pulse of 150 / min. The ECG confirms atrial
fibrillation. A CT head is immediately arranged and reported as normal. What is the most appropriate
initial management?

Aspirin
Aspirin + dipyridamole
Alteplase
Warfarin
Aspirin + warfarin
Rate control should also be initiated. He is outside the thrombolysis window so alteplase is not an option. The
2004 RCP guidelines recommend that anticoagulation should be commenced 14 days after an ischaemic
stroke. Earlier anticoagulation may exacerbate any secondary haemorrhage.
Dipyridamole should not be used in the acute phase.
Discuss and give feedback

Stroke: management
External links
NICE
2010 Clopidogrel and dipyridamole guidelines
SIGN
2008 Stroke guidelines
Age and Ageing
Interesting article on managing blood pressure during acute stroke
RCP
Stroke guidelines
NICE
2008 TIA and stroke guidelines
Clinical Knowledge Summaries
Stroke and TIA guidelines

MRCP part 1 Made Easy‫‏‬‎‫‏‬by M. Habayeb & A. Murad

466
Chapter: Neurology

Question 173 of 246

A 27-year-old female presents complaining of generalised weakness. Examination of her face reveals bilateral
ptosis, dysarthric speech and a slow-relaxing grip. What is the most likely diagnosis?

Myotonic dystrophy
Myasthenia gravis
Multiple sclerosis
Ataxic telangiectasia
Friedreich's ataxia

MRCP part 1 Made Easy‫‏‬‎‫‏‬by M. Habayeb & A. Murad

467
Chapter: Neurology

Question 173 of 246

A 27-year-old female presents complaining of generalised weakness. Examination of her face reveals bilateral
ptosis, dysarthric speech and a slow-relaxing grip. What is the most likely diagnosis?

Myotonic dystrophy
Myasthenia gravis
Multiple sclerosis
Ataxic telangiectasia
Friedreich's ataxia

Dystrophia myotonica - DM

 distal weakness initially

 autosomal dominant

 diabetes

 dysarthria

The slow-relaxing grip may be noticed on initial hand-shake with the patient and is typical of myotonic
dystrophy. Dysarthric speech is secondary to myotonia of the tongue and pharynx

Discuss and give feedback

Myotonic dystrophy

MRCP part 1 Made Easy‫‏‬‎‫‏‬by M. Habayeb & A. Murad

468
Chapter: Neurology

Question 174 of 246

A 36-year-old man presents to the emergency department with a severe left-sided headache with pain around
the left eye. He has had several similar episodes over the last 2 weeks, lasting 40-60 minutes each. The
headaches are associated with a runny nose. On examination, there is redness and tearing of his left eye.

What is the most appropriate acute management?

Acetazolamide
High flow oxygen
Paracetamol and naproxen
Urgent CT head
Verapamil

MRCP part 1 Made Easy‫‏‬‎‫‏‬by M. Habayeb & A. Murad

469
Chapter: Neurology

Question 174 of 246


A 36-year-old man presents to the emergency department with a severe left-sided headache with pain around
the left eye. He has had several similar episodes over the last 2 weeks, lasting 40-60 minutes each. The
headaches are associated with a runny nose. On examination, there is redness and tearing of his left eye.
What is the most appropriate acute management?

Acetazolamide
High flow oxygen
Paracetamol and naproxen
Urgent CT head
Verapamil

●This patient presents with a cluster headache. Cluster headaches are so named as clusters of headaches occur
frequently over a period of weeks followed by pain free periods of months to years.

♦The clinical features include :unilateral headaches almost always affecting the same side, tearing and
redness of the affected eye, rhinorrhoea and miosis +/- ptosis.

♦The pathophysiology of cluster headaches is unclear. They occur more common in men (5:1) and smokers.

♦Abortive management of cluster headaches involves the use of 100% oxygen at at least 12 litres per minute
via a non-rebreathable mask and/or a subcutaneous or nasal triptan.

♦First line long-term preventative management of cluster headaches is verapamil.


It is not recommended to offer paracetamol, NSAIDS, opioids, ergots or oral triptans for the acute treatment
of a cluster headache.

(Source: NICE headache guidelines) http://www.nice.org.uk/guidance/cg150/chapter/1-recommendation


Discuss and give feedback

Cluster headache

External links

NICE

2012 Headache guidelines

SIGN

2008 Diagnosis and management of headache in adults

MRCP part 1 Made Easy‫‏‬‎‫‏‬by M. Habayeb & A. Murad

470
Chapter: Neurology

Question 175 of 246

A 63-year-old woman with motor neuron disease is reviewed in clinic. Which one of the following
interventions will have the greatest effect on survival?

Regular chest physiotherapy


Total parental nutrition
Riluzole
Antioxidant supplementation
Non-invasive ventilation

MRCP part 1 Made Easy‫‏‬‎‫‏‬by M. Habayeb & A. Murad

471
Chapter: Neurology

Question 175 of 246

A 63-year-old woman with motor neuron disease is reviewed in clinic. Which one of the following
interventions will have the greatest effect on survival?

Regular chest physiotherapy


Total parental nutrition
Riluzole
Antioxidant supplementation
Non-invasive ventilation

Motor neuron disease - treatment: NIV is better than riluzole

Discuss and give feedback

Motor neuron disease: management

External links

NICE

2016 Motor neurone disease: assessment and management

MRCP part 1 Made Easy‫‏‬‎‫‏‬by M. Habayeb & A. Murad

472
Chapter: Neurology

Question 176 of 246

A 70-year-old man is investigated for involuntary, jerking movements of his arms. His symptoms seem to
resolve when he is asleep. Damage to which one of the following structures may lead to hemiballism?

Substantia nigra
Red nucleus
Subthalamic nucleus
Globus pallidus
Frontal lobe

MRCP part 1 Made Easy‫‏‬‎‫‏‬by M. Habayeb & A. Murad

473
Chapter: Neurology

Question 176 of 246

A 70-year-old man is investigated for involuntary, jerking movements of his arms. His symptoms seem to
resolve when he is asleep. Damage to which one of the following structures may lead to hemiballism?

Substantia nigra
Red nucleus
Subthalamic nucleus
Globus pallidus
Frontal lobe

Hemiballism is caused by damage to the subthalamic nucleus

Discuss and give feedback

Hemiballism

MRCP part 1 Made Easy‫‏‬‎‫‏‬by M. Habayeb & A. Murad

474
Chapter: Neurology

Question 177 of 246

A 63-year-old man is diagnosed as having restless legs syndrome. What is the most relevant blood test to
perform?

ESR
Ferritin
Blood glucose
Urea and electrolytes
Liver function tests

MRCP part 1 Made Easy‫‏‬‎‫‏‬by M. Habayeb & A. Murad

475
Chapter: Neurology

Question 177 of 246

A 63-year-old man is diagnosed as having restless legs syndrome. What is the most relevant blood test to
perform?

ESR
Ferritin
Blood glucose
Urea and electrolytes
Liver function tests

A case could be made for all the above tests but a low serum ferritin is most likely to be a cause of secondary
restless legs syndrome

Discuss and give feedback

Restless legs syndrome

MRCP part 1 Made Easy‫‏‬‎‫‏‬by M. Habayeb & A. Murad

476
Chapter: Neurology

Question 178 of 246

A 23-year-old woman is referred to the neurology clinic after developing a unilateral hand tremor. Over the
past 12 months her family report changes in her behaviour and mood associated with some speech problems.
On examination a tremor is noted in the right-hand at rest. There also appears to be paucity of movement and
some bradykinesia. Dark circular marks are also noted around the iris. The patient reports that her uncle died
of liver cirrhosis at the age of 40 years. Given the likely diagnosis, what is the mode of inheritance?

Autosomal dominant
Mitochondrial
X-linked recessive
Autosomal recessive
Polygenic

MRCP part 1 Made Easy‫‏‬‎‫‏‬by M. Habayeb & A. Murad

477
Chapter: Neurology

Question 178 of 246

A 23-year-old woman is referred to the neurology clinic after developing a unilateral hand tremor. Over the
past 12 months her family report changes in her behaviour and mood associated with some speech problems.
On examination a tremor is noted in the right-hand at rest. There also appears to be paucity of movement and
some bradykinesia. Dark circular marks are also noted around the iris. The patient reports that her uncle died
of liver cirrhosis at the age of 40 years. Given the likely diagnosis, what is the mode of inheritance?

Autosomal dominant
Mitochondrial
X-linked recessive
Autosomal recessive
Polygenic

Wilson's disease - autosomal recessive

This patient has Wilson's disease as evidence by the neuropsychiatric symptoms, Kayser-Fleischer rings and
family history of liver disease.

Discuss and give feedback

Wilson's disease

Wilson's disease is an autosomal recessive disorder characterised by excessive copper deposition in the
tissues. Metabolic abnormalities include increased copper absorption from the small intestine and decreased
hepatic copper excretion. Wilson's disease is caused by a defect in the ATP7B gene located on chromosome
13.
The onset of symptoms is usually between 10 - 25 years. Children usually present with liver disease whereas
the first sign of disease in young adults is often neurological disease
Features result from excessive copper deposition in the tissues, especially the brain, liver and cornea:
 liver: hepatitis, cirrhosis
 neurological: basal ganglia degeneration, speech and behavioural problems are often the first
manifestations. Also: asterixis, chorea, dementia
 Kayser-Fleischer rings
 renal tubular acidosis (esp. Fanconi syndrome)
 haemolysis
 blue nails

MRCP part 1 Made Easy‫‏‬‎‫‏‬by M. Habayeb & A. Murad

478
Chapter: Neurology

Diagnosis

 reduced serum caeruloplasmin


 reduced serum copper (counter-intuitive, but 95% of plasma copper is carried by ceruloplasmin)
 increased 24hr urinary copper excretion

Management:

 penicillamine (chelates copper) has been the traditional first-line treatment

 trientine hydrochloride is an alternative chelating agent which may become first-line treatment in the
future

 tetrathiomolybdate is a newer agent that is currently under investigation

External media

Wilson's disease

Osmosis - YouTube

MRCP part 1 Made Easy‫‏‬‎‫‏‬by M. Habayeb & A. Murad

479
Chapter: Neurology

Question 179 of 246

Which one of the following medications is most useful for helping to prevent attacks of Meniere's
disease?

Promethazine
Prochlorperazine
Betahistine
Chlorphenamine
Cinnarizine

MRCP part 1 Made Easy‫‏‬‎‫‏‬by M. Habayeb & A. Murad

480
Chapter: Neurology

Question 179 of 246

Which one of the following medications is most useful for helping to prevent attacks of Meniere's
disease?

Promethazine
Prochlorperazine
Betahistine
Chlorphenamine
Cinnarizine

Discuss and give feedback

Meniere's disease

External links

Clinical Knowledge Summaries

Meniere's disease guidelines

MRCP part 1 Made Easy‫‏‬‎‫‏‬by M. Habayeb & A. Murad

481
Chapter: Neurology

Question 180 of 246

A 23-year-old man with a history of migraine presents for review. His headaches are now occurring about
once a week. He describes unilateral, throbbing headaches that may last over 24 hours. Neurological
examination is unremarkable. Other than a history of asthma he is fit and well. What is the most suitable
therapy to reduce the frequency of migraine attacks?

Propranolol
Zolmitriptan
Topiramate
Amitriptyline
Pizotifen

MRCP part 1 Made Easy‫‏‬‎‫‏‬by M. Habayeb & A. Murad

482
Chapter: Neurology

Question 180 of 246


A 23-year-old man with a history of migraine presents for review. His headaches are now occurring about
once a week. He describes unilateral, throbbing headaches that may last over 24 hours. Neurological
examination is unremarkable. Other than a history of asthma he is fit and well. What is the most suitable
therapy to reduce the frequency of migraine attacks?

Propranolol
Zolmitriptan
Topiramate
Amitriptyline
Pizotifen

Migraine
 acute: triptan + NSAID or triptan + paracetamol
 prophylaxis: topiramate or propranolol

Pizotifen is used less commonly nowadays due to side-effects such as weight gain. Propranolol should be
avoided in asthmatics
Discuss and give feedback

Migraine: management
External links
SIGN
2008 Diagnosis and management of headache in adults
Clinical Knowledge Summaries
Migraine guidelines
NICE
2012 Headache guidelines
British Association for the Study of Headache
Headache guidelines

MRCP part 1 Made Easy‫‏‬‎‫‏‬by M. Habayeb & A. Murad

483
Chapter: Neurology

Question 181 of 246

Which one of the following statements regarding the stopping of anti-epileptic drugs (AED) is most
correct?

Can be considered if seizure free for > 5 years, with AEDs being stopped over 2-3 months
Can be considered if seizure free for > 2 years, with AEDs being stopped over 2-3 months
Can be considered if seizure free for > 1 year, with AEDs being stopped over 2-3 months
Can be considered if seizure free for > 5 years, with AEDs being stopped over 8-12
months
Can be considered if seizure free for > 1 year, with AEDs being stopped over 8-12 months

MRCP part 1 Made Easy‫‏‬‎‫‏‬by M. Habayeb & A. Murad

484
Chapter: Neurology

Question 181 of 246

Which one of the following statements regarding the stopping of anti-epileptic drugs (AED) is most
correct?

Can be considered if seizure free for > 5 years, with AEDs being stopped over 2-3 months
Can be considered if seizure free for > 2 years, with AEDs being stopped over 2-3
months
Can be considered if seizure free for > 1 year, with AEDs being stopped over 2-3 months
Can be considered if seizure free for > 5 years, with AEDs being stopped over 8-12
months
Can be considered if seizure free for > 1 year, with AEDs being stopped over 8-12 months

The above reflects 2004 NICE guidelines and should be done under the guidance of a specialist.
Benzodiazepines should be withdrawn over a longer period.

Discuss and give feedback

Epilepsy: treatment

External links

NICE

2012 Epilepsy guidelines

Royal College of Physicians

2013 Modern management of epilepsy

MRCP part 1 Made Easy‫‏‬‎‫‏‬by M. Habayeb & A. Murad

485
Chapter: Neurology

Question 182 of 246

A 30-year-old lady presents with acute occipital headache associated with vomiting, photophobia and stiff
neck. There is no history of rash or fever. She has a past medical history of phaeochromocytoma for which
she had surgery. She reports that her father died of kidney cancer and her brother is blind in his right eye due
to a bleed in the eye.

What is the unifying diagnosis?

Polycystic kidney disease


Von Hippel Lindau disease
Multiple endocrine neoplasia type 1
Tuberous sclerosis
Alport syndrome

MRCP part 1 Made Easy‫‏‬‎‫‏‬by M. Habayeb & A. Murad

486
Chapter: Neurology

Question 182 of 246

A 30-year-old lady presents with acute occipital headache associated with vomiting, photophobia and stiff
neck. There is no history of rash or fever. She has a past medical history of phaeochromocytoma for which
she had surgery. She reports that her father died of kidney cancer and her brother is blind in his right eye due
to a bleed in the eye.

What is the unifying diagnosis?

Polycystic kidney disease


Von Hippel Lindau disease
Multiple endocrine neoplasia type 1
Tuberous sclerosis
Alport syndrome

This patient has von Hippel Lindau disease which is characterised by retinal and cerebellar haemangiomas,
renal cysts with transformation to renal cell carcinoma, and phaeochromocytoma. Neurological signs can be
due to compression by haemangiomas, or an acute headache due to intracerebral or subarachnoid
haemorrhage as in this patient.

Explanation for other options:

 1. Polycystic kidney disease is associated with subarachnoid haemorrhage due to berry aneurysms, but
the other pointers in the history of a phaeochromocytoma and a family history of retinal bleeding and
renal cell carcinoma do not fit with PKD.

 3. Multiple endocrine neoplasia type 1 is associated with parathyroid disease, pancreatic endocrine
tumours and pituitary tumours.

 4. Tuberous sclerosis is associated with hamartomas of the brain, eye and kidneys but would have
characteristic skin signs present such as angiofibroma, shagreen patches and adenoma sebaceum.

 5. Alport syndrome is not associated with intracerebral bleeding.

Discuss and give feedback

Von Hippel-Lindau syndrome

MRCP part 1 Made Easy‫‏‬‎‫‏‬by M. Habayeb & A. Murad

487
Chapter: Neurology

Question 183 of 246

A 24-year-old female presents to her GP due to increased frequency of migraine attacks. She is now having
around four migraines per month. Which type of medication would it be most appropriate to prescribe to
reduce the frequency of migraine attacks?

Specific 5-HT2 agonist


5-HT1 antagonist
Tricyclic antidepressant
Beta-blocker
Specific 5-HT1 agonist

MRCP part 1 Made Easy‫‏‬‎‫‏‬by M. Habayeb & A. Murad

488
Chapter: Neurology

Question 183 of 246

A 24-year-old female presents to her GP due to increased frequency of migraine attacks. She is now having
around four migraines per month. Which type of medication would it be most appropriate to prescribe to
reduce the frequency of migraine attacks?

Specific 5-HT2 agonist


5-HT1 antagonist
Tricyclic antidepressant
Beta-blocker
Specific 5-HT1 agonist

Migraine

 acute: triptan + NSAID or triptan + paracetamol

 prophylaxis: topiramate or propranolol

♦Topiramate is also recommended by NICE as first-line prophylaxis against migraine. However, given that
she is female and of child-bearing age a beta-blocker (such as propranolol) is a better choice.

Migraine: management

It should be noted that as a general rule 5-HT receptor agonists are used in the acute treatment of migraine
whilst 5-HT receptor antagonists are used in prophylaxis. NICE produced guidelines in 2012 on the
management of headache, including migraines.
Acute treatment

 first-line: offer combination therapy with an oral triptan and an NSAID, or an oral triptan and
paracetamol

 for young people aged 12-17 years consider a nasal triptan in preference to an oral triptan

 if the above measures are not effective or not tolerated offer a non-oral preparation of
metoclopramide* or prochlorperazine and consider adding a non-oral NSAID or triptan.

MRCP part 1 Made Easy‫‏‬‎‫‏‬by M. Habayeb & A. Murad

489
Chapter: Neurology

Prophylaxis

 prophylaxis should be given if patients are experiencing 2 or more attacks per month. Modern
treatment is effective in about 60% of patients.
 NICE advise either topiramate or propranolol 'according to the person's preference, comorbidities and
risk of adverse events'. Propranolol should be used in preference to topiramate in women of child
bearing age as it may be teratogenic and it can reduce the effectiveness of hormonal contraceptives
 if these measures fail NICE recommend 'a course of up to 10 sessions of acupuncture over 5-8 weeks'
or gabapentin
 NICE recommend: 'Advise people with migraine that riboflavin (400 mg once a day) may be effective
in reducing migraine frequency and intensity for some people'
 for women with predictable menstrual migraine treatment NICE recommend either frovatriptan (2.5
mg twice a day) or zolmitriptan (2.5 mg twice or three times a day) as a type of 'mini-prophylaxis'
 pizotifen is no longer recommend. Adverse effects such as weight gain & drowsiness are common

*caution should be exercised with young patients as acute dystonic reactions may develop

External links
SIGN
2008 Diagnosis and management of headache in adults
Clinical Knowledge Summaries
Migraine guidelines
NICE
2012 Headache guidelines
British Association for the Study of Headache
Headache guidelines

External media

Migraine

Podmedics - YouTube

MRCP part 1 Made Easy‫‏‬‎‫‏‬by M. Habayeb & A. Murad

490
Chapter: Neurology

Question 184 of 246

A 42-year-old woman with a history of myasthenia gravis is admitted to the Emergency Department. She is
currently taking pyridostigmine but there has been a significant worsening of her symptoms following
antibiotic treatment for a chest infection. On examination she is dyspnoeic and cyanotic with quiet breath
sounds in both lungs. Other than respiratory support, what are the two treatments of choice?

IV methylprednisolone or plasmapheresis
IV methylprednisolone or intravenous immunoglobulins
Plasmapheresis or atropine
IV methylprednisolone or atropine
Plasmapheresis or intravenous immunoglobulins

MRCP part 1 Made Easy‫‏‬‎‫‏‬by M. Habayeb & A. Murad

491
Chapter: Neurology

Question 184 of 246

A 42-year-old woman with a history of myasthenia gravis is admitted to the Emergency Department. She is
currently taking pyridostigmine but there has been a significant worsening of her symptoms following
antibiotic treatment for a chest infection. On examination she is dyspnoeic and cyanotic with quiet breath
sounds in both lungs. Other than respiratory support, what are the two treatments of choice?

IV methylprednisolone or plasmapheresis
IV methylprednisolone or intravenous immunoglobulins
Plasmapheresis or atropine
IV methylprednisolone or atropine
Plasmapheresis or intravenous immunoglobulins

This patient is having a myasthenic crisis. Opinions vary as to whether plasmapheresis or intravenous
immunoglobulins should be given first-line. Plasmapheresis usually works quicker but involves more
expensive equipment

Discuss and give feedback

Myasthenia gravi

External links

Postgraduate Medical Journal

Review of myasthenia gravis

External media

Myasthenia gravis - causes, symptoms, treatment, pathology

Osmosis - YouTube

MRCP part 1 Made Easy‫‏‬‎‫‏‬by M. Habayeb & A. Murad

492
Chapter: Neurology

Question 185 of 246

Which one of the following anti-epileptic drugs is most likely to cause visual field defects?

Lamotrigine
Phenytoin
Ethosuximide
Vigabatrin
Pregabalin

MRCP part 1 Made Easy‫‏‬‎‫‏‬by M. Habayeb & A. Murad

493
Chapter: Neurology

Question 185 of 246

Which one of the following anti-epileptic drugs is most likely to cause visual field defects?

Lamotrigine
Phenytoin
Ethosuximide
Vigabatrin
Pregabalin

V for Vigabatrin - V for Visual field defects

Discuss and give feedback

Vigabatrin

MRCP part 1 Made Easy‫‏‬‎‫‏‬by M. Habayeb & A. Murad

494
Chapter: Neurology

Question 186 of 246

A 59-year-old man presents with recurrent attacks of vertigo and dizziness. These attacks are often
precipitated by a change in head position and typically last around half a minute. Examination of the cranial
nerves and ears is unremarkable. His blood pressure is 120/78 mmHg sitting and 116/76 mmHg standing.
Given the likely underlying disorder, what is the most appropriate next step to help confirm the
diagnosis?

Epley manoeuvre
Tilt table test
24 hour ECG monitoring
MRI of the cerebellopontine angle
Dix-Hallpike manoeuvre

MRCP part 1 Made Easy‫‏‬‎‫‏‬by M. Habayeb & A. Murad

495
Chapter: Neurology

Question 186 of 246

A 59-year-old man presents with recurrent attacks of vertigo and dizziness. These attacks are often
precipitated by a change in head position and typically last around half a minute. Examination of the cranial
nerves and ears is unremarkable. His blood pressure is 120/78 mmHg sitting and 116/76 mmHg standing.
Given the likely underlying disorder, what is the most appropriate next step to help confirm the
diagnosis?

Epley manoeuvre
Tilt table test
24 hour ECG monitoring
MRI of the cerebellopontine angle
Dix-Hallpike manoeuvre

This patient has classical symptoms of benign paroxysmal positional vertigo. A positive Dix-Hallpike
manoeuvre is an appropriate next step and would help support the diagnosis.

The change in blood pressure on standing is not significant.

Discuss and give feedback

Benign paroxysmal positional vertigo

External links

YouTube

Hallpike Test and Epley Maneuver

Clinical Knowledge Summaries

Benign paroxysmal positional vertigo guidelines

MRCP part 1 Made Easy‫‏‬‎‫‏‬by M. Habayeb & A. Murad

496
Chapter: Neurology

Question 187 of 246

A 55-year-old man presents complaining of visual disturbance. Examination reveals a right congruous
homonymous hemianopia with macula sparing. Where is the lesion most likely to be?

Right optic nerve


Right optic radiation
Left optic tract
Left occipital cortex
Optic chiasm

MRCP part 1 Made Easy‫‏‬‎‫‏‬by M. Habayeb & A. Murad

497
Chapter: Neurology

Question 187 of 246

A 55-year-old man presents complaining of visual disturbance. Examination reveals a right congruous
homonymous hemianopia with macula sparing. Where is the lesion most likely to be?

Right optic nerve


Right optic radiation
Left optic tract
Left occipital cortex
Optic chiasm

Visual field defects:

 left homonymous hemianopia means visual field defect to the left, i.e. lesion of right optic tract

 homonymous quadrantanopias: PITS (Parietal-Inferior, Temporal-Superior)

 incongruous defects = optic tract lesion; congruous defects= optic radiation lesion or occipital cortex

Discuss and give feedback

Visual field defects

MRCP part 1 Made Easy‫‏‬‎‫‏‬by M. Habayeb & A. Murad

498
Chapter: Neurology

Question 188 of 246

A 65-year-old female is admitted with a right hemiparesis. Examination reveals she is in atrial fibrillation. CT
confirms an ischaemic stroke and aspirin 300mg is commenced. If the patient is well and develops no new
problems at what point should warfarin be started?

After 14 days
Immediately
After 7 days
Following a repeat CT at 28 days to exclude secondary haemorrhage
Following a repeat CT at 14 days to exclude secondary haemorrhage

MRCP part 1 Made Easy‫‏‬‎‫‏‬by M. Habayeb & A. Murad

499
Chapter: Neurology

Question 188 of 246

A 65-year-old female is admitted with a right hemiparesis. Examination reveals she is in atrial fibrillation. CT
confirms an ischaemic stroke and aspirin 300mg is commenced. If the patient is well and develops no new
problems at what point should warfarin be started?

After 14 days
Immediately
After 7 days
Following a repeat CT at 28 days to exclude secondary haemorrhage
Following a repeat CT at 14 days to exclude secondary haemorrhage

The 2004 RCP guidelines recommend that anticoagulation should be commenced 14 days after an ischaemic
stroke. Earlier anticoagulation may exacerbate any secondary haemorrhage

Discuss and give feedback

Stroke: management

External links

NICE

2010 Clopidogrel and dipyridamole guidelines

SIGN

2008 Stroke guidelines

Age and Ageing

Interesting article on managing blood pressure during acute stroke

RCP

Stroke guidelines

NICE

2008 TIA and stroke guidelines

Clinical Knowledge Summaries

Stroke and TIA guidelines

MRCP part 1 Made Easy‫‏‬‎‫‏‬by M. Habayeb & A. Murad

500
Chapter: Neurology

Question 189 of 246

Which one of the following statements regarding restless legs syndrome is incorrect?

Movements may be seen during sleep


May be secondary to uraemia
Affects approximately 5% of the general population
Family history is found in up to 50% of patients
It is three times as common in females

MRCP part 1 Made Easy‫‏‬‎‫‏‬by M. Habayeb & A. Murad

501
Chapter: Neurology

Question 189 of 246

Which one of the following statements regarding restless legs syndrome is incorrect?

Movements may be seen during sleep


May be secondary to uraemia
Affects approximately 5% of the general population
Family history is found in up to 50% of patients
It is three times as common in females

Males and females are thought to be equally affected, with only one study showing a slightly increased
incidence in females

Discuss and give feedback

Restless legs syndrome

MRCP part 1 Made Easy‫‏‬‎‫‏‬by M. Habayeb & A. Murad

502
Chapter: Neurology

Question 190 of 246

Which one of the following conditions is least recognised as a cause of a seventh nerve palsy?

Acoustic neuroma
Herpes zoster
HIV
Systemic lupus erythematosus
Diabetes mellitus

MRCP part 1 Made Easy‫‏‬‎‫‏‬by M. Habayeb & A. Murad

503
Chapter: Neurology

Question 190 of 246

Which one of the following conditions is least recognised as a cause of a seventh nerve palsy?

Acoustic neuroma
Herpes zoster
HIV
Systemic lupus erythematosus
Diabetes mellitus

Discuss and give feedback

Facial nerve

MRCP part 1 Made Easy‫‏‬‎‫‏‬by M. Habayeb & A. Murad

504
Chapter: Neurology

Question 191 of 246

A 44-year-old male is seen in Neurology clinic after presenting with a subacute history of a headache. His
headache seems to be worse on recumbency and coughing/sneezing.

On examination, upper and lower limbs are normal with intact cranial nerves. On closer examination of the
oropharynx, there is evidence of palatal myoclonus. A CT scan confirms a space occupying lesion. Where is
the lesion likely located?

Temporal lobe
Olivary nucleus
Parietal lobe
Occipital lobe
Frontal lobe

MRCP part 1 Made Easy‫‏‬‎‫‏‬by M. Habayeb & A. Murad

505
Chapter: Neurology

Question 191 of 246

A 44-year-old male is seen in Neurology clinic after presenting with a subacute history of a headache. His
headache seems to be worse on recumbency and coughing/sneezing.

On examination, upper and lower limbs are normal with intact cranial nerves. On closer examination of the
oropharynx, there is evidence of palatal myoclonus. A CT scan confirms a space occupying lesion. Where is
the lesion likely located?

Temporal lobe
Olivary nucleus
Parietal lobe
Occipital lobe
Frontal lobe

Palatal myoclonus is a specific feature of hypertrophic olivary degeneration. This is caused by a lesion in the
triangle of Guillain and Mollaret (triangle linking the inferior olivary nucleus, red nucleus and the
contralateral dentate nucleus). An MRI brain is the gold standard imaging for this lesion.

The other answers are not a part of the triangle of Mollaret.

Discuss and give feedback

Brain lesions

MRCP part 1 Made Easy‫‏‬‎‫‏‬by M. Habayeb & A. Murad

506
Chapter: Neurology

Question 192 of 246

A 45-year-old female is diagnosed with a glioma in the parietal lobe after being investigated for new onset
seizures. Which one of the following features is she most likely to develop?

Visual agnosia
Auditory agnosia
Acalculia
Inability to generate a list
Expressive (Broca's) aphasia

MRCP part 1 Made Easy‫‏‬‎‫‏‬by M. Habayeb & A. Murad

507
Chapter: Neurology

Question 192 of 246

A 45-year-old female is diagnosed with a glioma in the parietal lobe after being investigated for new onset
seizures. Which one of the following features is she most likely to develop?

Visual agnosia
Auditory agnosia
Acalculia
Inability to generate a list
Expressive (Broca's) aphasia

Discuss and give feedback

Brain lesions

MRCP part 1 Made Easy‫‏‬‎‫‏‬by M. Habayeb & A. Murad

508
Chapter: Neurology

Question 193 of 246

A 71-year-old man is admitted to the Emergency Department. His family report that since yesterday he has
been very 'clumsy' and unsteady on his feet. This morning he started to complain of numbness down his left
side. On examination you notice that he has a right-sided Horner's syndrome and horizontal nystagmus.
Examination of the peripheral nervous system confirms the sensory loss on the left side. Where is the lesion
most likely to be?

Lateral sinus thrombosis


Posterior cerebral artery
Posterior inferior cerebellar artery
Middle cerebral artery
Anterior inferior cerebellar artery

MRCP part 1 Made Easy‫‏‬‎‫‏‬by M. Habayeb & A. Murad

509
Chapter: Neurology

Question 193 of 246

A 71-year-old man is admitted to the Emergency Department. His family report that since yesterday he has
been very 'clumsy' and unsteady on his feet. This morning he started to complain of numbness down his left
side. On examination you notice that he has a right-sided Horner's syndrome and horizontal nystagmus.
Examination of the peripheral nervous system confirms the sensory loss on the left side. Where is the lesion
most likely to be?

Lateral sinus thrombosis


Posterior cerebral artery
Posterior inferior cerebellar artery
Middle cerebral artery
Anterior inferior cerebellar artery

Discuss and give feedback

Lateral medullary syndrome

MRCP part 1 Made Easy‫‏‬‎‫‏‬by M. Habayeb & A. Murad

510
Chapter: Neurology

Question 194 of 246

A 63-year-old man who is known to have small cell lung carcinoma presents with gradually worsening
muscle weakness. This initially affected his legs but is now spreading to the arms. He also complains of a dry
mouth and erectile dysfunction. Neurological examination show bilateral leg and arm weakness associated
with hyporeflexia. Antibodies to which one of the following are most likely to be responsible for these
findings?

RNA-binding protein Nova-1


NMDA-receptors
Muscarinic acetylcholine receptors
Nicotinic acetylcholine receptors
Voltage gated calcium channels

MRCP part 1 Made Easy‫‏‬‎‫‏‬by M. Habayeb & A. Murad

511
Chapter: Neurology

Question 194 of 246

A 63-year-old man who is known to have small cell lung carcinoma presents with gradually worsening
muscle weakness. This initially affected his legs but is now spreading to the arms. He also complains of a dry
mouth and erectile dysfunction. Neurological examination show bilateral leg and arm weakness associated
with hyporeflexia. Antibodies to which one of the following are most likely to be responsible for these
findings?

RNA-binding protein Nova-1


NMDA-receptors
Muscarinic acetylcholine receptors
Nicotinic acetylcholine receptors
Voltage gated calcium channels

Discuss and give feedback

Lambert-Eaton syndrome

MRCP part 1 Made Easy‫‏‬‎‫‏‬by M. Habayeb & A. Murad

512
Chapter: Neurology

Question 195 of 246

A 76-year-old man is admitted with a right hemiparesis. On examination his blood pressure is 120/78 mmHg,
pulse 84 bpm and oxygen saturations 96% on room air. A CT scan excludes intracerebral haemorrhage and he
is given aspirin 300mg. What is the most appropriate management with regards to oxygen therapy in the
first 12 hours following admission?

35% via Venturi mask


24% via Venturi mask
No oxygen therapy
28% via Venturi mask
2 litres/minute via nasal cannulae

MRCP part 1 Made Easy‫‏‬‎‫‏‬by M. Habayeb & A. Murad

513
Chapter: Neurology

Question 195 of 246

A 76-year-old man is admitted with a right hemiparesis. On examination his blood pressure is 120/78 mmHg,
pulse 84 bpm and oxygen saturations 96% on room air. A CT scan excludes intracerebral haemorrhage and he
is given aspirin 300mg. What is the most appropriate management with regards to oxygen therapy in the
first 12 hours following admission?

35% via Venturi mask


24% via Venturi mask
No oxygen therapy
28% via Venturi mask
2 litres/minute via nasal cannulae

Both the NICE stroke guidelines and British Thoracic Society oxygen guidelines do not support giving
oxygen in this scenario.

Discuss and give feedback

Oxygen therapy

The British Thoracic Society published guidelines on emergency oxygen therapy in 2008. The following
selected points are taken from the guidelines. Please see the link provided for the full guideline.

In patients who are critically ill (anaphylaxis, shock etc) oxygen should initially be given via a reservoir
mask at 15 l/min. Hypoxia kills. The BTS guidelines specifically exclude certain conditions where the
patient is acutely unwell (e.g. myocardial infarction) but stable.

Oxygen saturation targets

 acutely ill patients: 94-98%

 patients at risk of hypercapnia (e.g. COPD patients): 88-92% (see below)

 oxygen should be reduced in stable patients with satisfactory oxygen saturation

MRCP part 1 Made Easy‫‏‬‎‫‏‬by M. Habayeb & A. Murad

514
Chapter: Neurology

Management of COPD patients

 prior to availability of blood gases, use a 28% Venturi mask at 4 l/min and aim for an oxygen
saturation of 88-92% for patients with risk factors for hypercapnia but no prior history of respiratory
acidosis

 adjust target range to 94-98% if the pCO2 is normal

Situations where oxygen therapy should not be used routinely if there is no evidence of hypoxia:

 myocardial infarction and acute coronary syndromes

 stroke

 obstetric emergencies

 anxiety-related hyperventilation

External links

British Thoracic Society

2008 Emergency oxygen therapy guidelines

External media

Oxygen Therapy and Delivery - How to Prescribe Oxygen

Oxford Medical Education – YouTube

MRCP part 1 Made Easy‫‏‬‎‫‏‬by M. Habayeb & A. Murad

515
Chapter: Neurology

Question 196 of 246

A 35-year-old man with a history of migraines has been admitted to the medical take with a right-sided
hemiparesis. On closer questioning, there is a family history of stroke and migraines. A CT brain reveals
multiple hypodensities within the basal ganglia and temporal lobes, out of keeping for his age. A diagnosis of
CADASIL is suspected. What is the pathophysiology of this condition?

NOTCH3 mutation
NOD2/CARD15 mutation
FXN mutation
GLA mutation
SCN5a mutation

MRCP part 1 Made Easy‫‏‬‎‫‏‬by M. Habayeb & A. Murad

516
Chapter: Neurology

Question 196 of 246

A 35-year-old man with a history of migraines has been admitted to the medical take with a right-sided
hemiparesis. On closer questioning, there is a family history of stroke and migraines. A CT brain reveals
multiple hypodensities within the basal ganglia and temporal lobes, out of keeping for his age. A diagnosis of
CADASIL is suspected. What is the pathophysiology of this condition?

NOTCH3 mutation
NOD2/CARD15 mutation
FXN mutation
GLA mutation
SCN5a mutation

♦CADASIL (also known as cerebral autosomal dominant arteriopathy with subcortical infarcts and
leukoencephalopathy) is the most common cause of hereditary cerebral small-vessel disease and vascular
cognitive impairment in young adults. It is, as implied by the name, an autosomal dominant condition caused
by a NOTCH3 mutation on chromosome 19. Characteristically, this presents with a migraine with aura, and a
family history of such, which can ultimately lead to stroke-like features and is a recognised important cause of
stroke in the young. Brain imaging will reveal multiple subcortical white matter lesions, particularly in the
anterior temporal lobes and basal ganglia, out of keeping with the patient's age.

♦NOD2/CARD15 mutations are associated with Crohn's, FXN mutations are associated with Friedreich's
ataxia and GLA mutations with Fabry disease. SCN5a mutations are associated with Brugada Syndrome.

Discuss and give feedback

Stroke by anatomy

MRCP part 1 Made Easy‫‏‬‎‫‏‬by M. Habayeb & A. Murad

517
Chapter: Neurology

Question 197 of 246

A 61-year-old man complains of a four month history of neck and arm pain. The pain is described as being
like 'electric shocks' and is worse when he turns his head. There is no history of trauma and no other obvious
trigger. He is otherwise fit and well and takes no other medication. On examination he has decreased
sensation on the dorsal aspect of the thumb and index finger. What is the most likely underlying diagnosis?

C4 radiculopathy
C5 radiculopathy
C6 radiculopathy
C7 radiculopathy
T1 radiculopathy

MRCP part 1 Made Easy‫‏‬‎‫‏‬by M. Habayeb & A. Murad

518
Chapter: Neurology

Question 197 of 246

A 61-year-old man complains of a four month history of neck and arm pain. The pain is described as being
like 'electric shocks' and is worse when he turns his head. There is no history of trauma and no other obvious
trigger. He is otherwise fit and well and takes no other medication. On examination he has decreased
sensation on the dorsal aspect of the thumb and index finger. What is the most likely underlying diagnosis?

C4 radiculopathy
C5 radiculopathy
C6 radiculopathy
C7 radiculopathy
T1 radiculopathy

Discuss and give feedback

Dermatomes

MRCP part 1 Made Easy‫‏‬‎‫‏‬by M. Habayeb & A. Murad

519
Chapter: Neurology

Question 198 of 246

A 23-year-old female has a lumbar puncture to exclude a subarachnoid haemorrhage following a negative CT
scan. Which one of the following factors would be most likely to influence the incidence of post-lumbar
puncture headache?

Position of the patient


Increased fluid intake post procedure
Opening pressure of CSF
Bed rest following the procedure
Replacing the stylet

MRCP part 1 Made Easy‫‏‬‎‫‏‬by M. Habayeb & A. Murad

520
Chapter: Neurology

Question 198 of 246

A 23-year-old female has a lumbar puncture to exclude a subarachnoid haemorrhage following a negative CT
scan. Which one of the following factors would be most likely to influence the incidence of post-lumbar
puncture headache?

Position of the patient


Increased fluid intake post procedure
Opening pressure of CSF
Bed rest following the procedure
Replacing the stylet

Discuss and give feedback

Post-lumbar puncture headache

MRCP part 1 Made Easy‫‏‬‎‫‏‬by M. Habayeb & A. Murad

521
Chapter: Neurology

Question 199 of 246

A 64-year-old man who is under investigation for parkinsonian symptoms is brought to the GP by his wife.
She is concerned her husband is becoming increasingly agitated. The GP prescribes haloperidol. One week
later the GP is called out to see the patient as his parkinsonian symptoms have deteriorated markedly. What is
the most likely underlying diagnosis?

Lewy body dementia


Normal pressure hydrocephalus
Progressive supranuclear palsy
Multiple system atrophy
Dementia pugilistica

MRCP part 1 Made Easy‫‏‬‎‫‏‬by M. Habayeb & A. Murad

522
Chapter: Neurology

Question 199 of 246

A 64-year-old man who is under investigation for parkinsonian symptoms is brought to the GP by his wife.
She is concerned her husband is becoming increasingly agitated. The GP prescribes haloperidol. One week
later the GP is called out to see the patient as his parkinsonian symptoms have deteriorated markedly. What is
the most likely underlying diagnosis?

Lewy body dementia


Normal pressure hydrocephalus
Progressive supranuclear palsy
Multiple system atrophy
Dementia pugilistica

♦Patients with Lewy body dementia are extremely sensitive to neuroleptic agents.

Discuss and give feedback

Lewy body dementia

MRCP part 1 Made Easy‫‏‬‎‫‏‬by M. Habayeb & A. Murad

523
Chapter: Neurology

Question 200 of 246

A 62-year-old man is seen in the rapid access transient ischaemic attack clinic following three episodes over
the past two weeks of transient left sided weakness. What is the most appropriate advice to give with
regards to driving?

Cannot drive for 12 months


Cannot drive until investigations complete
Inform DVLA but can continue driving
Cannot drive for 3 months
Cannot drive for 1 month

MRCP part 1 Made Easy‫‏‬‎‫‏‬by M. Habayeb & A. Murad

524
Chapter: Neurology

Question 200 of 246

A 62-year-old man is seen in the rapid access transient ischaemic attack clinic following three episodes over
the past two weeks of transient left sided weakness. What is the most appropriate advice to give with
regards to driving?

Cannot drive for 12 months


Cannot drive until investigations complete
Inform DVLA but can continue driving
Cannot drive for 3 months
Cannot drive for 1 month

DVLA advice post multipler TIAs: cannot drive for 3 months

Discuss and give feedback

DVLA: neurological disorders

MRCP part 1 Made Easy‫‏‬‎‫‏‬by M. Habayeb & A. Murad

525
Chapter: Neurology

Question 201 of 246

A 73-year-old woman presents with episodic confusion and headaches for the past week. She has a
history of alcohol excess and a background of atrial fibrillation and type 2 diabetes mellitus. Her
daughter reports that she has been having frequent spells of confusion over the past few days. Last
year she was assessed for frequent falls. Her current medications include bisoprolol, metformin and
warfarin. Neurological examination is unremarkable and her blood sugar is 6.7 mmol/l. What is the
most likely diagnosis?

Korsakoff's syndrome
Wernicke's encephalopathy
Extradural haematoma
Subarachnoid haemorrhage
Subdural haematoma

526
MRCP part 1 Made Easy‫‏‬‎‫‏‬by M. Habayeb & A. Murad
Chapter: Neurology

Question 201 of 246

A 73-year-old woman presents with episodic confusion and headaches for the past week. She has a
history of alcohol excess and a background of atrial fibrillation and type 2 diabetes mellitus. Her
daughter reports that she has been having frequent spells of confusion over the past few days. Last
year she was assessed for frequent falls. Her current medications include bisoprolol, metformin and
warfarin. Neurological examination is unremarkable and her blood sugar is 6.7 mmol/l. What is the
most likely diagnosis?

Korsakoff's syndrome
Wernicke's encephalopathy
Extradural haematoma
Subarachnoid haemorrhage
Subdural haematoma

Fluctuating confusion/consciousness? - subdural hematoma

This patient has a number of risk factors for a subdural haematoma including old age, alcoholism and
anticoagulation. Korsakoff's syndrome and Wernicke's encephalopathy do not usually cause
headaches.

Discuss and give feedback

Head injury: types of traumatic brain injury

External links

NICE

2014 Head injury guidelines

526
MRCP part 1 Made Easy‫‏‬‎‫‏‬by M. Habayeb & A. Murad
Chapter: Neurology

Question 202 of 246

A 34-year-old man who is known to suffer from complex partial seizures is reviewed in the
neurology clinic. He has not been able to tolerate either carbamazepine or sodium valproate. What is
the most appropriate next line drug?

Phenytoin
Lamotrigine
Ethosuximide
Topiramate
Clonazepam

526
MRCP part 1 Made Easy‫‏‬‎‫‏‬by M. Habayeb & A. Murad
Chapter: Neurology

Question 202 of 246

A 34-year-old man who is known to suffer from complex partial seizures is reviewed in the
neurology clinic. He has not been able to tolerate either carbamazepine or sodium valproate. What is
the most appropriate next line drug?

Phenytoin
Lamotrigine
Ethosuximide
Topiramate
Clonazepam

Discuss and give feedback

Epilepsy: treatment

526
MRCP part 1 Made Easy‫‏‬‎‫‏‬by M. Habayeb & A. Murad
Chapter: Neurology

Question 203 of 246

A 31-year-old woman presents with a 4 month history of headache. She has brought a headache diary
which demonstrates that her symptoms are present on around 20-25 days of each month. The
headache is typically unilateral and she is currently taking paracetamol 1g qds and ibuprofen 400mg
tds everyday to try and relieve her symptoms. A diagnosis of medication overuse headache is
suspected. What is the most appropriate management?

Add metoclopramide + start propranolol


Gradually withdraw analgesics + start propranolol
Abruptly stop analgesics
Gradually withdraw analgesics
Continue analgesics + start propranolol

526
MRCP part 1 Made Easy‫‏‬‎‫‏‬by M. Habayeb & A. Murad
Chapter: Neurology

Question 203 of 246

A 31-year-old woman presents with a 4 month history of headache. She has brought a headache diary
which demonstrates that her symptoms are present on around 20-25 days of each month. The
headache is typically unilateral and she is currently taking paracetamol 1g qds and ibuprofen 400mg
tds everyday to try and relieve her symptoms. A diagnosis of medication overuse headache is
suspected. What is the most appropriate management?

Add metoclopramide + start propranolol


Gradually withdraw analgesics + start propranolol
Abruptly stop analgesics
Gradually withdraw analgesics
Continue analgesics + start propranolol

Medication overuse headache

 simple analgesia + triptans: stop abruptly

 opioid analgesia: withdraw gradually

This answer may seem counterintuitive but it is line with recent guidelines from SIGN, please see the
link provided.

Discuss and give feedback

Medication overuse headache

External links

SIGN

2008 Diagnosis and management of headache in adults

526
MRCP part 1 Made Easy‫‏‬‎‫‏‬by M. Habayeb & A. Murad
Chapter: Neurology

Question 204 of 246

A 25-year-old woman presents with recurrent attacks of 'dizziness'. These attacks typically last
around 30-60 minutes and occur every few days or so. During an attack 'the room seems to be
spinning' and the patient often feels sick. These episodes are often accompanied by a 'roaring'
sensation in the left ear. Otoscopy is normal but Weber's test localises to the right ear. What is the
most likely diagnosis?

Acoustic neuroma
Vestibular neuritis
Benign paroxysmal positional vertigo
Multiple sclerosis
Meniere's disease

526
MRCP part 1 Made Easy‫‏‬‎‫‏‬by M. Habayeb & A. Murad
Chapter: Neurology

Question 204 of 246

A 25-year-old woman presents with recurrent attacks of 'dizziness'. These attacks typically last
around 30-60 minutes and occur every few days or so. During an attack 'the room seems to be
spinning' and the patient often feels sick. These episodes are often accompanied by a 'roaring'
sensation in the left ear. Otoscopy is normal but Weber's test localises to the right ear. What is the
most likely diagnosis?

Acoustic neuroma
Vestibular neuritis
Benign paroxysmal positional vertigo
Multiple sclerosis
Meniere's disease

In sensorineural hearing loss Weber's test localises to the contralateral ear.

Discuss and give feedback

Meniere's disease

External links

Clinical Knowledge Summaries

Meniere's disease guidelines

526
MRCP part 1 Made Easy‫‏‬‎‫‏‬by M. Habayeb & A. Murad
Chapter: Neurology

Question 205 of 246

Which one of the following is least likely to produce a lymphocytosis in the cerebrospinal fluid?

Systemic lupus erythematous


Guillain-Barre syndrome
Viral encephalitis
Partially treated bacterial meningitis
Behcet's syndrome

526
MRCP part 1 Made Easy‫‏‬‎‫‏‬by M. Habayeb & A. Murad
Chapter: Neurology

Question 205 of 246

Which one of the following is least likely to produce a lymphocytosis in the cerebrospinal fluid?

Systemic lupus erythematous


Guillain-Barre syndrome
Viral encephalitis
Partially treated bacterial meningitis
Behcet's syndrome

Discuss and give feedback

Cerebrospinal fluid: raised lymphocytes

526
MRCP part 1 Made Easy‫‏‬‎‫‏‬by M. Habayeb & A. Murad
Chapter: Neurology

Question 206 of 246

Which of the following features is least likely to be found in a patient with tuberous sclerosis?

Shagreen patches
Café-au-lait spots
Retinal hamartomas
Axillary freckling
Renal angiomyolipomata

526
MRCP part 1 Made Easy‫‏‬‎‫‏‬by M. Habayeb & A. Murad
Chapter: Neurology

Question 206 of 246

Which of the following features is least likely to be found in a patient with tuberous sclerosis?

Shagreen patches
Café-au-lait spots
Retinal hamartomas
Axillary freckling
Renal angiomyolipomata

Axillary freckling is seen in neurofibromatosis.

Discuss and give feedback

Tuberous sclerosis

External links

Patient.info

Tuberous sclerosis review

526
MRCP part 1 Made Easy‫‏‬‎‫‏‬by M. Habayeb & A. Murad
Chapter: Neurology

Question 207 of 246

A 25-year-old female with a history of depression presents to her GP with a two day history of
numbness affecting the C6 distribution in her right arm. There is no history of neck pain or injury.
Neurological examination confirms reduced sensation in that dermatome but is otherwise
unremarkable. She reports no similar episodes previously although does describe an episode three
months ago of reduced vision and painful movements in her right eye. This resolved spontaneously
and she did not seek medical attention. What is the most likely diagnosis?

Somatisation disorder
Huntington's disease
Multiple sclerosis
Syringomyelia
Conversion disorder

526
MRCP part 1 Made Easy‫‏‬‎‫‏‬by M. Habayeb & A. Murad
Chapter: Neurology

Question 207 of 246

A 25-year-old female with a history of depression presents to her GP with a two day history of
numbness affecting the C6 distribution in her right arm. There is no history of neck pain or injury.
Neurological examination confirms reduced sensation in that dermatome but is otherwise
unremarkable. She reports no similar episodes previously although does describe an episode three
months ago of reduced vision and painful movements in her right eye. This resolved spontaneously
and she did not seek medical attention. What is the most likely diagnosis?

Somatisation disorder
Huntington's disease
Multiple sclerosis
Syringomyelia
Conversion disorder

The symptoms three months ago were likely due to optic neuritis, a common presenting feature of
multiple sclerosis.

Whilst she has a depression this does not necessarily mean that her symptoms are due to either a
conversion or somatisation disorder. Depression is obviously very common and may indeed be one of
the subtle manifestations of multiple sclerosis.

Discuss and give feedback

Multiple sclerosis: features

External media

Multiple sclerosis - causes, symptoms, diagnosis, treatment, pathology

526
MRCP part 1 Made Easy‫‏‬‎‫‏‬by M. Habayeb & A. Murad
Chapter: Neurology

Question 208 of 246

Neuropathic pain characteristically responds poorly to opioids. However, if standard treatment


options have failed which opioid is it most appropriate to consider starting?

Tramadol
Morphine
Codeine
Oxycodone
Buprenorphine

526
MRCP part 1 Made Easy‫‏‬‎‫‏‬by M. Habayeb & A. Murad
Chapter: Neurology

Question 208 of 246

Neuropathic pain characteristically responds poorly to opioids. However, if standard treatment


options have failed which opioid is it most appropriate to consider starting?

Tramadol
Morphine
Codeine
Oxycodone
Buprenorphine

Discuss and give feedback

Neuropathic pain

526
MRCP part 1 Made Easy‫‏‬‎‫‏‬by M. Habayeb & A. Murad
Chapter: Neurology

Question 209 of 246

A 65-year-old man who is known to have metastatic colorectal cancer presents for review. Since last
been seen he reports being generally stiff and on examination is noted to have diffuse hypertonia.
Which antibodies are most likely to be responsible for this presentation?

Anti-GAD
Anti-Ri
Anti-Hu
Anti-La
Anti-Yo

526
MRCP part 1 Made Easy‫‏‬‎‫‏‬by M. Habayeb & A. Murad
Chapter: Neurology

Question 209 of 246

A 65-year-old man who is known to have metastatic colorectal cancer presents for review. Since last
been seen he reports being generally stiff and on examination is noted to have diffuse hypertonia.
Which antibodies are most likely to be responsible for this presentation?

Anti-GAD
Anti-Ri
Anti-Hu
Anti-La
Anti-Yo

This patient has developed stiff person's syndrome.

Discuss and give feedback

Paraneoplastic syndromes affecting nervous system

526
MRCP part 1 Made Easy‫‏‬‎‫‏‬by M. Habayeb & A. Murad
Chapter: Neurology

Question 210 of 246

Which one of the following factors indicates a poor prognosis in patients with multiple
sclerosis?

Relapsing-remitting disease
Presence of sensory symptoms
Young age of onset
Male sex
Long interval between first two relapses

526
MRCP part 1 Made Easy‫‏‬‎‫‏‬by M. Habayeb & A. Murad
Chapter: Neurology

Question 210 of 246

Which one of the following factors indicates a poor prognosis in patients with multiple
sclerosis?

Relapsing-remitting disease
Presence of sensory symptoms
Young age of onset
Male sex
Long interval between first two relapses

Discuss and give feedback

Multiple sclerosis: prognostic features

526
MRCP part 1 Made Easy‫‏‬‎‫‏‬by M. Habayeb & A. Murad
Chapter: Neurology

Question 211 of 246

A 24-year-old woman with Charcot-Marie-Tooth disease (type 1) asks how likely it is that any future
children will have the disease. What is the most accurate answer?

Three times as likely as background population


25%
Between 5 - 10%
Same as background population
50%

526
MRCP part 1 Made Easy‫‏‬‎‫‏‬by M. Habayeb & A. Murad
Chapter: Neurology

Question 211 of 246

A 24-year-old woman with Charcot-Marie-Tooth disease (type 1) asks how likely it is that any future
children will have the disease. What is the most accurate answer?

Three times as likely as background population


25%
Between 5 - 10%
Same as background population
50%

Charcot-Marie-Tooth disease (hereditary sensorimotor neuropathy type I) is an autosomal


dominant condition and therefore 50% of children will be affected.

Discuss and give feedback

HSMN

526
MRCP part 1 Made Easy‫‏‬‎‫‏‬by M. Habayeb & A. Murad
Chapter: Neurology

Question 212 of 246

A 29-year-old female with progressive dementia and myoclonus is seen in the memory clinic. On
examination, the patient has marked myoclonus with impairment of the concentration and memory
aspects of the Addenbrooke's test. An MRI reveals a 'hockey stick sign'. A few weeks later the patient
develops akinetic mutism and paresis of vertical upgaze. What is the likely diagnosis given the
clinical and radiological findings?

Progressive supranuclear palsy


Variant Creutzfeldt-Jakob disease
Lewy body dementia
Wilson's disease
Huntington's disease

526
MRCP part 1 Made Easy‫‏‬‎‫‏‬by M. Habayeb & A. Murad
Chapter: Neurology

Question 212 of 246

A 29-year-old female with progressive dementia and myoclonus is seen in the memory clinic. On
examination, the patient has marked myoclonus with impairment of the concentration and memory
aspects of the Addenbrooke's test. An MRI reveals a 'hockey stick sign'. A few weeks later the patient
develops akinetic mutism and paresis of vertical upgaze. What is the likely diagnosis given the
clinical and radiological findings?

Progressive supranuclear palsy


Variant Creutzfeldt-Jakob disease
Lewy body dementia
Wilson's disease
Huntington's disease

This is a case of variant Creutzfeldt-Jakob disease (vCJD). The typical presentation is that of a
younger patient with progressive dementia (less rapid the sporadic CJD) with myoclonus and, in the
later stages, mutism and vertical upgaze palsy (found in 50%). An MRI brain reveals a characteristic
'hockey stick sign' where the pulvinar region and dorsomedial thalamus are hyperintense on T2-
weighted imaging (or pulvinar sign where the pulvinar region is hyperintense only). CSF protein for
14-3-3 and periodic sharp wave complexes on the EEG are more commonly seen in sporadic CJD.

Wilson's disease often has extrapyramidal signs with other features such as liver disease and Kayser-
Fleischer rings. Progressive supranuclear palsy would present with a downgaze vertical gaze palsy
ore commonly and Lewy body dementia would have more Parkinsonian features. There is no obvious
family history of Huntington's here and there would be caudate atrophy on the MRI.

Discuss and give feedback

Creutzfeldt-Jakob disease

526
MRCP part 1 Made Easy‫‏‬‎‫‏‬by M. Habayeb & A. Murad
Chapter: Neurology

Question 213 of 246

A 44-year-old woman presents with a three month history of worsening involuntary movements of
the head. These are worse when she is stressed and improved by alcohol. They are not present when
she is sleep. There are no other neurological symptoms of note and neurological examination is
unremarkable other than spotaneous movements of the head which are worse when she looks to either
side. Her father had a similar complaint but never sought medical attention. What is the most likely
diagnosis?

Parkinson's disease
Cerebellar tremor
Huntington's disease
Multiple sclerosis
Essential tremor

Essential tremor is the most common cause of titubation (head tremor). Whilst the majority of
patients will complain of hand tremor titubation may occur in isolation. The tremor is characteristic
as it is worse on movement and during stress and relieved by alcohol and sleep. The family history is
also a pointer.

Discuss and give feedback

Essential tremor

526
MRCP part 1 Made Easy‫‏‬‎‫‏‬by M. Habayeb & A. Murad
Chapter: Neurology

Question 213 of 246

A 44-year-old woman presents with a three month history of worsening involuntary movements of
the head. These are worse when she is stressed and improved by alcohol. They are not present when
she is sleep. There are no other neurological symptoms of note and neurological examination is
unremarkable other than spotaneous movements of the head which are worse when she looks to either
side. Her father had a similar complaint but never sought medical attention. What is the most likely
diagnosis?

Parkinson's disease
Cerebellar tremor
Huntington's disease
Multiple sclerosis
Essential tremor

Essential tremor is the most common cause of titubation (head tremor). Whilst the majority of
patients will complain of hand tremor titubation may occur in isolation. The tremor is characteristic
as it is worse on movement and during stress and relieved by alcohol and sleep. The family history is
also a pointer.

Discuss and give feedback

Essential tremor

526
MRCP part 1 Made Easy‫‏‬‎‫‏‬by M. Habayeb & A. Murad
Chapter: Neurology

Question 214 of 246

A 63-year-old female is reviewed in the rapid access transient ischaemic attack clinic. For the past
three weeks she has been having episodes of transient loss of vision in the right eye. Carotid
ultrasound reveals a 48% stenosis of her right carotid artery and an ECG shows sinus rhythm. She
was started on aspirin 300mg od by her own GP after the first episode. What is the most
appropriate management of this patient?

Warfarin
Clopidogrel
Carotid endarterectomy
Dipyridamole
Aspirin and dipyridamole

Antiplatelet
 TIA: clopidogrel
 ischemic stroke: clopidogrel

♦Carotid artery endarterectomy is recommend if the patient has suffered a stroke or TIA in the carotid
territory and is not severely disabled. It should only be considered if the carotid stenosis is greater
than 70% or 50%, depending on the reporting criteria used - please see below.

NICE Clinical Knowledge Summaries state the following:


Give an antiplatelet drug unless they are taking an anticoagulant drug, when they should be admitted
immediately without giving treatment

 If they are not taking an anticoagulant or an antiplatelet drug, immediately give either
clopidogrel 300 mg (off-label use) or aspirin 300mg.
The 2012 Royal College of Physicians National clinical guidelines for stroke now recommend
using clopidogrel following a TIA. This brings it in line with current stroke guidance.

Discuss and give feedback

Transient ischaemic attack


External links
Royal College of Physicians
2012 National clinical guideline for stroke

526
MRCP part 1 Made Easy‫‏‬‎‫‏‬by M. Habayeb & A. Murad
Chapter: Neurology

Question 215 of 246

A 72-year-old gentleman with a history of hypertension and stroke presents to the emergency
department accompanied by his wife. His wife reported him waking at 7am confused. Repetitive
questioning was a prominent feature. He was observed to be rubbing his hands together during the
event. He was able to recognise his wife and correctly reported his address and date of birth to
paramedics. The event lasted approximately 10 minutes. On arrival to the emergency department he a
Glasgow Coma Scale score of 13 (E3 M6 V4). A full neurological examination was otherwise
normal. What is the most likely diagnosis?

Epilepsy
Transient psychogenic amnesia
Transient global amnesia
Brain tumour
Transient ischaemic attack

526
MRCP part 1 Made Easy‫‏‬‎‫‏‬by M. Habayeb & A. Murad
Chapter: Neurology

Question 215 of 246


A 72-year-old gentleman with a history of hypertension and stroke presents to the emergency
department accompanied by his wife. His wife reported him waking at 7am confused. Repetitive
questioning was a prominent feature. He was observed to be rubbing his hands together during the
event. He was able to recognise his wife and correctly reported his address and date of birth to
paramedics. The event lasted approximately 10 minutes. On arrival to the emergency department he a
Glasgow Coma Scale score of 13 (E3 M6 V4). A full neurological examination was otherwise
normal. What is the most likely diagnosis?

Epilepsy
Transient psychogenic amnesia
Transient global amnesia
Brain tumour
Transient ischaemic attack

●The gentleman in this scenario presents with an episode of transient amnesia. The key to this
question is distinguishing between transient global amnesia and transient epileptic amnesia.

♦Transient global amnesia is a clinical syndrome of uncertain aetiology, characterised by a discrete


episode lasting for a few hours (always less than 24 hours) of anterograde amnesia, retrograde
amnesia, repetitive questioning with an absence of other cognitive or neurological impairments.

♦Diagnostic criteria (in addition to the above features) are as follows; reliable witness to episode,
an absence of head trauma or loss of consciousness at the onset, preserved personal identity and
absence of epileptic features.

♦Epilepsy can present with discreet episodes of amnesia. This syndrome is called transient epileptic
amnesia. Features that suggest epilepsy are; shorter duration (should be less than 1 hour), multiple
attacks, onset on waking from sleep and accompanying epileptic features - e.g. motor automatism,
stereotyped behaviours, limb shaking.

●The correct answer is epilepsy rather than transient epileptic amnesia due to the presence of motor
automatism (rubbing hands together) and the short duration of the event. The previous stroke may be
the underlying cause of his seizure. The mildly reduced score on the Glasgow Coma Scale may
indicate a post-ictal state.
Discuss and give feedback

Epilepsy: classification

526
MRCP part 1 Made Easy‫‏‬‎‫‏‬by M. Habayeb & A. Murad
Chapter: Neurology

External media

Epilepsy (generalized, partial) - simple, complex, tonic, clonic, causes, symptoms

Osmosis – YouTube

Epilepsy

Podmedics - YouTube

526
MRCP part 1 Made Easy‫‏‬‎‫‏‬by M. Habayeb & A. Murad
Chapter: Neurology

Question 216 of 246

A 40-year-old woman who is known to be HIV positive is admitted to the Emergency Department
following a seizure. Her partner reports that she has been having headaches, night sweats and a poor
appetite for the past four weeks. Blood tests and a CT head are arranged:

CD4 89 u/l
CT head Single homogenously-enhancing lesion in the right parietal lobe

What is the most likely diagnosis?

Primary CNS lymphoma


Tuberculosis
Progressive multifocal leukoencephalopathy
Cryptococcus
Toxoplasmosis

526
MRCP part 1 Made Easy‫‏‬‎‫‏‬by M. Habayeb & A. Murad
Chapter: Neurology

Question 216 of 246


A 40-year-old woman who is known to be HIV positive is admitted to the Emergency Department
following a seizure. Her partner reports that she has been having headaches, night sweats and a poor
appetite for the past four weeks. Blood tests and a CT head are arranged:

CD4 89 u/l
CT head Single homogenously-enhancing lesion in the right parietal lobe

What is the most likely diagnosis?

Primary CNS lymphoma


Tuberculosis
Progressive multifocal leukoencephalopathy
Cryptococcus
Toxoplasmosis

This is a difficult question. Toxoplasmosis is the most common cause of brain lesions in HIV
patients. However, around 80% of toxoplasmosis cases involve multiple lesions and the history is
suggestive of lymphoma. Cerebral tuberculosis is much less common than lymphoma in HIV.

Discuss and give feedback

HIV: neurocomplications
Focal neurological lesions

Toxoplasmosis
 accounts for around 50% of cerebral lesions in patients with HIV
 constitutional symptoms, headache, confusion, drowsiness
 CT: usually single or multiple ring enhancing lesions, mass effect may be seen
 management: sulfadiazine and pyrimethamine

© Image used on license from Radiopaedia


Cerebral toxoplasmosis: CT scan with contrast showing multiple ring enhancing lesions

526
MRCP part 1 Made Easy‫‏‬‎‫‏‬by M. Habayeb & A. Murad
Chapter: Neurology

© Image used on license from Radiopaedia


Cerebral toxoplasmosis: MRI (T1 C+) demonstrates multiple small peripherally enhancing nodules
located predominantly in the basal ganglia as well as the central portions of the cerebellar
hemispheres. Only a small amount of surrounding oedema is present.

Primary CNS lymphoma


 accounts for around 30% of cerebral lesions
 associated with the Epstein-Barr virus
 CT: single or multiple homogenous enhancing lesions
 treatment generally involves steroids (may significantly reduce tumour size), chemotherapy
(e.g. methotrexate) + with or without whole brain irradiation. Surgical may be considered for
lower grade tumours

Primary CNS lymphoma: Non-contrast CT demonstrates a hyper-attenuating mass adjacent to


the left lateral ventricle, with no calcification or hemorrhage.

526
MRCP part 1 Made Easy‫‏‬‎‫‏‬by M. Habayeb & A. Murad
Chapter: Neurology

Primary CNS lymphoma: MRI (T1 C+) demonstrates a large multilobulated mass in the right
frontal lobe. It homogeneously enhances and extends to involve the caudate and the periventricular
area. There is significant mass effect.

Differentiating between toxoplasmosis and lymphoma is a common clinical scenario in HIV patients.
It is clearly important given the vastly different treatment strategies. The table below gives some
general differences. Please see the Radiopaedia link for more details.
Toxoplasmosis Lymphoma
Multiple lesions Single lesion
Ring or nodular enhancement Solid (homogenous) enhancement
Thallium SPECT negative Thallium SPECT positive

Tuberculosis
 much less common than toxoplasmosis or primary CNS lymphoma
 CT: single enhancing lesion

Generalised neurological disease


Encephalitis
 may be due to CMV or HIV itself
 HSV encephalitis but is relatively rare in the context of HIV
 CT: oedematous brain

Cryptococcus
 most common fungal infection of CNS
 headache, fever, malaise, nausea/vomiting, seizures, focal neurological deficit
 CSF: high opening pressure, India ink test positive
 CT: meningeal enhancement, cerebral oedema
 meningitis is typical presentation but may occasionally cause a space occupying lesion

526
MRCP part 1 Made Easy‫‏‬‎‫‏‬by M. Habayeb & A. Murad
Chapter: Neurology

Progressive multifocal leukoencephalopathy (PML)


 widespread demyelination
 due to infection of oligodendrocytes by JC virus (a polyoma DNA virus)
 symptoms, subacute onset : behavioural changes, speech, motor, visual impairment
 CT: single or multiple lesions, no mass effect, don't usually enhance. MRI is better - high-
signal demyelinating white matter lesions are seen

AIDS dementia complex

 caused by HIV virus itself


 symptoms: behavioural changes, motor impairment
 CT: cortical and subcortical atrophy

External links
Radiopaedia
Toxoplasmosis vs lymphoma

526
MRCP part 1 Made Easy‫‏‬‎‫‏‬by M. Habayeb & A. Murad
Chapter: Neurology

Question 217 of 246

Which one of the following statements regarding epilepsy in pregnant women is correct?

All pregnant women on antiepileptic medication should take 400mcg a day of folic acid
Antiepileptic drug levels should be monitored throughout pregnancy
The dose of lamotrigine usually needs to be decreased during pregnancy
Pregnant women taking phenytoin should be given vitamin K in the last month of
pregnancy
Sodium valproate is most strongly associated with cleft palate

526
MRCP part 1 Made Easy‫‏‬‎‫‏‬by M. Habayeb & A. Murad
Chapter: Neurology

Question 217 of 246

Which one of the following statements regarding epilepsy in pregnant women is correct?

All pregnant women on antiepileptic medication should take 400mcg a day of folic acid
Antiepileptic drug levels should be monitored throughout pregnancy
The dose of lamotrigine usually needs to be decreased during pregnancy
Pregnant women taking phenytoin should be given vitamin K in the last month of
pregnancy
Sodium valproate is most strongly associated with cleft palate

Discuss and give feedback

Epilepsy: pregnancy and breast feeding

External links

NICE

2012 Epilepsy guidelines

SIGN

2015 Diagnosis and management of epilepsy in adults

526
MRCP part 1 Made Easy‫‏‬‎‫‏‬by M. Habayeb & A. Murad
Chapter: Neurology

Question 218 of 246

A 37-year-old woman with a history of type 2 diabetes mellitus and obesity presents after a late
period. The urinary hCG test is positive. Her current medication is as follows:

Orlistat 120mg tds


Simvastatin 40mg on
Aspirin 75mg od
Metformin 1g bd
Paracetamol 1g qds
Aqueous cream prn

Which one of her medications must be stopped straight away?

Paracetamol
Aspirin
Simvastatin
Orlistat
Metformin

526
MRCP part 1 Made Easy‫‏‬‎‫‏‬by M. Habayeb & A. Murad
Chapter: Neurology

Question 218 of 246

A 37-year-old woman with a history of type 2 diabetes mellitus and obesity presents after a late
period. The urinary hCG test is positive. Her current medication is as follows:

Orlistat 120mg tds


Simvastatin 40mg on
Aspirin 75mg od
Metformin 1g bd
Paracetamol 1g qds
Aqueous cream prn

Which one of her medications must be stopped straight away?

Paracetamol
Aspirin
Simvastatin
Orlistat
Metformin

Simvastatin is contraindicated in pregnancy and must be stopped immediately. Metformin is


sometimes used in pregnancy although many diabetic women are converted to insulin for the duration
of the pregnancy to try and maximise control and minimise complications.

Whilst orlistat is not a known teratogen it should be used with 'caution' in pregnancy according to the
BNF and the benefits are very likely outweighed by risks.

Discuss and give feedback

Prescribing in pregnant patients

Very few drugs are known to be completely safe in pregnancy. The list below largely comprises of
those known to be harmful. Some countries have developed a grading system - see the link.

Antibiotics

 tetracyclines

 aminoglycosides

 sulphonamides and trimethoprim

 quinolones: the BNF advises to avoid due to arthropathy in some animal studies

526
MRCP part 1 Made Easy‫‏‬‎‫‏‬by M. Habayeb & A. Murad
Chapter: Neurology

Other drugs

 ACE inhibitors, angiotensin II receptor antagonists

 statins

 warfarin

 sulfonylureas

 retinoids (including topical)

 cytotoxic agents

The majority of antiepileptics including valproate, carbamazepine and phenytoin are known to be
potentially harmful. The decision to stop such treatments however is difficult as uncontrolled epilepsy
is also a risk

External links

BNF

Co-amoxiclav

GP notebook

Prescribing in pregnancy

Australian government

Prescribing in pregnancy

UKTIS

Co-amoxiclav in pregnancy

526
MRCP part 1 Made Easy‫‏‬‎‫‏‬by M. Habayeb & A. Murad
Chapter: Neurology

Question 219 of 246

Which one of the following is least recognised as causing idiopathic intracranial hypertension?

Oral contraceptive pill


Tetracycline
Ciclosporin
Prednisolone
Vitamin A

526
MRCP part 1 Made Easy‫‏‬‎‫‏‬by M. Habayeb & A. Murad
Chapter: Neurology

Question 219 of 246

Which one of the following is least recognised as causing idiopathic intracranial hypertension?

Oral contraceptive pill


Tetracycline
Ciclosporin
Prednisolone
Vitamin A

Discuss and give feedback

Idiopathic intracranial hypertension

External links

Patient.info

Idiopathic intracranial hypertension review

526
MRCP part 1 Made Easy‫‏‬‎‫‏‬by M. Habayeb & A. Murad
Chapter: Neurology

Question 220 of 246

A 19-year-old presents as she would like to start a combined oral contraceptive pill. During the
history she states that in the past she has had migraine with aura. She asks why the combined oral
contraceptive pill is contraindicated. What is the most appropriate response?

Theoretical risk of ischaemic stroke


Significantly increased risk of ischaemic stroke
Increased frequency of migraines
Migraine is an independent risk factor for venous thromboembolism
Increased severity of migraines

526
MRCP part 1 Made Easy‫‏‬‎‫‏‬by M. Habayeb & A. Murad
Chapter: Neurology

Question 220 of 246


A 19-year-old presents as she would like to start a combined oral contraceptive pill. During the
history she states that in the past she has had migraine with aura. She asks why the combined oral
contraceptive pill is contraindicated. What is the most appropriate response?

Theoretical risk of ischaemic stroke


Significantly increased risk of ischaemic stroke
Increased frequency of migraines
Migraine is an independent risk factor for venous thromboembolism
Increased severity of migraines
Discuss and give feedback

Migraine: pregnancy, contraception and other hormonal factors


SIGN produced guidelines in 2008 on the management of migraine, the following is selected
highlights:

Migraine during pregnancy


 paracetamol 1g is first-line
 aspirin 300mg or ibuprofen 400mg can be used second-line in the first and second trimester

Migraine and the combined oral contraceptive (COC) pill


 if patients have migraine with aura then the COC is absolutely contraindicated due to an
increased risk of stroke (relative risk 8.72)

Migraine and menstruation


 many women find that the frequency and severity of migraines increase around the time of
menstruation
 SIGN recommends that women are treated with mefanamic acid or a combination of aspirin,
paracetamol and caffeine. Triptans are also recommended in the acute situation

Migraine and hormone replacement therapy (HRT)

 safe to prescribe HRT for patients with a history of migraine but it may make migraines worse

526
MRCP part 1 Made Easy‫‏‬‎‫‏‬by M. Habayeb & A. Murad
Chapter: Neurology

Question 221 of 246

Which one of the following statements regarding absence seizures is incorrect?

Typical age of onset of 3-10 years old


Sodium valproate and ethosuximide are first-line treatments
Seizures may be provoked by a child holding their breath
There is a good prognosis
Characteristic EEG changes are seen

526
MRCP part 1 Made Easy‫‏‬‎‫‏‬by M. Habayeb & A. Murad
Chapter: Neurology

Question 221 of 246

Which one of the following statements regarding absence seizures is incorrect?

Typical age of onset of 3-10 years old


Sodium valproate and ethosuximide are first-line treatments
Seizures may be provoked by a child holding their breath
There is a good prognosis
Characteristic EEG changes are seen

Seizures are characteristically provoked by hyperventilation

Discuss and give feedback

Absence seizures

External links

NICE

2012 Epilepsy guidelines

526
MRCP part 1 Made Easy‫‏‬‎‫‏‬by M. Habayeb & A. Murad
Chapter: Neurology

Question 222 of 246

Which one of the following is least associated with Miller-Fisher syndrome?

Anti-GQ1b antibodies
Areflexia
Ataxia
Postural hypotension
Ophthalmoplegia

526
MRCP part 1 Made Easy‫‏‬‎‫‏‬by M. Habayeb & A. Murad
Chapter: Neurology

question 222 of 246

Which one of the following is least associated with Miller-Fisher syndrome?

Anti-GQ1b antibodies
Areflexia
Ataxia
Postural hypotension
Ophthalmoplegia

Postural hypotension due to autonomic involvement is not a feature of Miller Fisher syndrome, but
may be seen in Guillain-Barre syndrome

Discuss and give feedback

Guillain-Barre syndrome

526
MRCP part 1 Made Easy‫‏‬‎‫‏‬by M. Habayeb & A. Murad
Chapter: Neurology

Question 223 of 246

A 67-year-old man who has a history of type 2 diabetes mellitus and benign prostatic hypertrophy
presents with burning pain in his feet. This has been present for the past few months and is getting
gradually worse. He has tried taking duloxetine but unfortunately has received no benefit. Clinical
examination is unremarkable other than diminished sensation to fine touch on both soles. What is the
most suitable initial management?

Carbamazepine
Amitriptyline
Pregabalin
Fluoxetine
Sodium valproate

526
MRCP part 1 Made Easy‫‏‬‎‫‏‬by M. Habayeb & A. Murad
Chapter: Neurology

Question 223 of 246


A 67-year-old man who has a history of type 2 diabetes mellitus and benign prostatic hypertrophy
presents with burning pain in his feet. This has been present for the past few months and is getting
gradually worse. He has tried taking duloxetine but unfortunately has received no benefit. Clinical
examination is unremarkable other than diminished sensation to fine touch on both soles. What is the
most suitable initial management?

Carbamazepine
Amitriptyline
Pregabalin
Fluoxetine
Sodium valproate

Amitriptyline would normally be first choice but given his history of benign prostatic hyperplasia it
is better to avoid amitriptyline due to the risk of urinary retention.
Diabetic neuropathy
NICE updated it's guidance on the management of neuropathic pain in 2013. Diabetic neuropathy is
now managed in the same way as other forms of neuropathic pain:
 first-line treatment: amitriptyline, duloxetine, gabapentin or pregabalin
 if the first-line drug treatment does not work try one of the other 3 drugs
 tramadol may be used as 'rescue therapy' for exacerbations of neuropathic pain
 topical capsaicin may be used for localised neuropathic pain (e.g. post-herpetic neuralgia)
 pain management clinics may be useful in patients with resistant problems
Gastroparesis
 symptoms include erratic blood glucose control, bloating and vomiting
 management options include metoclopramide, domperidone or erythromycin (prokinetic
agents)
External links
NICE
2015 Type 2 diabetes guidelines

NICE
2013 Neuropathic pain guidelines

526
MRCP part 1 Made Easy‫‏‬‎‫‏‬by M. Habayeb & A. Murad
Chapter: Neurology

Question 224 of 246

A 49-year-old man is prescribed procyclidine for Parkinson's disease. What is the mechanism of
action?

Antimuscarinic
Dopamine receptor agonist
Decarboxylase inhibitor
Dopamine receptor antagonist
Monoamine Oxidase-B inhibitor

526
MRCP part 1 Made Easy‫‏‬‎‫‏‬by M. Habayeb & A. Murad
Chapter: Neurology

Question 224 of 246

A 49-year-old man is prescribed procyclidine for Parkinson's disease. What is the mechanism of
action?

Antimuscarinic
Dopamine receptor agonist
Decarboxylase inhibitor
Dopamine receptor antagonist
Monoamine Oxidase-B inhibitor

Discuss and give feedback

Parkinson's disease: management

526
MRCP part 1 Made Easy‫‏‬‎‫‏‬by M. Habayeb & A. Murad
Chapter: Neurology

Question 225 of 246

Which one of the following features is least typically of motor neuron disease?

Fasciculation
Dysarthria
Increased muscle tone
Ataxia
Absent reflexes

526
MRCP part 1 Made Easy‫‏‬‎‫‏‬by M. Habayeb & A. Murad
Chapter: Neurology

Question 225 of 246

Which one of the following features is least typically of motor neuron disease?

Fasciculation
Dysarthria
Increased muscle tone
Ataxia
Absent reflexes

Ataxia is not a feature of motor neuron disease.

Discuss and give feedback

Motor neuron disease: features

External links

Royal College of Physicians

Motor neuron disease: diagnostic pitfalls

Postgraduate Medical Journal

Review of MND

526
MRCP part 1 Made Easy‫‏‬‎‫‏‬by M. Habayeb & A. Murad
Chapter: Neurology

Question 226 of 246

A 35-year-old man presents with progressive weakness of his hands. On examination you notice
wasting of the small muscles of the hand. A diagnosis of syringomyelia is suspected. Which one of
the following features would most support this diagnosis?

Hyper-reflexia in the upper limbs


Loss of vibration sensation in the hands
Loss of temperature sensation in the hands
Loss of light touch sensation in the hands
Fasciculation of the small muscles of the hand

526
MRCP part 1 Made Easy‫‏‬‎‫‏‬by M. Habayeb & A. Murad
Chapter: Neurology

Question 226 of 246

A 35-year-old man presents with progressive weakness of his hands. On examination you notice
wasting of the small muscles of the hand. A diagnosis of syringomyelia is suspected. Which one of
the following features would most support this diagnosis?

Hyper-reflexia in the upper limbs


Loss of vibration sensation in the hands
Loss of temperature sensation in the hands
Loss of light touch sensation in the hands
Fasciculation of the small muscles of the hand

Syringomyelia - spinothalamic sensory loss (pain and temperature)

Discuss and give feedback

Syringomyelia

526
MRCP part 1 Made Easy‫‏‬‎‫‏‬by M. Habayeb & A. Murad
Chapter: Neurology

Question 227 of 246

A 68-year-old man undergoes a lumbar puncture to investigate new confusion. His platelet count is
normal, and his clotting screen is within an acceptable range. The procedure is discussed with his
family as he is unable to consent and they are happy with the procedure but want to ensure that the
possibility of post lumbar puncture headache is minimalised. What factor has been demonstrated
to increase the likelihood of a post-LP headache?

Small needle gauge


Replacing the stylet during the procedure
Keeping the bevel of the needle parallel to the dura
Early mobilisation following procedure
Use of a Quincke (sharp) needle

526
MRCP part 1 Made Easy‫‏‬‎‫‏‬by M. Habayeb & A. Murad
Chapter: Neurology

Question 227 of 246

A 68-year-old man undergoes a lumbar puncture to investigate new confusion. His platelet count is
normal, and his clotting screen is within an acceptable range. The procedure is discussed with his
family as he is unable to consent and they are happy with the procedure but want to ensure that the
possibility of post lumbar puncture headache is minimalised. What factor has been demonstrated
to increase the likelihood of a post-LP headache?

Small needle gauge


Replacing the stylet during the procedure
Keeping the bevel of the needle parallel to the dura
Early mobilisation following procedure
Use of a Quincke (sharp) needle

Sharp needles have been found to be associated with a greater frequency of post-lumbar puncture
headaches when compared to atraumatic needles inserted using an introducer. Using a small needle,
replacing the stylet and maintaining the bevel parallel decreases the chance of developing a post-
lumbar puncture headache, whilst early mobilisation has no impact.

Johnson, Kimberley S., and Daniel J. Sexton. 'Lumbar Puncture: Technique, Indications,
Contraindications, and Complications in Adults.' UpToDate. 3 Feb. 2016.

Discuss and give feedback

Post-lumbar puncture headache

526
MRCP part 1 Made Easy‫‏‬‎‫‏‬by M. Habayeb & A. Murad
Chapter: Neurology

Question 228 of 246

A 51-year-old man with a history of schizophrenia is reviewed. He has developed parkinsonism


secondary to his antipsychotic medication. Which one of the following drugs is most useful in the
management of tremor?

Apomorphine
Cabergoline
Selegiline
Amantadine
Benzhexol

526
MRCP part 1 Made Easy‫‏‬‎‫‏‬by M. Habayeb & A. Murad
Chapter: Neurology

Question 228 of 246

A 51-year-old man with a history of schizophrenia is reviewed. He has developed parkinsonism


secondary to his antipsychotic medication. Which one of the following drugs is most useful in the
management of tremor?

Apomorphine
Cabergoline
Selegiline
Amantadine
Benzhexol

Benzhexol is now more commonly referred to as trihexyphenidyl. It is now mainly used for drug-
induced parkinsonism rather than idiopathic Parkinson's disease

Discuss and give feedback

Parkinson's disease: management

526
MRCP part 1 Made Easy‫‏‬‎‫‏‬by M. Habayeb & A. Murad
Chapter: Neurology

Question 229 of 246

A 45-year-old lady presents with a 2-month history of left-hand weakness. She has no past medical
history. On examination, there is a mild weakness of the left upper and lower limbs with a right sided
facial weakness which spares the forehead. Where is the lesion?

Right cerebrum
Left cerebrum
Right pons
Left pons
Cervical spinal cord

526
MRCP part 1 Made Easy‫‏‬‎‫‏‬by M. Habayeb & A. Murad
Chapter: Neurology

Question 229 of 246

A 45-year-old lady presents with a 2-month history of left-hand weakness. She has no past medical
history. On examination, there is a mild weakness of the left upper and lower limbs with a right sided
facial weakness which spares the forehead. Where is the lesion?

Right cerebrum
Left cerebrum
Right pons
Left pons
Cervical spinal cord

●This is a fairly challenging localisation question but is perhaps not as difficult as it first seems if
you work through the possibilities and eliminate options as you go.

♦A right cerebral lesion would give left upper and lower limb weakness. It would also cause a left
sided facial weakness

♦A left cerebral lesion would give right upper and lower limb weakness with right facial weakness

♦A cervical spinal cord lesion would not cause a facial weakness

►We are left with the pons. The pons is above the level of decussation of the corticospinal tracts so a
pontine lesion would cause a contralateral limb weakness. Based on this information alone you can
conclude that option 3 must be the correct answer without needing to understand why it has caused an
ipsilateral facial weakness

►The facial motor nucleus is located in the pons and supplies the ipsilateral facial muscles

●The difference between an upper and lower motor neuron CN7 lesion is not discussed here but is
worth learning
Discuss and give feedback

Brain lesions

526
MRCP part 1 Made Easy‫‏‬‎‫‏‬by M. Habayeb & A. Murad
Chapter: Neurology

Question 230 of 246

Which one of the following is least associated with myotonic dystrophy?

Dysphagia
Aortic regurgitation
Diabetes mellitus
Testicular atrophy
Learning difficulties

526
MRCP part 1 Made Easy‫‏‬‎‫‏‬by M. Habayeb & A. Murad
Chapter: Neurology

Question 230 of 246

Which one of the following is least associated with myotonic dystrophy?

Dysphagia
Aortic regurgitation
Diabetes mellitus
Testicular atrophy
Learning difficulties

Dystrophia myotonica - DM1

 distal weakness initially

 autosomal dominant

 diabetes

 dysarthria

Discuss and give feedback

Myotonic dystrophy

526
MRCP part 1 Made Easy‫‏‬‎‫‏‬by M. Habayeb & A. Murad
Chapter: Neurology

Question 231 of 246

A 50-year-old man develops chronic, severe pain after sustaining a brachial plexus injury as a result
of motorbike accident. He has had no benefit from paracetamol or ibuprofen. He has had a trial of
amitriptyline which was not successful. Following recent NICE guidelines, what is the most
appropriate medication to consider?

Sertraline
Topical lidocaine
Carbamazepine
Pregabalin
Buprenorphine

526
MRCP part 1 Made Easy‫‏‬‎‫‏‬by M. Habayeb & A. Murad
Chapter: Neurology

Question 231 of 246

A 50-year-old man develops chronic, severe pain after sustaining a brachial plexus injury as a result
of motorbike accident. He has had no benefit from paracetamol or ibuprofen. He has had a trial of
amitriptyline which was not successful. Following recent NICE guidelines, what is the most
appropriate medication to consider?

Sertraline
Topical lidocaine
Carbamazepine
Pregabalin
Buprenorphine

Discuss and give feedback

Neuropathic pain

526
MRCP part 1 Made Easy‫‏‬‎‫‏‬by M. Habayeb & A. Murad
Chapter: Neurology

Question 232 of 246

A 24-year-old woman presents for advice. Over the past few months she has been having increasing
problems with migraine around the time of menstruation. Her current migraine started around 24
hours ago and has not responded to a combination of paracetamol and aspirin. What is the most
appropriate next step to relieve her headache?

Codeine
Ergotamine
Sumatriptan
Venlafaxine
Norethisterone

526
MRCP part 1 Made Easy‫‏‬‎‫‏‬by M. Habayeb & A. Murad
Chapter: Neurology

Question 232 of 246


A 24-year-old woman presents for advice. Over the past few months she has been having increasing
problems with migraine around the time of menstruation. Her current migraine started around 24
hours ago and has not responded to a combination of paracetamol and aspirin. What is the most
appropriate next step to relieve her headache?

Codeine
Ergotamine
Sumatriptan
Venlafaxine
Norethisterone

Oral mefanamic acid would also be a suitable alternative.


Discuss and give feedback

Migraine: pregnancy, contraception and other hormonal factors


SIGN produced guidelines in 2008 on the management of migraine, the following is selected
highlights:

Migraine during pregnancy


 paracetamol 1g is first-line
 aspirin 300mg or ibuprofen 400mg can be used second-line in the first and second trimester

Migraine and the combined oral contraceptive (COC) pill


 if patients have migraine with aura then the COC is absolutely contraindicated due to an
increased risk of stroke (relative risk 8.72).

Migraine and menstruation


 many women find that the frequency and severity of migraines increase around the time of
menstruation
 SIGN recommends that women are treated with mefanamic acid or a combination of aspirin,
paracetamol and caffeine. Triptans are also recommended in the acute situation.

Migraine and hormone replacement therapy (HRT)


 safe to prescribe HRT for patients with a history of migraine but it may make migraines
worse.
526
MRCP part 1 Made Easy‫‏‬‎‫‏‬by M. Habayeb & A. Murad
Chapter: Neurology

Question 233 of 246

A 71-year-old man is reviewed following an ischaemic stroke. He is known to be intolerant of


clopidogrel. What is the most appropriate therapy to help reduce his chance of having a further
stroke?

Aspirin + dipyridamole. Stop dipyridamole after 2 years


Dipyridamole. Stop dipyridamole after 2 years
Aspirin lifelong
Warfarin
Aspirin + dipyridamole lifelong

526
MRCP part 1 Made Easy‫‏‬‎‫‏‬by M. Habayeb & A. Murad
Chapter: Neurology

Question 233 of 246

A 71-year-old man is reviewed following an ischaemic stroke. He is known to be intolerant of


clopidogrel. What is the most appropriate therapy to help reduce his chance of having a further
stroke?

Aspirin + dipyridamole. Stop dipyridamole after 2 years


Dipyridamole. Stop dipyridamole after 2 years
Aspirin lifelong
Warfarin
Aspirin + dipyridamole lifelong

Please see the 2010 NICE guidelines for more details. The 2-year limit has now been removed.

Discuss and give feedback

Stroke: management

External links

NICE

2010 Clopidogrel and dipyridamole guidelines

SIGN

2008 Stroke guidelines

Age and Ageing

Interesting article on managing blood pressure during acute stroke

RCP

Stroke guidelines

NICE

2008 TIA and stroke guidelines

Clinical Knowledge Summaries

Stroke and TIA guidelines

526
MRCP part 1 Made Easy‫‏‬‎‫‏‬by M. Habayeb & A. Murad
Chapter: Neurology

Question 234 of 246

Antiepileptic medication is known to increase the risk of congenital defects. Which one of the
following medications is thought to be associated with the lowest risk?

Levetiracetam
Sodium valproate
Phenytoin
Carbamazepine
Gabapentin

526
MRCP part 1 Made Easy‫‏‬‎‫‏‬by M. Habayeb & A. Murad
Chapter: Neurology

Question 234 of 246

Antiepileptic medication is known to increase the risk of congenital defects. Which one of the
following medications is thought to be associated with the lowest risk?

Levetiracetam
Sodium valproate
Phenytoin
Carbamazepine
Gabapentin

Discuss and give feedback

Epilepsy: pregnancy and breast feeding

External links

NICE

2012 Epilepsy guidelines

SIGN

2015 Diagnosis and management of epilepsy in adults

526
MRCP part 1 Made Easy‫‏‬‎‫‏‬by M. Habayeb & A. Murad
Chapter: Neurology

Question 235 of 246

You review a 65-year-old man who is currently taking antipsychotic medication. His carers have
noticed that his movements have been very slow over the past few weeks. Which one of the
following would suggest a diagnosis of Parkinson's disease rather than drug-induced
parkinsonism?

Rigidity
Masked face
Bilateral symptoms
Flexed posture
Restlessness of arms and legs

526
MRCP part 1 Made Easy‫‏‬‎‫‏‬by M. Habayeb & A. Murad
Chapter: Neurology

Question 235 of 246

You review a 65-year-old man who is currently taking antipsychotic medication. His carers have
noticed that his movements have been very slow over the past few weeks. Which one of the
following would suggest a diagnosis of Parkinson's disease rather than drug-induced
parkinsonism?

Rigidity
Masked face
Bilateral symptoms
Flexed posture
Restlessness of arms and legs

Rigidity and rest tremor are uncommon in drug-induced parkinsonism. Masked face and flexed
posture can be seen in both conditions. Bilateral symptoms are more common in drug-induced
parkinsonism. Restlessness of arms and legs (akathisia) is a common side-effect of antipsychotics.

Discuss and give feedback

Parkinson's disease: features

External media

Parkinsonian gait

mhdbelalalsabek – YouTube

Parkinson's disease

Osmosis – YouTube

526
MRCP part 1 Made Easy‫‏‬‎‫‏‬by M. Habayeb & A. Murad
Chapter: Neurology

Question 236 of 246

Each of the following are causes of peripheral neuropathy. Which one is associated with
predominately sensory loss?

Diphtheria
Hereditary sensorimotor neuropathies
Porphyria
Lead poisoning
Uraemia

526
MRCP part 1 Made Easy‫‏‬‎‫‏‬by M. Habayeb & A. Murad
Chapter: Neurology

Question 236 of 246

Each of the following are causes of peripheral neuropathy. Which one is associated with
predominately sensory loss?

Diphtheria
Hereditary sensorimotor neuropathies
Porphyria
Lead poisoning
Uraemia

Discuss and give feedback

Peripheral neuropathy

526
MRCP part 1 Made Easy‫‏‬‎‫‏‬by M. Habayeb & A. Murad
Chapter: Neurology

Question 237 of 246

A 65-year-old lady presents to clinic with a 6-month history of walking difficulty. She offers a more
recent history of urinary urgency. Examination reveals a short, shuffling gait, mild bilateral, postural
upper limb tremor, and extensor plantar response. She scored 20/30 on an MMSE. What is the most
likely diagnosis?

Idiopathic parkinson's disease


Dementia with Lewy bodies
Parkinson's disease with dementia
Normal pressure hydrocephalus
Vascular dementia

526
MRCP part 1 Made Easy‫‏‬‎‫‏‬by M. Habayeb & A. Murad
Chapter: Neurology

Question 237 of 246

A 65-year-old lady presents to clinic with a 6-month history of walking difficulty. She offers a more
recent history of urinary urgency. Examination reveals a short, shuffling gait, mild bilateral, postural
upper limb tremor, and extensor plantar response. She scored 20/30 on an MMSE. What is the most
likely diagnosis?

Idiopathic parkinson's disease


Dementia with Lewy bodies
Parkinson's disease with dementia
Normal pressure hydrocephalus
Vascular dementia

This case conforms to the classical triad of NPH.

In idiopathic Parkinson's disease, you would expect to see a unilateral upper limb onset

Dementia with Lewy bodies and Parkinson's disease with dementia both present with cognitive
impairment and parkinsonism. In the former, the cognitive impairment precedes the motor
manifestations or occurs within 1 year of the onset of motor manifestations

Discuss and give feedback

Normal pressure hydrocephalus

External links

YouTube

Typical NPH Gait

526
MRCP part 1 Made Easy‫‏‬‎‫‏‬by M. Habayeb & A. Murad
Chapter: Neurology

Question 238 of 246

Which one of the following causes of peripheral neuropathy is most associated with an axonal,
rather than demyelinating, pathology?

Paraprotein neuropathy
Guillain-Barre syndrome
Hereditary sensorimotor neuropathies (HSMN) type I
Amiodarone
Vasculitis

526
MRCP part 1 Made Easy‫‏‬‎‫‏‬by M. Habayeb & A. Murad
Chapter: Neurology

Question 238 of 246

Which one of the following causes of peripheral neuropathy is most associated with an axonal,
rather than demyelinating, pathology?

Paraprotein neuropathy
Guillain-Barre syndrome
Hereditary sensorimotor neuropathies (HSMN) type I
Amiodarone
Vasculitis

The other causes are associated with a demyelinating pathology

Discuss and give feedback

Peripheral neuropathy: demyelinating vs. axonal

526
MRCP part 1 Made Easy‫‏‬‎‫‏‬by M. Habayeb & A. Murad
Chapter: Neurology

Question 239 of 246

The following drugs commonly exacerbate myasthenia gravis, except:

Methotrexate
Gentamicin
Beta-blockers
Quinidine
Penicillamine

526
MRCP part 1 Made Easy‫‏‬‎‫‏‬by M. Habayeb & A. Murad
Chapter: Neurology

Question 239 of 246

The following drugs commonly exacerbate myasthenia gravis, except:

Methotrexate
Gentamicin
Beta-blockers
Quinidine
Penicillamine

Discuss and give feedback

Myasthenia gravis: exacerbating factors

External links

Royal College of Physicians (Edin)

Myasthenia gravis review

Myasthenia Gravis Association

Drugs which may aggravate myasthenia gravis

526
MRCP part 1 Made Easy‫‏‬‎‫‏‬by M. Habayeb & A. Murad
Chapter: Neurology

Question 240 of 246

A 67-year-old woman comes for review with her husband. Her husband complains that she is
constantly getting up from bed at night and pacing around the bedroom. She complains of 'antsy' legs
and a 'horrible, creeping sensation'. Her symptoms generally come on in the evening and are only
relieved by moving round. Given the likely diagnosis, what is the most appropriate treatment?

Ropinirole
Carbamazepine
Amitriptyline
Citalopram
Quinine

526
MRCP part 1 Made Easy‫‏‬‎‫‏‬by M. Habayeb & A. Murad
Chapter: Neurology

Question 240 of 246

A 67-year-old woman comes for review with her husband. Her husband complains that she is
constantly getting up from bed at night and pacing around the bedroom. She complains of 'antsy' legs
and a 'horrible, creeping sensation'. Her symptoms generally come on in the evening and are only
relieved by moving round. Given the likely diagnosis, what is the most appropriate treatment?

Ropinirole
Carbamazepine
Amitriptyline
Citalopram
Quinine

Restless leg syndrome - management includes dopamine agonists such as ropinirole

Discuss and give feedback

Restless legs syndrome

526
MRCP part 1 Made Easy‫‏‬‎‫‏‬by M. Habayeb & A. Murad
Chapter: Neurology

Question 241 of 246

A 45-year-old female with a past medical history of asthma is diagnosed as having essential tremor.
What is the most suitable management?

Amitriptyline
Propranolol
Sodium valproate
Carbamazepine
Primidone

526
MRCP part 1 Made Easy‫‏‬‎‫‏‬by M. Habayeb & A. Murad
Chapter: Neurology

Question 241 of 246

A 45-year-old female with a past medical history of asthma is diagnosed as having essential tremor.
What is the most suitable management?

Amitriptyline
Propranolol
Sodium valproate
Carbamazepine
Primidone

Essential tremor is an AD condition that is made worse when arms are outstretched, made
better by alcohol and propranolol

Propranolol is generally considered first-line in essential but given the history of asthma primidone
should be used

Discuss and give feedback

Essential tremor

526
MRCP part 1 Made Easy‫‏‬‎‫‏‬by M. Habayeb & A. Murad
Chapter: Neurology

Question 242 of 246

Which one of the following antibodies is associated with ocular opsoclonus-myoclonus in


patients with breast cancer?

Anti-Hu
Anti-La
Anti-GAD
Anti-Yo
Anti-Ri

526
MRCP part 1 Made Easy‫‏‬‎‫‏‬by M. Habayeb & A. Murad
Chapter: Neurology

Question 242 of 246

Which one of the following antibodies is associated with ocular opsoclonus-myoclonus in


patients with breast cancer?

Anti-Hu
Anti-La
Anti-GAD
Anti-Yo
Anti-Ri

Discuss and give feedback

Paraneoplastic syndromes affecting nervous system

526
MRCP part 1 Made Easy‫‏‬‎‫‏‬by M. Habayeb & A. Murad
Chapter: Neurology

Question 243 of 246

A 78-year-old man is referred to neurology outpatients. For the past six months he has been troubled
with memory impairment, hallucinations and a resting tremor. On walking into the clinic room he is
noted to have a festinating gait and an expressionless face. He scores 12 / 30 on the mini-mental state
examination (MMSE). Given the likely diagnosis, which one of the following tests is most likely to
confirm the diagnosis?

Serum copper levels


Cerebral angiography
MRI head
SPECT scan
PET scan

526
MRCP part 1 Made Easy‫‏‬‎‫‏‬by M. Habayeb & A. Murad
Chapter: Neurology

Question 243 of 246

A 78-year-old man is referred to neurology outpatients. For the past six months he has been troubled
with memory impairment, hallucinations and a resting tremor. On walking into the clinic room he is
noted to have a festinating gait and an expressionless face. He scores 12 / 30 on the mini-mental state
examination (MMSE). Given the likely diagnosis, which one of the following tests is most likely to
confirm the diagnosis?

Serum copper levels


Cerebral angiography
MRI head
SPECT scan
PET scan

This patient has Lewy body dementia. The findings on conventional imaging such as MRI are
generally non-specific.

Discuss and give feedback

Lewy body dementia

526
MRCP part 1 Made Easy‫‏‬‎‫‏‬by M. Habayeb & A. Murad
Chapter: Neurology

Question 244 of 246

A 24-year-old woman who is 14 weeks pregnant presents with a severe migraine. She has a long
history of migraine and stopped propranolol prophylaxis when she found out she was pregnant.
Unfortunately the headache has not responded to paracetamol 1g. What is the most appropriate
next step?

Ergotamine
Nasal zolmitriptan
Ibuprofen 400mg
Almotriptan 12.5mg
Codeine 30mg

526
MRCP part 1 Made Easy‫‏‬‎‫‏‬by M. Habayeb & A. Murad
Chapter: Neurology

Question 244 of 246


A 24-year-old woman who is 14 weeks pregnant presents with a severe migraine. She has a long
history of migraine and stopped propranolol prophylaxis when she found out she was pregnant.
Unfortunately the headache has not responded to paracetamol 1g. What is the most appropriate
next step?

Ergotamine
Nasal zolmitriptan
Ibuprofen 400mg
Almotriptan 12.5mg
Codeine 30mg

Discuss and give feedback


Migraine: pregnancy, contraception and other hormonal factors
SIGN produced guidelines in 2008 on the management of migraine, the following is selected
highlights:

Migraine during pregnancy


 paracetamol 1g is first-line
 aspirin 300mg or ibuprofen 400mg can be used second-line in the first and second trimester

Migraine and the combined oral contraceptive (COC) pill

 if patients have migraine with aura then the COC is absolutely contraindicated due to an
increased risk of stroke (relative risk 8.72)

Migraine and menstruation


 many women find that the frequency and severity of migraines increase around the time of
menstruation
 SIGN recommends that women are treated with mefanamic acid or a combination of aspirin,
paracetamol and caffeine. Triptans are also recommended in the acute situation

Migraine and hormone replacement therapy (HRT)


 safe to prescribe HRT for patients with a history of migraine but it may make migraines worse

526
MRCP part 1 Made Easy‫‏‬‎‫‏‬by M. Habayeb & A. Murad
Chapter: Neurology

Question 245 of 246

A 39-year-old man is diagnosed as having cluster headaches. He has received subcutaneous


sumatriptan on two occasions but would like to start medication to help prevent further attacks. Of the
following options, which one is the most suitable treatment?

Atenolol
Amitriptyline
Sodium valproate
Verapamil
Gabapentin

526
MRCP part 1 Made Easy‫‏‬‎‫‏‬by M. Habayeb & A. Murad
Chapter: Neurology

Question 245 of 246

A 39-year-old man is diagnosed as having cluster headaches. He has received subcutaneous


sumatriptan on two occasions but would like to start medication to help prevent further attacks. Of the
following options, which one is the most suitable treatment?

Atenolol
Amitriptyline
Sodium valproate
Verapamil
Gabapentin

Discuss and give feedback

Cluster headache

External links

NICE

2012 Headache guidelines

SIGN

2008 Diagnosis and management of headache in adults

526
MRCP part 1 Made Easy‫‏‬‎‫‏‬by M. Habayeb & A. Murad
Chapter: Neurology

Question 246 of 246

A 50-year-old lady presents to clinic with a 12-month history of limb weakness and falls. Her
husband offers a recent history of personality change; she has become disinhibited and sometimes
rude to strangers. Her father suffered from a similar illness. On examination, there is generalised
muscle wasting and shoulder girdle fasciculations. There is a generalised weakness with a brisk left
knee reflex, diminished right knee reflex and a bilateral extensor plantar response. The sensation is
normal throughout. Examination of the cranial nerves was normal. What is the most likely gene
involved?

FXN
NOTCH3
PMP22
C9ORF72
ATXN1

526
MRCP part 1 Made Easy‫‏‬‎‫‏‬by M. Habayeb & A. Murad
Chapter: Neurology

Question 246 of 246

A 50-year-old lady presents to clinic with a 12-month history of limb weakness and falls. Her
husband offers a recent history of personality change; she has become disinhibited and sometimes
rude to strangers. Her father suffered from a similar illness. On examination, there is generalised
muscle wasting and shoulder girdle fasciculations. There is a generalised weakness with a brisk left
knee reflex, diminished right knee reflex and a bilateral extensor plantar response. The sensation is
normal throughout. Examination of the cranial nerves was normal. What is the most likely gene
involved?

FXN
NOTCH3
PMP22
C9ORF72
ATXN1

♦The physical symptoms and signs suggest motor neurone disease evidenced by a mixture of upper
and lower motor neurone signs with normal sensation. The description of personality change
conforms to the description of early behavioural variant frontotemporal dementia.

C9ORF72 is associated with an autosomal dominant inheritance of motor neurone disease and
frontotemporal dementia

FXN is the gene for Friedreich Ataxia


NOTCH3 is the gene for CADASIL

PMP22 is the gene for CMT1A

ATXN1 is the gene for SCA1

Discuss and give feedback

Frontotemporal lobar degeneration

526
MRCP part 1 Made Easy‫‏‬‎‫‏‬by M. Habayeb & A. Murad

You might also like